Вы находитесь на странице: 1из 446

DeVita,

Hellman, and Rosenberg’s

Cancer
Principles & Practice of Oncology

REVIEW
4th Edition

Editors

Ramaswamy Govindan, MD
Professor
Department of Medicine
Division of Oncology
Alvin J. Siteman Cancer Center at Washington University School of Medicine
St. Louis, Missouri

Daniel Morgensztern, MD
Associate Professor
Department of Medicine
Division of Oncology
Alvin J. Siteman Cancer Center at
Washington University School of Medicine
St. Louis, Missouri
Acquisitions Editor: Julie Goolsby
Senior Product Development Editor: Emilie Moyer
Senior Editorial Assistant: Brian Convery
Production Project Manager: Bridgett Dougherty
Design Coordinator: Holly McLaughlin
Manufacturing Coordinator: Beth Welsh
Marketing Manager: Rachel Mante Leung
Prepress Vendor: Aptara, Inc.

4th edition

Copyright © 2016 Wolters Kluwer.

3rd edition Copyright © 2012 and 2nd edition Copyright © 2009 by Lippincott Williams & Wilkins, a Wolters Kluwer
business. Copyright © 2005 by Lippincott Williams & Wilkins.

All rights reserved. This book is protected by copyright. No part of this book may be reproduced or transmitted in any form or
by any means, including as photocopies or scanned-in or other electronic copies, or utilized by any information storage and
retrieval system without written permission from the copyright owner, except for brief quotations embodied in critical
articles and reviews. Materials appearing in this book prepared by individuals as part of their official duties as U.S.
government employees are not covered by the above-mentioned copyright. To request permission, please contact Wolters
Kluwer at Two Commerce Square, 2001 Market Street, Philadelphia, PA 19103, via email at permissions@lww.com, or via our
website at lww.com (products and services).

9 8 7 6 5 4 3 2 1

Printed in China

Library of Congress Cataloging-in-Publication Data

Names: Govindan, Ramaswamy, editor. | Morgensztern, Daniel, editor.


Title: Devita, Hellman, and Rosenberg’s Cancer, principles & practice of
oncology review / editors, Ramaswamy Govindan, Daniel Morgensztern.
Other titles: Cancer, principles & practice of oncology review
Description: 4th edition. | Philadelphia : Wolters Kluwer, [2016] |
Supplement to: DeVita, Hellman, and Rosenberg’s cancer / editors, Vincent
T. DeVita, Jr., Theodore S. Lawrence, Steven A. Rosenberg. 10th ed. [2015]
| Includes bibliographical references and index.
Identifiers: LCCN 2016011573 | ISBN 9781496310804
Subjects: | MESH: Neoplasms | Examination Questions
Classification: LCC RC261 | NLM QZ 18.2 | DDC 616.99/4–dc23
LC record available at http://lccn.loc.gov/2016011573

This work is provided “as is,” and the publisher disclaims any and all warranties, express or implied, including any
warranties as to accuracy, comprehensiveness, or currency of the content of this work.
This work is no substitute for individual patient assessment based upon healthcare professionals’ examination of each
patient and consideration of, among other things, age, weight, gender, current or prior medical conditions, medication
history, laboratory data, and other factors unique to the patient. The publisher does not provide medical advice or guidance
and this work is merely a reference tool. Healthcare professionals, and not the publisher, are solely responsible for the use of
this work including all medical judgments and for any resulting diagnosis and treatments.
Given continuous, rapid advances in medical science and health information, independent professional verification of
medical diagnoses, indications, appropriate pharmaceutical selections and dosages, and treatment options should be made
and healthcare professionals should consult a variety of sources. When prescribing medication, healthcare professionals are
advised to consult the product information sheet (the manufacturer’s package insert) accompanying each drug to verify,
among other things, conditions of use, warnings and side effects and identify any changes in dosage schedule or
contraindications, particularly if the medication to be administered is new, infrequently used or has a narrow therapeutic
range. To the maximum extent permitted under applicable law, no responsibility is assumed by the publisher for any injury
and/or damage to persons or property, as a matter of products liability, negligence law or otherwise, or from any reference to
or use by any person of this work.

LWW.com
TO OUR CONTRIBUTORS
CONTRIBUTORS

Foluso C. Ademuyiwa, MD, MPH


Assistant Professor
Internal Medicine
Washington University School of Medicine
St. Louis, Missouri

Douglas R. Adkins, MD
Professor
Internal Medicine–Medical Oncology
Washington University School of Medicine
St. Louis, Missouri

Rebecca Aft, MD, PhD


Professor
Department of Surgery
Washington University School of Medicine
St. Louis, Missouri

George Ansstas, MD
Assistant Professor
Department of Medicine
Division of Oncology
Washington University School of Medicine
St. Louis, Missouri

Pasquale W. Benedetto, MD
Leonard M. Miller Professor of Medicine
Department of Medicine
University of Miami
Professor of Medicine
Medicine/Division of Hematology–Oncology
University of Miami Sylvester Cancer Center
Miami, Florida

Pavan Kumar Bhamidipati, MBBS, MD


Clinical Fellow
Department of Medicine
Division of Oncology
Washington University School of Medicine
Barnes-Jewish Hospital
St. Louis, Missouri

Leigh M. Boehmer, PharmD, BCOP


Clinical Pharmacist
Medical Oncology
Barnes Jewish Hospital
St. Louis, Missouri

Ron Bose, MD, PhD


Associate Professor
Department of Medicine
Oncology Division
Medical Oncology Section
Department of Cell Biology & Physiology
Washington University School of Medicine
St. Louis, Missouri

Sara K. Butler, PharmD, BCPS, BCOP


Clinical Pharmacy Specialist
Medical Oncology
Barnes-Jewish Hospital
St. Louis, Missouri

Jian Li Campian, MD, PhD


Assistant Professor
Department of Medicine
Division of Oncology
Washington University School of Medicine
St. Louis, Missouri

Richard Chen, MD, PhD


Resident
Department of General
Barnes-Jewish Hospital
St. Louis, Missouri

Ling Chen, MSPH, PhD


Assistant Professor
Division of Biostatistics
Washington University School of Medicine
St. Louis, Missouri

Hak Choy, MD
Professor and Chairman
Nancy B. and Jake L. Hamon Distinguished Chair in Therapeutic Oncology Research
Department of Radiation Oncology
UT Southwestern Medical Center
Dallas, Texas

Amanda F. Cashen, MD
Associate Professor
Department of Medicine
Division of Oncology
Washington University School of Medicine
St. Louis, Missouri

Siddhartha Devarakonda, MD
Clinical Fellow
Division of Hematology/Oncology
Washington University School of Medicine
St. Louis, Missouri

Laura Divine, MD
Clinical Fellow
Department of Obstetrics and Gynecology
Division of Gynecology
Washington University School of Medicine
Barnes-Jewish Hospital
St. Louis, Missouri

Lingling Du, MD
Clinical Fellow
Department of Medicine
Division of Medical Oncology
Washington University School of Medicine
St. Louis, Missouri

Vinicius Ernani, MD
Clinical Fellow
Department of Hematology and Medical Oncology
Emory University School of Medicine
Atlanta, Georgia

Shirish M. Gadgeel, MD
Professor
Department of Oncology
Karmanos Cancer Institute
Wayne State University
Detroit, Michigan
Sarah B. Goldberg, MD, MPH
Assistant Professor
Department of Medicine (Medical Oncology)
Yale School of Medicine
New Haven, Connecticut

Hassan Hatoum, MD
Assistant Professor
Department of Internal Medicine
University of Oklahoma
Health Science Center/Stephenson Cancer Center
Oklahoma City, Oklahoma

Brian Hess, MD
Clinical Fellow
Hematology/Oncology
Washington University School of Medicine
St. Louis, Missouri

Angela C. Hirbe, MD, PHD


Assistant Professor
Department of Medicine
Division of Oncology
Washington University School of Medicine
St. Louis, Missouri

Jennifer Ivanovich, MS, MBA


Assistant Professor
Department of Surgery
Washington University School of Medicine
St. Louis, Missouri

Renuka Iyer, MD
Associate Professor of Oncology
Department of Internal Medicine
Co-Director Liver and Pancreas Tumor Center
Roswell Park Cancer Institute
Buffalo, New York

Tanner M. Johanns, MD, PhD


Clinical Fellow
Department of Internal Medicine
Division of Medical Oncology
Washington University School of Medicine
St. Louis, Missouri
Gregory P. Kalemkerian, MD
Professor
Internal Medicine
University of Michigan
Ann Arbor, Michigan

Jesse Keller, MD
Clinical Fellow
Department of Hematology & Oncology
Washington University School of Medicine
St. Louis, Missouri

Robert A. Kratzke, MD
John Skoglund Chair of Lung Cancer Research
Department of Medicine
University of Minnesota Medical School
Minneapolis, Minnesota

Aaron Laine, MD, PhD


Assistant Professor
Department of Radiation Oncology
UT Southwestern Medical Center
Dallas, Texas

Jessica C. Ley, BS, CCRP


Clinical Research Specialist
Internal Medicine–Medical Oncology
Washington University School of Medicine
St. Louis, Missouri

Gerald P. Linette, MD, PhD


Associate Professor
Department of Medicine
Division of Oncology
Washington University School of Medicine
St. Louis, Missouri

Cynthia X. Ma, MD, PhD


Associate Professor of Medicine
Department of Internal Medicine
Division of Oncology
Washington University School of Medicine
St. Louis, Missouri

Loren Michel, MD
Associate Professor of Medicine
Internal Medicine–Medical Oncology
Washington University School of Medicine
St. Louis, Missouri

Daniel Morgensztern, MD
Associate Professor
Department of Medicine
Division of Oncology
Alvin J. Siteman Cancer Center
Washington University School of Medicine
St. Louis, Missouri

Ashley Morton, MSN, RN, ANP-BC


Nurse Practitioner
Medical Oncology
Washington University School of Medicine
St. Louis, Missouri

David G. Mutch, MD
Vice Chairman of Gynecology
Obstetrics and Gynecology
Washington University School of Medicine
St. Louis, Missouri

Russell Pachynski, MD
Assistant Professor
Department of Medicine
Oncology Division
Medical Oncology Section
Washington University School of Medicine
St. Louis, Missouri

Dilan Patel, MD
Resident
Department of Medicine
Washington University School of Medicine
Barnes-Jewish Hospital
St. Louis, Missouri

Matthew A. Powell, MD
Associate Professor
Obstetrics and Gynecology
Washington University School of Medicine
St. Louis, Missouri

Suresh S. Ramalingam, MD
Assistant Dean for Cancer Research
Professor
Emory University School of Medicine
Deputy Director
Director of Medical Oncology
Hematology and Medical Oncology
Winship Cancer Institute
Atlanta, Georgia

Lee Ratner, MD, PhD


Professor of Medicine
Medicine and Molecular Microbiology
Washington University School of Medicine
St. Louis, Missouri

Thomas Regenbogen, MD
Clinical Fellow
Department of Medicine
Washington University School of Medicine
St. Louis, Missouri

Rizwan Romee, MD
Assistant Professor of Medicine
Medicine/Oncology
Washington University School of Medicine
St. Louis, Missouri

George R. Simon, MD
Professor
Department of Thoracic/Head and Neck Medical Oncology
Division of Cancer Medicine
Chief
Section of Experimental Therapeutics
The University of Texas MD Anderson Cancer Center
Houston, Texas

Preet Paul Singh, MD


Assistant Professor
Department of Medicine
Division of Oncology
Washington University School of Medicine
St. Louis, Missouri

Thomas E. Stinchcombe, MD
Associate Professor
Division of Hematology/ Oncology
University of North Carolina
Chapel Hill, North Carolina

Keith Stockerl-Goldstein, MD
Associate Professor of Medicine
Medicine/Oncology
Washington University School of Medicine
St. Louis, Missouri

Benjamin R. Tan, MD
Assistant Professor of Medicine
Department of Medicine
Division of Oncology
Washington University School of Medicine
St. Louis, Missouri

Kathryn Trinkaus, PhD


Research Statistician
Biostatistics Shared Resource
Siteman Cancer Center
Washington University School of Medicine
St. Louis, Missouri

Viralkumar Vaghani
School of Biomedical Informatics
University of Texas Health Science Center
Houston, Texas

Brian A. Van Tine, MD, PhD


Assistant Professor
Department of Internal Medicine
Washington University School of Medicine
St. Louis, Missouri

Ravi Vij, MD, MBA


Professor
Department of Medicine
Section of Bone Marrow Transplantation and Leukemia
Washington University School of Medicine
St. Louis, Missouri

Nina Wagner-Johnston, MD
Associate Professor of Oncology
John Hopkins Medicine
Director of Lymphoma Drug Development
Johns Hopkins University
Baltimore, Maryland

Andrea Wang-Gillam, MD, PhD


Associate Professor
Department of Medicine
Oncology Division
Medical Oncology Section
Washington University School of Medicine
St. Louis, Missouri

Saiama N. Waqar, MBBS, MSCI


Assistant Professor
Internal Medicine/Division of Oncology
Washington University School of Medicine
St. Louis, Missouri

Megan E. Wren, MD, FACP


Associate Professor
Department of Medicine
Washington University School of Medicine
Associate Program Director, Internal Medicine Residency
Department of Medicine
Barnes-Jewish Hospital
St. Louis, Missouri
PREFACE

We are pleased to publish the fourth edition of the companion review book for DeVita,
Hellman, and Rosenberg’s Cancer: Principles and Practice of Oncology, 10th edition (PPO). While
these review books are often seen as study aids for last-minute cramming for the board
examinations, we hope this book will enable readers to learn key points from each chapter of
the PPO textbook. Each chapter in the review book corresponds to one or more chapters in
the main textbook just as they were in the first three editions. We hope you find this book
useful and informative. Please do not hesitate to contact us with comments, criticisms, and
suggestions.

Ramaswamy Govindan, MD
Daniel Morgensztern, MD
PREFACE TO PREVIOUS EDITION

The past decade has witnessed numerous advances in cancer therapy. Even since the
publication of the previous edition of Cancer: Principles and Practice of Oncology (PPO), or
simply known as the “DeVita book,” several new drugs have been approved for cancer
therapy. Cancer Genome Sequencing projects are going ahead full steam. Molecular
mechanisms that underline the course of several cancer types and responses to specific
therapies are understood better than before. This companion review book, now in its third
edition, is an attempt to cull out the key learning pointsfrom the massive tome of “the DeVita
book” that captures all these advances in a timely manner. While these review books are
often seen as “study-aids” for last minute cramming for the board examinations, we hope this
book would serve to highlight key points from each chapter of PPO. Each chapter in the
review book corresponds to one or more chapters in the main textbook just as they were in
the first two editions. We hope you find this book useful and informative. Please do not
hesitate to contact me with comments, criticisms, and suggestions. You can reach me by
email at rgovinda@dom.wustl.edu.

Ramaswamy Govindan
ACKNOWLEDGMENTS

At the outset, I want to thank the contributors for their diligence, time, and patience.

My special thanks to Dr. Daniel Morgensztern for his hard work, dedication, and commitment
to make this project successful. I want to thank Julie Goolsby and Emilie Moyer from Wolters
Kluwer for their support.

Needless to say, these projects take a sizeable amount of time away from the family. I will
always be grateful to my wife Prabha and my two children, Ashwin and Akshay.

Ramaswamy Govindan

I would like to thank my mentor Ramaswamy Govindan for the opportunity to participate as
a co-editor on the fourth edition of the PPO review, the contributors for their excellent
chapters, and Julie Goolsby and Emilie Moyer from Wolters Kluwer for their constant
support. I am also very grateful to my parents Silvia and Felipe, my lovely wife Marcela and
my three delightful children Alan, David, and Michael.

Daniel Morgensztern
CONTENTS

1. Molecular Biology of Cancer • Part 1


Robert A. Kratzke

2. Molecular Biology of Cancer • Part 2


Viralkumar Vaghani and George R. Simon

3. Epidemiology of Cancer
Thomas E. Stinchcombe

4. Etiology of Cancer
Shirish M. Gadgeel

5. Cancer Screening
Megan E. Wren

6. Cancer Prevention
Lingling Du, Richard Chen, Daniel Morgensztern, and Rebecca Aft

7. Genetic Counseling
Jennifer Ivanovich

8. Principles of Radiation Oncology


Hak Choy and Aaron Laine

9. Systemic Therapy for Cancer


Sara K. Butler and Leigh M. Boehmer

10. Principles of Immunotherapy


Sarah B. Goldberg

11. Assessment of Clinical Responses


Vinicius Ernani and Suresh S. Ramalingam

12. Design and Analysis of Clinical Trials


Ling Chen and Kathryn Trinkaus

13. Cancer of the Head and Neck


Douglas R. Adkins, Jessica C. Ley and Loren Michel

14. Lung Cancer and Mesothelioma


Siddhartha Devarakonda and Saiama N. Waqar

15. Neoplasms of the Mediastinum


Gregory P. Kalemkerian

16. Esophagus and Stomach Cancer


Hassan Hatoum and Renuka Iyer

17. Pancreatic and Hepatobiliary Cancer


Thomas Regenbogen and Andrea Wang-Gillam

18. Small Bowel Cancers and Gastrointestinal Stromal Tumors (GIST)


Benjamin R. Tan, Jr.

19. Colorectal and Anal Cancers


Ashley Morton and Benjamin R. Tan, Jr.

20. Prostate, Bladder, and Kidney Cancer


Russell K. Pachynski

21. Testicular Cancer


Pasquale Benedetto

22. Vulvar, Vaginal, Uterine, and Cervical Cancer


Matthew A. Powell and Laura Divine

23. Ovarian Cancers


David G. Mutch

24. Breast Cancer


Foluso C. Ademuyiwa, Ron Bose, and Cynthia X. Ma

25. Cancers of the Central Nervous System


Tanner M. Johanns, Jian Li Campian and George Ansstas

26. Sarcomas
Brian A. Van Tine and Angela C. Hirbe

27. Cancer of the Skin and Melanoma


Gerald P. Linette

28. Lymphomas
Nina Wagner-Johnston and Dilan Patel

29. Acute Leukemias


Amanda F. Cashen
30. Chronic Leukemias
Pavan Bhamidipati and Keith Stockerl-Goldstein

31. Plasma Cell Neoplasms


Ravi Vij and Jesse Keller

32. Immunosuppression-Related Malignancies


Lee Ratner

33. Stem Cell Transplantation


Brian Hess and Rizwan Romee

34. Oncologic Emergencies


Preet Paul Singh

Index
1 Molecular Biology of Cancer • Part 1
Robert A. Kratzke

QUESTIONS
Each of the numbered items below is followed by lettered answers. Select the ONE lettered answer
that is BEST in each case unless instructed otherwise.

Question 1.1 Completion of the Human Genome Project has revealed that human cells have
a repertoire of genes of which approximate number?
A. 2,500 genes
B. 25,000 genes
C. 250,000 genes
D. 2,500,000 genes

Question 1.2 One of the reasons to use cancer cell culture experiments in preclinical studies
of cancers is:
A. Allows evaluation of cancer cell interaction with the tumor microenvironment.
B. Cell cultures are amenable to easily manipulated experimental techniques.
C. Adaptation of cancer cells to growth in culture corresponds exactly to cancer cell
growth in vivo.
D. Allows evaluation of cancer cell interaction with the native immune system.

Question 1.3 Which of the following is TRUE with regard to genetic mutations in cancer?
A. Gain-of-function mutations (oncogenes) are generally dominant at the cellular level.
B. Loss-of-functions mutations (tumor suppressor genes) are generally recessive at the
cellular level.
C. Ninety percent of germ line mutations in familial cancer syndromes are in tumor
suppressor genes.
D. All of the above.

Question 1.4 Which of the following proteins has inhibitory activity in the cell cycle?
A. Cyclin D1
B. E2F
C. p16INK4a
D. Cyclin-dependent kinase 4

Question 1.5 Which of the following contributes to cancer progression?


A. Autophagy
B. Apoptosis
C. Senescence
D. Angiogenesis

Question 1.6 Which of the following prevents successful invasion and metastasis ?
A. Senescence
B. Angiogenesis
C. Evasion of apoptosis
D. Self-sufficiency in growth signals

Question 1.7 Which of the following best describes the term “protooncogene”?
A. A normal cellular gene that has been transduced by a retrovirus that is then mutated
following viral replication.
B. A homologue of a known oncogenic element identified in prehistoric specimens.
C. A transforming viral gene that can cause malignant transformation in fibroblasts in
vitro.
D. The first oncogene discovered to be associated with human cancer.
E. A viral oncogene that, following infection, is the direct causative agent of human
cancer.

Question 1.8 The DNA damage checkpoints are located in which phase of the cell cycle?
A. G1/S
B. S/G2
C. M
D. All of the above

Question 1.9 Which of the following is a potential flaw in microarray studies?


A. Inadequate controls
B. Biased estimation of prediction accuracy
C. Correlation between clusters and clinical outcome
D. All of the above

Question 1.10 Which of the following statements about miRNAs is/are TRUE?
A. Consist of RNA 19 to 24 nucleotides in length
B. Cannot be evaluated in array format as part of clinical studies
C. Alters gene expression and protein translation
D. A and C

Question 1.11 The proteome is which of the following:


A. The set of all expressed gene products at a given time
B. The proteins expressed preferentially in malignant cells
C. The set of all proteins potentially expressed by the genome
D. The set of protonated peptides subject to matrix-assisted laser desorption ionization
time-of-flight analysis

Question 1.12 Which one of the following statement is CORRECT with regard to molecular
profiling using gene arrays and proteomics?
A. Gene arrays can predict protein–protein interactions.
B. Protein levels and protein function do not correspond directly with gene transcript
levels.
C. Polymerase chain reaction can be used to amplify biopsy material for use in gene
arrays, whereas no signal amplification technology is standard in protein arrays.
D. All of the above
E. B and C

Question 1.13 Which of the following is TRUE about the peptidome?


A. Included peptides must be less than 50,000 daltons.
B. Consists of fragments of larger proteins.
C. May not be amplified in the circulation.
D. Peptide fragments do not bind blood proteins, such as albumin.

Question 1.14 Which of the following statements is CORRECT?


A. Hereditary nonpolyposis colon cancer syndrome (HNPCC) is associated with a 25%
lifetime risk of developing colorectal cancer.
B. Approximately 10% of all cases of colorectal cancer are associated with HNPCC.
C. Microsatellite instability is associated with resistance to 5-fluorouracil chemotherapy.
D. None of the above.

Question 1.15 Which of the following is a DNA hypomethylating agent?


A. Suberoylanilide hydroxamic acid (SAHA)
B. 5-Azacytidine
C. Depsipeptide
D. A and C

Question 1.16 The presence of mutations in p53 has been associated with which of the
following properties on cells:
A. Loss of the G2 checkpoint following treatment with DNA-damaging agents
B. Enhanced capacity to undergo apoptosis following exposure to radiation
C. Increased capacity for DNA amplification
D. A and C

Question 1.17 Which of the following is an example of gene amplification found in cancer?
A. N-myc amplification in neuroblastoma
B. C-myc amplification in small cell lung cancer
C. Her2/neu amplification in breast cancer
D. All of the above

Question 1.18 Which of the following is TRUE regarding microsatellite instability in colon
cancer?
A. Approximately 15% of patients with hereditary nonpolyposis coli have mutations in
MLH1 or MSH2.
B. There is potential resistance to 5-fluorouracil.
C. It has a less favorable prognosis.
D. Evidence is in favor of it occurring only late in sporadic colon cancer cases.

Question 1.19 Which of the following is TRUE about excision repair mechanisms?
A. Increased expression of ERCC1 in non–small-cell lung cancer is associated with response
to cisplatin.
B. There are two nucleotide excision repair (NER) pathways.
C. Base excision repair is involved in response to damage from chemicals and radiographs.
D. B and C.

Question 1.20 ATR/CHK1 signaling is associated with all of the following statements,
A. Bone marrow failure
B. Predisposition to squamous cell carcinoma
C. Predisposition to acute leukemia
D. All of the above

Question 1.21 Which of the following syndromes are associated with abnormalities in the
double-strand repair?
A. Xeroderma pigmentosum
B. Fanconi anemia
C. Lynch syndrome
D. Bloom syndrome

Question 1.22 Decreased expression of Aurora B kinase could lead to the following:
A. Inability for sister chromatids to separate before anaphase
B. Rapid cell division
C. Aneuploidy
D. Increased disassembly of kinetochore proteins

Question 1.23 Which of the following characterizes cytogenetic abnormalities in most


human cancers?
A. Universal (monoclonal) population of cells containing identical cytogenetic
abnormalities
B. Completely normal karyotype
C. Heterogeneous complex karyotypes
D. Complete loss of X or Y chromosomes
Question 1.24 The “Hayflick phenomenon,” which is the name given to the limited
replicative potential of cells, is thought to arise from which of the following?
A. The sequential loss of genetic material from the ends of chromosomes (telomeres) with
each round of division
B. The gradual accumulation of uncorrected genetic defects passed on during division
leading to senescence or malignancy
C. The activation of telomerase in aging cells leading to enzymatic loss of genetic material
from telomeres
D. Dividing eukaryotic cells outgrowing their vascular supply

Question 1.25 Which of the following statements is TRUE regarding telomerase?


A. It has both a DNA and a protein component.
B. It is a DNA topoisomerase.
C. Telomerase protects the integrity of the chromosomal ends.
D. Overexpression of telomerase is found in all cancer specimens.

Question 1.26 Which of the following will induce a quiescent state?


A. Telomere shortening
B. Prolonged DNA damage
C. High-density growth
D. Oncogene activation

Question 1.27 All of the following is/are evidence for senescence as a tumor suppressor
mechanism, EXCEPT:
A. Several “tumor suppressor” proteins that are involved in senescence pathways (e.g.,
p16INK4a) are mutated in familial cancer syndromes.
B. Mice and humans with impaired p16INK4a and p53 function develop normally other
than an age-dependent decrease in cancer and decreased susceptibility to cancer in
response to carcinogen exposure.
C. Reestablishment of p53 activity in sarcoma and hepatocellular carcinoma has led to
cessation of tumor growth.
D. Growth arrest in lung epithelium has been demonstrated in response to oncogenic
events.
E. A, C, and D.

Question 1.28 Which of the following is/are potential therapeutic strategy (ies) to promote
apoptosis in cancer cells?
A. Inhibition of p16INK4a activity
B. Inhibition of p16INK4a activity via activation of DNA methyltransferases
C. Activation of MDM2
D. Promotion of p16INK4a-p53 interactions

Question 1.29 Breakage–fusion–bridge cycles lead to the following:


A. Methylation
B. Aneuploidy
C. Amplifications
D. Deletions
E. B, C, and D

Question 1.30 Telomerase-null mice are associated with which of the following?
A. Decreased sensitivity to radiation
B. Decreased sensitivity to chemotherapy that induces double-strand breaks (DSBs)
C. Decreased genomic stability in the presence of p53 deficiency
D. Decreased rate of spontaneous malignancy

Question 1.31 Which of the following are receptor tyrosine kinases?


A. Platelet-derived growth factor receptor
B. Insulin-like growth factor receptor 1
C. cKit
D. All of the above

Question 1.32 Which of the following is TRUE about receptor tyrosine kinases?
A. They are always monomeric.
B. Activation always requires tyrosine phosphorylation in all classes.
C. Different types of ligands can activate the same class of receptors.
D. Different types of ligands induce the same receptor conformational changes on binding.

Question 1.33 Which of the following is TRUE about receptor phosphotyrosine


phosphatases?
A. Several ligands for these have been described.
B. They rarely function to antagonize tyrosine kinases.
C. Inactivation could increase intracellular phosphorylation under certain circumstances.
D. Receptor phosphatases containing two catalytic domains are rare.
ANSWERS

Question 1.1 The answer is B.


One of the surprising findings of analysis of the sequenced human genome is the presence
of only approximately 25,000 expressed genes in human cells. However, only a few
thousand of these seem to be expressed at any given time. Previous estimates, based on
the known approximate size of human chromosomes, were that there were up to 100,000
or more individual genes expressed in human cells.

Question 1.2 The answer is B.


Preclinical and clinical studies into cancer cell pathogenesis and response to potential
treatments can be guided initially with cell culture experiments because these experiments
can be manipulated in a reproducible manner. However, cell culture experiments do not
allow for evaluation of the interaction between cancer cells and the tumor
microenvironment. Some techniques such as cellular assays that evaluate for cellular
migration and invasion through a Matrigel can mimic the tumor microenvironment, but
they cannot replicate all of the cellular–environmental interactions. In addition, cancer
cells can acquire genetic changes through multiple passages in culture such that they may
not completely reflect in vivo cancer cell–host interactions.

Question 1.3 The answer is D.


Ninety percent of germ line mutations in familial cancer syndromes are in tumor
suppressor genes. It is thought that many dominant oncogene mutations likely cause fetal
demise and therefore do not arise in familial cancer syndromes. Dominant mutations in
cancer genes denote oncogenes and make up 90% of all cancer genes.

Question 1.4 The answer is C.


The cell cycle is composed of the phases G0, G1, S, G2, and M. Regulation of the cell cycle
depends on an interplay of multiple proteins. Cyclin-dependent kinases are
serine/threonine kinases that are ubiquitous in the cell cycle. They generally require
phosphorylation and association with cyclins for activity. Cyclin-dependent kinase 4
phosphorylates retinoblastoma protein, allowing depression of transcription factors (of
which E2F is an example) essential to progression of the cell cycle. A series of inhibitors
(INK4 family) act to decrease the activity of cyclins, of which p16INK4a is a member.

Question 1.5 The answer is D.


Angiogenesis encompasses a cascade of events that allow a tumor’s vascular supply to be
enhanced, thereby increasing cellular proliferation. Apoptosis and autophagy are two
distinct mechanisms of cell death. Apoptosis is regulated via caspase cascades and
mitochondrial-mediated pathways. Autophagy often is induced via nutrient starvation (as
sensed by the mTor kinase) and involves autodigestion of intracellular organelles.
Senescence, the permanent arrest of cell division, is regulated via the ARF-p53 and
p16INK4a-retinoblastoma pathways.
Question 1.6 The answer is A.
Senescence is the permanent arrest of cell division, which would not contribute to cancer
progression via invasion and metastasis. The other choices are all required for such
progression. In addition, insensitivity to growth inhibitory signals and limitless replicative
potential is also necessary for the complex processes of invasion and metastasis.

Question 1.7 The answer is A.


It has been known for almost a century that certain viruses can cause malignant
transformation. The responsible genetic elements identified in the viral genomes are
called viral oncogenes (v-oncogenes). It was found that homologues of these v-oncogenes
existed in the eukaryotic genome from which they had likely originated. These normal
human genetic elements were given the name protooncogene. One of the first of these,
Src, was identified in the avian Rous sarcoma virus that had been studied as a
transforming virus decades earlier. In most human cancers, mutations or aberrant
expressions of oncogenes have been identified.

Question 1.8 The answer is A.


There are three major DNA checkpoints, occurring at G1/S, G2/M, and S. These pathways
promote cell cycle arrest and coordinate the recruitment of repair proteins to the sites of
DNA damage.

Question 1.9 The answer is D.


Microarray technology is dependent on unbiased and rigorous statistical analysis, and as
such, is susceptible to user bias. To help address the continuing need for standardization
guidelines in the clinical use of microarray technology, the Minimum Information About A
Microarray Experiment (MIAME) was published as a demonstration that microarray data
can be independently verified, thereby reducing bias. The answers listed all may
contribute to flawed interpretation of microarray data.

Question 1.10 The answer is D.


MicroRNA (miRNA) consists of an RNA sequence of 19 to 24 nucleotides in length and
participates in cell processes such as apoptosis and development via the effect on gene
expression and protein translation. Recently, miRNA array signatures have been evaluated
in clinical samples from patients with lung cancer, chronic lymphocytic leukemia, and
breast cancer. In a recent lung cancer study, the miRNA profile correlated with survival.

Question 1.11 The answer is C.


The term “proteome” is generally taken to refer to all potentially expressed proteins
encoded by the genome. However, many operational definitions specify only those
proteins expressed at a given time or under certain conditions, so option “A” denotes a
relatively common use of the term as well. Increasingly, the term “proteome” is used in
conjunction with a modifier such as “serum,” cellular,” or even “malignant” to denote a
more limited set of conditions. Protonation is a preparative step before proteomic
evaluation by current means. Most proteomic analyses require cleavage of the proteins in
the samples followed by protonation of the resulting peptides before matrix-assisted laser
desorption ionization time-of-flight analysis.

Question 1.12 The answer is E.


Current gene array technology is focused mostly around the detection of gene transcript
levels. Small quantities of nuclear material from minimal biopsy material can be amplified
via polymerase chain reaction. However, protein–protein interactions and
posttranslational processing cannot be predicted. Multiple technologies have been used to
detect “proteomic signatures,” which include two-dimensional gel electrophoresis, affinity
tagging, liquid chromatography coupled with mass spectrometry, and antibody arrays.
Limitations of proteomic technologies include disruption of the native three-dimensional
protein conformations in addition to the need for larger amounts of biopsy material
needed compared with genomic arrays. This requires sensitive proteomic technology, such
as antibodies in the femtomolar range.

Question 1.13 The answer is A.


The term “peptidome” refers to the low–molecular-weight range of the proteome that
consists of peptides or protein fragments generally less than 50,000 daltons. It is thought
that these peptide fragments are secreted from various tumors and exist in the circulation
bound to carrier proteins. Studies are under way to determine if the secreted pattern of
such fragments can be correlated with disease states. A confounding factor in peptidome
analysis may be the susceptibility of peptides to protease degradation in the blood after
secretion from the tumor. It remains to be seen if consistent patterns can be determined
after protease degradation.

Question 1.14 The answer is C.


HNPCC is associated with a 60% to 80% lifetime risk of developing colorectal cancer and
accounts for 2% to 5% of all cases of colorectal cancer. In both sporadic and HNPCC
cases, microsatellite instability has been associated with a more favorable prognosis but
potential resistance to 5-fluorouracil chemotherapy.

Question 1.15 The answer is B.


SAHA and depsipetide are examples of histone deacetylase inhibitors whereas 5-
azacytidine and 5-Aza-2′-deoxycytidine are DNA hypomethylating agents.

Question 1.16 The answer is D.


Mutations of p53 are among the most commonly identified in human cancers. Wild-type
p53 has a function in signaling cells with damaged DNA to undergo programmed cell
death (apoptosis). In the presence of an inactivating mutation of p53, commonly used
genotoxic agents, such as radiation and chemotherapy, may prove less effective with the
intact p53 signal. This has been demonstrated in preclinical models and correlates with
some human clinical trial data. It must be noted, however, that many common
malignancies harboring p53 mutations, such as small cell lung cancer and ovarian cancer,
are sensitive to chemotherapy and radiation. The capacity of p53 to trigger apoptosis is
associated with the cell-cycle checkpoints that have been identified as critical nodal
moments at which the cell may “choose” to continue to divide or, if sufficiently damaged,
progress down a path to cell death and deletion of the potentially damaged clone.
Interestingly, p53 mutant transgenic mice are viable and develop normally. However,
they have an accelerated rate of tumor formation under certain tumorigenic stimuli.

Question 1.17 The answer is D.


Gene amplification as a mechanism resulting in overexpression of gene products involved
in tumorigenesis or tumor progression has been reported in several cancers. It is known
that N-myc amplification in neuroblastoma can be a useful prognostic factor aiding in
designing therapy. In small cell lung cancer, c-myc amplification has been identified in up
to 10% of specimens, a percentage that may increase after treatment. Her2/neu
amplification occurs in up to 30% of breast cancers and is useful in predicting response to
Her2/neu-targeted therapy.

Question 1.18 The answer is B.


Microsatellite instability refers to frequent mutations in regions of simple repeat
sequences. Early studies in kindreds with HNPCC demonstrated mutations in MSH2 and
MLH1, which are mismatch repair genes. Approximately 85% of patients with HNPCC
have mutations in MSH2 and MLH1. Microsatellite instability is associated with a
resistance to 5-fluorouracil treatment, more favorable prognosis, and lack of p53
mutations. The protein MSH2 complexes with MSH6 to recognize mismatched bases, and
subsequent recruitment of MLH1 and PMS2 initiates the steps of repair (excision, DNA
synthesis, and ligation).

Question 1.19 The answer is D.


The mismatch repair machinery of the cell includes two major pathways: nucleotide
excision repair (NER) and base excision repair (BER). These pathways respond to lesions
created by DNA-damaging agents, such as chemicals and radiation from numerous sources.
Early discoveries in the NER pathway were made via the study of xeroderma
pigmentosum, where patients exhibit extreme sun sensitivity. Two NER pathways have
been described: The first involves global genome scanning for lesions and the second
detects lesions that interfere with RNA polymerases. In BER, the repair machinery
responds mainly to chemical and x-ray damage and base loss by using DNA glycosylates to
remove damaged bases and a complex of APEX1 endonuclease, PARP, DNA
polymerase/ligase, and XRCC1 to recognize abasic sites. As of now, no inherited cancer
syndromes caused by mutations in BER have been discovered.

Question 1.20 The answer is D.


DSBs can promote major cytogenetic abnormalities and are repaired via homologous
recombination or nonhomologous end-joining. ATR/CHK1 signaling leads to activation of
the Fanconi anemia repair pathway. Defects in Fanconi anemia lead to bone marrow
failure, congenital abnormalities, predisposition to acute leukemias and squamous cell
cancer, and increased sensitivity to DNA cross-linking agents such as cisplatin and
mitomycin C.

Question 1.21 The answer is B.


Xeroderma pigmentosum, Lynch syndrome, and Bloom syndrome are caused by
abnormalities in the NER, mismatch repair, and helicase, respectively. Fanconi anemia is
caused by abnormalities in Fanc genes, involved in the double-strand DNA repair.

Question 1.22 The answer is A.


The mitotic spindle is central to cell division, and Aurora kinase B is intricately involved
in the process of cell division. The linking of sister chromatids before anaphase is lost
partly because of the action of Aurora B kinase. Aurora B kinase also phosphorylates other
kinetochore proteins to aid disassembly of kinetochore-microtubule attachments not under
proper tension (as part of spindle checkpoint activation). Amplification of Aurora B kinase
has been associated with sporadic tumors.

Question 1.23 The answer is C.


Human cancers are largely characterized by complex karyotypes with multiple different
populations. In particular, solid tumors such as lung and colon cancer exhibit great
heterogeneity that is likely the result of the high level of genomic instability found in
cancers. Some less common malignancies, such as CML, are characterized by the relatively
uniform presence of an abnormal karyotype, often with a limited number of detectable
breakpoints and translocations. In fact, the presence of a uniform karyotypic abnormality
raises the possibility of a germ line cytogenetic defect that may be further evaluated by
examining the karyotype of noncancerous circulating lymphocytes from the same patient.
The absence of any karyotypic abnormality in cancer cell populations is less common but
can occur. For example, the presence of a normal karyotype is a well-recognized
observation in acute myelogenous leukemia and gives important clinical information
conferring an intermediate prognosis. The loss of a single X or Y chromosome is a normal
variant commonly observed in older patients. It carries little clinical significance.

Question 1.24 The answer is A.


Hayflick and Moorehead documented a landmark finding of cell biology when they
observed that normal eukaryotic cells would only replicate approximately 50 to 100 times
before dying. This limited replicative capacity of nontransformed cells is called the
“Hayflick phenomenon.” It is now known that the ends of the chromosomes, called
“telomeres,” gradually shorten, losing 25 to 200 base pairs with each round of division.
After 50 to 100 rounds of division, the telomeres reach a critically shortened length,
triggering cell senescence. Telomeres can be maintained by the enzymatic activity of
telomerase, which function to restore and maintain telomeres at their full length.
Activation of telomerase is a key mechanism by which some cells, such as cancer cells,
evade senescence. Although gene defects occur spontaneously in cells and will lead to
programmed cell death, this usually occurs in the immediate generation after the defect is
established. Tumor growth does require neovascularization, but the “Hayflick
phenomenon” applies to programmed senescence in nontransformed cells.

Question 1.25 The answer is C.


Telomerase is the DNA polymerase that synthesizes the repeating six base pair motif
(TTAGGG) that comprises the ends of all chromosomes. It is a nucleoprotein with both a
protein and RNA component. With cell division, the telomere ends become progressively
shorter until a critical length is reached and programmed cell death is initiated.
Synthesizing and repairing the ends of the shortening telomeres allows cells to maintain
the integrity of the chromosomal ends and is important for cells that need to divide
without reaching senescence, such as cancer cells. In light of this, it is not surprising that
the majority of cancer cells overexpress telomerase, but there are cancers that appear to
have invoked alternative mechanisms to repair telomeres. Indeed, telomerase-deficient
(deleted) mice can be induced to develop tumors.

Question 1.26 The answer is C.


Senescence refers to a state in cultured cells where growth arrest is permanent.
Senescence differs from quiescence in that quiescent cells can reenter the cell cycle in
response to appropriate mitogenic cues. Quiescence is generally induced by serum
starvation, growth factor deprivation, high-density growth, and transient DNA damage.
Senescence can be induced by prolonged DNA damage, oxidative stress, telomere
shortening, and oncogene activation. Additional differences include uninducible c-fos
expression in response to serum stimulation and increased PAI expression in senescence
versus quiescence.

Question 1.27 The answer is B.


Cell-cycle inhibitors that lead to senescence include p16INK4a and p53, among others. It
follows that mice and humans with impaired p16INK4a and p53 function develop
normally other than an age-dependent increase in cancer and increased susceptibility to
cancer in response to carcinogen exposure. Aside from the other correct answers listed,
other evidence is supportive. Minimal residual disease data demonstrate frequent
oncogenic events as evidence for a need for tumor suppression. Mice are highly
susceptible to inactivation of p16INK4a and/or p53 as evidenced by experiments
demonstrating development of primary tumors at 8 weeks of age (normal life span is
approximately 100 weeks).

Question 1.28 The answer is D.


Knockout of INK4a and ARF loci separately and in combination leads to an increase in
spontaneous tumors. Silencing of p16INK4a via hypermethylation has been well described
and is associated with numerous types of cancers. Inhibition of methylation is thought to
increase expression of p16INK4a. Gene therapy as a method to replace p16INK4a activity
has been achieved in the laboratory. ARF leads to MDM2 activation and subsequent p53
degradation. Inhibition of MDM2 activity is thought to increase p53 levels and lead to
apoptosis and senescence. p53 and p16INK4a do not interact directly.

Question 1.29 The answer is E.


Chromosomal rearrangements are thought to contribute to the malignant phenotype of
many different cancers. Such rearrangements are the result of breakage–fusion–bridge
cycles. Unprotected chromatids can fuse to form a dicentric chromosome. The fused
chromosomes break during anaphase and create atelomeric chromosomes. Atelomeric
chromosomes can fuse with other chromosomes and perpetuate the cycle. Amplification,
deletions, and aneuploidy can arise this way. Methylation is an epigenetic phenomenon
that is not thought to be directly related to chromosome fusion.

Question 1.30 The answer is C.


It is thought that one of the mechanisms of cancer pathogenesis is related to short
telomeres that contribute to genomic instability followed by increased telomerase
activity. Paradoxically, genomic instability can increase with decreased telomerase
activity in the presence of concomitant p53 deficiency, suggesting that telomerase
inhibition by itself is likely not sufficient as a cancer therapeutic. However, increased
sensitivity to radiation and DSB-inducing chemotherapeutics is associated with decreased
telomerase activity and suggests possible additive or synergistic effects.

Question 1.31 The answer is D.


Phosphorylation of tyrosine residues by tyrosine kinases is an important signal for cell
stimulation and cancer growth. Each transmembrane tyrosine kinase is activated by its
associated ligand. Increasingly, tyrosine kinases are an important target for novel
therapeutics as well. Examples of this class of agents already in common use include both
imatinib mesylate (Gleevec) and gefitinib (Iressa). All of the examples listed above are
receptor tyrosine kinases except for Akt. Akt is a threonine/serine kinase and a key
component of the downstream phosphatidylinositol 3 kinase-signaling pathway through
which many of the tyrosine kinases transmit their signaling.

Question 1.32 The answer is C.


Receptor tyrosine kinases generally consist of an extracellular binding domain, a
transmembrane domain, and an intracellular kinase domain. These receptors bind ligands
that are usually peptides or proteins. Most are monomeric with the exception of the
insulin-receptor family, which consists of heterodimers covalently bound by disulfide
bonds. Six major classes exist. Activation of the receptor generally requires
phosphorylation of a tyrosine on the receptor, but the insulin binding to the insulin
receptor is an exception to this rule and activation is generated by an insulin-induced
conformational change. Outside of this, most receptor tyrosine kinases are activated by
oligomerization, which brings intracellular kinase domains into proximity to allow cross-
phosphorylation. Ligands generally stimulate oligomerization. Although receptor classes
have been defined by particular ligands, it is accepted that particular receptor classes may
bind more than one class of ligand. A particular ligand may have different conformational
changes in the same receptor, leading to distinct downstream signal pathway activation.

Question 1.33 The answer is B.


Receptor phosphotyrosine phosphatases (RPTPs) are similar in structure to receptor
tyrosine kinases in that they consist of an extracellular domain, a single transmembrane
domain, and an intracellular catalytic domain (generally two domains). Although no true
ligands for RPTPs have been described, some RPTPs contain structural repeats that
suggest adhesion molecule recognition. Although RPTPs act as phosphatases, they do not
always function in opposition to tyrosine kinases. Particular phosphorylation events can
be inhibitory to tyrosine kinases, and activation of phosphatase activity could then
increase tyrosine kinase global activity.

Corresponding chapters in Cancer: Principles & Practice of Oncology, Tenth Edition: 1 (The Cancer Genome), 2 (Hallmarks of
Cancer: An Organizing Principle for Cancer Medicine), and 3 (Molecular Methods in Cancer).
2 Molecular Biology of Cancer • Part 2
Viralkumar Vaghani and George R. Simon

QUESTIONS
Each of the numbered items below is followed by lettered answers. Select the ONE lettered answer
that is BEST in each case unless instructed otherwise.

Question 2.1 The correct order of mitosis is:


A. Prometaphase, metaphase, anaphase, telophase, prophase
B. Prometaphase, metaphase, prophase, anaphase, telophase
C. Prophase, prometaphase, metaphase, anaphase, telophase
D. Telophase, anaphase, prophase, metaphase, prometaphase

Question 2.2 The cyclin responsible for facilitating the progress of the cell into and through
mitosis is:
A. Cyclin A
B. Cyclin B
C. Cyclin D
D. Cyclin E

Question 2.3 Which of the following are means of regulating cyclin-dependent kinases
(CDKs)?
A. Positive regulatory acetylation
B. Negative regulatory phosphorylation
C. CDK inhibitors
D. B and C
E. A and C

Question 2.4 Which of the following is necessary for the G1/S-phase transition?
A. Cyclin B-CDK1 complex accumulation
B. Cyclins E and A accumulation
C. Dephosphorylation of T14 and Y15
D. CDK1 activation along with binding of CDC20 to anaphase promoting
complex/cyclosome (APC/C)

Question 2.5 Which of the following has no role in cell-cycle exit?


A. Downregulation of CDKs and cyclins
B. Activation of APC/C
C. Reduction in protein synthesis
D. A and C
E. B and C

Question 2.6 Activation of antimitogenic signaling involves the following molecules:


A. Transforming growth factor (TGF)-β
B. Interferon (IFN)-α
C. Upregulation of CDK inhibitors and downregulation of cyclins
D. All of the above.

Question 2.7 Select the FALSE statement regarding microRNAs?


A. microRNAs encode protein kinases that regulate cell cycle.
B. They regulate mRNA expression.
C. There is at least five clusters of microRNAs targeting mRNA that encodes cell-cycle
regulatory proteins.
D. miR-15a/16 cluster targets cyclin E1 and cyclin D1.

Question 2.8 Of the genes listed below, select the gene that is involved in both cell-cycle
regulation and DNA repair?
A. TP53
B. NBS1
C. PTTG1
D. CHK2

Question 2.9 Replicative senescence is initiated by means of:


A. Downregulation of cyclins
B. p53 activation
C. Progressive telomere shortening
D. Upregulation of IFNs

Question 2.10 Genes that encode negative regulators of growth and proliferation are:
A. Protooncogenes
B. Oncogenes
C. S-phase genes
D. Tumor suppressor genes

Question 2.11 Select the syndrome that is associated with a mutation in the gene encoding
Nbs1, leading to microcephaly and a strong predisposition to lymphoid malignancies?
A. Nijmegen disease
B. Von Hippel–Lindau (VHL) syndrome
C. Familial malignant melanoma syndrome
D. Li–Fraumeni syndrome
Question 2.12 Type I cell death is also known as:
A. Autophagy
B. Apoptosis
C. Necrosis
D. Autolysis

Question 2.13 Which of the following proteins serve as are ligands for the death receptor
pathway?
A. TNFα
B. Fas
C. Noxa
D. A and C
E. A and B

Question 2.14 Taxanes specifically target this BH3-only Bcl-2 protein:


A. Nbk/Bik
B. Bim
C. Puma
D. Bad

Question 2.15 RAF is inhibited by which of these therapeutic agents that induce apoptosis?
A. Sorafenib
B. Imatinib mesylate
C. Bortezomib
D. Taxanes

Question 2.16 Bcl-2 small molecule inhibitors like ABT-737 induce apoptosis by:
A. Binding to the BH3-binding pocket.
B. Inhibiting tyrosine kinase activity of Bcr/Abl restoring Bim function.
C. Stimulating Bim expression
D. Blocking proteasome degradation of Bim

Question 2.17 Angiogenesis inhibitors theoretically induce apoptosis by which of the


following means?
A. Tyrosine kinase inhibition
B. Histone deacetylase inhibition
C. Inhibition of antiapoptotic proteins
D. Nutrient deprivation

Question 2.18 Type II programmed cell death, autophagy, is regulated by:


A. mTOR in the PI3-kinase/AKT pathway
B. Bax/Bak
C. BH3-only proteins
D. APC/C

Question 2.19 Autophagy has a role in the following:


A. Recycling of normal cellular components.
B. Protection against aging.
C. Cell lysis caused by surrounding inflammation.
D. A and B.
E. A and C.

Question 2.20 Select the property that is specific to apoptosis?


A. Rapid cellular swelling
B. Lysosome activation
C. Loss of plasma membrane integrity
D. Release of intracellular components to the extracellular compartment

Question 2.21 The Warburg effect describes:


A. Exponential cell growth in response to an exogenous stimulant.
B. Inefficient energy production by most cancer cells, resulting in rapid adenosine
triphosphate (ATP) depletion and necrotic cell death.
C. Initiation of neoplasia requires two somatic mutations for initiation of sporadic
neoplasms and hereditary neoplasms require a genetic plus a somatic mutation.
D. Sculpting of normal human tissues as a result of cell death.

Question 2.22 Bevacizumab is:


A. A monoclonal antibody to basic fibroblast growth factor (bFGF).
B. A monoclonal antibody to vascular endothelial growth factor (VEGF).
C. A monoclonal antibody to phospholipid lipid growth factor (PLGF).
D. A monoclonal antibody to epidermal growth factor receptor (EGFR).

Question 2.23 The first step in tumor angiogenesis is:


A. A localized degradation of the surrounding basement membrane of a parental venule.
B. Paracrine signaling causing increased leakiness of a parental venule’s basement
membrane.
C. An inflammatory response causing recruitment and activation of leukocytes that
mediate angiogenesis
D. Release of endothelial cell (EC) progenitors from the bone marrow

Question 2.24 Which of the following functions is ascribed to pericytes?


A. Paracrine signaling
B. Mediation of resistance to antiangiogenic therapy
C. Enhances endothelial survival
D. All of the above
Question 2.25 Tumor vasculature is characterized by?
A. Increased dilation
B. Increased red blood cell (RBC) flow
C. Increased tortuosity
D. A and B
E. A and C

Question 2.26 Functions ascribed to VEGF are?


A. Stimulation of endothelial cell division
B. Migration of endothelial cells.
C. Enhancing endothelial cell survival.
D. A and B
E. A, B and C

Question 2.27 VEGF is inducible by many factors, of which the most important may be:
A. H1F1α
B. TSP-1
C. pVHL
D. p53

Question 2.28 Which of the following is thought to promote tumor angiogenesis?


A. TSP-1
B. IL-6, IL-8
C. DLL4
D. Vasohibin

Question 2.29 Metronomic low-dose chemotherapy describes:


A. Continuous low-dose treatment with chemotherapeutics
B. Pulse dosing with low doses of chemotherapeutics
C. Continuous low-dose treatment with intermittent pulse dosings of chemotherapeutics
D. Treatment of refractory tumors with continuous low doses of chemotherapeutics in
conjunction with metronidazole

Question 2.30 Which of the following is/are potential toxicity(ies) of antiangiogenic agents?
A. Hypertension
B. Aplastic anemia
C. Arteriothrombotic event
D. A and B
E. A and C

Question 2.31 Which of the following factors is/are associated with risk of metastasis?
A. Tumor grade
B. Depth of invasion beyond normal cellular compartments
C. Lymphovascular invasion
D. All of the above

Question 2.32 The percentage of tumor cells that can give rise to metastases is?
A. <0.01%
B. 1%
C. 20%
D. >80%

Question 2.33 Genes that can mediate tumorigenic functions and secondarily serve
metastatic-specific functions either in a general way or with particular organ selectivity is
best known as:
A. Tumorigenic genes
B. Metastasis progression genes
C. Metastasis virulence genes
D. Protooncogenes

Question 2.34 Cells that mediate the breakdown of the basement membrane allowing for
tumor invasion are known as:
A. Carcinoma-associated fibroblasts
B. Pericyte-derived fibroblast
C. Tumor-associated macrophages (TAMs)
D. Dendritic cells

Question 2.35 Cellular loss of this molecule leads to decreased cellular attachment and
enhanced tumor invasion/motility:
A. E-cadherin
B. β-Catenin
C. α-Tubulin
D. Ankyrin

Question 2.36 The following are the selective pressures experienced by circulating tumor
cells (CTCs)
A. Shear stress
B. Nitric oxide
C. Nutrient deprivation
D. A, B, and C

Question 2.37 Which of the following molecules are known to assist in the extravasation of
CTCs?
A. Ezrin
B. VEGF
C. CXCL12
D. CXCR4
E. All of the above

Question 2.38 Osteoblastic bone metastases are characteristic of which type of cancer?
A. Breast
B. Prostate
C. Lung
D. Renal cell

Question 2.39 Which of the following cancers rarely metastasize to the liver?
A. Colon cancer
B. Breast cancer
C. Prostate cancer
D. Melanoma

Question 2.40 The stochastic model of tumor heterogeneity is best described as:
A. Every cell has equal potential to initiate and sustain tumor growth, but most cells do
not proliferate extensively because of the low cumulative probability of permissive
events.
B. Cancer stem cells (CSCs) are biologically distinct from the bulk cell population, which
does not possess tumor-initiating activity.
C. Tumors are heterogeneous secondary to random, acquired mutations.
D. Tumors are pressured into heterogeneity by variable pressures applied to the tumor.

Question 2.41 The hierarchy model of tumor heterogeneity is best described as:
A. Every cell has equal potential to initiate and sustain tumor growth, but most cells do
not proliferate extensively because of the low cumulative probability of permissive
events.
B. CSCs are biologically distinct from the bulk cell population, which does not possess
tumor-initiating activity.
C. Tumors are heterogeneous secondary to random, acquired mutations.
D. Tumors are pressured into heterogeneity by variable pressures applied to the tumor.

Question 2.42 The drug vemurafenib (PLX4032) targets which gene in the MAPK/ERK
pathway?
A. RAS
B. BRAF
C. BIM
D. ERK

Question 2.43 Which of the following agents targets tumor metabolic pathways?
A. Pemetrexed
B. 5 FU
C. Temsirolimus
D. All of the above
ANSWERS

Question 2.1 The answer is C.


Mitosis is divided into prophase, prometaphase, metaphase, anaphase, and telophase.
Most of the internal membranous components of the cell are dissembled and dispersed in
prophase. Prometaphase prepares the cell for metaphase by forming bivalent attachments
to the spindle driving them to the cellular equator. Paired chromatids align along the
spindle during metaphase, followed by anaphase where sister chromatids pull to opposite
poles. During telophase, prophase is reversed. Mitosis is completed by daughter cells
separating during cytokinesis.

Question 2.2 The answer is B.


Cyclins are the required positive regulatory subunits of CDKs. B-type cyclins, along with
CDK1, are responsible for getting cells into and through mitosis. Cyclin B1 accumulates
during S and G2 phase and then is degraded at the metaphase–anaphase transition.

Question 2.3 The answer is D


After phosphorylation of the T loop, there is an increase in CDK-cyclin contacts and
changes in the binding site. Proteins from the INK4 (p15, p16, p18, and p19), Cip/Kip
(p21Cip1, p27Kip1, and p57Kip2), and Rb (p107 and p130) families all act as inhibitors of
CDKs. Function of CDKs is dependent on cellular availability of cyclins for them to have
enzymatic activity. Last, nuclear import/export regulation further regulates the activity of
CDKs.

Question 2.4 The answer is B.


Accumulation of cyclins E and A via transcriptional induction and interaction with CDK2
allows for entry into S phase from G1 and is regulated by proteins from the Cip/Kip
family. Cyclin B-CDK1 similarly accumulates allowing for the transition into M phase
from G2. Entry into M phase is signaled by the dephosphorylation of T15 and Y15
resulting in the activation of CDK1. Activation of CDK1 and binding of CDC20 to APC/C
is the trigger for which sister chromatids separate and move to opposite poles during
anaphase.

Question 2.5 The answer is D.


Positive cell-cycle machinery is dismantled after the reduction of CDKs and cyclins. Cell-
cycle exit is also usually associated with a transient increase in the number of CDK
inhibitors, such as those from the INK4, Cip/Kip, and Rb protein families. Cells in
quiescence also have a reduced rate of protein synthesis. Protein synthesis is largely
dependent on growth factors and mitogens activating the cell through the mitogen-
activated protein kinase/extracellular signaling-regulated kinase pathway and the
phosphoinositide 3 (PI3) kinase/AKT pathway.

Question 2.6 The answer is D


TGF-β is a cytokine that binds to a class of transcription factors known as SMADs that
translocate to the nucleus, where they complex with DNA-binding transcription factors
and coactivators to transactivate specific genes. IFNs are another class of cytokines that
have antiproliferative effects by means of upregulation of CDK inhibitors and
downregulation of cyclins. INK family proteins are CDK inhibitors of which p15
specifically inhibits CDK4 and CDK6 directly and cyclin E-CDK2 and cyclin A-CDK2
indirectly. The final outcome of antimitogenic signaling involves the upregulation of CDK
inhibitors and downregulation of cyclins.

Question 2.7 The answer is A.


MicroRNAs regulate mRNA expression by degradation or inhibition of translation. Some
of them target mRNA encoding cell-cycle regulatory proteins but none of them directly
encode proteins that regulate cell cycle.

Question 2.8 The answer is B.


All of these genes are involved in regulation of cell cycle. The NBS1 gene regulates
checkpoints in cell cycles and also is involved in DNA repair. Mutation involving the NBS1
gene results in the Nijmegen breakage syndrome 1. Patients diagnosed with this syndrome
have increased risk of developing non-Hodgkins lymphoma.

Question 2.9 The answer is C.


Secondary to the topology of telomeres and the requirements of DNA replication, each
cell cycle causes progressive shortening of the telomeres. In most, if not all, somatic cells,
when sufficient telomere attrition has been reached, cells enter into a chronic checkpoint
response, which is the molecular basis of senescence. Germ line cells forego this
checkpoint by means of a special replicase, telomerase. For a somatic cell to undergo
malignant transformation, the senescence barrier must be overcome allowing for
unlimited proliferative capacity.

Question 2.10 The answer is D.


Tumor suppressor genes usually encode negative regulators of growth and proliferation,
thereby conferring protection to cells against malignancy. Protooncogenes encode genes
in which mutations may cause gain of function or an enhanced level of function leading to
malignancy. Mutations of both tumor suppressor genes and protooncogenes may lead to
uncontrolled proliferation and malignancy.

Question 2.11 The answer is A.


Li–Fraumeni syndrome is caused by an inherited mutation in TP53, the gene encoding the
checkpoint effector p53. Nijmegen disease is caused by mutation of the gene encoding
Nbs1, which is required for the activation of Chk1 and Chk2 kinases. This disease is
characterized by microcephaly, immunodeficiency, increased sensitivity to radiation, and
a predisposition to lymphoid malignancies. VHL is caused by a mutation in the pVHL
tumor suppressor gene causing development of sporadic hemangioblastomas and clear-cell
renal carcinomas. Familial malignant melanoma syndrome is caused by mutation in
CDKN4A, the gene encoding p16.

Question 2.12 The answer is B.


Apoptosis is a genetically programmed means of rapid and efficient killing of unnecessary
or damaged cells. It is characterized by cell shrinkage, blebbing of the plasma membrane,
chromatin condensation, and intranucleosomal DNA fragmentation without being
followed by surrounding inflammation.

Question 2.13 The answer is E


The death receptor pathway is involved in the modulation of apoptosis. It involves the
ligands TNFα, Fas, TRAIL, and their receptors. Noxa is a member of the BH3-only protein
family and can induce apoptosis.

Question 2.14 The answer is B.


Taxanes specifically induce apoptosis by stimulating Bim expression. Nbk/Bik stimulates
apoptosis by inhibiting protein synthesis. Puma and Noxa mediate apoptosis by p53
activation. After growth factor withdrawal, Bad stimulates apoptosis.

Question 2.15 The answer is A.


Sorafenib directly inhibits RAF, thereby causing MAP kinase inhibition and restoring
apoptotic function. Imatinib mesylate, by means of blocking the constitutively active
tyrosine kinase activity of the Bcr/Abl fusion gene, restores Bim and Bad apoptotic
function. Bortezomib is a proteasome inhibitor that blocks Bim degradation, knowing it to
induce apoptosis. Taxanes induce apoptosis by stimulating Bim expression.

Question 2.16 The answer is A.


Bcl-2 inhibitors like ABT-737 binds to the BH3-binding pocket of Bcl-2, Bcl-xL, and Bcl-w,
thereby blocking the antiapoptotic effect of Bcl-2. ABT-737 exhibits some activity against
some human lymphomas and small cell lung cancers in vitro. Inhibition of the tyrosine
kinase activity of Bcr/Abl is accomplished through imatinib mesylate. Taxanes stimulate
Bim expression, restoring apoptosis. Bortezomib blocks the proteasomal degradation of
Bim.

Question 2.17 The answer is D.


Tumor cells generally have a reduced metabolic capacity that is frequently coupled with
high-energy demand sustaining rapid cell growth. One means for specifically targeting
tumor cells is through therapeutic nutrient deprivation. This is the basis for the use of
angiogenesis inhibitors.

Question 2.18 The answer is A.


Autophagy is regulated by mTOR in the PI3-kinase/AKT pathway that functions to link
nutrient availability to cellular metabolism. Defective autophagy has been implicated in
carcinogenesis in certain human breast, ovarian, and prostate tumors. Type I programmed
cell death, apoptosis, is regulated by Bax/Bak and BH3-only proteins. APC/C is the
anaphase promoting complex/cyclosome that regulates sister chromatid separation in
mitosis.

Question 2.19 The answer is D.


Autophagy is involved in recycling normal cellular constituents, damaged
protein/organelle removal, innate immunity by removal of cells infected by intracellular
pathogens, acquired immunity by promoting T-cell survival/proliferation, and protection
against aging. Cell lysis secondary to surrounding inflammation or insult is a feature of
necrosis.

Question 2.20 The answer is B.


Lysosome activation is a feature of apoptosis, as is cell shrinkage, blebbing of the plasma
membrane, chromatin condensation, and intranucleosomal DNA fragmentation without
being followed by surrounding inflammation. Necrosis is characterized by rapid swelling
of the cell, loss of plasma membrane integrity, and release of intracellular components
into the extracellular compartment resulting in an acute inflammatory response.

Question 2.21 The answer is B.


The Warburg effect describes an observation where most cancer cells produce energy by
anaerobic glycolysis even when oxygen is plentiful. This is an inefficient way of producing
energy that generates less ATPs per mole of glucose metabolized, which explains the high
glucose requirement by tumors.

Question 2.22 The answer is B.


Bevacizumab is the first successful antiangiogenic agent that has shown activity in
randomized phase III trials. It is a monoclonal antibody to VEGF. It has been shown to be
effective in combination with chemotherapy for the treatment of metastatic colorectal
cancer, breast cancer, renal cell carcinoma, and non–small-cell lung cancer.

Question 2.23 The answer is A.


Tumor angiogenesis is first mediated by several proteolytic enzymes, such as matrix
metalloproteinases, and urokinase plasminogen activator, causing a focal degradation of a
venule’s basement membrane. This is likely a consequence of various proangiogenic
growth factors secreted by the tumor-cell population or reactive stromal cells.

Question 2.24 The answer is B.


The single layer of periendothelial smooth muscle cells is formed by pericytes. These cells
modulate EC function and are critical for the development of a mature vessel network. In
addition to regulation of vessel diameter and permeability, they provide mechanical
support and survival of ECs through paracrine signaling. Because of their ability to
maintain EC survival, they have become an important target for antiangiogenic therapy
and it is hypothesized that they may mediate resistance to this therapy.

Question 2.25 The answer is E.


The vasculature of solid tumors has several notable abnormalities. These vessels are
classically much dilated with little to no basement membrane and have excessive
tortuosity. A decreased number of pericytes portrays a relative lack of support and
excessive vascular leakiness. These features contribute to the highly heterogeneous and
often sluggish flow through solid tumors and create areas of relative nutrient deprivation
and hypoxia. The increased leak from these vessels also allows for extravasation of high–
molecular-weight plasma proteins, leading to areas of elevated interstitial fluid pressures.

Question 2.26 The answer is E.


VEGF promotes tumor angiogenesis is various ways, including stimulation of EC division,
inducing EC locomotion/migration, enhancing EC survival by upregulating various
inhibitors of apoptosis, and mobilizing endothelial progenitor cells from the bone marrow
to sites of angiogenesis. Further, VEGF is approximately 50,000 times more potent than
histamine in increasing vascular permeability. VEGF is expressed in most if not all of
human cancers, and increased levels carry a poor prognosis.

Question 2.27 The answer is A.


H1F1α is upregulated by hypoxia. H1F1α then, in turn, activates numerous other genes,
of which VEGF may be the most important.

Question 2.28 The answer is B.


IL-6/8 is thought to be secondarily angiogenic. TSP-1, DLL4, and vasohibin are all shown
to inhibit angiogenesis. Embryologic studies have shown that Notch/DLL4 is important in
early angiogenesis; however, in the adult, it has been shown to be an inhibitor of tumor
angiogenesis.

Question 2.29 The answer is A.


The effectiveness of antiangiogenic therapy is profoundly reversed during episodes of
drug-free breaks. This is thought to be mediated by the rapid mobilization and homing of
ECs to the drug-treated tumors. This has led to the concept of metronomic low-dose
chemotherapy. By reducing or eliminating altogether drug-free periods with continuous
low-dose chemotherapy, the rapid reversal of antiangiogenic therapy may be prevented.

Question 2.30 The answer is E.


Antiangiogenic therapy is not without side effects. Anti-VEGF therapy has been shown to
lead to specific toxicities, such as hypertension, proteinuria, bowel perforation,
hemorrhage, arteriothrombotic events, and others. Among these, aplastic anemia has not
been documented.

Question 2.31 The answer is D.


Tumor grade, depth of invasion, and lymphovascular invasion all carry a higher risk of
metastasis. Proper immune function would hinder the development of metastasis because
CTCs would be targeted for destruction.

Question 2.32 The answer is A


By using radioactive nucleotides incorporated into tumor DNA, it has been shown that less
than 0.01% of tumor cells can give rise to metastases.

Question 2.33 The answer is B.


Metastasis progression genes promote tumor metastases among other functions.
Tumorigenic genes are genes that promote primary tumor growth. Metastasis virulence
genes confer protection to tumor cells in the distant environment but confer no protection
to the primary tumor.

Question 2.34 The answer is C.


TAMs can comprise a large percentage of the bulk of a tumor mass. Through secretion of
FGF, EGF receptor ligands, and PDGF, they can stimulate tumor cell growth and motility.
Furthermore, by producing uPA, MMP7, and MMP9, TAMs help degrade basement
membrane allowing tumor cells to invade and metastasize.

Question 2.35 The answer is A.


Invasion starts with loss of cellular adhesion. This is largely mediated by loss of E-
cadherin, the prototypic protein of the cadherin family. Cadherins are transmembrane
glycoproteins that mediate cellular attachment. They anchor the cell through attachments
to the actin cytoskeleton via another family of proteins called the catenins.

Question 2.36 The answer is D.


CTCs experience multiple stressors in the circulation, leading to a short half-life. Of these,
physical stressors such as shear forces or mechanical injury may limit CTC life span. Also,
EC-mediated stressors such as nitric oxide may induce apoptosis in CTCs. DARC, a Duffy
blood group glycoprotein, interacts with KAI1 expressed on CTCs causing them to
undergo senescence.

Question 2.37 The answer is E.


Ezrin links the cell membrane to the actin cytoskeleton and has been found to assist in
extravasation of CTCs via activation of the MAPK pathway. VEGF has been shown to
assist in CTC extravasation by causing disruptions in EC junctions and increased vascular
permeability. CTC expression of CXCR4 interacts with CXCL12 expressed on certain
organs, such as lung, liver, bone, and lymph nodes, allows for selective extravasation.

Question 2.38 The answer is B.


Breast, lung, and kidney cancer typically cause osteolytic lesions on metastasizing to
bone. Unlike these lesions, prostate cancer causes osteoblastic lesions on metastases to
bone. In contrast with osteolytic metastasis, osteoblastic lesions result from the
preferential stimulation of osteoblasts or the inhibition of osteoclasts. This is mediated
through many different molecules, including bone morphogenetic proteins such as WNT,
TGF-β, IGF, PDGF, FGF, and VEGF.

Question 2.39 The answer is C.


Because of the microenvironment the liver provides and the dual blood supply (from the
portal vein and hepatic artery), the liver is a common site of metastasis. Colon cancer
generally seeds the liver via the portal system, whereas breast, lung, and melanoma
approach the liver via the systemic circulation through the hepatic artery.

Question 2.40 The answer is A.


In the stochastic model of tumor heterogeneity, every cell within the tumor has equal
potential but low probability of initiating tumor growth.

Question 2.41 The answer is B.


The hierarchy model of tumor heterogeneity hypothesizes the existence of functionally
distinct classes of cells within the tumor, and only those with unique self-renewal ability
produce new tumor growth.

Question 2.42 The answer is B.


PLX4032 is an orally available inhibitor of the mutated BRAF gene. It has shown activity
in patients with malignant melanoma that carry the BRAF V600E mutation. About 60% of
all patients with melanomas have the V600E BRAF mutation.

Question 2.43 The answer is D.


Pemetrexed is a folate antimetabolite that targets enzymes involved in purine and
pyrimidine synthesis. 5-Fluorouracil is an antimetabolite that targets thymidylate
synthase. Temsirolimus is mammalian target of rapamycin (mTOR) inhibitor that targets
the PI3-kinase pathway. The PI3-kinase pathway is involved in regulation of glucose
metabolism.

Corresponding Chapters in Cancer: Principles & Practice of Oncology, Tenth Edition: 1 (The Cancer Genome), 2 (Hallmarks of
Cancer: An Organizing Principle for Cancer Medicine), and 3 (Molecular Methods in Cancer).
3 Epidemiology of Cancer
Thomas E. Stinchcombe

QUESTIONS
Each of the numbered items below is followed by lettered answers. Select the ONE lettered answer
that is BEST in each case unless instructed otherwise.

Question 3.1 Which of the following is CORRECT about cohort studies?


A. Cohort studies only follow subjects prospectively
B. Subjects are free of disease at the time of study entry
C. The incidence of the disease in the exposed group is compared to members of the
general population who have been matched for selected characteristics
D. Cohort studies are useful for estimating the prevalence of a disease

Question 3.2 Potential weaknesses in case-control studies include:


A. Selection biases related to choosing and enrolling subjects
B. Recall biases related to previous exposures
C. Cases may not be representative of disease
D. All of the above

Question 3.3 Confounding factors and interaction (also known as effect modification) are
common in epidemiologic studies. Which of the following statements is TRUE in regards to
confounding and interaction?
A. Confounder is not associated with exposure
B. Confounder is associated with the disease and dependent on the exposure
C. Confounder may be an intermediate step between exposure and disease
D. Confounder is not a true relationship and interaction is true relationship between an
exposure and disease

Question 3.4 Statistical methods are required to evaluate the role of chance, and a usual way
to estimate chance is to calculate the upper and lower limits of a 95% confidence interval
around a point estimate for relative risk. Which of the following are TRUE related to the use
and interpretation of confidence intervals? (Select two correct responses)
A. If the confidence interval does not include 1, the observed association is statistically
significant. If the confidence includes 1, then the observed relationship is not
statistically significant
B. The width of the confidence interval is directly related to the number of participates in
a study or sample size
C. The choice of 95% confidence interval is commonly used, but the upper and lower
limits of the interval (e.g., 90% or 99%) can certainly be calculated and interpreted
accordingly
D. All of the above

Question 3.5 Dr. Bush wishes to study the relationship between diet soda consumption and
the development of gastric cancer. He selects the cases of gastric cancer detected over the
last 15 years, and then to obtain his controls he selects for patients who referred to the
gastroenterology clinic. This is as an example of:
A. Information bias
B. Selection bias
C. Misclassification bias
D. All of the above

Question 3.6 For which of the following analytical studies is the group the unit of analysis?
A. Ecologic
B. Cross-sectional
C. Cohort
D. Case-control

Question 3.7 Genome-wide association studies (GWAS) have revealed an association


between CHRNA3 and CHRNA5, and lung cancer
A. True
B. False

Question 3.8 Which of the following cancer is increasing in incidence?


A. Lung cancer among women
B. Colorectal cancer
C. Breast cancer
D. Melanoma

Question 3.9 The discontinuation of the routine use hormone replacement therapy following
the publication of the Women’s Health Initiative study is thought to contribute to the
declining rate of which of these cancers
A. Breast cancer
B. Lung cancer
C. Colorectal cancer
D. Stomach cancer

Question 3.10 Which of the following is TRUE about the Surveillance, Epidemiology, and
End Results (SEER) Program?
A. It covers approximately 10% of the US population
B. Staging, treatment and outcome assessment is highly standardized
C. It provides the most precise measure of long-term trends in cancer incidence
D. All of the above
ANSWERS

Question 3.1 The answer is B.


Patients in cohort studies are free of the specific disease and then are followed for a
certain period of time, often years. A cohort study can prospective or retrospective based
on past exposure (e.g., occupational cohort based on employment records). Cohort studies
are useful for determining risk of disease while cross-sectional studies are useful for
determining the prevalence of disease. In a cohort study subjects in the exposed group are
compared to subjects in the unexposed group rather than subjects in the general
population which is typical for a case-control study.

Question 3.2 The answer is D.


Case-control studies provide a method of estimating the odds ratio or relative risk, and
more efficient in terms of time and number of subjects than cohort studies. The need for
follow-up is avoided as well. However, they have several deficiencies that need to be
recognized. Case-control studies are susceptible to biases. Specific biases include recall
(the status of the subject, case or control, may influence how they recall and report
previous exposures), selection (individuals in the study, as cases or controls, are different
from the target population), and misclassification (occurs if the ascertainment of disease
by observer is influenced by the exposure status; for example a physician may examine
the subject if he or she knows the subject has been exposed and likely at a higher risk, and
the exposed subject visits the doctor more often if he or she thinks they are at higher risk
related to an exposure)

Question 3.3 The answer is D.


Confounding factors and interaction (also known as effect modification) are confusing and
both may be present in the same study. In order to be confounding factor the factor must
fulfill three criteria: (1) be associated with exposure; (2) be associated with disease
independent of exposure; and (3) not be an intermediate step between exposure and
disease (i.e., not be in casual pathway). Interaction is another variable that impacts risk of
disease (i.e., may be synergistic with other risk factor or protective factor). Confounding
is considered a nuisance which the investigator seeks to remove through epidemiologic
methods, and interaction is a more detailed relationship between an exposure and disease.

Question 3.4 The answers is D.


Confidence intervals are a method of evaluating chance, and a 95% confidence interval
means that the confidence interval will include the correct value of the measure 95% of
the time. The width of the confidence interval is directly related to the sample size, and
with increasing sample size the width of the confidence interval decreases. If the
confidence interval includes 1 it is not considered statistically significant and if does not
include 1 then it is considered statistically significant. While the 95% confidence interval
is the most frequently used confidence interval other confidence intervals can be used.
Question 3.5 The answer is B.
The patient population that has been referred to the gastroenterology clinic may be
significantly different than the general population in terms of the incidence of gastric
cancer and/or soda consumption.

Question 3.6 The answer is A.


Ecologic studies uses groups of people as the unit and are performed when measures of
exposure and/or outcome are available. An example would be the relationship between
dietary fat intake and the incidence of a type of cancer by country. The other types of
studies use the individual as the unit of study.

Question 3.7 The answer is True.


These genes encode neuronal nicotinic acetylcholine receptor subunits, and different
genotypes of these subunits appear to influence an individual’s addiction to tobacco which
leads to different smoking exposure and lung cancer risk.

Question 3.8 The answer is D.


The incidence of lung cancer among women increased throughout the 1990’s, stabilized
from 2000 to 2005, and declined in the period of 2005 to 2010. This pattern reflects the
trends of cigarette use among women. The rate of colorectal cancer (CRC) is declining,
possibility related to down decreasing rates of tobacco use, increased use of endoscopic
removal of adenoma preventing the development of CRC and intermittent used of
nonsteroidal anti-inflammatory agents. The incidence of breast cancer is declining since
2000, and this may be due to a reduction in the use of hormone replacement therapy. The
rate of melanoma is increasing most likely due to the effects of previous sun exposure and
increased awareness and surveillance of pigmented skin lesions.

Question 3.9 The answer is A.


The decline in the use of hormone replacement therapy (HRT) is thought to be a
contributing factor in the incidence of breast cancer. HRT may reduce the rate of CRC and
the discontinuation of the widespread use of HRT may adversely affect CRC trends. The
rate of gastric cancer is decreasing due to historic improvements in nutrition and the
declining prevalence of chronic Helicobacter pylori infection and the declining rates in lung
cancer are related to the decreasing prevalence of cigarette use.

Question 3.10 The answer is D.


The SEER program started in 1973 collects highly standardized assessment of staging,
treatment and outcomes, and includes 10% of the US population. This program provides
the most precise information about cancer incidence trends. It does not capture behavioral
risks that affect cancer risk which is captured by the Health Interview Survey and in the
annual reports by the Behavioral Risk Factor Surveillance System.

Corresponding chapters in Cancer: Principles & Practice of Oncology, Tenth edition: 11 (Epidemiologic Methods) and 12
(Trends in United States Cancer Mortality).
4 Etiology of Cancer
Shirish M. Gadgeel

QUESTIONS
Each of the numbered items below is followed by lettered answers. Select the ONE lettered answer
that is BEST in each case unless instructed otherwise.

Question 4.1 Which of the following statements regarding the relationship between
cigarette smoking and lung cancer are CORRECT? (Select two correct responses)
A. The duration of smoking is a strong risk factor for developing lung cancer in smokers.
B. Smoking is only associated with a risk of developing squamous cell lung cancer and not
adenocarcinoma of the lung.
C. The risk of developing lung cancer in ex-smokers drops to the level of never smokers 2
years after smoking cessation.
D. The risk of developing lung cancer increases with the number of cigarettes smoked.

Question 4.2 Which of the following statements regarding the role of specific tobacco
products and tobacco related carcinogens in the development of cancer are CORRECT?
A. Polycyclic aromatic hydrocarbons in cigarette smoke are major causative factors for
development of lung cancer.
B. Nicotine is an important carcinogen in the causation of laryngeal cancer.
C. Cigarettes with lower tar yields are associated with reduced risk of diseases from
cigarette smoking.
D. Smokeless tobacco products don’t contain high amounts of carcinogens.

Question 4.3 Which of the following is TRUE regarding cigarette smoking causing cancer?
A. DNA adducts caused by carcinogens in cigarette smoke are responsible for mutations
that can eventually cause cancer.
B. Nicotine can reduce apoptosis and promote angiogenesis.
C. Compounds that promote inflammation may play a role in cancer development
D. All of the above.

Question 4.4 The HPV vaccine Gardasil


A. Is a quadrivalent vaccine containing virus-like particles (VLPs) from four different types
of HPV.
B. Protects from all HPV-causing cervical cancer.
C. Is recommended only for sexually active women.
D. Is recommended for treating cervical cancer.

Question 4.5 The role of EBV in cancer is characterized by the following:


A. EBV DNA is present in the tumor of all Burkitt lymphoma patients.
B. EBV is universally present in all CNS lymphomas in AIDS patients.
C. Nasopharyngeal carcinoma is the only nonlymphoid malignancy associated with EBV.
D. Involvement in all Hodgkin lymphoma.

Question 4.6 Association of Hepatitis C and Hepatitis B viruses with hepatocellular


carcinoma (HCC) is characterized by
A. Coinfection with Hepatitis D is necessary for development of HCC in patients infected
with Hepatitis B.
B. Antiviral therapy for Hepatitis C does not reduce the risk of HCC.
C. Surveillance for HCC, with ultrasound, is recommended in Hepatitis C infected
individuals with cirrhosis.
D. In the United States, more than 60% of HCC cases are associated with Hepatitis B.

Question 4.7 Role of inflammation in carcinogenesis has the following feature:


A. Inflammation is involved in several steps of carcinogenesis but not in malignant
transformation.
B. Inflammation and angiogenesis are two independent steps in cancer formation and
progression.
C. Inflammation can cause cancer cell proliferation and cancer cell invasion.
D. None of the above.

Question 4.8 Data on markers of inflammation and prognosis of cancer patients shows
A. Glasgow Prognostic Score (GPS) is based on absolute white cell count, neutrophil to
lymphocyte ratio, and platelet count.
B. Glasgow Prognostic Score predicts for higher risk of death in prostate cancer patients.
C. In a study of renal cancer patients, inflammatory markers did not correlate with overall
survival.
D. Elevated C reactive protein levels are associated with higher disease free survival in
breast cancer patients.

Question 4.9 Carcinogenic effects of chemicals are characterized by


A. Most chemical carcinogens undergo metabolic activation by enzymes such as
cytochrome P450s.
B. Direct DNA damage leading to mutations or chromosomal alterations.
C. Induction of epigenetic changes such as altered DNA methylation.
D. All the above.

Question 4.10 Chemical carcinogenesis is characterized by


A. Individual susceptibility that varies based on genetic polymorphisms.
B. Biomarkers are available to assess cancer risk of an individual from chemical
carcinogenesis.
C. Direct correlation with the chemical’s ability to cause DNA damage.
D. None of the above.

Question 4.11 Examples of chemical carcinogens include


A. Benzene has been linked to myelodysplastic syndrome and acute myeloid leukemia.
B. Heterocyclic amines are associated with breast and colon cancer.
C. Aryl aromatic amines found in tobacco smoke and in dyes have been associated with
bladder cancer.
D. All of the above.

Question 4.12 Cellular response to correct and prevent damage from ionizing radiation (IR)
is characterized by
A. DNA damage induced by IR can be repaired by mechanisms such as homologous
recombination repair pathway and nonhomologous end joining pathway.
B. If the DNA is damaged by IR, there are no mechanisms in place to prevent cells with
DNA damage from entering into critical phases of cell cycle.
C. All cells in the body are equally sensitive to radiation-induced cell killing.
D. IR-induces cell death through apoptosis alone.

Question 4.13 Risk of cancer from ionizing radiation (IR) is characterized by


A. Most cancers in areas of atomic bomb explosion occurred within 20 years of the
exposure.
B. Older individuals are at the highest risk of developing cancer in areas of massive IR
exposure.
C. The largest source of IR exposure to the population in the US is exposure from
radiology studies.
D. The most sensitive tissues for development of cancer from IR are thyroid, lung, breast,
and bone marrow.

Question 4.14 The following is TRUE about ultraviolet (UV) light:


A. UVA light induces reactive oxygen species which in turn causes DNA damage.
B. Risk of melanoma appears to be linked to cumulative lifetime exposure to UV light.
C. UV light initiates carcinogenesis only by inducing DNA lesions.
D. Melanoma occurs only in sun exposed areas of the skin.

Question 4.15 Alcohol consumption and risk of cancer (Select two correct responses)
A. Alcohol is not a carcinogen since it only induces cancer indirectly by causing
inflammation.
B. Types of alcoholic drinks can influence the risk of cancer.
C. Mechanisms of inducing cancers include direct damage to cells, modulation of DNA
methylation, and causing DNA adducts.
D. Alcohol can increase the risk of breast cancer.

Question 4.16 Which statement is CORRECT concerning dietary influence on risk of cancer
A. Dietary fat intake is an important contributor to breast cancer incidence.
B. Cancer risk is more related to the types of foods consumed than the total caloric intake.
C. Data supports a link between consumption of fat-containing animal products and
incidence of prostate cancer.
D. A higher intake of vegetable fat was associated with greater risk of breast cancer in
premenopausal women.

Question 4.17 Fruit and vegetable consumption and risk of cancer are characterized by
A. There is an inverse relationship between consumption of fruits and vegetables and risk
of colon cancer.
B. An inverse relationship between intake of fruits and vegetables and risk of estrogen
receptor negative breast cancer has been shown by the Nurse’s Health Study.
C. Consumption of fruit and vegetables reduces total cancer incidence.
D. None of the above.

Question 4.18 Data on the influence of specific nutrients and risk of cancer shows
A. Individuals with higher 25 (OH) D levels have a lower risk of colorectal cancer.
B. Higher intake of beta carotene reduces the risk of lung cancer.
C. In the Selenium and Vitamin E Cancer Prevention Trial (SELECT) trial selenium was
found to protect from development of prostate cancer.
D. High intake of calcium protects development of prostate cancer.

Question 4.19 Obesity and physical activity as risk factors for breast cancer is characterized
by
A. Low level of physical activity is a risk factor for breast cancer in postmenopausal
women but not premenopausal women.
B. Obesity is associated with reduced risk of breast cancer in postmenopausal women.
C. Obesity is associated with reduced risk of breast cancer in premenopausal women.
D. None of the above.

Question 4.20 Obesity and physical activity are associated with following cancers:
A. Obesity but not low physical activity is associated with risk of colon cancer.
B. Obesity is not a risk factor for endometrial cancer.
C. Higher lifetime physical activity is associated with lower risk for pancreatic cancer.
D. Obesity lowers the risk of gallbladder cancer.
ANSWERS

Question 4.1 The answers are A and D.


A common index of cancer risk is pack-years, or the number of packs of cigarettes smoked
per day multiplied by the number of years smoked in the lifetime. In general, the higher
the number of pack-years, the greater the cancer risk. Risk for lung cancer declines with
smoking cessation and the longer a former smoker remains off of cigarettes the more the
risk declines. However, excepting those smokers who quit with relatively few pack-years
accumulated (typically before age 40), cancer risk rarely approaches that of a never
smoker. Smoking increases the risk of developing all histologic types of lung cancer
including adenocarcinoma, squamous cell carcinoma, large cell carcinoma, and small cell
carcinoma.

Question 4.2 The answer is A.


Cigarette smoke consists of many carcinogens. Data from carcinogenicity studies, product
analyses, and biochemical and molecular biologic investigations support a significant role
for certain carcinogens in specific types of tobacco-induced cancers. In this regard,
evidence suggests a significant role for polycyclic aromatic hydrocarbons and N-
nitrosamines as causative factors in lung cancer. Nicotine itself is not known to initiate
cancer formation, but recent evidence suggests that nicotine through nicotinic receptors
may promote cellular proliferation. After tar yields from cigarettes were recognized as an
important contributor to smoking caused diseases, many manufactures reduced the tar
yield of cigarettes. However, these reductions in tar yields have not led to changes in
disease risk from cigarette smoking. Smokeless tobacco contains substantial level of
carcinogens, most prominently N-nitrosamines.

Question 4.3 The answer is D.


Many cigarette smoke carcinogens are not active and require metabolic activation to
transform them into active carcinogens. Cytochrome P450 enzymes convert these
compounds into electrophilic entities that can covalently bind to DNA-forming DNA
adducts. The cigarette smoke carcinogens can undergo detoxification by glutathione-S-
transferases and uridine diphosphate-glucuronosyltransferases. Therefore, cancer
susceptibility may be determined by the balance achieved between activation of the
carcinogens in cigarette smoke and their detoxification. In addition, polymorphisms in
genes encoding these enzymes may influence an individual’s risk of cancer from these
carcinogens. Cellular repair systems can remove the DNA adducts formed by smoke
carcinogens and repair the DNA, but they are not involved in metabolic activation of
carcinogens in cigarette smoke. Nucleotide excision repair enzymes are a component of
these repair systems. Nicotine, although not a carcinogen, is known to reduce apoptosis
and increase angiogenesis and thus can promote cancer growth. Compounds in cigarette
smoke that are toxic to the cilia, that can cause inflammation and that have oxidizing
properties can influence likelihood of development of cancer.
Question 4.4 The answer is A.
Gardasil vaccine for HPV consists of VLPs (virus like particles) from HPV-16, HPV-18,
HPV-6, and HPV-11. These VLPs present the epitopes that are necessary for the
development of a high titer neutralizing antisera. HPV-16 and HPV-18 are associated with
approximately 70% of cervical cancers and HPV-6 and HPV-11 are associated with 90% of
all genital warts. There are HPV types other than HPV-16 and HPV-18 that can cause
cervical cancer and therefore vaccinated individuals at risk should continue to receive
cervical cancer screening. In addition, the VLPs that are in the vaccine may not be
expressed in latently infected keratinocyte stem cell residing on the epithelial basement
membrane and therefore may not eradicate infections existing at the time of vaccination.
The vaccine does not treat cervical cancer. The current recommendations of the US
Centers for Disease Control and Prevention are that both boys and girls should be
vaccinated.

Question 4.5 The answer is B.


Although nearly all cases of endemic Burkitt lymphoma contain EBV DNA in the tumor,
only about 20% of sporadic cases arising in immunocompetent individuals contain EBV.
EBV is universally present in all CNS lymphomas in AIDS patients and therefore a PCR
test for EBV together with radiologic findings in these patients can obviate the need for a
biopsy. EBV is associated with nasopharyngeal carcinoma and a small percentage (5% to
15%) of gastric cancers. EBV is associated with about 50% of Hodgkin lymphomas.

Question 4.6 The answer is C.


Hepatitis D virus (HDV) infection occurs only in individuals infected with HBV. HDV
infection results in more severe complications than infection with HBV alone, with a
higher likelihood and more rapid progression to cirrhosis and HCC. However, coinfection
with HDV is not necessary for HCC to occur in patients infected with Hepatitis B. A meta-
analysis of eight randomized trials has shown that antiviral therapy for Hepatitis C virus
reduces the risk of HCC by 50%. The American Association for the Study of Liver
Diseases, as well as the European and Asian Pacific Associations for the Study of the Liver,
endorse surveillance in HCV-infected individuals with cirrhosis using ultrasound every 6
months. Viral eradication does not fully eliminate the risk of HCC, and thus, continued
surveillance is still recommended in cirrhotic patients. In the United States, more than
60% of HCC are associated with Hepatitis C infection.

Question 4.7 The answer is C.


Transformation is the process by which the cellular and molecular makeup of a cell is
altered as it becomes malignant. Numerous factors are involved in the process of cell
transformation, including inflammation. Angiogenesis is tightly linked to chronic
inflammation and cancer. Angiogenesis is one of the molecular events that bridge the gap
between inflammation and cancer. Inflammation can cause several aspects of
carcinogenesis including cellular proliferation and cancer cell invasion.
Question 4.8 The answer is B.
Glasgow prognostic score (GPS) combines plasma C reactive protein and albumin.
Prostate cancer patients with elevated GPS had a higher risk of death overall as well as
high grade disease. In a study of 416 renal cell cancer patients, inflammatory markers
such as elevated neutrophil count, elevated platelet count and high neutrophil to
lymphocyte ratio correlated with shorter overall survival. Elevated C-reactive protein
levels correlate with reduced disease free survival in breast cancer patients.

Question 4.9 The answer is D.


Most chemical carcinogens first undergo metabolic activation by cytochrome P450s or
other metabolic pathways so that they react with DNA and/or alter epigenetic
mechanisms. This activation process generates reactive carcinogenic intermediates that
can bind to DNA and cause mutations and other alterations. Chemical carcinogens can
cause both DNA damage resulting in mutations or chromosomal alteration and epigenetic
changes. Both these changes can contribute to carcinogenesis.

Question 4.10 The answer is A.


Genetic polymorphisms may modify an individual’s susceptibility to the carcinogenic
effects of chemicals. No single biomarker has been considered to be sufficiently validated
for use as a cancer risk marker in an individual as it relates to risk from chemical
carcinogenesis. The carcinogenic effect of chemicals is not only related to its ability to
cause DNA damage but is a composite of the net result of carcinogen exposure, activation,
lack of detoxification, lack of effective DNA repair, and lack of programmed cell death.

Question 4.11 The answer is D.


Epidemiologic and experimental studies have linked benzene to hematologic toxicity,
including aplastic anemia, myelodysplastic syndrome, and acute myeloid leukemia.
Heterocyclic amines are formed from the overheating of food with creatine, such as meat,
chicken, and fish. Heterocyclic amines, estimated based on consumption of well-done
meat, have been associated with breast and colon cancer, presumably through metabolic
activation mechanisms and DNA damage. Aryl aromatic amines have been implicated in
bladder carcinogenesis, especially in occupationally exposed cohorts (e.g., dye workers)
and tobacco smokers.

Question 4.12 The answer is A.


Both homologous recombination and nonhomologous end joining pathways can repair
damage from IR. To prevent cells with damaged DNA from entering into these critical
stages of the cell cycle, cells can activate cell-cycle checkpoints. The major sensor of
radiation-induced damage in cells is the ataxia-telangiectasia mutated (ATM) kinase,
which, following activation, can phosphorylate more than 700 proteins in cells. Two ATM
substrates, p53 and Chk2, are critical for the activation of cell cycle arrests at multiple
sites in the cell cycle. Terminally differentiated and stationary cells, such as kidney, lung,
brain, muscle, and liver cells, are generally more resistant to radiation-induced killing
than are cells with a high turnover rate, such as different epithelial cells, spermatogonia,
and hair follicles. However, the spleen and thymus, which consist of mostly nondividing
cells, are among the most radiosensitive tissues, implying that the rate of cell
proliferation is not the sole determiner of the radiation sensitivity of a tissue. IR can
induce cell death in tissues by many different mechanisms. Apoptosis can occur rapidly in
a p53-dependent manner or later in a p53-independent manner. Cell death induced by IR
may in some cases be associated with autophagy. Finally, tissue can undergo necrotic cell
death following exposure to IR.

Question 4.13 The answer is D.


Twenty years after the atomic bomb explosions in Japan during World War II, significant
increases in the incidence of thyroid cancer and leukemia were observed. However, it
took almost 50 years before solid tumors appeared in the population as a result of
radiation exposure from the atomic bombs. Young children were the most vulnerable to
radiation exposure, with much higher rates of thyroid cancer in younger children. The
largest source of radiation exposure to the US population is radon, which is a natural
radioactive gas formed as a decay product of radium in the decay chain of uranium. The
most sensitive tissues for the development of secondary cancer have been found to be
bone marrow (leukemia), the thyroid, breast, and lung.

Question 4.14 The answer is A.


UVA light can induce reactive oxygen species which in turn can form single strand breaks
and base lesions in the DNA of exposed cells. The risk of nonmelanoma skin cancers
relates to cumulative lifetime exposure to UV light. However, the risk of melanoma is
related to high sunlight exposure during childhood. UV light is a potent carcinogen since it
not only induces DNA lesions but also suppresses the immune system. Melanoma can
develop in both sun exposed and shielded areas of the skin.

Question 4.15 The answers are C and D.


Although alcohol can cause liver cancer by causing alcoholic hepatitis and cirrhosis,
alcohol is also a carcinogen and can cause direct damage to cells, modulation of DNA
methylation, and acetaldehyde, the main metabolite of alcohol can form DNA adducts. For
most cancer sites, no important difference in associations was found with the type of
alcoholic beverage, suggesting a critical role of ethanol in carcinogenesis. A small but
significant risk for breast cancer has been found even with one drink per day.

Question 4.16 The answer is C.


Although evidence suggests that a high intake of animal fat early in adult life may
increase the risk of premenopausal breast cancer, this is not likely to be due to fat per se
because vegetable-fat intake was not related to risk. The most important impact of diet on
the risk of cancer is mediated through body weight. Thus, caloric intake is more
important than specific food types with regards to cancer risk. At present, the available
evidence most strongly suggests an association between animal-fat consumption and risk
of prostate cancer, particularly the aggressive form of this disease.

Question 4.17 The answer is B.


The results from the largest studies, the Nurses’ Health Study and the Health
Professionals’ Follow-Up Study, suggested no important association between the
consumption of fruits and vegetables and the incidence of cancers of the colon or rectum
during 1,743,645 person-years of follow-up. In a recent analysis within the Nurses’ Health
Study, an inverse association was seen between vegetable intake and the risk of estrogen
receptor–negative breast cancer. The comprehensive report of the World Cancer Research
Fund and the American Institute for Cancer Research, published in 2007 and titled Food,
Nutrition, Physical Activity, and the Prevention of Cancer: A Global Perspective, reached the
consensus based on the available evidence: “findings from cohort studies conducted since
the mid-1990s have made the overall evidence, that vegetables or fruits protect against
cancers, somewhat less impressive. In no case now is the evidence of protection judged to
be convincing”.

Question 4.18 The answer is A.


A meta-analysis, including five nested case-control studies with prediagnostic serum,
suggested a reduction of colorectal cancer risk by about half among individuals with
serum 25 (OH) D levels of more than 82 nmol/L compared to individuals with less than
30 nmol/L. A subsequent meta-analysis including eight studies confirmed these
associations. In a detailed analysis of prospective studies, no association was seen between
the intake of beta carotene and the risk of lung cancer. In a recent meta-analysis,
plasma/serum selenium was also inversely correlated with prostate cancer. In the
SELECT, no protective effect of selenium was found for prostate cancer. However, the
trial was terminated prematurely after 4 years, which is a short period in which to expect
a reduction in cancer. In multiple studies, a high intake of calcium or dairy products has
been associated with an increased risk of prostate cancer.

Question 4.19 The answer is C.


Low level of physical activity is an established breast cancer risk factor among
postmenopausal women and, to a lesser extent, premenopausal women. Obesity appears
to have a paradoxical relationship with breast cancer risk in that it is associated with an
elevated risk in postmenopausal women but may offer some protection for breast cancer
in premenopausal women.

Question 4.20 The answer is C.


Both low physical activity and obesity are associated with risk of colon cancer.
Epidemiologic studies have established a strong association between obesity and
endometrial cancer risk. A meta-analysis of 28 studies of pancreatic cancer showed that
higher total lifetime physical activity and occupational activity were associated with a
lower risk. Obesity was associated with an increased risk of gallbladder cancer.
Corresponding chapters in Cancer: Principles & Practice of Oncology, Tenth Edition: 4 (Tobacco), 5 (Oncogenic Viruses), 6
(Inflammation), 7 (Chemical Factors), 8 (Physical Factors), 9 (Dietary Factors), and 10 (Obesity and Physical Activity).
5 Cancer Screening
Megan E. Wren

QUESTIONS
Each of the numbered items below is followed by lettered answers. Select the ONE lettered answer
that is BEST in each case unless instructed otherwise.

Question 5.1 Studies of screening for cancer are subject to several types of bias. If screening
detects a cancer earlier (before it becomes symptomatic), but treatment has no effect on the
course of the disease, then the subject will seem to live longer than if he/she had presented
symptomatically (i.e., the cancer is known for a longer period of time, but the time of death
is not altered). This type of bias is known as
A. Lead-time bias.
B. Length bias.
C. Selection bias.
D. Overdiagnosis bias.

Question 5.2 In planning a prostate cancer screening program you are concerned that a
prostate specific antigen (PSA) threshold of 4 ng/mL has a sensitivity of just 24% for the
diagnosis of prostate cancer. You can anticipate that lowering the PSA threshold to 2.5
ng/mL threshold will
A. Decrease the positive predictive value (PPV).
B. Decrease the rate of true-positive results.
C. Decrease the sensitivity.
D. Increase the positive predictive value (PPV).
E. Increase the specificity.

Question 5.3 In the PLCO Cancer Screening Trial men were randomized to receive annual
PSA testing for 6 years or usual care. At the conclusion of the 13 year follow-up period it was
found that there was a high rate of PSA testing among men randomized to the control arm.
This “drop-in” had what effect on the results?
A. Increased the positive predictive value (PPV) of the PSA test.
B. Increased the relative risk reduction (RRR) of the screening arm as compared with the
control arm.
C. Reduced the sensitivity of the PSA test.
D. Reduced the statistical power of the study to detect differences in outcome between the
two arms.
Question 5.4 The U.S. Preventive Services Task Force (USPSTF)
A. Is composed of government employees with limited clinical experience.
B. Formulates recommendations based on expert opinion more than on evidence based
medicine.
C. Weighs benefits and harms of screening tests in the context of cost effectiveness.
D. Weighs benefits and harms of screening tests without considering cost effectiveness.

Question 5.5 A new screening test for ovarian cancer was developed. It was tested in a
tertiary care academic medical center in a group of women with breast cancer (BRCA)
mutations and was found to have a sensitivity of 70%, specificity of 90%, and positive
predictive value (PPV) of 10%. If a woman in the study population is found to have a
positive (abnormal) test result, how would you interpret it?
A. There is a 30% chance that this represents a false-positive result.
B. There is a 10% chance that this represents a false-positive result.
C. There is a 9 in 10 chance that this represents a false-positive result.
D. You can be 90% sure that your patient has the disease.

Question 5.6 If the use of this test (from Question 5.5) is expanded to the general
population, it is expected that
A. The test sensitivity will be lower.
B. The test specificity will be lower.
C. The PPV will be lower.
D. The test specificity will be higher.

Question 5.7 In analyzing a randomized controlled trial of a screening test which is the best
indicator of effectiveness?
A. Absolute reduction in mortality
B. Evidence of stage shift
C. Increased 5-year survival
D. Relative reduction in mortality

Question 5.8 Which statement regarding screening for breast cancer is TRUE?
A. The monthly breast self-examination (BSE) is a crucial component of breast cancer
screening programs.
B. The BSE has been shown to be ineffective for breast cancer screening.
C. Increasing the interval of mammographic screening from 1 year to 2 years results in a
halving of the mortality benefit.
D. Mammography has a higher positive predictive value (PPV) for women aged 40 to 49
as compared with women aged 50 to 59.

Question 5.9 The American Cancer Society (ACS) recommends annual screening
mammography and MRI starting at age 30 for women at high risk for breast cancer including
which of the following? (Select two correct responses)
A. Women with a history of fibrocystic breast disease
B. Women with a history of mantle radiation for Hodgkin disease
C. Women with a known BRCA mutation
D. Women with increased mammographic breast density

Question 5.10 A 19-year-old woman presents for a health maintenance visit. She has been
sexually active for 4 years and has had one episode of chlamydia. What is the recommended
approach to cervical cancer screening for her?
A. Begin annual screening with cervical cytology (Pap smear) plus HPV cotesting.
B. Begin annual screening with cervical cytology alone.
C. No cervical cancer screening at this time; start screening at age 21 with cervical
cytology alone.
D. No cervical cancer screening at this time; start screening at age 21 with cervical
cytology and HPV cotesting every 3 years.

Question 5.11 A 40-year-old woman had her first screening mammogram and was noted to
have increased mammographic breast density. Current evidence suggests that the best
recommendation for future breast cancer screening would be
A. Annual mammography alone.
B. Annual mammography + annual MRI.
C. Annual mammography + monthly breast self-examination (BSE).
D. Annual mammography + monthly breast self-examination (BSE) + annual MRI.

Question 5.12 The United States Preventive Services Task Force (USPSTF), the American
College of Obstetricians and Gynecologists (ACOG), and the American College of Physicians
(ACP) agree on which one of the following recommendations for ovarian cancer screening in
women ages 20 to 65 years without oophorectomy?
A. Annual pelvic examination for all women
B. Annual serum CA-125 measurement
C. No population-based screening is recommended
D. Transvaginal ultrasound every 5 years
E. Two-stage screening: annual CA-125, then transvaginal ultrasound if elevated or rising
CA-125

Question 5.13 A 44-year-old woman had a total hysterectomy for fibroids and menorrhagia.
What is the recommended approach to cervical cancer screening for her?
A. Cervical cytology testing should be discontinued.
B. Cervical cytology testing should be performed annually indefinitely.
C. Cervical cytology testing should be performed every 1 to 3 years until age 65 years.
D. Cervical cytology testing plus HPV cotesting should be performed every 5 years until
age 65 years.

Question 5.14 In the Mayo Lung Project randomized controlled trial more than 9,200 male
smokers were randomized to intensive screening (sputum cytology and CXR every 4 months
for 6 years) or a control group (same tests performed annually). After nearly 20 years of
follow-up there were more lung cancers diagnosed in the intensive screening arm versus the
control arm (585 vs. 500) but the intensive screening arm did not show improvement in lung-
cancer mortality (4.4 lung cancer deaths per 1,000 person-years in the intensively screened
arm vs. 3.9 per 1,000 person-years in the control arm). This is best explained by
A. Lead-time bias.
B. Overdiagnosis.
C. Selection bias.
D. Stage shift

Question 5.15 Based on the results of the National Lung Screening Trial (NLST) several
organizations recommended consideration of lung cancer screening with low-dose
computerized tomography (LDCT) for patients who would have qualified for the trial.
Eligible patients had to have which two of the following criteria? (Select two correct
responses)
A. Age less than 60 years
B. At least a 30 pack-year smoking history
C. At least a 50 pack-year smoking history
D. Current smoker
E. Relatively good health

Question 5.16 Which of the following statements regarding prostate cancer screening is
TRUE?
A. Intensity of prostate cancer screening has been shown to correlate with the degree of
mortality benefit.
B. Prostate cancer screening and therapy are associated with significant harms.
C. PSA levels are unaffected by benign prostate diseases.
D. Routine annual PSA testing is recommended for all men ages 55 to 74 years.

Question 5.17 A 50-year-old-man has no family history of colorectal cancer or polyps. He


inquires about CT colonography (“virtual colonoscopy”) for screening as compared to
traditional colonoscopy (endoscopy). Which one is TRUE?
A. CT colonography does not require a full bowel preparation
B. CT colonography requires sedation
C. CT colonography is only diagnostic; a colonoscopy would be required if any polyps
need removal
D. CT colonography is only half a sensitive as colonoscopy

Question 5.18 More frequent screening for colorectal cancer (CRC) is recommended in
certain high risk individuals. Which two of the following groups need to have screening
increased to colonoscopy every 5 years starting by age 40? (Select two correct responses)
A. First-degree relative with CRC or adenomatous polyp at age ≥60 years
B. One first-degree relative with CRC or adenomatous polyps diagnosed at age <60 years
C. One second-degree relative with colorectal cancer
D. Two or more first-degree relatives with CRC
E. Two second-degree relatives with CRC

Question 5.19 Which of the following CRC screening tests can directly prevent colorectal
cancer?
A. CT colonography (“virtual colonoscopy”)
B. Double-contrast barium enema
C. Fecal immunochemical testing (FIT)
D. High-sensitivity fecal occult blood testing (FOBT)
E. Optical colonoscopy (endoscopy)
ANSWERS

Question 5.1 The answer is A.


Lead-time bias is the interval between the diagnosis of disease at screening and when it
would have been detected because of the development of symptoms. Length bias is the
overrepresentation among screen-detected cases of those with a long preclinical period
(thus, less rapidly fatal), leading to the incorrect conclusion that screening was beneficial.
Selection bias occurs when enrollees in a clinical study differ from the general population.
Volunteers who choose to participate in screening programs tend to be healthier than the
general population.

Question 5.2 The answer is A.


Lowering the PSA threshold will lead to more positive test results, both true positives and
false positives. Finding more true positives results in an increased sensitivity of the test.
Finding more false positives results in a decreased sensitivity of the test. Note that for a
given screening test, sensitivity and specificity are inversely related. The increase in false
positives (the decrease in sensitivity) results in a decrease in the positive predictive value
(PPV).

Question 5.3 The answer is D.


Sensitivity and specificity are characteristics of a test itself. The PPV is related to the test
sensitivity, test specificity, and population prevalence of the disease. “Drop-in” or
“contamination” by screening in the control arm potentially decreased the mortality in the
control arm, minimizing the difference between the groups (RRR). Both drop-ins and
drop-outs reduce the statistical power of a clinical trial to detect differences in outcome
between the two arms.

Question 5.4 The answer is D.


The USPSTF is a panel of experts in prevention and evidence based medicine, and are
primary care providers specializing in internal medicine, pediatrics, family practice,
gynecology and other fields. They adhere to high standards for recommending a screening
test, focusing on whether there is evidence to show that the benefits outweigh the harms
of a screening test. Cost effectiveness is not considered.

Question 5.5 The answer is C.


The sensitivity of a test reflects its ability to detect a known disease, and the specificity
reflects the ability of a test to give a normal result when the disease is known to be
absent. From the point of view of the clinician, the presence or absence of a disease is
unknown, and the pertinent question is the accuracy of the test result. The PPV is an
estimate of the accuracy of the test in predicting the presence of disease; the negative
predictive value (NPV) is an estimate of the accuracy of the test in predicting the absence
of disease. The PPV represents the proportion of all positive tests that are true positives:
TP/(TP+FP). A PPV of 10% means that 10% of positive test results are true positives, so
90% are false positives.

Question 5.6 The answer is C.


The sensitivity and specificity are characteristics of the test itself and are not affected by
the population characteristics. On the other hand, the PPV and NPV values are influenced
by the disease prevalence in the population being tested. The general population will have
a lower prevalence of breast cancer than a group of women with known BRCA mutations.

Question 5.7 The answer is A.


Absolute risk reduction conveys more information than relative risk reduction. A large
relative risk reduction applied to a very small risk translates to a small number of lives
saved. For example, reducing mortality from one in a million to two in a million is an
impressive sounding 50% risk reduction but only saves one more life per million patients.
Stage shift is the diagnosis of a cancer at an earlier stage than would have occurred in the
absence of screening. This is necessary but not sufficient for a screening test to be
effective in terms of reducing mortality. Survival time after diagnosis can be inflated by
lead-time bias. Mortality rates are a better measure of effectiveness.

Question 5.8 The answer is B.


Large trials of careful BSE instruction have shown a lack of mortality benefit, so BSE is no
longer a standard component of breast cancer screening programs (the American Cancer
Society lists BSE as an option). Mammography has a lower PPV for women in their 40s
due to a lower prevalence of breast cancer and a lower sensitivity of mammography (due
to increased breast density in premenopausal women). Biennial screening only marginally
decreases the number of lives saved while halving the false-positive rate as compared
with annual screening.

Question 5.9 The answers are B and C.


Annual mammography and MRI starting at age 30 is recommended for women at high
risk due to a known BRCA mutation, women who are untested but have a first-degree
relative with a BRCA mutation, women who had been treated with radiation to the chest
for Hodgkin disease, and women with an elevated lifetime risk (> 20% to 25%) based on
breast cancer risk estimation models.
It has yet to be determined whether supplemental imaging reduces breast cancer
mortality in women with increased breast density.

Question 5.10 The answer is C.


Screening guidelines recommend that cervical cancer screening with cytology not begin
until age 21 and HPV testing not begin until age 30. Although HPV infections are common
in young women, the overwhelming majority of these HPV infections and associated CIN
will spontaneously regress. Screening young women could lead to overdiagnosis,
aggressive treatment, and unnecessary harm from ablative surgical procedures.
Question 5.11 The answer is A.
Large trials of careful BSE instruction have shown a lack of mortality benefit for BSE. MRI
screening is only indicated in women at very high risk for breast cancer.

Question 5.12 The answer is C.


The USPSTF, ACOG, and ACP all discourage routine screening for ovarian cancer in the
general population. The currently available screening tests do not have adequate
sensitivity and specificity, especially considering the relatively low prevalence in the
population.

Question 5.13 The answer is A.


Women without a cervix are not at risk for cervical cancer unless there was a history of
cervical cancer (in that case the cytology testing is for follow-up of the cancer, not for
screening). Vaginal cuff smears are unnecessary; they have an extremely low likelihood of
detecting vaginal dysplasia, and the false-positive rate is high.

Question 5.14 The answer is B.


Overdiagnosis is an extreme form of length bias and refers to the detection of tumors that
fulfill the histologic criteria for malignancy but are not biologically destined to harm the
patient within his/her lifespan. Lead-time bias is the interval between the diagnosis of
disease at screening and when it would have been detected because of the development of
symptoms. Selection bias occurs when enrollees in a clinical study differ from the general
population. Volunteers who choose to participate in screening programs tend to be
healthier than the general population. In stage shift a cancer diagnosis is made at an
earlier stage than would have occurred in the absence of screening.

Question 5.15 Answers are B and E.


The NLST eligibility criteria includes
• Age 55 to 74 years;
• At least a 30 pack-year smoking history;
• Currently smoke or have quit within the past 15 years;
• Relatively good health (i.e., able to tolerate therapy for lung cancer and have the life
expectancy benefit from it).

Question 5.16 The answer is B.


Prostate cancer screening and therapy are associated with significant harms. Experts do
not agree as to whether the benefits of screening for this disease outweigh the harms.
Professional organizations recommend that screening should only be done in the context
of fully informed consent. Prostate cancer mortality rates have declined in many countries
that have not widely adopted screening suggesting that improvements in treatment have
contributed to the observed decline in prostate cancer mortality. Aging and noncancerous
diseases of the prostate are associated with rising PSA levels.
Question 5.17 The answer is C.
CT colonography is only diagnostic and a colonoscopy would be required if any polyps
need removal. CT colonography involves the same preparation as a colonoscopy, but does
not involve sedation. The sensitivity of CT colonography for the detection of polyps ≥6
mm appears to be comparable to that of optical (endoscopic) colonoscopy.

Question 5.18 The answers are B and D.


Patients with two or more first-degree relatives with CRC, or one first-degree relative
with CRC or adenomatous polyps diagnosed at age <60 years need to have colonoscopy
every 5 years, beginning at age 40 years or 10 years younger than the earliest diagnosis
in the family, whichever comes first. Those with one first-degree relative with CRC or an
adenomatous polyp at age ≥60 years, or two second-degree relatives with CRC need to
start screening at age 40 but can otherwise follow the guidelines for average-risk persons.

Question 5.19 The answer is E.


Endoscopy with polypectomy can remove precancerous lesions (adenomatous polyps) thus
reducing the incidence of subsequent CRC.

Corresponding chapter in Cancer: Principles & Practice of Oncology, Tenth edition: 34 (Cancer Screening).
6 Cancer Prevention
Lingling Du, Richard Chen, Daniel Morgensztern, and Rebecca Aft

QUESTIONS
Each of the numbered items below is followed by lettered answers. Select the ONE lettered answer
that is BEST in each case unless instructed otherwise.

Question 6.1 Which of the following statements regarding nicotine replacement therapy
(NRT) is TRUE?
A. NRT increases the odds of quitting smoking compared to placebo.
B. NRT is associated with increased risk of carcinogenesis in the general population.
C. a and b
D. None of the above

Question 6.2 Which of the following statement is TRUE?


A. Continued smoking after the diagnosis of localized prostate cancer does not increase the
risk of recurrence.
B. The adverse effect of smoking is not acutely reversible. Therefore smoking cessation is
not recommended before surgical resection for esophageal cancer.
C. Current smoking does not increase the risk of postoperative death in lung cancer
patients undergoing surgery.
D. Current smoking increases the toxicities associated with radiotherapy in patients with
head and neck cancer.

Question 6.3 Varenicline (Chantix) is associated with which of the following side effects?
A. Hypertension
B. Anorexia
C. Suicidal ideation
D. Elevated liver enzymes

Question 6.4 Which of the following is recommended as second-line pharmacotherapy for


tobacco dependence?
A. Clonidine
B. Nortriptyline
C. Both a and b
D. None of the above
Question 6.5 Which of the following neoplasias is an intraepithelial marker for invasive
cancer?
A. Cervical ectropion
B. Junctional nevus
C. Seborrheic keratosis
D. Leukoplakia

Question 6.6 In clinical trials, systemic retinoids and beta carotene were shown to reduce
cancer incidence in this group of high-risk individuals
A. Skin cancer in patients with xeroderma pigmentosum
B. Lung cancer in heavy smokers
C. Cervical cancer in patients with cervical dysplasia
D. Colorectal cancer in patients with adenoma

Question 6.7 Which of the following was associated with selenium supplementation in
randomized clinical trials?
A. Decreased risk of squamous cell skin cancer
B. Reduced incidence of prostate cancer
C. Increased incidence of nonmelanoma skin cancer
D. Increased risk of prostate cancer

Question 6.8 Which of the following statements is/are TRUE regarding calcium and vitamin
D?
A. Calcium reduces the risk of recurrent colorectal adenoma.
B. The combination of vitamin D and calcium decreased the risk of developing colon
cancer in postmenopausal women in the Women’s Health Initiative (WHI) trial.
C. The combination of vitamin D and calcium reduced the risk of breast cancer in
postmenopausal women in the WHI trial.
D. High serum level of vitamin D is associated with decreased risk of pancreatic cancer.

Question 6.9 Which of the statements is/are TRUE regarding nonsteroidal anti-inflammatory
drugs (NSAIDs) in the prevention of colorectal cancer?
A. Chemopreventive effects of low-dose aspirin on colorectal cancer development were
demonstrated in clinical trials, and its overall benefits outweigh the risks in adults at
average risks of developing colorectal cancer.
B. High-dose sulindac prevents the development of new polyps, although it does not cause
the regression of established polyps.
C. In randomized trials, both rofecoxib and celecoxib were shown to reduce the risk of
metachronous colorectal adenomas and colorectal cancer.
D. Considering the risks associated with these agents, screening strategy alone is
recommended in average-risk adults.

Question 6.10 Which of the statement is TRUE regarding tamoxifen and raloxifene?
A. Tamoxifen reduces the risk of ER-negative breast cancer.
B. Raloxifene reduces breast cancer-specific mortality rates.
C. Tamoxifen is more effective than raloxifene in reducing breast cancer incidence.
D. Both tamoxifen and raloxifene decrease the risk of in situ breast neoplasms.

Question 6.11 The risk of which of the following toxicities is increased in patients treated
with raloxifene?
A. Endometrial cancer
B. Vertebral fracture
C. Stroke
D. Coronary artery heart disease

Question 6.12 Which of the following statement is TRUE regarding the role of 5-alpha-
steroid reductase inhibitors in preventing prostate cancer?
A. Finasteride does not reduce the incidence of prostate cancer.
B. Patients treated with finasteride or dutasteride who develop prostate cancer are more
likely to have tumors of a high Gleason score compared to patients treated with
placebo.
C. Finasteride decreases the survival after prostate cancer diagnosis compared to placebo.
D. Dutasteride increases the risk of prostate cancer.

Question 6.13 Which of the following is TRUE regarding statins?


A. Simvastatin was shown to decrease the overall cancer risk in several large randomized
trials.
B. Statins reduces the risk of breast cancer and colon cancer in randomized studies.
C. Both a and b.
D. None of the above.

Question 6.14 Which of the following is TRUE?


A. Bisphosphonates reduces the risk of recurrent breast cancer in postmenopausal women
with breast cancer.
B. Metformin decreases the risk of invasive breast cancer in patients with type 2 diabetes
mellitus in large randomized studies.
C. Both a and b.
D. None of the above.

Question 6.15 Which of the following is TRUE regarding diet-derived natural products?
A. There have been no large prospective trials demonstrating the efficacy of natural
products in reducing cancer risk.
B. Berry formulations reduce esophageal dysplasia and oral leukoplakia in phase II studies.
C. Both a and b.
D. None of the above.
Question 6.16 Helicobacter pylori infection is associated with increased risk of which of the
following?
A. Esophageal adenocarcinoma
B. Gastric cardia carcinoma
C. Esophageal squamous cell carcinoma
D. Intestinal-type gastric adenocarcinoma

Question 6.17 In performing risk-reducing salpingo-oophorectomy for high-risk patients:


A. An open approach is required to perform an en-bloc resection of ovaries, uterus, and
nearby pelvic lymph nodes.
B. BRCA mutation confers a high risk of uterine cancer.
C. Patients may be subsequently protected from all breast cancers.
D. Patients with Lynch syndrome should undergo simultaneous hysterectomy.

Question 6.18 In patients with medullary thyroid carcinoma (MTC), which of the following
preoperative basal calcitonin levels is an indication for ipsilateral central and lateral neck
dissection?
A. 15 to 20 pg/mL
B. 20 to 50 pg/mL
C. 50 to 200 pg/mL
D. 200 to 500 pg/mL

Question 6.19 What is TRUE regarding the risk of endometrial cancer for patients with
Lynch syndrome?
A. The risk for women with Lynch syndrome varies from 20% to 60% and is higher than
development of ovarian cancer.
B. Clinically, these cancers are frequently poorly differentiated, present at an advanced
stage, and associated with poor prognosis.
C. Risk reducing hysterectomy should be performed shortly after menarche.
D. Estrogen replacement is contraindicated in women with Lynch syndrome undergoing
bilateral salpingo-oophorectomy.

Question 6.20 What is the best treatment of a patient with FAP presenting with
approximately 300 colorectal polyps and 15 rectal adenomas?
A. Observation
B. Total proctocolectomy with permanent ileostomy (TPC)
C. Proctocolectomy with ileal pouch-anal anastomosis (IPAA)
D. Total colectomy with ileorectal anastomosis (IRA)

Question 6.21 Which of the following malignancies are associated with germ line mutations
in the RET protooncogene?
A. Multiple endocrine neoplasia (MEN) 2A
B. MEN 2B
C. Sporadic medullary thyroid carcinoma
D. All the above
ANSWERS

Question 6.1 The answer is A.


NRT increases the odds of quitting smoking by approximately twofold compared with
placebo. Studies have also shown that NRT is not associated with increased risk of
carcinogenesis in the general population.

Question 6.2 The answer is D.


Studies have demonstrated that current smoking increases the risk of recurrences in
multiple cancers, including head and neck cancer, lung cancer, prostate cancer, and
bladder cancer. Stopping smoking before surgery is advised since it reduces postoperative
morbidity and mortality. Current smoking also increases radiotherapy-associated toxicities
in multiple types of cancers.

Question 6.3 The answer is C.


Although recent safety studies have shown no difference in neuropsychiatric side effects
between varenicline and NRTs, varenicline carries a FDA-issue box warning that some
patients develop suicidal thoughts after its use. Anorexia, hypertension, and elevated liver
enzymes are not commonly seen with varenicline.

Question 6.4 The answer is C.


Both clonidine and nortriptyline are recommended as second-line pharmacotherapies for
tobacco dependence.

Question 6.5 The answer is D.


Intraepithelial neoplasias could be used as surrogate end points in chemopreventive
studies. Intraepithelial neoplasias include colorectal adenoma, Barrett esophagus, actinic
keratosis of skin, dysplastic nevus, cervical intraepithelial neoplasia, leukoplakia, prostate
intraepithelial neoplasia, bronchial dysplasia, and pancreatic intraepithelial neoplasia.
Cervical ectropion, junctional nevus, and seborrheic keratosis are not intraepithelial
neoplasias and do not confer increased risk of developing cancer.

Question 6.6 The answer is A.


In smokers, the Alpha-Tocopherol, Beta-Carotene Trial (ATBC) and the Carotene and
Retinol Efficacy Trial (CARET) showed increased risk of lung cancer in patients receiving
beta-carotene with or without retinol. Although topical all-trans retinoic acid (ATRA)
might cause regression of cervical dysplasia, this effect was not seen in studies involving
systemic retinoids or beta-carotene. Prospective studies found no protective effect of beta-
carotene supplementation. Retinoids and beta-carotene have been studied extensively in
both clinical and epidemiologic trials. Systemic retinoids reduce skin cancer incidence by
63% during therapy in patients with xeroderma pigmentosum, although the protective
effect is lost after discontinuation of the treatment.
Question 6.7 The answer is C.
Selenium increased the incidence of total nonmelanoma skin cancer and squamous cell
skin cancer in patients with a history of nonmelanoma skin cancer. In the SELECT trial,
the incidence of prostate cancer was not significantly different between the selenium arm
and placebo arm, or between the selenium+vitamin E arm and the placebo arm.

Question 6.8 The answer is A.


Calcium decreased the risk of recurrent colorectal adenoma in a randomized study. In the
WHI trial, the combination of vitamin D and calcium did not decrease the risk of
developing either colon cancer or breast cancer. There has been no evidence to suggest
that high levels of vitamin D decrease the risk of pancreatic cancer.

Question 6.9 The answer is D.


Several studies have shown protective effects from NSAIDs in patients with colorectal
cancer. Although regular use of aspirin was associated with protective effects in
population base studies, the benefit of low-dose aspirin was not yet clear in randomized
clinical trials. The limited benefit of NSAIDS, including low-dose aspirin, in preventing
colorectal cancer, does not justify their use in average-risk individuals. Sulindac, at high
dose, was associated with both regression of existing polyps and prevention of new polyps
in patients with FAP, but the protection was transient and incomplete. Although the
cyclooxygenase-2 inhibitors rofecoxib and celecoxib were shown to reduce the risks of
metachronous adenoma, the studies were terminated early because of their side effects,
being unable to detect the reduction of colorectal cancer incidence. Although NSAIDs
reduce the risk of colorectal adenoma and colorectal cancer, these agents are associated
with significant gastrointestinal and cardiovascular toxicities, which outweigh the
benefits. Therefore, the best strategy for the general population remains effective
screening.

Question 6.10 The answer is C.


Tamoxifen is more effective than raloxifene in decreasing the breast cancer incidence.
Neither tamoxifen nor raloxifene reduces the risk of ER-negative breast cancer or breast
cancer-specific mortality. Tamoxifen, but not raloxifene, reduces the risk of in situ breast
neoplasm.

Question 6.11 The answer is C.


Raloxifene is a nonsteroidal selective estrogen receptor modulator that binds to estrogen
receptor leading to estrogen agonist effects in some tissues and estrogen antagonist effects
in others. Raloxifene has estrogen agonist activity in the bone but is less active than
tamoxifen in the uterus. Unlike tamoxifen, raloxifene was not associated with increased
risk of uterine cancer. Because of raloxifene’s agonist activity in bone, it reduced the risk
of vertebral fracture. Raloxifene does not increase the risk of coronary artery heart
disease, but studies showed increased risk of stroke.
Question 6.12 The answer is B.
In randomized placebo-controlled trials, patients treated with either finasteride or
dutasteride had reduced incidence of prostate cancer. However, patients who received
either drug were more likely to develop tumors of a high Gleason score compared to
those received placebo. Neither the overall survival nor the survival after prostate cancer
diagnosis was different between the finasteride group and the placebo group.

Question 6.13 The answer is D.


Although the Women’s Health Initiative study found that lovastatin conferred a lower risk
of colorectal cancer, there has been no strong evidence that statins decrease general
cancer risks. The Nurse’s Health Study showed no association with statin use and risk of
breast cancer.

Question 6.14 The answer is A.


Bisphophonates reduce the risk of recurrent breast cancer in postmenopausal women with
breast cancer in randomized studies. Although the Women’s Health Initiative observed a
lower incidence of breast cancer in diabetic women treated with metformin, this
association has not been validated in large randomized studies.

Question 6.15 The answer is C.


No diet-derived natural products have been studied in large randomized trials regarding
their role in reducing cancer risk. In human phase II trials, berry formulations decreased
esophageal dysplasia and oral leukoplakia.

Question 6.16 The answer is D.


H. pylori infection increases the risk of intestinal-type gastric adenocarcinoma. Studies
have showed that the eradication of H. pylori reduces the risk of gastric cancer in high-
risk populations. Some studies suggested that H. pylori infections might be associated
with a reduced risk of esophageal adenocarcinoma and gastric cardia carcinoma.

Question 6.17 The answer is D.


In most women, bilateral salpingo-oophorectomy may be performed safely using a
laparoscopic approach, and the uterus can be spared. The BRCA mutation does not
increase the risk of uterine cancer. However, patients with mutations of the mismatch-
repair genes in Lynch syndrome have an increased rate of ovarian and endometrial
cancers. After risk-reducing salpingo-oophorectomy (RRSO), patients may be subsequently
protected from estrogen-positive breast cancers, however, there is no protection from
estrogen-negative cancer.

Question 6.18 The answer is B.


Based on current guidelines, surgical therapy for patients with MTC can be stratified
according to their preoperative basal calcitonin levels. For levels <20 pg/mL, a total
thyroidectomy alone is needed. For levels 20 to 50 pg/mL, an ipsilateral central and
lateral neck dissection is added. For 50 to 200 pg/mL, the contralateral central neck is
explored, and the contralateral lateral neck is dissected for levels between 200 and 500
pg/mL. Mediastinal exploration is reserved for patients with imaging showing disease.

Question 6.19 The answer is A.


In women with Lynch syndrome (LS), endometrial cancers have been reported at rates of
20% to 60%. Ovarian cancers are detected at rates of 12%. Endometrial and cancers
associated with LS have clinically favorable features, including well-differentiated tumors
that are early in stage and have survival of 90%. Risk-reducing surgery is offered once the
woman has completed childbearing. Estrogen replacement is not contraindicated in
women who have otherwise undergone only bilateral salpingo-oophorectomy.

Question 6.20 The answer is D.


Because of the virtually universal risk of developing cancer by the age of 40 years in the
absence of therapy, patients with FAP should undergo surgery, with the timing depending
on the extent of disease. Among the surgical options, TPC is rarely used and most patients
will be treated with IRA or IPAA. One of the main factors in deciding whether to remove
the rectum is the risk of rectal cancer. Therefore, IRA may be considered in patients with
less than 1,000 colorectal polyps and less than 20 rectal adenomas. In contrast, patients
with a higher number of rectal adenomas, adenomas larger than 3 cm, or severe dysplasia
should undergo proctectomy in addition to colectomy, through IPAA.

Question 6.21 The answer is D.


MEN 2A, MEN 2B, familial medullary thyroid carcinoma and sporadic medullary
carcinoma are associated with germ line mutations in the RET gene.

Corresponding Chapters in Cancer: Principles & Practice of Oncology, Tenth Edition: 31 (Tobacco Use and the Cancer Patient),
32 (Role of Surgery in Cancer Prevention), and 33 (Cancer Risk Reducing Agents).
7 Genetic Counseling
Jennifer Ivanovich

QUESTIONS
Each of the numbered items below is followed by lettered answers. Select the ONE lettered answer
that is BEST in each case unless instructed otherwise.
Refer to the following pedigree for Questions 7.1 to 7.4.

Question 7.1 The family depicted has Lynch syndrome (previously referred to as the
hereditary nonpolyposis colon cancer [HNPCC]) syndrome. This syndrome is characterized by
a high risk to develop colon and endometrial cancer and an increased risk to develop various
other tumor types. Before her death, individual III-2 had genetic testing of the MLH1 and
MSH2 genes, and an MSH2 gene mutation was identified. Specifically, the mutation is c.942
+ 3A > T. MLH1 and MSH2 gene mutations account for 80% to 90% of families with
HNPCC. Individual II-1, depicted with the arrow, has not been diagnosed with cancer. He has
undergone frequent colonoscopy screening. He has also had genetic testing and was found to
carry the family MSH2 gene mutation. The finding that individual II-1 has not been diagnosed
with cancer can be explained by which of the following concepts?
A. Genetic testing laboratory error
B. Clinical variability
C. Genetic heterogeneity
D. Reduced penetrance

Question 7.2 Assuming individual II-3 has the family MSH2 gene mutation, what is the
calculated probability Diane (individual IV-1) has inherited the MSH2 gene mutation?
A. 0%
B. 12.5%
C. 25%
D. 50%

Question 7.3 Diane (individual IV-1) is 24 years of age. She approaches her primary care
physician about her medical management, given her family history of the Lynch syndrome.
She has a healthy body weight and does not smoke. Both her paternal grandmother (II-3) and
her father (III-6) refuse to have genetic testing. In addition to her annual Pap smear, which of
the following cancer screening is most appropriate for Diane?
A. Colonoscopy screening to begin at 50 years of age
B. Sigmoidoscopy screening to begin by 25 years of age
C. Colonoscopy screening to begin by 25 years of age
D. Colonoscopy and mammography screening to begin by 25 years of age

Question 7.4 Diane (individual IV-1) decides to pursue genetic testing to aid in her medical
management. She states that her father will not undergo genetic testing. What specific
genetic test should be ordered?
A. Full sequencing of the MSH2 gene
B. Full sequencing of the MSH2 and MLH1 genes
C. Mutation specific analysis of the MSH2 gene
D. Deletion analysis using multiplex ligation-dependent probe amplification

Question 7.5 Which of the following factors suggest that a family may have hereditary
cancer?
A. Young age at diagnosis
B. Bilateral cancer in an affected family member
C. Multiple affected generations
D. All of the above
Refer to the following pedigree for Questions 7.6 to 7.8.
Olivia is a 38-year-old healthy woman who presents to her primary care physician
concerned about her family history of cancer. She has no chronic health concerns. Both
her mother’s and father’s family are of Ashkenazi Jewish ancestry. Olivia pursues an
evaluation with a clinical geneticist for assessment and genetic testing.

Question 7.6 The clinical geneticist diagnoses the family with which cancer predisposition
syndrome?
A. HNPCC syndrome
B. Hereditary breast cancer 1 (BRCA1) syndrome
C. Li–Fraumeni syndrome
D. Hereditary BRCA2 syndrome
E. None of the above

Question 7.7 Genetic testing is recommended to begin with which family member(s)?
A. Olivia, III-2
B. Olivia’s oldest sibling, individual III-4
C. Olivia and her two siblings should be tested at the same time
D. Olivia’s maternal first cousin, individual III-1

Question 7.8 Which of the following statements regarding prophylactic oophorectomy is


TRUE?
A. Prophylactic bilateral salpingo-oophorectomy (BSO) is the most effective therapy to
reduce the risk of ovarian cancer in women with BRCA1 or BRCA2 gene mutation.
B. BSO in BRCA1 or BRCA2 gene mutation carriers is indicated at age 30.
C. BSO in BRCA1 or BRCA2 gene mutation carriers has no effect on the risk of developing
breast cancer.
D. All the above.

Question 7.9 Jennifer is a 38-year-old woman who was diagnosed with stage IIA ductal
carcinoma at 30 years of age. She undergoes direct genetic testing of the BRCA1 and BRCA2
genes and is found to have a BRCA2 gene variant. Specifically the variant is Q713L (2366A >
T), which results in the substitution of leucine for glutamine at amino acid 713 of the BRCA2
protein. The functional significance is unknown. Jennifer’s sister, Angela, is 40 years of age
and has no personal history of cancer. Which is the most appropriate genetic testing approach
for Angela?
A. Genetic testing is recommended for Angela because it will clarify her breast cancer risk.
B. Genetic testing is not recommended for Angela because it will not clarify her breast
cancer risk.
C. Genetic testing is recommended for Angela because it will help clarify Jennifer’s genetic
test results.
D. A and C.

Question 7.10 Which of the following statements regarding people with a BRCA1 or BRCA2
gene mutation are TRUE?
A. The majority of BRCA1 gene mutation carriers who develop breast cancer develop
estrogen receptor–positive breast tumors, and BRCA2 gene mutation carriers most often
develop estrogen receptor–negative breast tumors.
B. BRCA1 and BRCA2 gene mutation carriers have an increased risk to develop pancreatic
cancer.
C. BRCA2 gene mutations are associated with an increased risk for follicular thyroid cancer
D. Prophylactic bilateral mastectomy decreases the risk of breast cancer by more than 90%
among women with a BRCA1 or BRCA2 gene mutation.
Refer to the following pedigree for Questions 7.11 and 7.12.

Question 7.11 Pauline, the family proband as designated by the arrow, is a healthy 36-year-
old woman who presents to her gynecologist concerned about her breast cancer risk. She
requests genetic testing. No other family member has undergone genetic testing. The
gynecologist orders BRCA1 and BRCA2 gene testing. No mutation is identified. Which of the
following statements is the CORRECT interpretation of Pauline’s testing?
A. Pauline’s test result is a true negative.
B. Pauline’s test result is a variant of uncertain significance.
C. Pauline’s test result is uninformative.
D. Pauline’s test result is positive.

Question 7.12 Pauline’s sister, individual III-2, is currently undergoing neoadjuvant


chemotherapy for an aggressive ductal carcinoma of the breast. She is not interested in
pursuing genetic testing at this time given the demands of her treatment. What breast health
screening recommendations do you make for Pauline given her negative genetic testing?
A. Mammography beginning at age 40 years
B. Mammography and breast ultrasound beginning at age 40 years
C. Mammography and breast ultrasound beginning now
D. Mammography and breast MRI beginning now

Question 7.13 Which of the following malignancies does NOT have a significant increase in
incidence among people with Lynch syndrome?
A. Colon cancer
B. Cervical cancer
C. Ovarian cancer
D. Endometrial cancer

Question 7.14 A 28-year-old woman with bilateral ductal cancer presents to the medical
oncologist for assessment and treatment. Her medical history is notable for intussusception at
6 years of age. On her physical examination, the medicine resident notices small blue/black
hyperpigmented macules on her lips, buccal mucosa, and fingertips. What is her diagnosis?
A. Hereditary BRCA1 syndrome
B. Hereditary BRCA2 syndrome
C. Li–Fraumeni syndrome
D. Peutz–Jeghers syndrome
E. Cowden syndrome
ANSWERS

Question 7.1 The answer is D.


Penetrance may be defined as the proportion of individuals with a mutant genotype who
show any manifestation of a given disorder. A disorder that is expressed in less than 100%
of individuals with the mutant genotype is said to have reduced penetrance. For example,
if only 60% of individuals with the genotype express clinical features, the disease is
considered to be 60% penetrant. Expression in hereditary cancer syndromes occurs when
an individual is diagnosed with one of the associated malignancies or benign lesions.
Almost all cancer syndromes show reduced penetrance. This concept is important to
understand because many families believe if a family member has not developed a specific
cancer, he/she cannot pass on the gene mutation to a child.

Question 7.2 The answer is C.


The Lynch syndrome is an autosomal-dominant disease. Calculation of Diane’s probability
to have inherited the gene mutation is based on autosomal-dominant inheritance. By
assuming individual II-3 has the MSH2 gene mutation, the calculated probability Diane’s
father (III-6) has inherited the mutation is 50%. The probability Diane has inherited it
from her father is 50%. Thus, Diane’s calculated probability to have inherited the MSH2
gene mutation is 25% (50% × 50%). Most hereditary cancer syndromes identified to date
are inherited in an autosomal-dominant manner with reduced penetrance. In a clinical
setting, it would be important not to assume that individual II-3 has the mutation.
Although she has been diagnosed with colon cancer, the cardinal feature of the Lynch
syndrome, individuals may be diagnosed with an associated cancer type and not have the
causal gene mutation. That is, an individual may develop a sporadic cancer that does not
result from the underlying family gene mutation. If individual II-3 does not have the
MSH2 gene mutation, her sporadic colon cancer would be a phenocopy, a mimic of the
syndrome phenotype. II-3’s daughter, who died of breast cancer at 39 years of age, does
not provide any insight as to whether II-3 has the gene mutation. It remains under
investigation if breast cancer is one of the associated cancer types in the Lynch syndrome.

Question 7.3 The answer is C.


The Lynch syndrome is associated with a 70% to 80% lifetime risk for colon cancer and a
20% to 60% risk for endometrial cancer. There are also increased risks for gastric
(intestinal type), ovarian, urinary tract, and central nervous system tumors, with the level
of risk varying for each cancer type. Colonoscopy screening is recommended to begin by
20 to 25 years of age or 10 years before the youngest age of cancer diagnosis and
repeated every 1 to 2 years. Sigmoidoscopy is not recommended because of the
predominance of right-sided colon cancers. Approximately two-thirds of all colon cancers
in the Lynch syndrome occur in the ascending colon. Consensus recommendations also
include consideration of endometrial cancer surveillance with transvaginal ultrasound and
endometrial biopsy; however, the efficacy of this screening regimen is unknown.
Although Diane’s (IV-1) paternal aunt (III-5) died of breast cancer at a young age, there
are insufficient data to prove breast cancer is one of the associated cancers in the Lynch
syndrome. As such, intensified mammography screening is not warranted. Intensified
surveillance recommendations are available for many hereditary cancer syndromes.
Limited data exist to prove the efficacy of these screening protocols. However, until
larger data sets are developed, consensus recommendations are issued to help guide
clinicians in the care of high-risk families.

Question 7.4 The answer is C.


The MSH2 gene mutation, c.942 + 3A > T, identified in an affected family member,
results in an A > T change at the third nucleotide of the splice-donor site of intron 5 and
causes an in-frame deletion of exon 5. The mutation can be detected using sequencing.
Mutation specific analysis for the MSH2 gene c.942 + 3A > T mutation is the only test
that is needed. Full sequencing of the MSH2 gene is unnecessary and may result in excess
cost to the individual pursuing genetic testing. Approximately 20% of MSH2 gene
mutations are large deletions or gene rearrangements. These mutations cannot be
detected using sequencing. Southern blot analysis and multiplex ligation-dependent probe
amplification are techniques used to detect large deletions. A working knowledge of the
types of mutations that have been described for a given gene, as well as an understanding
of the limitations of the techniques used by a given laboratory, is crucial in overseeing the
clinical genetic testing of a family.
Diane (IV-1) has the right to pursue clinical genetic testing to aid in her medical
management. If her testing is positive for the MSH2 gene mutation, then her father and
paternal grandmother may gain information they have previously chosen not to obtain. In
advance of ordering the testing, it is crucial to discuss if and how Diane is going to
communicate her test results with her family and their potential reactions. By facilitating
this discussion, the clinician acknowledges some of the psychosocial implications
associated with hereditary disease and providing anticipatory guidance as to how Diane
may address potential reactions from her family.

Question 7.5 The answer is D.


Several factors suggest the possibility of hereditary disease including young age at
diagnosis, bilateral organ involvement in an affected family member, multiple affected
family members in multiple generations, rare tumor histology such as medullary thyroid
cancer, and unusual presentation of a tumor such as male breast cancer.

Question 7.6 The answer is E.


The family cancer history meets the diagnostic criteria for the hereditary diffuse gastric
cancer syndrome (HDGC). HDGC is an autosomal-dominant, highly penetrant cancer
predisposition syndrome that results from mutations in the E-cadherin gene (CDH1) in
one-third of families. The genetic etiology is unknown for the remaining families. This
syndrome is associated with a 70% cumulative lifetime risk for diffuse gastric cancer, an
adenocarcinoma that causes thickening of the gastric wall (linitis plastica) without a
distinct mass. There is also a 40% lifetime risk for lobular breast cancer among female
mutation carriers. The youngest reported case of gastric cancer is 14 years with a median
age at diagnosis of 38 years. Intensified surveillance with upper endoscopy and breast
mammography with MRI is recommended. Prophylactic gastrectomy may also be
considered given the limitation in detecting diffuse lesions. Precancerous lesions have
been detected in a small series of “prophylactic” gastrectomy specimens.
It is a challenge for healthcare providers to stay well informed of advances in their own
medical discipline, let alone to maintain a working knowledge of the advances for other
specialties and rare syndromes. Working collaboratively with a clinical geneticist and
genetic counselor may help to facilitate the identification of families with hereditary
cancer. Although these syndromes may be considered “rare,” they often go unrecognized
and the opportunity for positive medical intervention for a family is lost. The first step in
the identification of hereditary cancer begins with a thorough collection and evaluation of
the family cancer history. Detailed reviews of many hereditary cancer syndromes can be
found at http://www.ncbi.nlm.nih.gov/books/NBK1116/.

Question 7.7 The answer is D.


The most informative approach to genetic testing is to begin with an affected family
member, diagnosed with one of the associated cancers for the suspected cancer
predisposition syndrome. If an affected family undergoes testing first and if a gene
mutation is discovered, then the underlying genetic basis for the cancer predisposition in
the family has been identified, the testing is informative, and other family members may
pursue mutation specific analysis. If a gene mutation is not identified, then the underlying
genetic basis for disease has not been identified, the testing is noninformative, and
consequently predictive genetic testing is not available for other family members. For this
family, if individual III-A does not have a CDH1 gene mutation, the family would still be
considered to have the hereditary diffuse gastric cancer syndrome, however an
informative genetic test would not be available to help guide Olivia’s medical care.

Question 7.8 The answer is A.


BSO in BRCA carriers is the most effective procedure to reduce the risk of ovarian cancer
and is generally recommended upon completion of childbearing. BSO also decreases the
risk of subsequent breast cancer.

Question 7.9 The answer is B


Missense variants result in the substitution of one amino acid for another. These variants
occur frequently in the BRCA1 and BRCA2 genes. The functional significance cannot be
determined with gene sequencing alone. Functional analysis of the BRCA1 and BRCA2
proteins is not available clinically. Angela has a 50% probability to have the BRCA2 gene
variant, but because the functional significance is unknown, testing Angela does not
provide her with any clinically relevant information. Furthermore, because Angela has not
been diagnosed with cancer, genetic analysis does not provide any new information to
clarify Jennifer’s test results. If Angela had been diagnosed with early-onset breast cancer,
then analysis for the BRCA2 gene variant would have been offered for research purposes
only to help determine whether the variant is tracking with disease in the family.
For this family, genetic testing did not provide any new information to facilitate their
medical decision making. Individuals with these types of gene variants are often left
intellectually and emotionally frustrated by the testing. Imagine deciding to pursue
genetic testing with the intention of using the results to guide your personal and family
medical care only to learn the testing provides no clear answer. As clinical genetic testing
quickly becomes available following gene discovery, clinicians must contend with the
identification of variants of unknown functional significance. The development of
functional, rather than descriptive, testing will help to address this challenging clinical
issue. Until such techniques are developed, it is important to inform individuals of the
possibility of identifying these types of variants of unknown before the initiation of
genetic testing.

Question 7.10 The answer is B and D.


BRCA1 and BRCA2 gene mutations have a high risk for breast and ovarian cancer and an
increased risk for other cancer types, including pancreatic cancer. Studies evaluating the
efficacy of prophylactic mastectomy, conducted specifically among female BRCA1 and
BRCA2 gene mutations, have demonstrated a greater than 90% reduction in the risk to
develop breast cancer. Women with a BRCA1 gene mutation most often develop estrogen
receptor–negative breast cancers and women with BRCA2 gene mutations develop
estrogen receptor–positive breast tumors. Follicular thyroid cancer is an associated cancer
in the Cowden syndrome, and has not been noted to occur more frequently among
individuals with a BRCA1 or BRCA2 gene mutation.

Question 7.11 The answer is C.


Pauline’s testing is uninformative because the underlying genetic basis for cancer
predisposition has not yet been identified in her family. When possible, genetic testing
should always begin with an affected family member. In essence, when evaluating a
family with genetic disease, testing should begin with the person who expresses features
of the inherited condition. In the case of hereditary cancer syndromes, expression is
defined as a person who is diagnosed with one of the associated malignancies or benign
lesions. If Pauline’s sister undergoes clinical BRCA1 and BRCA2 genetic testing, then these
results may change the interpretation of Pauline’s testing. For example, if Pauline’s sister’s
testing is positive for a mutation, then Pauline’s testing would be interpreted as a true
negative. If Pauline’s sister’s testing is negative, then Pauline’s testing was of no clinical
value because the basis for disease remains unknown. Without prior knowledge of the
family’s gene mutation, Pauline’s negative testing may have provided a false and
dangerous sense of security.

Question 7.12 The answer is D.


The underlying genetic basis for cancer predisposition remains unknown. Even though
Pauline had negative genetic testing, it is not known whether she inherited the disease
causing gene mutation. As such, Pauline, and her paternal family members, should
continue to be considered at high risk and followed accordingly. The clinician must return
to the family cancer history to determine her medical follow-up. The family history is
consistent with the clinical diagnosis of either the hereditary BRCA1 or BRCA2 syndrome.
Pauline should follow the same recommendations as families with a confirmed diagnosis
of one of these syndromes, until genetic testing proves informative for her family.
Intensified surveillance with mammography and breast MRI screening, beginning by 25
years of age, is recommended. In studies of individuals with an increased breast cancer
risk, the combination of mammography and breast MRI proved to be more effective in
identifying abnormal breast lesions than mammography alone. The combination of
mammography and breast ultrasound for women with an increased breast cancer risk is
still under investigation. In a 1997 study published by Giradello and colleagues in the New
England Journal of Medicine, approximately one-third of all ordering physicians
misinterpreted the results of APC gene testing. The majority of errors arose as physicians
misinterpreted a negative test result when the genetic basis for disease had not previously
been established in a given family. Ongoing genetics education is needed for practitioners
who assume the genetics care and testing of families with hereditary disease.

Question 7.13 The answer is B.


The Lynch syndrome is primarily associated with a high risk for colon and endometrial
cancer and an increased risk for other tumors including ovary, stomach, small intestine,
hepatobiliary tract, urinary tract, brain, and skin. Cervical cancer is not one of the known
associated cancer types. The life time risk for colon cancer is estimated to range from 60%
to 80% and for uterine cancer as high as 60% lifetime risk.

Question 7.14 The answer is D.


Peutz–Jeghers syndrome is characterized by the association of hyperpigmented macules,
increased cancer risk, and gastrointestinal polyposis. Blue/black macules may be found
around the mouth, buccal mucosa, nostrils, eyes, and fingertips. These lesions may fade
over time. Hamartomatous polyps (Peutz–Jeghers type) most commonly occur in the
small bowel but can also develop in the colon or the stomach. Recurrent obstruction and
intussusception may occur. Individuals with Peutz–Jeghers syndrome are at increased risk
for various tumors, including breast, colorectal, ovarian, gastric, and pancreatic cancers.
Affected females are also at increased risk for adenoma malignum of the cervix and a
benign ovarian tumor, sex cord tumors with annular tubules (SCTAT). Peutz–Jeghers
syndrome results from mutations in the STK11 gene found on chromosome 19. The Peutz–
Jeghers syndrome serves as an example of the importance of a detailed physical
examination, in combination with the family cancer history, to identify individuals with a
hereditary cancer syndrome. Other physical features that provide important diagnostic
clues include the mucosal neuromas of the lips and tongue in the multiple endocrine
neoplasia syndrome type 2B, trichilemmomas or papillomatous lesions in the Cowden
syndrome, and sebaceous adenomas in the Lynch syndrome.

Corresponding Chapter in Cancer: Principles & Practice of Oncology, Tenth Edition: 35 (Genetic Counseling).
8 Principles of Radiation Oncology
Hak Choy and Aaron Laine

QUESTIONS
Each of the numbered items below is followed by lettered answers. Select the ONE lettered answer
that is BEST in each case unless instructed otherwise.

Question 8.1 Which of the following describes the specific pathways for the repair of
double-stranded DNA breaks?
A. The repair may be by homologous recombination (HR) or nonhomologous end-joining
(NHEJ) pathway.
B. The HR repair pathway functions by degrading the single strand at each side of the
break and then annealing the two ends.
C. The NHEJ pathway functions by replicating the missing genetic information from the
homologous DNA template.
D. NHEJ is a minor component of mechanism for repair of double-stranded DNA breaks in
mammalian cells.

Question 8.2 Which of the following is TRUE about ionizing events in tissues caused by x-
rays?
A. Radiation dose describes the quantity of energy deposited.
B. The direct effect of radiation is primarily due to injury to cellular membranes.
C. The relative biological effectiveness (RBE) describes the ratio of doses required to give
the same amount of killing under normoxic and hypoxic conditions.
D. Indirect action of ionizing radiation where hydroxyl radicals damage target tissues is
commonly seen.

Question 8.3 Which of the following statements describes the DNA damage/repair process
associated with ionizing radiation?
A. Single-stranded breaks of the DNA are thought to represent the lethal event.
B. All of the radiation-induced, double-stranded breaks are rejoined in cells within 2 hours.
C. In mammalian cells, choice of repair can be biased by phase of the cell cycle and by
abundance of repetitive DNA.
D. Nonhomologous end-joining (NHEJ) is effective in rejoining DNA double-strand break
because it is an error free process.

Question 8.4 Which of the following statements describes the dose-related cellular response
to radiation?
A. In mammalian cells, there is generally a linear and quadratic relationship between the
cell killing and the dose given.
B. The term “D0” describes the quasi-threshold dose, defining the width of the shoulder
C. Densely ionizing radiation leads to an extended shoulder on the survival curve.
D. In general, postirradiation conditions that accelerate the cell division are the ones most
favorable to repair of potentially lethal damage.

Question 8.5 Which of the following statements is TRUE regarding independent events (4
R’s) that occur during fractionated radiotherapy?
A. Repopulation refers to spontaneous repopulation and induced cell proliferation or
recruitment of cells after irradiation.
B. Repair explains the linear portion of the radiation survival curve showing that cells can
repair some of the radiation damage.
C. Redistribution explains migration of cells away from an irradiation source.
D. Reoxygenation describes the decrease of tumor oxygen levels during fractionated
radiation.

Question 8.6 Which of the following acute effects are related to radiation-induced cell
death?
A. Cystitis
B. Esophagitis
C. Dermatitis
D. Proctitis
E. All of the above

Question 8.7 The abscopal effect of radiation is


A. Attributed to the activation of antigen and cytokine release.
B. Commonly seen because immune system evasion by cancer cells is rarely present.
C. Seen more frequently with lower doses and greater fractions.
D. Activation of the humoral immune response.
E. Double-stranded breaks in DNA in nonirradiated cells

Question 8.8 Which of the following best describes the modern linear accelerator?
A. A modern linear accelerator (LINAC) can deliver energies up to 1 MeV.
B. Generate Bremsstrahlung x-rays by bombarding a target with high-energy electrons
C. Primarily uses 60Cobalt which is always “on” and must be kept in a shielded position
until the beam is needed for treatment.
D. The focal point of the gantry’s rotation is called the field edge.

Question 8.9 Which of the following statements describes the role of oxygen in radiation
effects?
A. A greater dose of radiation is required for cell killing in an oxic condition compared
with a hypoxic condition.
B. A randomized trial showed that epoetin b improved survival in patients with head and
neck cancer.
C. Oxygen need not be present at the time of irradiation for oxygen enhancement of
radiotherapy to occur.
D. Hyperbaric oxygen often shows a dramatic increase in curability with standard
fractionated radiotherapy.
E. The oxygen enhancement ratio (OER) has more relevance on the exponential portion of
the cell survival curve.

Question 8.10 Which of the following best describes the concept of altered fractionation?
A. Accelerated fractionation does not reduce the overall treatment time.
B. Hyperfractionation refers to a radiotherapy schedule that uses multiple daily treatments
more than 6 hours apart with reduced fraction size and increased number of fractions.
C. Standard fractionation is commonly defined as 3 to 5 Gy/day
D. The goal of accelerated fractionation is to complete radiation before accelerated
reoxygenation occurs

Question 8.11 Which of the following statements regarding interaction of chemotherapy and
radiation therapy is TRUE?
A. Rationale for combining radiation therapy with radiosensitizing chemotherapy is
primarily to confer an additive effect.
B. Gemcitabine is a potent radiosensitizer and special consideration is not required when
administered with radiation for treatment of lung and head and neck cancers.
C. Paclitaxel radiosensitizing effect to due to G1 cell-cycle arrest
D. Mechanism of radiosensitization by cisplatin may be due to its ability to inhibit DNA
repair of radiation induced DNA double-strand breaks.

Question 8.12 Which of the following statements is TRUE regarding intensity-modulated


radiation therapy (IMRT)?
A. It is an uncomplicated method to deliver radiation in a nonconformal manner.
B. Is delivered only using a limited number of beam orientations.
C. Optimization of each individual beamlet can create sharp dose gradients.
D. Does not take into account organs at risk (OARs) when developing the treatment plan.

Question 8.13 Which of the following statements best describes the tissue effects from
radiation?
A. Early or acute effects typically occur within months after irradiation.
B. Large α/β> ratio has a small “shoulder” in the low-dose portion.
C. The frequencies of late effects depend strongly on radiation fraction size.
D. Typical human tumors and early-responding normal tissues have a small α/β ratio.

Question 8.14 Which of the following statements describes of the interaction of x-rays with
biologic material?
A. In modern treatment with greater than 4 MeV photons, photoelectric effect dominates
the interaction.
B. In Compton scattering, an incoming x-ray transfers all its energy to an inner orbit
electron, which is ejected from the atom. A photon is produced as an outer shell
electron fills the vacant hole.
C. In photoelectric effect, energy from the x-ray is both absorbed and scattered. The
photon emerges with reduced energy and a change in direction.
D. In pair production, an electron and a positron are produced.

Question 8.15 Which of the following statements describes the depth dose characteristics of
radiation?
A. Higher-energy photons deposit more dose to the skin surface.
B. For a given energy, electrons generally penetrate deeper in tissue compared with
photons.
C. Electron beams deposit less dose to the skin surface as the incident electron energy
increases.
D. Depth of maximum dose increases as the energy of the incident beam increases.

Question 8.16 Which of the following statements is TRUE regarding brachytherapy?


A. Isotopes with properties of long half-lives and high energy are typically used for
permanent implants.
B. Brachytherapy refers to placement of radioactive sources next to or inside the tumor, or
within body or surgical cavities.
C. The fluence drops in intensity as 1 over the distance from the source.
D. A potential advantage of brachytherapy is its ability to produce conformal treatments
with high normal tissue dose.

Question 8.17 Which of the following can be a subacute toxicity from radiation therapy?
A. Pneumonitis
B. Myelitis.
C. Brain necrosis.
D. B and C.

Question 8.18 Which of the following best describes the treatment planning process?
A. At the time of image acquisition for planning, tumor motion caused by respiration must
be determined.
B. Immobilization is not important because it does not add to accuracy.
C. Three-dimensional dose distribution in each patient is easily measured.
D. Intensity-modulated radiation therapy (IMRT) cannot control the shaping of the dose
distribution.

Question 8.19 Which of the following is TRUE regarding charged particle beams?
A. Charged particle beam therapy includes photon-based therapy.
B. Protons have shown to confer a definite clinical benefit over photon-based therapies for
most clinical cancer applications.
C. Ability of charged particles to stop at a given depth gives it a potential advantage for
treatment of tumors in close proximity to critical structures.
D. Charged particle therapy is relatively inexpensive in terms of the cost involved with
production and operation of such facility.
ANSWERS

Question 8.1 The answer is A.


There are two generally accepted mechanisms of repair for DNA double-strand breaks.
They are broadly known as the homologous recombination (HR) or nonhomologous end-
joining (NHEJ) pathways. HR functions by replicating the missing genetic information
from the homologous DNA template. NHEJ functions by degrading the single strand at
each side of the break and then annealing the two ends in a region of microhomology.
NHEJ may result in loss or gain in genetic information and is mutation prone. Some have
suggested that HR and NHEJ have overlapping complementary roles. NHEJ is a dominant
mechanism for repairing double-stranded DNA breaks in mammalian cells. Several
investigators have demonstrated enhanced radiosensitivity of human cell lines by
inhibiting the proteins required for NHEJ.

Question 8.2 The answer is D.


All other statements are false. The indirect effects of x-rays predominate the biologic
effects seen with x-rays. The photons cause ejection of fast electrons. The ejected fast
electrons can directly damage the DNA (direct action) or, more commonly, the fast
electrons interact with plentiful water molecules to produce hydroxyl radicals (OH–),
which in turn can damage the biologic target (indirect action). Radiation dose is the
energy deposited per mass of tissue. Although injury to cellular membranes contributes to
radiation response, the primary effect is due to DNA damage. The RBE describes the ratio
of doses required to yield an equivalent biologic event.

Question 8.3 The answer is C.


Although single-stranded and double-stranded breaks are observed from ionizing
radiation, it is the double-stranded breaks that are thought to represent the lethal event.
Although the majority of the radiation-induced, double-stranded breaks are rejoined in
cells within 2 hours after exposure, the process can continue for 24 hours. In mammalian
cells, choice of repair is biased by phase of cell cycle and by abundance of repetitive DNA.
HRR is used primarily in late S to G2 phase of cell cycle, and NHEJ predominates in the
G1 phase of cell cycle. However, this is not an absolute, and factors in addition to cell
cycle phase are important in determining which mechanism will be used to repair DNA
strand breaks. NHEJ while effective is known to be highly error prone.

Question 8.4 The answer is A.


D0 is related to the slope of the exponential survival curve and represents a dose that is
required to reduce the surviving fraction to 0.37% or 37% of the original population. Dq
is the quasi-threshold dose. Mammalian cell lines display a “shoulder” in the low-dose
region and the exponential relation at higher doses. The “shoulder” represents a reduced
efficiency of cell killing. The linear quadratic model rather than the linear model best
describes the cellular response to radiation for low-ionizing radiation, while the linear
model best describes response to densely ionizing radiation. The shoulder disappears for
densely ionizing radiation. The postirradiation conditions that suppress the cell division
are the ones most favorable to potentially lethal damage repair.

Question 8.5 The answer is A.


Repair explains the shoulder of the radiation survival curve showing that cells can repair
some of the radiation damage. The majority of sublethal damage repair occurs within 6
hours after irradiation. Redistribution explains differences in cell-cycle radiation
sensitivity, with the mitotic (M) phase being the most sensitive. Cells gradually increase
in resistance as they proceed through the late G1 and S phases. Repopulation refers to
spontaneous repopulation and induced cell proliferation or recruitment of cells after
irradiation. This may occur in some tumors but occurs less than that in normal tissues.
Reoxygenation explains how a proportion of hypoxic cells present in a tumor return to
preradiation level. Some tumor cells may reoxygenate after radiotherapy and the
proportion of the hypoxic cells that was present before irradiation may be seen. Tumor
oxygen levels increaser during a course of fractionated radiation.

Question 8.6 The answer is E.


Acute effects of radiation usually occur in organs dependent on rapid self-renewal and are
mostly caused by radiation-induced cell death during mitosis. In the case of nausea and
fatigue however, these toxicities are most likely related to the release of cytokines.

Question 8.7 The answer is A.


Abscopal effects are not commonly seen due to an inherent characteristic of cancer cells
for immune system evasion. The abscopal effect is more commonly seen with higher doses
and fewer fractions of radiation than conventional fractionation. The activation of tumor-
specific T cells is the primary mechanism immunogenic tumor cell death. Double-stranded
breaks are a direct action on irradiated cells.

Question 8.8 The answer is B.


A modern LINAC can deliver energies up to 25 MeV. The focal point of the gantry’s
rotation is called the isocenter. The 60Cobalt teletherapy unit is a megavoltage machine
that relies on radioactive cobalt, not electrons, to produce a photon beam. A modern
LINAC uses accelerated electrons to bombard a high-Z target to generate Bremsstrahlung
x-rays.

Question 8.9 The answer is E.


Greater doses of radiation are required for cell killing in hypoxic conditions. The oxygen
enhancement is seen more readily during the exponential portion of the survival curve
because of the reduced capacity for cells to repair sublethal damage under hypoxic
conditions. Tumor cells growing in a physiologic hypoxic condition have reduced capacity
to repair sublethal damage. The OER is the ratio of dose required for equivalent cell
killing in the absence of oxygen compared with the dose required in the presence of
oxygen. The range of OER varies from 2.5 to 3.5. Oxygen must be present at the time of
irradiation for oxygen enhancement of radiotherapy to occur. OER is more important for
radiation that damages cells via hydroxyl radical intermediates. Hyperbaric oxygen often
does not show a dramatic increase in curability with standard fractionated radiotherapy.
However, hyperbaric oxygen appears to increase curability in a small number of fractions.
A meta-analysis showed that hyperbaric oxygen improves the local control of solid tumors
by approximately 10%. Unfortunately, a randomized trial published recently showed that
epoetin b did not improve cancer control or survival in patients with head and neck
cancer.

Question 8.10 The answer is B.


Hyperfractionation refers to a radiotherapy schedule that uses multiple daily treatments
more than 6 hours apart with reduced fraction size and increased number of fractions.
Accelerated fractionation reduces the overall treatment time. It delivers the same dose
with the same fraction size but in a shorter overall treatment time in an attempt to
complete radiation before accelerated repopulation occurs. Standard fractionation for
radiation therapy is the delivery of one treatment of 1.8 to 2.25 Gy/day.

Question 8.11 The answer is D.


The two proposed reasons for combining chemotherapy with radiation therapy include
radiosensitization and spatial additivity. The former concept theoretically may yield
synergistic effect which is greater than simply adding the two together. Gemcitabine is a
potent radiosensitizer due to the induction of dATP depletion and redistribution of the
cells into the early S phase. Although full dose gemcitabine can be safely combined with
radiation therapy for the treatment of locally advanced pancreatic carcinoma in cases
without involved lymph nodes, including the regional lymph nodes in the radiation field
can be extremely toxic. Cisplatin, 5-fluorouracil, and paclitaxel are also radiosensitizers.
Paclitaxel radiosensitizing is thought to be attributable to the redistribution of cells into
G2/M. Mechanism of cisplatin radiosensitization is related to its ability to cause inter- and
intrastrand DNA cross links. Two possible explanations are that cisplatin inhibits repair of
radiation induced DNA double-strand breaks or increases the number of lethal radiation-
induced double-strand breaks.

Question 8.12 The answer is C.


IMRT is a highly complex, computer intensive treatment planning process that typically
utilizes multiple beam orientations to provide many degrees of freedom and to create
highly sculpted dose distributions. Each treatment beam is broken down into beamlets to
facilitate optimization. When developing a treatment plan, the target coverage in addition
to sparing of excess dose to OARs is usually taken into account.

Question 8.13 The answer is C.


Early or acute effects typically occur within weeks after irradiation. They often occur in
tissues that have rapid turnover, and it is thought to result from the depletion of the
clonogenic or stem cells within that tissue. The frequencies of late effects depend strongly
on radiation fraction size. There are fewer late effects with smaller fraction size. Large α/
β ratio has a large “shoulder” in the low-dose portion. The α/β ratio represents the dose
at which the quadratic (b) and linear (a) components of cell kill are equivalent. The dose
response to radiation can be described by the formula S = exp (–αD – βD2), where S is
surviving fraction and D represents dose. Typical human tumors and early-responding
normal tissues have large α/β ratios (9 to 13 Gy).

Question 8.14 The answer is A.


The relative probabilities of photoelectric, Compton, and pair production interaction
depend on the photon energy and the atomic number of the irradiated material. In
modern treatment machines with greater than 4 MeV photons, the Compton interactions
and pair productions are commonly seen. At diagnostic equipment energy range (25 kVp),
photoelectric effect predominates. In photoelectric effect, an incoming x-ray transfers all
its energy to an inner orbit electron, which is ejected from the atom. A photon is
produced as an outer shell electron fills the vacant hole. In Compton scattering, energy
from the x-ray is both absorbed and scattered. The photon emerges with reduced energy
and a change in direction. In pair production, an electron and a positron are produced,
which then deposit energy through collisions with other electrons. The threshold energy
for pair production is 1.02 MeV.

Question 8.15 The answer is D.


Higher-energy photons deposit less dose to the skin surface. Thus, it is called the skin-
sparing effect of high-energy photons. Depth of maximum dose increases as the energy of
the incident beam increases. It is often desirable to use high-energy photons (>10 MeV)
to reach deeply located tumors. For a given energy, electrons do not penetrate deeper in
tissue compared with photons. It is for this reason that electron beams are often used to
treat superficially located tumors such as skin cancer. Electron beams, unlike photons,
deposit more dose to the skin surface as the incident electron energy increases.

Question 8.16 The answer is B.


Isotopes with properties of very short half-lives and low energy are used for permanent
implants, such as for prostate cancer treatment with iodine-125 with a half-life of 59.4
days and an x-ray energy of 27 to 35 keV. Brachytherapy is a form of treatment that uses
direct placement of radioactive sources or materials within tumors (interstitial
brachytherapy) or within body or surgical cavities (intracavitary brachytherapy), either
permanently or temporarily. Relative to external beam therapy energies, the emitted
spectra are of low energy, but high doses can be delivered within few centimeters of the
source. The fluence drops in intensity as 1 over the square of the distance from the source.
The ability to irradiate tumors from close range can lead to conformal treatments with
potentially lower normal tissue doses.

Question 8.17 The answer is A.


Radiation related adverse effects can be divided into acute, subacute, and chronic (late)
effects. Acute effects tend to occur in organs (typically within the field of radiation) that
depend on rapid self renewal. Acute effects commonly occur and typically are self
limiting. Examples include mucositis, esophagitis, diarrhea, and skin reaction. Subacute
toxicities typically occur 2 weeks to 3 months after radiation has been completed.
Radiation-induced pneumonitis is usually a subacute toxicity. Late effects from radiation
therapy are usually observed after 6 months from completion of therapy. Some examples
include radiation myelitis, brain necrosis, and bowel obstruction.

Question 8.18 The answer is A.


Immobilization is critically important because it adds to the accuracy of daily setup and
treatment. A planning computed tomography (CT) scan is obtained after immobilization
has been completed. At the time of image acquisition for planning, tumor motion caused
by respiration must be determined. The planning volume must account for respiratory
movement and uncertainty of tumor position. Three-dimensional dose distribution in each
patient is not easily measured and it must be predicted from computer calculation.
Intensity-modulated radiation therapy can have a high degree of control on the shaping of
the dose distribution. The computer determines the intensity profiles to achieve the
desired dose distribution.

Question 8.19 The answer is C.


Charged particles include protons and carbons. These particles differ from photons in that
they interact only modestly with tissue until they reach the end of their path where they
then deposit majority of their energy and stops (Bragg peak). This ability to stop at given
depth gives them the potential advantage of treating tumors that are close to critical
structures. While proton therapy may have potential use in clinical settings demanding
such characteristics, it’s widespread use for all cancer is not yet warranted. Furthermore,
in the era of IMRT (Intensity-Modulated Radiation Therapy) and IGRT (Image-Guided
Radiation Therapy), despite theoretical and dosimetric advantages conferred by charged
particles, whether this will allow higher dose to be delivered more safely, and yield a
clinical advantage over highly conformal photon techniques is not yet fully determined.
The high cost of development and operation of charged particle therapy facilities has also
limited its widespread development and use.

Corresponding chapter in Cancer: Principles & Practice of Oncology, Tenth Edition: 13 (Essentials of Radiation Therapy).
9 Systemic Therapy for Cancer
Sara K. Butler and Leigh M. Boehmer

QUESTIONS
Each of the numbered items below is followed by lettered answers. Select the ONE lettered answer
that is BEST in each case, unless instructed otherwise.

Question 9.1 Interindividual pharmacokinetic/pharmacodynamic variability can be a result


of which of the following principles?
A. Dose proportionality
B. Feedback-controlled dosing
C. Therapeutic drug monitoring
D. Sex dependence

Question 9.2 A 35-year-old female is currently being treated with pazopanib (CYP3A4
substrate). She is admitted for seizure management and the consult team recommends
initiation of phenytoin. What effect may concurrent administration have on pazopanib
concentrations?
A. Decrease pazopanib concentration
B. Increase pazopanib concentration
C. No change in pazopanib concentration
D. None, if the pazopanib is taken with food

Question 9.3 Which of the following scenarios best identifies a prognostic marker?
A. KRAS mutation testing in setting of colon cancer
B. Dihydropyrimidine dehydrogenase (DPD) deficiency in the setting of 5-FU therapy
C. MMR protein expression deficiency in setting of colon cancer
D. UGT1A1*28 polymorphism in setting of irinotecan therapy

Question 9.4 HG is a 55-year-old patient with BRAF mutation positive, metastatic melanoma
on vemurafenib therapy. Despite an initial dramatic response, his disease relapsed after
roughly 6 month of treatment. Which of the following explanations describes the most likely
mechanism of resistance?
A. KRAS mutation
B. MEK mutation
C. T790M mutation
D. T315I mutation
Question 9.5 Which of the following medication:pharmacogenomic assay pairs are correctly
matched (select two correct responses)?
A. 6-mercaptopurine:thiopurine methyltransferase (TPMT) activity
B. Tamoxifen:uridine diphosphate glucuronosyltransferase (UGT) 1A1 activity
C. 5-fluorouracil:dihydropyrimidine dehydrogenase (DPD) activity
D. Afatinib:anaplastic lymphoma kinase (ALK) gene rearrangement

Question 9.6 Which of the following is CORRECT regarding hemorrhagic cystitis?


A. Bladder protection with mesna is required for all patients receiving cyclophosphamide
B. Hemorrhagic cystitis is caused by indirect damage to the bladder wall by either
cyclophosphamide or ifosfamide
C. Mesna works by providing a free sulfhydryl group that binds to toxic metabolites
D. Mesna should be given in divided doses to total 100% of the total alkylating agent dose

Question 9.7 RT is a 50-year-old female with progressive disease following two cycles of
carboplatin and paclitaxel for advanced ovarian cancer. Based on our current understanding,
which of the following mechanisms may best explain platinum-resistant tumor cells (select
two correct responses)?
A. Decreased damage tolerance
B. Reduced cellular accumulation
C. Inactivation of autophagy
D. Intracellular detoxification

Question 9.8 PL is a-65 year-old patient with newly diagnosed metastatic non–small cell
lung adenocarcinoma scheduled to begin treatment with cisplatin and pemetrexed in 1 week.
Which of the following medications should be started today to help minimize hematologic
toxicities with pemetrexed?
A. Vitamin B6 and Vitamin B12
B. Folic acid and dexamethasone
C. Folinic acid and Vitamin B6
D. Vitamin B12 and folic acid

Question 9.9 Which of the following statements is CORRECT regarding anthracycline


cardiotoxicity?
A. Acute doxorubicin cardiotoxicity is irreversible, and its incidence can be reduced by
increasing infusion rates.
B. Chronic and delayed anthracycline cardiotoxicity is more common and severe because it
is irreversible.
C. The incidence of cardiomyopathy secondary to anthracyclines is only related to the
cumulative dose.
D. Available evidence suggests the superiority of β-blocker use to prevent anthracycline
cardiotoxicity.
Question 9.10 NB is a 68-year-old patient with hormone-refractory metastatic prostate
cancer with progressive disease, including brain metastases, after docetaxel treatment. In
discussing the possible role for cabazitaxel in this situation with the patient, which of the
following statements are CORRECT (select two correct responses)?
A. Cabazitaxel penetrates the blood–brain barrier
B. Cabazitaxel is an excellent substrate for the multidrug resistance P-glycoprotein pump
C. Cabazitaxel binds to and stabilized the α-tubulin subunit causing cell-cycle arrest
D. Cabazitaxel is active against prostate cancers that are sensitive and resistant to
docetaxel

Question 9.11 Which of the following kinase inhibitors were originally developed as a direct
inhibitor of a mutated/amplified tyrosine kinase (select two correct responses)?
A. Imatinib for the treatment of GIST
B. Erlotinib for the treatment of non–small cell lung cancer
C. Lapatinib for the treatment of HER2-positive breast cancer
D. Vandetanib in the treatment of medullary thyroid cancer

Question 9.12 Which of the following is a CORRECT statement regarding the role of VEGF
inhibition in the treatment of kidney cancer?
A. VEGF inhibitors like pazopanib or sunitinib are effective in the treatment of kidney
cancer due to known kinase mutations in VEGF.
B. The mechanism in which mTOR inhibitors are effective in the treatment of kidney
cancer is solely based on the antiproliferative impact against endothelial cells resulting
in anti-angiogensis.
C. The increase activation of the hypoxia inducible factor (HIF) pathway results in
activation of downstream target genes such as VEGF.
D. Inhibition of VEGF via bevacizumab has resulted in greater disease response than
inhibition of VEGF via multikinase inhibitors such as sunitinib or sorafenib.

Question 9.13 LB is a 63-year-old patient with newly diagnosed metastatic melanoma. His
tumor was sent for mutational analysis and was found to be BRAF V600E mutation positive.
Which of the following is a CORRECT statement regarding treatment of his tumor?
A. Due to sorafenib’s inhibition of RAF, it can be used as initial therapy.
B. BRAF inhibition is the most important part of his therapy and treatment with a MEK
inhibitor should not be considered.
C. BRAF inhibitors are only approved in the second-line setting and LB should be initiated
on either cytotoxic chemotherapy or immunotherapy with ipilimumab.
D. High serum levels of BRAF inhibitors are important to allow for achieving adequate
pathway inhibition.

Question 9.14 Which of the following is a principle of epigenetic therapy?


A. Epigenetic changes cause alterations in the sequencing of targeted genes.
B. Epigenetic therapy is mostly utilized and has the greatest efficacy in solid tumors.
C. The impact of epigenetic therapy is not seen immediately due to efficacy being based on
cellular reprogramming.
D. Combination therapy with HDAC inhibitors and demethylating agents should not be
pursued due overlapping toxicities.

Question 9.15 PM is an 82-year-old male with newly diagnosed multiple myeloma who
presents for discussion of management of his disease. Past medical history is significant for
diabetes, peripheral neuropathy, hyperthyroidism, and hypertension. Which of the following
would be the most appropriate management of his cancer?
A. Bortezomib 1.3 mg/m2 subcutaneously on days 1, 4, 8, and 11 and dexamethasone.
B. Carfilzomib 20 mg/m2 IV on days 1, 2, 8, 9, 15, and 16 and dexamethasone.
C. Bortezomib 1.3 mg/m2 IV on days 1, 4, 8 and 11 and dexamethasone.
D. Carfilzomib 27 mg/m2 IV on days 1, 2, 8, 9, 15, and 16 and dexamethasone.

Question 9.16 Poly (ADP-ribose) polymerase (PARP) signals the presence of DNA damage
and facilitates DNA repair. Which of the following is a role for PARP inhibition? (Select two
correct responses).
A. Chemopotentiation
B. Serine/threonine kinase inhibition
C. Synthetic lethality with BRCA 1/2 mutations
D. Increased hypoxia-inducible factor-1 function

Question 9.17 Asparaginase is commonly used in combination with methotrexate as part of


acute lymphoblastic leukemia treatment protocols. Which of the following statements
regarding administration of these drugs is TRUE?
A. Asparaginase should be given concurrently with methotrexate in order to inhibit
methotrexate’s clearance and increase its cytotoxicity.
B. Asparaginase should be given immediately before methotrexate in order to decrease the
risk of methotrexate neurotoxicity.
C. Asparaginase should be given 12 hours before methotrexate in order to prime cancer
cells for methotrexate’s antimetabolite activity.
D. Asparaginase and methotrexate should be given sequentially at least 24 hours apart
secondary to methotrexate antagonism.

Question 9.18 SB is a patient with metastatic prostate cancer who presents to the clinic with
hypokalemia and peripheral edema. He recently started on abiraterone 1,000 mg po daily
after disease progression on docetaxel. Which of the following would be the most appropriate
treatment options?
A. Switch the patient to enzalutamide therapy.
B. Decrease the dose of abiraterone to 750 mg po daily.
C. Initiate prednisone 5 mg po twice daily.
D. Initiate spironolactone 50 mg po daily.

Question 9.19 Which of the following statements regarding antiangiogenesis agents is


CORRECT?
A. Lenalidomide was the first angiogenesis inhibitor approved by the FDA for cancer
treatment.
B. Vorinostat, celecoxib, and bortezomib may all be referred to as exclusive angiogenesis
inhibitors.
C. Proangiogenic factors include endostatin, angiostatin, and thrombospondin.
D. Resistance to vascular endothelial growth factor inhibitors has been seen secondary to
increased expression of proangiogenic factors.

Question 9.20 TB is a 72-year-old male with progressive CLL who is about to initiate
therapy with ofatumumab. Which of the following are considered advantages to ofatumumab
over rituximab therapy?
A. Ofatumumab induces more antibody-dependent cell-mediated toxicity (ADCC) than
rituximab.
B. Ofatumumab causes more complement activation than rituximab.
C. Ofatumumab is a humanized monoclonal antibody whereas rituximab is chimeric.
D. Ofatumumab demonstrated about an 80% response rate in treatment refractory disease.
ANSWERS

Question 9.1 The answer is D.


A number of analyses have suggested that many anticancer drugs exhibit significant
pharmacokinetic sexual dimorphism. Dose proportionality is when drug concentrations
change in strict proportionality to the dose of drug administered. In this scenario,
pharmacokinetic parameters are constant. Therapeutic drug monitoring and feedback-
controlled dosing are both dose-adaptation principles developed to overcome
interindividual pharmacokinetic variability.

Question 9.2 The answer is A.


Anticonvulsant drugs such as phenytoin and carbamazepine can induce drug-metabolizing
enzymes (e.g., CYP450 isoenzymes) and thereby increase anticancer agent clearance.
Suppression of CYP activity, with agents such as ketoconazole, may lead to a rise in
plasma concentrations. It is not advisable to attempt to manipulate the extent of drug
absorption, by way of ingesting food, to overcome drug–drug interactions.

Question 9.3 The answer is C.


A prognostic biomarker identifies subpopulations of patients with different disease
outcomes, independent of treatment. In stage II colon cancer patients, MMR protein
expression deficiency leads to a more favorable natural disease course. The remaining
options represent predictive biomarkers, which identify subpopulations of patients most
(or least) likely to have a response to a given therapy.

Question 9.4 The answer is B.


KRAS is a downstream effector of the EGFR pathway, and colorectal cancer patients with
a KRAS mutation do not receive benefit from cetuximab or panitumumab therapy.
Activating BRAF mutations result in uncontrolled activity of the MAPK pathway via MEK
activation. Secondary MEK mutations have been implicated in acquired resistance to
vemurafenib. T790M and T315I point mutations, respectively, are associated with
acquired resistance to EGFR and BCR-ABL tyrosine kinase inhibitors.

Question 9.5 The answers are A and C.


TPMT deficiency results in higher intracellular activation of 6MP to form active
thioguanine nucleotides, resulting in severe, possibly fatal hematologic toxicity from
standard doses of therapy. DPD deficiency may result in higher 5-FU blood levels leading
to severe, possibly fatal toxicities. UGT1A1 is an essential enzyme involved in irinotecan
metabolism. Polymorphisms, notably the *28 allele, have been associated with reduced
expression of UGT1A1 and greater rates of irinotecan toxicity. ALK-positive NSCLC
patients are highly sensitive to therapy with ALK-targeted inhibitors, including crizotinib
and ceritinib.

Question 9.6 The answer is C.


Bladder protection with mesna is required for all patients receiving ifosfamide, and only
those select patients receiving high-dose cyclophosphamide. Mesna prevents hemorrhagic
cystitis by providing a free sulfhydryl group that conjugates with toxic metabolites
(particularly acrolein) of ifosfamide and cyclophosphamide. The dose of mesna should be
60% of the alkylating agent dose, given in divided doses every 4 hours.

Question 9.7 The answers are B and D.


Inhibition of transport mechanisms has a marked effect on intracellular platinum
accumulation, which may also be affected by enhanced efflux. Inactivation of the
platinum compounds may occur through cellular thiols and heavy metal binding via
metallothionein. Increased DNA damage tolerance may permit persistence of cancer cells
and autophagy reversal has been shown to sensitize tumors to the effects of cytotoxic
drugs.

Question 9.8 The answer is D.


Supplementation with Vitamin B12 and folic acid is required to minimize hematologic
toxicities with both pemetrexed and pralatrexate. Other side effects reduced by
supplementation include: mucositis, skin rash, reversible transaminasemia, anorexia and
fatigue, and GI toxicity. Vitamin B12 and folic acid supplementation should start at least 1
week prior to chemotherapy and be continued for at least 21 days after therapy.

Question 9.9 The answer is B.


Acute doxorubicin cardiotoxicity is reversible, and its incidence can be significantly
reduced by decreasing doxorubicin infusion rates. The incidence of anthracycline
cardiotoxicity is dependent on both cumulative dose and the schedule of drug
administration. Recent meta-analytic data has shown relatively similar efficacy in
preventing anthracycline cardiotoxicity regardless of which prophylactic treatment was
used (e.g., β-blockers, dexrazoxane, statins, angiotensin antagonists).

Question 9.10 The answers are A and D.


Cabazitaxel is a very poor substrate for the multidrug resistance P-glycoprotein efflux
pump, which makes it useful for treatment of docetaxel-resistant prostate cancer. It is a
semisynthetic taxoid derivative which binds to and stabilized the β-tubulin subunit causing
inhibition of microtubule depolymerization and cell cycle arrest in the G2/M phase.

Question 9.11 The answers are A and C.


Imatinib and lapatinib were both developed based on their direct inhibition of
KIT/PDGFR mutations and HER2 kinase gene amplification, respectively. Erlotinib was
developed in more of cytotoxic approach in NSCLC where an all comer approach was
taken. It was not until after FDA approval where the EGFR mutation status was taken into
consideration when prescribing erlotinib to NSCLC patients. It is known that many
medullary thyroid cancers have activating mutations targeting RET, but there are no
direct RET inhibitors available. Vandetanib does have off-target activity against RET, but
it is unknown if this is driving the response rate or if the VEGF receptor inhibitor is the
main driver.

Question 9.12 The answer is C.


There are no known driver mutations in VEGF that have been shown in kidney cancer.
The loss of function in the Von Hippel–Lindau (VHL) tumor suppressor gene results in
activation of oncogenes within the hypoxia-inducible factor (HIF) pathway. HIF activation
results in downstream activation of VEGF which is why it is thought that anti-VEGF drugs
have activity in kidney cancer. The mechanism of mTOR inhibitors in kidney cancer is
poorly understood. The antiproliferative activity against endothelial cells suggests an
antiangiogenic mechanism, but mTOR inhibitors have also shown to inhibit the growth of
kidney cell lines without tumor angiogenesis impact.

Question 9.13 The answer is D.


Since sorafenib is a multikinase inhibitor the drug concentration needed for complete
inhibition of RAF resulted in intolerable adverse events. This was an initial challenge as
well with an older formulation of vemurafenib. Approved BRAF inhibitors have high
enough serum concentrations to inhibit the BRAF pathway. Since MEK is part of the
RAS/RAF pathway, inhibition of MEK can be utilized as monotherapy or in combination
with a BRAF inhibitor. Patients with BRAF V600 mutations should be treated with
targeted therapy initially and chemotherapy or immunotherapy should be reserved for
progression of disease.

Question 9.14 The answer is C.


Epigenetic therapy is based on restoring normal function of the affected genes and
proteins and not targeting specific genetic alterations. Thus far, the majority of research
and clinical efficacy has been in the treatment of hematologic malignancies. Since many of
the epigenetic therapies do not have overlapping toxicity profiles, combination therapies
are being explored. An immediate response is typically not seen with epigenetic therapy
since it takes time for cellular restoration to occur.

Question 9.15 The answer is A.


Bortezomib is approved for first-line therapy for multiple myeloma either as a single
agent or in combination. Carfilzomib is only indicated for relapsed or refractory disease;
thus, would not be appropriate initial therapy. The patient already has a history of
peripheral neuropathy. Carfilzomib is associated with a lower incidence of neuropathy
than bortezomib, but this toxicity can be minimized when bortezomib is given
subcutaneously instead of intravenously.

Question 9.16 The answers are A and C.


Preclinical studies have shown that combining PARP inhibitors with anticancer treatment
modalities that result in DNA instability (e.g., radiation therapy and platinum
chemotherapy agents) potentiates their cytotoxicity. In the presence of BRCA mutations,
PARP inhibitors cause impaired homologous recombination, loss of repair mechanisms,
and cell death via synthetic lethality. PARP inhibition has also been implicated in
decreased production of hypoxia-inducible factor-1 function that may contribute to cancer
cell death.

Question 9.17 The answer is D.


Asparaginase has been shown to antagonize the cytotoxic antimetabolite activity of
methotrexate when administered either concurrently with or immediately before the
drug. The proposed mechanism is via decreased methotrexate polyglutamation leading to
lower levels of unbound intracellular drug. It is recommended, therefore, that the two
drugs be given sequentially at least 24 hours apart.

Question 9.18 The answer is C.


Abiraterone is a potent and irreversible inhibitor of CYP17. This blockage can result in
raising adrenocorticotrophic hormone (ACTH) levels including corticosterone and
deoxycorticosterone. Administration of corticosteroids such as prednisone should always
be given with abiraterone to reduce the toxicity of mineralocorticoid excess.

Question 9.19 The answer is D.


Bevacizumab was the first angiogenesis inhibitor to be approved by the FDA for cancer
treatment following a phase III trial showing a survival benefit. Drugs that only exhibit
antiangiogenic properties are known as exclusive inhibitors. Drugs such as vorinostat,
celecoxib, and bortezomib all inhibit angiogenesis as a secondary function and thus are
known as inclusive inhibitors. Proangiogenic factors include, but are not limited to,
vascular endothelial growth factor, platelet-derived growth factor, placental growth
factor, and transforming growth factor-β.

Question 9.20 The answer is B.


Ofatumumab is a fully human monoclonal antibody that binds to a different epitope on
CD20 than rituximab leading to increases in complement activation, but less antibody-
dependent cell-mediated toxicity (ADCC) than rituximab. Treatment for refractory CLL
resulted in about a 50% overall response with the majority of patients having bulky
disease.

Corresponding chapters in Cancer: Principles & Practice of Oncology, Tenth Edition: 15 (Pharmacokinetics and
Pharmacodynamics), 16 (Pharmacogenomics), 17 (Alkylating Agents), 18 (Platinum Analogs), 19 (Antimetabolites), 20
(Topoisomerase-Interacting Agents), 21 (Antimicrotubule Agents), 22 (Kinase Inhibitors as Anticancer Drugs), 23 (Histone
Deacetylase Inhibitors and Demethylating Agents), 24 (Proteasome Inhibitors), 25 (Poly[ADP-Ribose] Polymerase Inhibitors),
26 (Miscellaneous Chemotherapeutic Agents), 27 (Hormonal Agents) and 28(Antiangiogenesis Agents), 29 (Monoclonal
Antibodies).
10 Principles of Immunotherapy
Sarah B. Goldberg

QUESTIONS
Each of the numbered items below is followed by lettered answers. Select the ONE lettered answer
that is BEST in each case unless instructed otherwise.

Question 10.1 Which of the following types of immunity is most responsible for the host
response to tumor development?
A. Humoral immunity
B. B-cell–mediated immunity
C. T-cell–mediated immunity
D. Antibody-mediated immunity

Question 10.2 Currently, what is the proposed mechanism for using immunotherapy to
destroy cancer cells?
A. Develop antibodies to target growth factors on cancer cells
B. Increase levels of immune lymphocytes
C. Develop antibodies to directly destroy the cancer cells
D. Increase levels of antigen-presenting cells

Question 10.3 A 55-year-old woman with metastatic melanoma enters a clinical trial with a
cancer vaccine. Approximately 4 weeks after starting treatment she develops abdominal pain
and watery diarrhea. Which of the following is the most likely diagnosis and the etiology?
A. Autoimmune colitis from activation of T cells against antigen found in cells lining the
gastrointestinal tract
B. Autoimmune colitis from antibody production targeting antigen found in cells lining the
gastrointestinal tract
C. Infectious colitis from immunosuppression due to aberrant T-cell production
D. Infectious colitis from contamination of the vaccine product with a virus capable of
infecting the gastrointestinal tract

Question 10.4 Expression of the Epstein–Barr virus (EBV) latent gene EBNA is seen in which
of the following cancers?
A. Nasopharyngeal carcinoma
B. Burkitt lymphoma
C. T-cell lymphoma
D. All of the above

Question 10.5 A 20-year-old sexually active woman inquires about the role of the human
papilloma virus (HPV) vaccine in relation to cervical cancer. Which of the following
statements regarding the HPV vaccine is CORRECT?
A. It prevents the development of cervical cancer in all patients
B. It induces regression of cervical cancer tumors
C. It prevents infection with HPV 16 and 18
D. It prevents the development of cervical cancer in patients infected with HPV 16 and 18

Question 10.6 Which of the following is an approach to cancer immunotherapy?


A. Passive transfer of activated immune cells with antitumor activity
B. Active immunization to enhance antitumor reactions
C. Nonspecific stimulation of immune reactions
D. All of the above

Question 10.7 Which of the following is an example of an immunotherapy that works by


nonspecific stimulation of immune reactions, and in which disease has it shown to be
effective?
A. Cancer vaccine in hepatocellular carcinoma
B. Adoptive immunotherapy in lung cancer
C. IL-2 in melanoma
D. Anti-CTLA4 in pancreatic cancer

Question 10.8 A 48-year-old man presents with hematuria, and CT imaging reveals a 5-cm
kidney mass. Imaging of his chest shows multiple small bilateral lung nodules consistent with
metastatic renal cell cancer. He is asymptomatic and in otherwise good health. You discuss
treatment options with him including the use of IL-2 therapy. Which of the following
statements is TRUE regarding the use of IL-2 in this situation?
A. IL-2 is very well-tolerated with minimal side effects in the majority of patients
B. IL-2 is FDA-approved for multiple cancer types given the excellent outcomes, including
kidney cancer, melanoma, and lung cancer.
C. The chance of tumor regression with the use of IL-2 in kidney cancer is approximately
75%.
D. Although tumor response is seen in approximately 20% of patients with kidney cancer
treated with IL-2, some patients can have durable benefit without tumor progression for
several years.

Question 10.9 Which of the following results in inhibition of T cells?


A. CTLA-4
B. CD28
C. B7-1
D. B7-2
Question 10.10 Of over 1,000 vaccine treatments that have been studied, what is the
approximate overall objective response rate to vaccines for cancer treatment?
A. <5%
B. 10%
C. 30%
D. 50%

Question 10.11 The purpose of checkpoint molecules such as CTLA-4 and PD-1 are to:
A. Enhance the immune response against infectious agents by activating T cells.
B. Enhance the immune response against cancer by activating T cells.
C. Protect normal tissues against autoimmunity by inhibiting T cells.
D. Protect normal tissues from tumorigenesis by stimulating the immune system.

Question 10.12 A 67-year-old man presents with abdominal pain and weight loss and is
found to have multiple liver masses and diffuse lymphadenopathy. Biopsy of a liver lesion
confirms metastatic melanoma. He is otherwise healthy with no other significant medical
problems. Which of the following is accurate regarding the use of immunotherapeutic agents
in metastatic melanoma?
A. Nivolumab is an inhibitor of the checkpoint molecule CTLA-4, and its use can result in
significant tumor regression
B. Nivolumab often results in autoimmune toxicity while ipilimumab is rarely associated
with side effects
C. Both nivolumab and ipilimumab can result in durable tumor regression
D. The combination of nivolumab and ipilimumab does not appear to result in additional
toxicity than either agent alone

Question 10.13 Which of the following has been associated with death in trials of nivolumab
for lung cancer?
A. Neutropenia
B. Pneumonitis
C. Hepatitis
D. Hypophysitis

Question 10.14 Which is TRUE regarding cancer vaccines?


A. Cancer vaccines have a high rate of efficacy but their use is limited due to toxicity
B. Cancer vaccines have a high rate of efficacy and are currently used in multiple tumor
types to induce tumor regression.
C. Cancer vaccines have a low rate of efficacy because of their inability to generate
antitumor T cells
D. Cancer vaccines have a low rate of efficacy despite their ability to generate antitumor T
cells

Question 10.15 Which of the following is TRUE regarding sipuleucel-T?


A. Sipuleucel-T is a vaccine loaded with prostatic acid phosphatase linked to GM-CSF.
B. The response rate to sipuleucel-T is approximately 50%.
C. Sipuleucel-T significant prolongs time to disease progression and overall survival in men
with prostate cancer.
D. All of the above.

Question 10.16 What is the purpose of adoptive cell transfer therapies?


A. Decreasing the number of reactive T cells ex vivo and transfer back to the patient
B. Increasing the number of cytokines ex vivo and transfer back to the patient
C. Activating and expanding tumor-reactive T cells and transfer back to the patient
D. Modulating CD4+ tumor-specific T cells before transferring back to the patient

Question 10.17 Which of the following is CORRECT regarding adoptive cell therapy?
A. Adoptive cell therapy is only effective if the lymphocytes are genetically modified prior
to infusing them into the patient.
B. Both nonviral- and viral-based gene delivery platforms have been used to genetically
modify cells for use in adoptive cell therapy
C. Targeting antigens that are expressed only on tumor cells and not on normal tissue can
decrease the toxicity associated with on-target, off-tumor effects of the lymphocytes
D. B and C only

Question 10.18 Which of the following indicates the potential utility of anti-CD19 CAR T
cells?
A. They have been used in many types of solid tumors because of the high prevalence of
CD19 on solid tumor cells.
B. Because they are found on almost all malignant B cells, they have been tested in
patients with hematologic malignancies.
C. Toxicity associated with this therapy is typically long term due to graft-versus-host
disease.
D. B and C only
ANSWERS

Question 10.1 The answer is C.


Initially, it was thought that the action of antibodies or humoral immunity was mainly
responsible for the response to malignant cell development. However, it has been
discovered that malignant cell development is closely related to the cellular arm of the
immune response, specifically a response to T cells. B cells do not play a major role in the
immune response to tumor cells.

Question 10.2 The answer is B.


For existing malignancies, the main mechanism for immunotherapy is to try to increase
the level of immune lymphocytes. These immune lymphocytes would be responsible for
recognizing the cancer antigens and destroying the cancer cells. Although the
development of antibodies that target growth factors on cancer cells may help in reducing
the overall tumor burden, this does not work by directly destroying the cancer cells.

Question 10.3 The answer is A.


Autoimmune adverse events are the typical toxicities associated with immunotherapy
including vaccine therapy. Colitis occurs from T-cell activation against antigen that is
found in both the tumor and the intestinal lining. Infectious colitis is possible, however
vaccine therapy does not result in immunosuppression and vaccines are not made with
live virus that can infect the host.

Question 10.4 The answer is D.


Expression of the Epstein–Barr virus latent gene EBNA is seen in Burkitt lymphoma,
nasopharyngeal carcinoma, and T-cell lymphoma.

Question 10.5 The answer is C.


HPV vaccines protect women from contracting the strains of HPV that are commonly
associated with the development of cervical cancer: HPV 16 and 18. The quadrivalent
HPV vaccine (Gardasil) also offers protection against HPV 6 and 11, which cause genital
warts in 90% of cases. HPV vaccination does not prevent cervical cancer in everyone,
since only 70% of cervical cancer is related to HPV. If a patient already has cervical
cancer, it will not provide a tumor response, and if the patient has already contracted
HPV16 or 18, it will not offer any additional protection to the patient.

Question 10.6 The answer is D.


Three main approaches to cancer immunotherapy include nonspecific stimulation of
immune reactions through stimulation of effector cells and/or inhibition of regulatory
cells, active immunization using cancer vaccines, and adoptive immunotherapy, defined as
the passive transfer of activated immune cells with antitumor activity.

Question 10.7 The answer is C.


IL-2 is glycoprotein produced by activated peripheral blood lymphocytes and recombinant
IL-2 in high doses can result in tumor cell regression. The FDA-approved IL-2 for the
treatment of melanoma and renal cell carcinoma given the activity that has been shown in
clinical trials. Its activity stems from the nonspecific stimulation of the immune system.
Although anti-CTLA-4 therapies also work by nonspecific stimulation of immune reactions,
they have not been shown to be effective in pancreatic cancer. Cancer vaccines and
adoptive immunotherapy have not been shown to be effective in lung cancer or
hepatocellular carcinoma.

Question 10.8 The answer is D.


IL-2 has a fairly low response rate of 20%, however it can result in durable responses in
some patients with renal cell cancer; in fact, it is the only known curative treatment for
this patient population. It also has efficacy in melanoma, but has not proven to be
effective in other cancers. It does carry a high rate of toxicity due to an inflammatory
response mediated by cytokine release, with symptoms including fever, hypotension,
renal insufficiency, hepatic dysfunction, and pulmonary edema.

Question 10.9 The answer is A.


When CTLA-4 is engaged on the T cell, the T-cell function is impaired. When CD28 is
engaged, T cells are stimulated. B7-1 and B7-2 are ligands that are costimulatory on the T
cells.

Question 10.10 The answer is A.


The results of treatment with cancer vaccines have been overall poor. Of over a 1,000
vaccine treatments studied in cancer, the objective response rate is only 3.4%. The
objective response rate used has been criticized by many, suggesting that the response
rate may be even lower.

Question 10.11 The answer is C.


Both CTLA-4 and PD-1 are receptors on T cells that inhibit T cells in order to prevent
overstimulation of the immune system and autoimmunity. Inhibiting their function with
therapeutic antibodies (i.e., ipilimumab and nivolumab) can activate the immune system
against cancer but can also lead to autoimmune side effects.

Question 10.12 The answer is C.


The CTLA-4 inhibitor ipilimumab and the PD-1 inhibitor nivolumab both can result in
significant and durable tumor regression in patients with metastatic melanoma. However,
they can both also cause autoimmune toxicity, and the combination of the two drugs
appears to have a higher rate of toxicity, with one trial showing a 53% rate of grade 3 or
4 adverse events.

Question 10.13 The answer is B.


Because of the activation and proliferation of T-cells achieved by inhibition of PD-1,
nivolumab is associated with many immune-mediated toxicities. In the early studies with
nivolumab in lung cancer, there were several patients who died from pneumonitis.
Hepatitis and hypophysitis (as well as many other immune-mediate adverse reactions) can
also be seen with PD-1 inhibitors, however these have not been shown to be fatal.
Cytopenias (including neutropenia) are rare with immunotherapeutics.

Question 10.14 The answer is D.


Cancer vaccines have shown the ability to generate antitumor T cells, however they have
not proven the ability to reduce tumor size or improve cancer survival, except in a few
rare instances. Therefore they are not used in most tumor types, although they continue
to be studied particularly in combination with other immunotherapeutics.

Question 10.15 The answer is A.


Sipuleucel-T is a cancer vaccine that is FDA-approved for use in metastatic prostate
cancer. It consists of a prostatic acid phosphatase linked to GM-CSF. Interestingly, it has
been shown in a randomized trial to improve survival in men with prostate cancer,
however the response rate is extremely low and it does not improve time to disease
progression compared to placebo.

Question 10.16 The answer is C.


Adoptive cell transfer therapies function by stimulating T cells ex vivo by activating and
expanding self/tumor-reactive T cells before transfer back to the patient. CD8+ T cells
have been subject to much research because they are potent effectors of the adaptive
tumor response. CD4+ T cells are more controversial, because they can either help or
hinder antitumor immune responses.

Question 10.17 The answer is D.


Adoptive cell therapy involves the transfer of lymphocytes with anticancer activity to a
patient with cancer. It has been shown to have activity in patients with some types of
metastatic cancer (particularly melanoma) in studies using genetically modified and
nongenetically modified lymphocytes. Genetic modification of lymphocytes can be
performed using nonviral- or viral-based gene delivery platforms, although the viral-based
platforms are more common. Toxicity increases if the antigen targeted is found on both
tumor and normal tissue, making the ideal target one that is found exclusively on tumor
cells.

Question 10.18 The answer is B.


CARs are fusion proteins that contain an antigen recognition domain linked to a T-cell
activation domain. Because CD19 is found on most malignant B cells but not on normal
cells (except for B cells), it is a good target for treating many B-cell hematologic
malignancies. Therefore, immunotherapy with anti-CD19 CAR T cells has been tested in
patients with B-cell hematologic malignancies. CD19 is not typically found on solid
tumors so its utility is limited in that patient population. Toxicity seen in trials with this
therapy has been transient and has included hypotension and neurologic toxicity. Long-
term toxicity such as GVHD has not been observed.

Corresponding chapter in Cancer: Principles & Practice of Oncology, Tenth edition: 14 (Cancer Immunotherapy).
11 Assessment of Clinical Responses
Vinicius Ernani and Suresh S. Ramalingam

QUESTIONS
Each of the numbered items below is followed by lettered answers. Select the ONE lettered answer
that is BEST in each case unless instructed otherwise.

Question 11.1 Which of the following drug acts on a targeted mutation?


A. Docetaxel
B. Navelbine
C. Erlotinib
D. Pemetrexed

Question 11.2 In children with lymphoblastic leukemia, folic acid was observed to:
A. Cause disease proliferation.
B. Cause disease regression.
C. Have no effect.
D. Improve survival.

Question 11.3 Which of the following drug is an antimetabolite chemotherapeutic agent?


A. Gemcitabine
B. Temozolomide
C. Mitoxantrone
D. Bevacizumab

Question 11.4 In patients with advanced or metastatic disease, which of the following has
curative potential with primary induction chemotherapy?
A. Germ cell cancer
B. Non–small cell lung cancer
C. Gastric cancer
D. Colorectal cancer

Question 11.5 The goals of therapy in incurable, advanced, metastatic disease include:
A. Obtain a complete response
B. Palliate tumor-related symptoms
C. Deliver higher doses of chemotherapy
D. Never offer therapy
Question 11.6 Neoadjuvant chemotherapy can be utilized in which of the following cancers?
A. Burkitt lymphoma
B. Hodgkin lymphoma
C. Multiple Myeloma
D. Lung cancer

Question 11.7 Which of the following is TRUE regarding the purpose of administering
adjuvant chemotherapy?
A. To increase quality of life.
B. To reduce local and distant recurrence.
C. To use as primary therapy instead of surgery.
D. To use as primary therapy instead of radiation.

Question 11.8 Which of the following play essential role in activating apoptosis?
A. Bcl-2
B. NF-κB
C. Caspases
D. Bcl-xL

Question 11.9 The most important indicator of the effectiveness of chemotherapy is:
A. Achievement of a minor response.
B. Achievement of a partial response.
C. Achievement of a complete response.
D. Achievement of stable disease.

Question 11.10 According to RECIST criteria 1.1, a partial response is defined as:
A. At least a 30% reduction in measurable tumor mass.
B. At least a 10% reduction in measurable tumor mass.
C. Improvement in quality of life.
D. Improvement in survival.

Question 11.11 Anticancer activity of bortezomib is due to the inhibition of:


A. Proteasome.
B. Bcl-2.
C. Thymidylate synthase.
D. MDM2.

Question 11.12 In early-stage colon cancer treated with adjuvant chemotherapy, the
majority of relapses occur:
A. Within the first 5 years.
B. Within the first 3 years.
C. Within the first 2 years.
D. Within the first year.

Question 11.13 Gompertzian kinetics are characterized by:


A. Clinically undetectable tumors are at their lowest growth fraction.
B. The growth fraction is stable over time.
C. The growth fraction increases exponentially with time.
D. The growth fraction decreases exponentially with time.

Question 11.14 Which of the following is the most common genetic change associated with
development of resistance to tyrosine kinase inhibitors (TKI) in patients with epidermal
growth factor receptor (EGFR) mutation?
A. T790M mutation
B. PTEN overexpression
C. D761Y mutation
D. MET gene mutation

Question 11.15 Which of the following is TRUE regarding intention to treat (ITT) analysis?
A. Increases the risk of bias
B. Should not be used in a randomized control trial
C. Small effect on overall survival and progression-free survival
D. All patients randomized to a group will be analyzed

Question 11.16 In patients with colorectal cancer the genetic change associated with
resistance to cetuximab is:
A. BRAF mutation.
B. Wild-type KRAS.
C. EGFR-TK mutation.
D. NFκB activation.

Question 11.17 Dose-dense chemotherapy was developed after the observation that:
A. The growth of cells is significantly lower in the early part of the growth curve.
B. The log cell kill is higher in smaller volume tumors, resulting in more rapid growth
between chemotherapy cycles.
C. Growth between chemotherapy cycles is halted.
D. More frequent administration of chemotherapy led to less toxicity.

Question 11.18 In most cases, normal bone marrow and gastrointestinal (GI) precursor cells
are able to counteract the effects of cytotoxic chemotherapy because:
A. They are not exposed to the chemotherapeutic agent.
B. They have intact mechanisms for apoptosis and cell-cycle arrest.
C. They have rapid turnover.
D. They possess resistance mechanisms.
Question 11.19 Which of the following is TRUE concerning p53?
A. It is a targetable mutation for therapy
B. It is a potent inducer of apoptosis.
C. It causes S-phase arrest in the cell cycle.
D. It can decrease the activity of cytotoxic chemotherapy.

Question 11.20 The final stage of the programmed-cell death pathway is mediated by:
A. Bcl-2.
B. p53.
C. Tumor necrosis factor.
D. Caspase cascade.

Question 11.21 Which of the following is inhibited by imatinib?


A. EGFR
B. BRAF
C. KRAS
D. Bcr-abl

Question 11.22 Cetuximab use in colorectal cancer is characterized by:


A. Approval in the first-line setting.
B. Limited utility as monotherapy.
C. High toxicity.
D. Response rates of 21% to 23% when used with irinotecan.

Question 11.23 Which of the following is characteristic of a randomized discontinuation


trial design?
A. Phase 1 study design
B. Phase 2 study design
C. Phase 3 study design
D. Preferred for drugs with preexisting predictive biomarker

Question 11.24 Palbociclib is an inhibitor of :


A. CDK4 and CDK6.
B. FGFR.
C. C KIT.
D. VEGFR2.

Question 11.25 Which of the following is a nonmeasurable lesion, according to RECIST


criteria?
A. 3.5 cm × 2.0 cm pancreatic adenocarcinoma
B. 1.7 cm × 1.5 cm invasive ductal carcinoma of the breast
C. 0.8 cm × 0.7 cm right paratracheal lymph node
D. 5.5 cm × 4.0 cm rectal adenocarcinoma
Question 11.26 Computed tomography contrast perfusion and dynamic-contrast magnetic
resonance imaging may be useful in evaluating:
A. Response to alkylating agents.
B. Response to antiangiogenic agents.
C. Degree of tumor necrosis.
D. Overall survival.

Question 11.27 Which of the following is TRUE regarding the use of time to progression as
an end point in clinical trials?
A. Uses a smaller sample size when compared with using overall survival as an end point.
B. Not confounded by subsequent therapies.
C. The study can be completed in a shorter amount of time.
D. All of the above.
ANSWERS

Question 11.1 The answer is C.


Erlotinib is a reversible tyrosine kinase inhibitor, which acts on the epidermal growth
factor receptor (EGFR).

Question 11.2 The answer is A.


Folic acid has been observed to have a significant proliferative effect in children with
lymphoblastic leukemia. This led to the development of folic acid analogues as
antineoplastic agents.

Question 11.3 The answer is A.


Gemcitabine is considered an antimetabolite because of the ability to inhibit the normal
pathways involved in purine and pyrimidine synthesis. Temozolomide is an alkylating
agent. Mitoxantrone is an anthracycline. Bevacizumab is an angiogenesis inhibitor.

Question 11.4 The answer is A.


Germ cell cancers are curable with primary induction chemotherapy in a subset of patients
presenting with advanced or metastatic disease. In the metastatic setting, non–small cell
lung cancer, gastric cancer, and colorectal cancer are not considered curable with
conventional chemotherapy alone.

Question 11.5 The answer is B.


In most cases, the goals of chemotherapy in patients with advanced or metastatic
malignancies include palliation of tumor-related symptoms, improvement in quality of
life, and prolongation of time to tumor progression. Eradication of all evidence of disease,
although it may occur, is not generally achievable in this setting.

Question 11.6 The answer is D.


Neoadjuvant chemotherapy is used in a variety of locally advanced solid tumors in an
effort to confer possible future surgical resectability. Chemotherapy is the primary
treatment modality in patients with hematologic malignancies.

Question 11.7 The answer is B.


Adjuvant chemotherapy is used as an adjunct to surgery and/or radiation to reduce the
risk of local or distant recurrence from occult micrometastatic disease. Randomized phase
III data support the use of adjuvant chemotherapy in breast cancer, colorectal cancer,
non–small cell lung cancer, gastric cancer, Wilms tumor, and osteogenic sarcoma.

Question 11.8 The answer is C.


Activation of caspases leads to cell death, whereas activation of Bcl-2, Bcl-xL, and NF-κB
lead to inhibition of apoptosis.
Question 11.9 The answer is C.
The most important indicator of the effectiveness of chemotherapy is the rate of complete
response. Despite improvements in response rates and overall survival in certain
metastatic diseases, unless a complete response is achieved with chemotherapy, the
patient is not considered cured.

Question 11.10 The answer is A.


A partial response is defined as at least a 30% improvement in measurable tumor mass.
Although there are data to suggest that quality of life and survival may improve when a
partial response is achieved, it is not consistently applicable across diseases.

Question 11.11 The answer is A.


Bortezomib exerts its anticancer activity by inhibiting the 26S proteasome. The
proteasome–ubiquitin complex degrades several intracellular regulatory proteins including
IκB that is an inhibitor of NFκB. Activation of NFκB promotes cell proliferation and
inhibition of apoptosis.

Question 11.12 The answer is B.


The majority of relapses in early-stage colon cancer treated with adjuvant chemotherapy
occur within the first 3 years. Therefore, adjuvant trials may use 3-year disease-free
survival as the primary end point.

Question 11.13 The answer is D.


Gompertzian kinetics provides a model of tumor growth whereby the growth fraction of a
tumor decreases exponentially over time. Clinically undetectable tumors are at their
highest growth fraction. Therefore, chemotherapy would result in a significantly higher
fractional cell kill.

Question 11.14 The answer is A.


T790M mutation is the most common cause for development of resistance to EGFR TKIs
such as gefitinib and erlotinib in patients with activating mutations in the TK domain of
the EGFR gene. In addition, MET gene amplification, loss of PTEN expression and D761Y
mutation has been implicated in the development of resistance to EGFR TKI.

Question 11.15 The answer is D.


ITT analysis includes all patients assigned to a group at the time of randomization
regardless of what occurred subsequently. When evaluating a randomized controlled trial,
it is important that the OS as well as the PFS analyses are always by intention to treat
(ITT). It avoids the bias introduced by omitting dropouts and noncompliant patients.

Question 11.16 The answer is A.


Both KRAS and BRAF V600E mutations are associated with development of resistance to
cetuximab in the treatment of patients with colorectal cancer. EGFR-TK mutations are
uncommon in patients with colorectal cancer, and NFκB activation results in pro-apoptotic
signaling; however, it has not been shown to be associated with development of resistance
to cetuximab in patients with colorectal cancer.

Question 11.17 The answer is B.


The growth of cells is significantly faster in the early part of the growth curve enabling
chemotherapy to cause a greater log cell kill. Dose-dense chemotherapy, by administering
cytotoxic drugs more frequently, can produce greater effects on smaller-volume tumors.

Question 11.18 The answer is B.


Normal bone marrow and GI precursor cells possess intact genetic machinery to undergo
apoptosis, repair DNA damage, and halt cell-cycle progression. Therefore, cells that have
been damaged by chemotherapy undergo programmed cell death and are not allowed to
continue on through the cell cycle.

Question 11.19 The answer is B.


p53 is a tumor-suppressor gene that mediates apoptosis in cells with DNA damage. It plays
a role in G1 and G2 arrest of the cell cycle after exposure to cytotoxic agents.

Question 11.20 The answer is D.


Caspase activation occurs through the activity of the intrinsic and extrinsic pathways of
apoptosis. Bcl-2 is involved in the intrinsic pathway, and tumor necrosis factor mediates
the extrinsic pathway. p53 is a tumor suppressor gene involved in the early activation of
the programmed-cell death pathway.

Question 11.21 The answer is D.


Imatinib inhibits bcr-abl, platelet-derived growth factor, and c-kit. It is used to treat
chronic myeloid leukemia and GI stromal tumors.

Question 11.22 The answer is D.


Cetuximab is a chimeric antibody with promising activity in colorectal cancer and other
solid tumors. Currently, it is used in the second- or third-line setting, either alone or in
combination with chemotherapy. In irinotecan-resistant patients, reported overall
response rates with the cetuximab/irinotecan combination are 21% to 23%. Toxicity is
usually manageable and is mostly related to rash.

Question 11.23 The answer is B.


Randomized discontinuation design is a novel phase 2 study design and is useful in
identifying the target population for a drug. For a drug with an established predictive
biomarker, there is no benefit to using this study design since the target population is
already defined by the biomarker.
Question 11.24 The answer is A.
Palbociclib is a selective inhibitor of cyclin-dependent kinases CDK4 and CDK6. It has
been approved for the treatment of ER positive and HER2 negative advanced metastatic
cancer in combination with letrozole.

Question 11.25 The answer is C.


The RECIST criteria use evaluation of tumor size to define a response. Measurable disease
is a lesion with longest diameter ≥10 mm on CT or on skin if using calipers and nodes
with a short axis of ≥15 mm.

Question 11.26 The answer is B.


Computed tomography contrast perfusion has been studied in assessing blood flow to
rectal tumors treated with bevacizumab. The functionality of vascular endothelial growth
factor receptor blockers has also been studied using dynamic-contrast magnetic resonance
imaging. The utility of these tests in assessing response to alkylating agents, tumor
necrosis, or effect on overall survival is not known.

Question 11.27 The answer is D.


Time to tumor progression as an end point for studies involving metastatic cancer has
certain advantages over using overall survival: Sample size can be smaller, subsequent
therapies do not confound the data, and the study may be able to be completed in a
shorter amount of time.

Corresponding chapter in Cancer: Principles & Practice of Oncology, Tenth Edition: 30 (Assessment of Clinical Response).
12 Design and Analysis of Clinical Trials
Ling Chen and Kathryn Trinkaus

QUESTIONS
Each of the numbered items below is followed by lettered answers. Select the ONE lettered answer
that is BEST in each case unless instructed otherwise.

Question 12.1 Which of the following is TRUE concerning a clinical trial?


A. The sample size of a clinical trial is fixed and determined before the trial begins.
B. Clinical trials are prospectively planned experiments involving only animals.
C. Using tumor registry data to compare the survival rates of prostate cancer patients
treated with surgery to those of patients who received radiotherapy is an example of
clinical trial.
D. Clinical trials test a clearly stated hypothesis using a predefined analysis plan.

Question 12.2 Phase I trials of cytotoxic agents are designed to determine a dose that is
appropriate for use in phase II trials. Which of the following statements is TRUE about phase
I trials?
A. Phase I trials involve the first use of a drug, device, or procedure in humans in a new
disease setting or a new combination of therapies.
B. Patients for phase I trials are usually not chosen on the basis of their likelihood of
having a favorable response to the experimental treatment.
C. Phase I trials do not require statistical justification because the sample sizes are usually
small.
D. A and B.

Question 12.3 Which of the following regarding the conventional 3 + 3 phase I designs is
TRUE?
A. Participants are often those with end-stage disease who have many treatment options.
B. Such studies are usually performed by starting with a high dose.
C. Cohorts of three to six patients are treated at each dose level, and the initial dose is
chosen to cause little or no toxicity.
D. The optimal dose recommended for phase I is chosen using a dose-toxicity curve
generated from a statistical model fitted to the full data.

Question 12.4 Traditional phase I trials are designed to determine a dose that is appropriate
for use in phase II trials. Which of the following statement on traditional phase I trials is
TRUE?
A. Many patients may receive subtherapeutic doses and it does not allow for dose
escalation in the individual patient.
B. All phase I trials are completed in a short time period.
C. Dose escalation in the individual patient is allowed.
D. Phase I trial provides substantial information regarding cumulative toxicity from the
study drug

Question 12.5 A drug, device, or other treatment may be considered for phase II testing
when:
A. The maximum tolerable dose is known but there is genuine doubt about efficacy and
patients with the condition of interest and a likely favorable outcome can be recruited
as study participants.
B. A standard treatment exists for comparison.
C. Accrual is rapid enough to allow recruitment of 20 to 30 patients.
D. An appropriate control group can be recruited at the same time as the experimental
group.

Question 12.6 which of the following statements regarding phase II trials is CORRECT?
A. Phase II trials are traditionally performed in patients with a wide variety of tumor
types.
B. Two-stage designs allow for early elimination of ineffective drugs.
C. Patient eligibility should not be restricted by biomarker testing.
D. Phase II trials cannot help develop predictive biomarker assays for the study drug.

Question 12.7 Phase II trials of a single agent are designed to determine whether the
experimental drug has any antitumor activity against a given type of tumor and the
antitumor effect is often measured as an objective response based on tumor shrinkage.
However, such a trial is NOT appropriate for the question of patient welfare because:
A. The welfare of patients with cancer is often measured by survival or symptom control,
but there is no necessary relationship between such beneficial end points and tumor
response.
B. Survival is inherently a comparative end point that should be assessed in a
prognostically comparable set of patients.
C. Comparing the survival times of responders and nonresponders is not a valid means of
showing a treatment is beneficial to patients because responders may have more
favorable prognostic factors than nonresponders.
D. All of the above.

Question 12.8 Patients in a phase II trial are often recruited in a two-stage manner that
allows early stopping if the null hypothesis is unlikely to be rejected, thus saving patients
from unnecessary exposure to inactive agents. Which of the following is TRUE of a two-stage
design?
A. The optimal two-stage design minimizes the average sample size and thus optimizes the
protection of patients given an inactive agent.
B. Simon two-stage designs are often applied in phase II trials on molecularly targeted
drugs and therapeutic cancer vaccines.
C. If both stages are completed, the minimax design usually enrolls more patients
compared with the optimal design.
D. Simon two-stage designs are often used in phase II trials comparing treatment groups to
an internal control group to draw conclusions about survival.

Question 12.9 Unlike cytotoxic agents that work by killing tumor cells, cytostatic agents
work via biologic effects that modify the environment of tumor growth; that is, inhibition of
a molecular target. Which of the following is TRUE regarding early-phase trials of cytostatic
agents such as molecularly targeted drugs and cancer therapeutic vaccines?
A. Phase I trials on cytostatic agents are often designed to identify maximum tolerated
dose.
B. Heavily treated patients with end-stage disease greatly benefit from a cancer vaccine.
C. The response rate based on tumor shrinkage is the best primary end point measuring
treatment efficacy in phase II trials on cytostatic agents.
D. Because of the nature of low toxicity profiles in cytostatic agents, factorial designs and
randomized studies have appealing features and are used to evaluate multiple drugs in a
single trial.

Question 12.10 Which of the following best describes Simon phase 2.5 design on
molecularly targeted drugs and cancer therapeutic vaccines?
A. The design compares progression-free survival times of the same patient from the
current trial with his/her previous trial.
B. The design first treats all eligible patients with two to four courses of the experimental
drugs. Patients are then evaluated, and the experimental drug is either continued or
discontinued, depending on their response to the initial treatment.
C. The design is similar to a phase III trial except that it allows a relatively large type I
(false positive) error and aims to identify a relatively large difference, thus requiring
fewer patients.
D. The design will compare time to progression of patients in the study with a
prospectively identified and prognostically comparable historical control.

Question 12.11 Which of the following is TRUE of phase III trials?


A. The primary end points should be demonstrably related to patient benefit such as tumor
shrinkage.
B. Eligibility criteria should be limited to a group of patients with certain conditions.
C. Conditions of treatment should be suited to community patient care.
D. A quality-of-life measure should be included to measure benefit to patients.

Question 12.12 Which of the following is TRUE of randomized treatment assignment?


A. Trial participants are a representative sample of patients with the disease of interest.
B. Randomization may fail to balance unknown prognostic factors in small studies.
C. Randomization is unnecessary if controls can be matched closely enough to trial
participants.
D. Randomization is effective only if patients and treating physicians are unaware of
treatment assignment for the duration of the study.

Question 12.13 Randomization is generally unnecessary in which of the following scenarios?


A. Study of patients with a disease that will uniformly and rapidly progress to an end
stage.
B. Phase III trial
C. Screening study on multiple cytostatic agents, such as molecularly targeted drugs or
cancer therapeutic vaccines
D. Phase II trial on cytotoxic agents using time to progression as the primary efficacy end
point

Question 12.14 Which of the following is TRUE of power and hypothesis testing in clinical
trials?
A. Statistical power mainly depends on the anticipated size of treatment effect. Trials
designed to detect a large effect will demand small sample sizes.
B. Statistical power is also heavily influenced by the prespecified type I error (probability
of erroneously claiming a “positive” result), and a two-sided type I error of 0.05 has
been widely accepted, especially in phase III trials.
C. Confidence intervals do not provide information about the size and direction of
treatment effect, and it is less informative than significance testing.
D. Because data from small randomized trials can be pooled for meta-analysis, the
calculation of sample size for a randomized clinical trial is no longer critical.

Question 12.15 Which of the following is TRUE for therapeutic equivalence (or
noninferiority) trials?
A. Therapeutic equivalence trials are designed to test similarity, as measured by a clinical
end point, between the experimental drugs and the standard treatment.
B. Because it is impossible to prove equivalence in true sense, in practice therapeutic
equivalence will be accepted if the difference in efficacy is larger than or smaller than a
prespecified amount δ.
C. The traditional superiority trials can be used to prove equivalence
D. A trial designed to demonstrate therapeutic equivalence requires fewer patients because
it usually allows a relatively large type I error.

Question 12.16 Therapeutic equivalence trials often express results as an estimate of


difference between two treatments with a 95% confidence interval. Which of the following
statements about the 95% confidence interval is CORRECT?
A. The confidence interval is a range of values consistent with the hypothesized difference,
given the precision with which the trial can measure that difference.
B. The trial estimates that in 5 of 100 cases the true difference will fall outside the 95%
confidence interval.
C. It is unlikely that the true difference is larger than the lower limit or smaller than the
upper limit of the confidence interval.
D. If the confidence interval includes 0, then the trial has shown that the two treatments
do not differ.

Question 12.17 Which one of the following describing the pros and cons of Bayesian
methods in planning clinical trials is TRUE?
A. In the traditional statistical (frequentist) methods, the treatment effect is regarded as a
fixed but unknown parameter, and the likelihood of the parameter is described using a
probability derived from observed frequencies in a defined distribution. In contrast, a
Bayesian statistical method assumes that the treatment effect itself is a random quantity
drawn from a prior distribution.
B. Bayesian methods are more widely applied in planning phase III trials than phase I or
phase II trials.
C. Bayes’ theorem can be considered as a substitute for randomization.
D. Bayesian clinical trials require fewer patients than frequentist trials.

Question 12.18 The intention-to-treat analysis is considered the primary analysis of data
from a clinical trial. Which of the following describes an intention-to-treat analysis?
A. An ineligible patient is not included in an intention-to-treat analysis because the study
treatment is not appropriate for that patient.
B. All patients who give consent and are randomized are included in the intention-to-treat
analysis in the study arm to which they were randomized.
C. Data from patients who are ineligible or who die or withdraw from the study before
completion should be analyzed separately from data from eligible patients who
complete the trial.
D. Lack of compliance can distort the results from patients who fail to complete the trial,
so these data should be dropped from an intention-to-treat analysis.

Question 12.19 An interim analysis is any assessment of data performed during patient
enrollment or follow-up period of a trial. Which of the following is TRUE for planning
clinical trials with interim analyses?
A. Multiple looks at outcome data over the course of a trial will alter type I and II errors.
Unless they are included in the design, multiple looks will cause the operating
characteristics of the trial to deviate from the planned values.
B. Interim outcome information is generally available to participating physicians and study
leaders.
C. The method of stochastic curtailment is designed to calculate conditional power (the
projected probability of rejecting the null hypothesis at the end of study), conditional
on the accumulated information, and to stop the trial if the conditional power is small
(e.g., 0.2). In such a “futility” analysis, the number of interim analyses does not need to
be fixed in advance.
D. A data-safety committee is usually put in charge of the interpretation of interim
analyses. The committee also includes the study leaders to notify the patients and
medical community in a timely manner.

Question 12.20 One of the most important features that make survival data different from
other kinds of data is the presence of censoring. That is, the actual survival times for some
patients cannot be observed during the study. Which of the following is TRUE for the analysis
of survival data?
A. The usual statistical methods are appropriate for analyzing survival data.
B. Two methods are most frequently used for the estimation of survival curves. The life
table method requires relatively large sample size, and the Kaplan–Meier product limit
method is appropriate for any sample size.
C. Traditional summary statistics, such as means and standard deviations, can provide a
sufficient summary for survival data without censoring.
D. The statistical power of clinical trials using survival as a primary end point will be
mainly determined by the total number of patients rather than the actual number of
events including death or tumor progression.

Question 12.21 An important assumption in the analysis of survival data is noninformative


censoring. That is, censoring is not related to the outcome of interest. For a clinical trial with
cancer-specific survival as the primary end point, which of the following patients is more
likely to suffer from informative censoring?
A. Alive at the time of study closeout
B. Withdrew from the trial because of disease progression
C. Died of accident
D. Moved out of the region

Question 12.22 Which of the following is characteristic of phase 0 trials?


A. Phase 0 trials do not require an assay for measuring the pharmacodynamic endpoint.
B. Patients receive multiple doses of the experimental drug.
C. Phase 0 trials identify whether the experimental drug has inhibitory effect against its
specified molecular target.
D. Drug doses used in these trials are expected to cause significant toxicity.

Question 12.23 Which of the following is TRUE regarding meta-analysis?


A. Meta-analysis combines the results of several studies that address a set of related
research hypothesis.
B. Meta-analysis on therapeutic studies usually includes both randomized and
nonrandomized trials.
C. Meta-analysis based on summarized p-values is more precise than meta-analysis based
on individual patient data.
D. Meta-analysis is an alternative to properly designed and sized randomized clinical trials.
ANSWERS

Question 12.1 The answer is D.


Sample size may be altered during the course of a trial of any phase on the basis of
outcomes observed during the trial. Clinical trials are prospectively planned experiments
involving human subjects. Using tumor registry data to compare the survival rates of
prostate cancer patients treated with surgery to those of patients who received
radiotherapy is an example of observational study.

Question 12.2 The answer is D.


Statistical power is not the focus of phase I trial design, but the issues (e.g., patient
safety) are major, and long-term consequences (e.g., loss of patient benefit and waste of
resources) are severe if resources are incorrectly directed toward ineffective treatments or
away from effective treatments. Therefore, careful statistical design is needed if sound
and sufficient information is to be drawn from small and often heterogeneous patient
samples.

Question 12.3 The answer is C.


Participants are often those with end-stage disease whose treatment options have been
exhausted. Patients with advanced disease that is resistant to standard therapy are often
included in such trials. Such studies are usually performed by starting with a low dose that
is not expected to produce serious toxicity in any patients. The conventional 3+3 phase I
design is totally algorithm based, and the dose recommended for phase II trials is chosen
among the fixed and prespecified levels. The recommended dose, commonly referred to as
the maximum tolerated dose, is usually taken as the highest level for which the dose-
limiting toxicity rate is less than 33%.

Question 12.4 The answer is A.


Traditional phase I study design has several inherent limitations. Many patients may
receive subtherapeutic doses and it does not allow for dose escalation in the individual
patient. Furthermore, it may require longer period of time to complete than some of the
newer trial designs and may not be effective in determining cumulative toxicity.

Question 12.5 The answer is A.


Phase II trials may be single-arm trials testing a single treatment, so no direct comparison
of treatments may be involved. The ideal sample size depends on many factors, including
the nature and variability of the primary outcome, the expected effect size, and the
procedures to be used to test hypotheses. An appropriate primary outcome is determined
by the nature of the treatment being tested and the goals of the study.

Question 12.6 The answer is B.


Phase II trials are traditionally restricted to a particular tumor type because the biologic
response of interest is that of the tumor itself. Phase II trials are designed to establish the
experimental drug’s activity in a broad population of unselected patients. The two-stage
study design is useful in eliminating ineffective drugs at an early stage. With the entry of
molecularly targeted drugs, it is important to identify and develop assays for predictive
biomarkers. Furthermore, phase II trials in which patient selection is driven by biomarker
testing will have to be designed to identify the target patient population.

Question 12.7 The answer is D.


There is no biologically inherent relationship between response and survival. For
example, a meta-analysis on randomized clinical trials by Torri and colleagues showed
that a large improvement in response rate only corresponded to a minor increase in
survival. In addition, survival end points are comparative in nature and require taking the
distribution of potential prognostic factors into consideration.

Question 12.8 The answer is A.


The optimal two-stage design minimizes the average sample size and thus optimizes the
protection of patients given an inactive agent. In some cases molecularly targeted drugs
and therapeutic cancer vaccines are not expected to be toxic to normal tissue. Simon and
colleagues have suggested that some therapeutic cancer vaccines do not require a toxicity
trial. For some molecularly targeted drugs, preclinical studies provide a target serum
concentration, in such cases the phase I trial can be designed to estimate the relationship
between the dose administered and serum concentration. The minimax design have
somewhat larger average sample size compared to the optimal design, but in some cases it
is preferable because the small increase in average sample size is more than compensated
for by a large reduction in maximum sample size. Trials of cytostatic agents often choose
time to progression as the primary end point. When testing cytotoxic agents, the use of
Simon two-stage designs is limited to situations where outcome assessment can be made
shortly after patients are treated. These designs usually are not practical for end points
requiring a long period of follow-up. Phase II trials usually do not have an internal control
group.

Question 12.9 The answer is D.


Because cytostatic agents usually have a good safety profile, phase I trials on such drugs
are often designed to identify a biologically active dose rather than maximum tolerated
dose. Because therapeutic cancer vaccines work via stimulation of tumor reactive T cells,
heavily treated patients with end-stage disease are less likely to benefit from a cancer
vaccine. Cytostatic agents, such as antiangiogenesis factors, growth modulators, or cancer
vaccines, usually selectively work on molecular targets to modulate tumor environment.
They may prolong patient survival without causing tumor shrinkage. Therefore, time to
progression is often used as the primary end point for such trials.

Question 12.10 The answer is C.


Option A represents a design that used growth modulation index for the assessment of
efficacy, option B describes a randomized discontinuation design, and option D is
frequently used for trials on the combination of cytotoxic agents. Simon phase 2.5 designs
are similar to phase III trials except that they allow a relatively large type I error and are
able to identify only a relatively large difference.

Question 12.11 The answer is C.


The primary end points for evaluating effectiveness of a treatment should be direct
measures of patient welfare. Thus tumor shrinkage is usually not an appropriate end point
for phase III trials because it may have little or no relation to patient benefit. Eligibility
criteria should be broad to enhance generalizability of results. Validated quality-of-life
instruments are useful parts of phase III trials, especially if they are directly related to
areas of concern to patients. Many areas of patient benefit (e.g., longer survival, lower
risk of relapse, accessibility, or lower cost) may not be readily measured by these
instruments.

Question 12.12 The answer is B.


A random sample helps to provide a fair, unbiased comparison of the treatments within
the population of patients from whom participants are recruited, although this population
may not be representative of all patients with the disease. Randomization may not result
in equal, or nearly equal, numbers of patients with a particular characteristic in each
treatment group, but it does distribute biasing characteristics according to a known
probability distribution so that they do not obscure identification of treatment effects, if
any, by statistical tests. Matching is not a substitute for randomization; in fact, matching
too closely can mask real treatment effects. Blinding of participants or treating physicians
to treatment assignment can reduce bias introduced in the process of treatment, whereas
randomization reduces bias in patient selection. Randomized assignment to treatment can
be used with or without blinding.

Question 12.13 The answer is A.


Randomization is unnecessary for patients with a disease that will uniformly and rapidly
progress to an end stage. In such a setting of homogeneity, a historical control usually is
sufficient. In phase III trials, screening studies of multiple cytostatic agents and phase II
trials, including those of cytotoxic agents, randomization allows the investigators to
conclude with confidence that any observed differences between study groups are due to
the treatment being studied.

Question 12.14 The answer is B.


Statistical power depends on the anticipated size and the variability of the treatment
effect. Trials designed to detect a large effect that varies substantially from patient to
patient may demand large sample sizes. Because a confidence interval provides
information about the size and direction of treatment effect, it is more informative than
significance testing, especially for “negative” trials. It is true that meta-analysis can
combine the results of small randomized studies and may address questions that cannot be
answered by any individual study alone. However, by no means can meta-analyses replace
carefully designed and adequately powered randomized trials. This is because different
trials can be heterogeneous in terms of diagnostic and staging procedures, supportive
care, and methods of patient evaluation and follow-up. Such heterogeneity may obscure
small-to-moderate therapeutic effects and undermine the ground for any pooled analyses.

Question 12.15 The answer is A.


Because it is impossible to prove equivalence in true sense, in practice therapeutic
equivalence will be accepted if the difference in efficacy is within a prespecified level δ.
The traditional superiority trials cannot be used to prove equivalence because a
“negative” finding may be simply due to inadequate sample size. Claim D may be true for
bioequivalence studies but not for therapeutic equivalence trials. In the former, for
example, the objective is to show biologic equivalence of two proprietary preparations of
a drug, and a relatively large type I error or large tolerable difference δ is acceptable.
For trials targeting equivalence in therapeutic effect (e.g., mortality rate), however,
usually a much smaller δ is of interest, and this will require a very large sample size.

Question 12.16 The answer is B.


The confidence interval is a range of values consistent with the observed difference, not
the hypothesized difference, given the precision with which the trial can measure that
difference. It is likely that the true difference is smaller than the lower limit or larger
than the upper limit of the confidence interval. If the confidence interval includes 0, then
the trial has failed to show difference.

Question 12.17 The answer is A.


Because Bayesian methods can incorporate information from preclinical studies and
sources outside of the trial, they are appealing in planning phase I and II trials. In these
trials, information regarding the experimental treatment is sparse, and study design is
based on various (usually subjective) assumptions. However, phase III trials intend to
provide reliable and objective guidance for decision making in a given disease, so the
subjective nature of prior distributions in Bayesian methods limits their popularity in
planning phase III trials. Bayes theorem cannot be considered as a substitute for
randomization. Randomization is just as important for the validity of Bayesian methods as
for frequentist methods. Bayesian sample size calculation depends on the prior
distribution used. If the prior distribution for the treatment effect is a skeptical prior, then
the sample size for Bayesian methods may be much larger than the conventional sample
size.

Question 12.18 The answer is B.


Omitting patients who die or withdraw without completing the study is a serious source
of bias, as is dropping patients for noncompliance or deviation from protocol. These may
be patients for whom the treatment does not work or is associated with serious adverse
consequences. There is no sound way to identify and eliminate this bias in the context of
the trial, so the intention-to-treat analysis includes all consenting, randomized patients in
the study arm to which they were randomized.

Question 12.19 The answer is C.


Multiple looks at outcome data over the course of a trial will not alter type I and II errors
and are useful for directing decisions during the trial. Interim outcome information is
generally available only to data-monitoring committee. For phase III trials, it is
mandatory to have an independent data-safety committee that has the power to
recommend termination of the study based on safety concerns, outstanding benefit, or
futility. Because the study leaders have a conflict of interest with respect to the decision
to continue the study, they are not part of the committee.

Question 12.20 The answer is B.


The usual statistical methods are not appropriate for analyzing survival data because they
ignore the censored observations. Even if survival data have no censoring, a survival
curve still provides a better approach for data summary because it takes the length of
follow-up time into consideration. In addition, survival times are usually skewed to the
right, and the conventional means and standard deviations may not be adequate for data
description. The statistical power of clinical trials using survival as a primary end point
will be mainly determined by the actual number of events (i.e., death or progression)
rather than the total number of patients.

Question 12.21 The answer is B.


For this study, noninformative censoring means that the probability distributions of
cancer-specific survival are similar for both censored and noncensored patients. However,
patients who have experienced disease progression are more likely to have a poor
prognosis.

Question 12.22 The answer is C.


Phase 0 trials are first in human trials that help eliminate ineffective and toxic drugs at a
very early stage of clinical development. A small sample of patients receive a single, low
dose of the study drug at which it is not expected to cause toxic adverse effects. These
trials provide preliminary pharmacokinetic and pharmacodynamic information to inform
decisions concerning further evaluation of the study drug. These studies do require the
prior development of an assay to measure the pharmacodynamics end point for the study
drug.

Question 12.23 The answer is A.


Meta-analysis combines and summarizes the results from several therapeutic trials. To
achieve meaningful results and avoid bias, meta-analysis includes only randomized clinical
trials including unpublished studies. Studies are not restricted to any particular geographic
region. Collaboration of the individual study investigators is required for meta-analysis.
Meta-analysis based on individual patient data is more precise than meta-analysis based
on summarized p-values. Meta-analysis is not an alternative to properly designed and
sized randomized clinical trials.

Corresponding chapter in Cancer: Principles & Practice of Oncology, Tenth edition: 36 (Design and Analysis of Clinical Trials).
13 Cancer of the Head and Neck
Douglas R. Adkins, Jessica C. Ley and Loren Michel

QUESTIONS
Each of the numbered items below is followed by lettered answers. Select the ONE lettered answer
that is BEST in each case unless instructed otherwise.

Question 13.1 A 65-year-old man with a 40 pack-year history of smoking presented with
hoarseness and a neck mass. The patient did not aspirate and had not lost weight. The patient
was found to have T4N2cM0 squamous cell carcinoma (SCC) of the glottic larynx. T4
classification was based on limited cortical erosion of the thyroid cartilage. A total
laryngectomy and bilateral neck dissections followed by adjuvant radiation therapy was
recommended by the otolaryngologist. However, the patient preferred a chance to preserve
the larynx. Which treatment approach offers the best chance to preserve the patient’s larynx
and prevent disease recurrence?
A. Radiation therapy alone
B. Induction chemotherapy followed by radiation therapy
C. Concurrent chemotherapy and radiation therapy
D. Chemotherapy alone

Question 13.2 Which of the following statements about the epidermal growth factor
receptor (EGFR) in head and neck SCC (HNSCC) is TRUE?
A. Randomized trials demonstrated that oral tyrosine kinase inhibitors (TKI) of EGFR such
as erlotinib and monoclonal antibody inhibitors of EGFR such as cetuximab resulted in
similar tumor control.
B. Activating EGFR mutations are frequent in HNSCC.
C. The EGFR inhibitor cetuximab improved overall survival when added to definitive
radiation therapy or to palliative platin-based chemotherapy in HNSCC.
D. High EGFR expression is associated with sensitivity to radiation therapy.

Question 13.3 Which of the following are risk factors for HNSCC?
A. Tobacco use
B. Alcohol use
C. Fanconi anemia
D. All of the above

Question 13.4 Which of the following characteristics are typical features of human
papilloma virus (HPV)-related HNSCC?
A. Most patients have limited or no smoking history and most have primary tumors that
involve the oropharynx (base of tongue or palatine tonsils).
B. Survival outcomes are substantially better than in patients with smoking-associated
HNSCC.
C. The role of the HPV in subsites other than the oropharynx is not clear.
D. All of the above.

Question 13.5 A 56-year-old man presented to his primary care physician with a 3-month
history of right ear pain and two masses on the right side of his neck. On examination, he
was found to have a 3-cm right palatine tonsil mass and two enlarged lymph nodes under the
upper part of the right sternocleidomastoid muscle. One lymph node was 2 cm, and the other
was 1 cm in diameter. A fine-needle aspiration (FNA) of one of the neck lymph nodes was
performed and it was consistent with p16+ SCC. Neck and chest computed tomography (CT)
showed no additional lymphadenopathy or distant metastasis. What is the stage of his
disease?
A. Stage II
B. Stage III
C. Stage IVA
D. Stage IVC

Question 13.6 Which of the following statements is TRUE?


A. Radical neck dissections should be performed for the management of all locally
advanced HNSCC.
B. Complications from neck dissections may include hematoma, seroma, lymphedema,
wound infections and dehiscence, carotid artery exposure and rupture, and damage to
the VII, X, XI, and XII cranial nerves.
C. In a surgically treated tumor without radiographic evidence of lymph node metastasis,
an elective neck dissection should be performed if the risk of occult metastasis to the
neck nodes is greater than 50%.
D. For patients who have had a neck dissection for HNSCC, there is no benefit from
postoperative adjuvant radiation therapy.

Question 13.7 Which of the following statements is TRUE?


A. The combination of cisplatin and 5FU improves overall survival in comparison to single-
agent methotrexate in patients with metastatic HNSCC.
B. Cetuximab has no activity in metastatic or recurrent HNSCC.
C. The combination of cisplatin and 5FU results in a higher tumor response rate than
single-agent methotrexate in patients with metastatic HNSCC.
D. Higher chemotherapy doses improve overall survival.

Question 13.8 A 66-year-old man presented to his otolaryngologist with a 4-cm left floor of
mouth SCC. A 3-cm left neck node was felt on examination. CT of the neck also showed that
the primary tumor invaded into the mandible. There was no evidence of distant metastasis on
CT of the chest. What is the most appropriate initial therapy for this patient?
A. Surgery
B. Radiation therapy
C. Concurrent chemotherapy and radiation therapy
D. Chemotherapy

Question 13.9 A 54-year-old woman recently underwent resection of a SCC of the


supraglottis with bilateral neck dissections. The pathology showed a T2 lesion completely
excised to negative margins but lymphovascular and perineural invasion were present. Two
of the 28 lymph nodes contained SCC with extracapsular extension on the right. What is the
most appropriate adjuvant therapy for this patient?
A. Observation
B. Radiation alone.
C. Concurrent chemotherapy and radiation therapy.
D. Chemotherapy alone.

Question 13.10 What is the benefit of cetuximab in patients with HNSCC?


A. Concurrent cetuximab and radiation therapy resulted in improved overall survival
compared to radiation therapy alone in stage I to IV disease.
B. Cetuximab is equally effective as cisplatin when given concurrently with radiation
therapy.
C. Addition of cetuximab to concurrent cisplatin and radiation therapy improved overall
survival and reduced relapse risk.
D. Concurrent cetuximab and radiation therapy resulted in improved overall survival
compared to radiation therapy alone in nonmetastatic stage III to IV HNSCC.

Question 13.11 Which of the following statements about the treatment of laryngeal SCC is
TRUE?
A. Radiation therapy alone resulted in worse overall survival compared with concurrent
chemotherapy and radiation therapy in patients with locally advanced laryngeal SCC.
B. Concurrent chemotherapy and radiation therapy resulted in better local tumor control
and laryngeal preservation rate than radiation therapy alone for locally advanced SCC
of the larynx.
C. A total laryngectomy is required for the treatment of locally advanced SCC of the
larynx.
D. Large-volume laryngeal SCC is defined as tumor that extends more than 1 cm onto the
base of tongue and/or extension through the thyroid cartilage into adjacent soft tissue
and is best treated with nonsurgical therapy.

Question 13.12 True or False: There is an overall survival benefit with adjuvant
chemotherapy given after resection of a SCC of the larynx.
A. True
B. False

Question 13.13 Which of the following statements are TRUE? (Select two correct responses)
A. The three subtypes of nasopharyngeal carcinoma are World Health Organization (WHO)
type I keratinizing SCC; WHO type II nonkeratinizing SCC; and WHO type III
undifferentiated carcinoma or lymphoepithelioma. Type III nasopharyngeal SCC is due
to EBV infection and has a much better prognosis than Type I nasopharyngeal SCC.
B. Nasopharyngeal carcinoma is usually treated with chemotherapy and radiation.
C. Common presenting symptoms of nasopharyngeal carcinomas include hoarseness,
painful lower neck mass, and weight loss.
D. The majority of nasopharyngeal carcinomas present with distant metastatic disease.

Question 13.14 A 56-year-old woman presented to her primary care physician with left-
sided facial swelling and facial drooping. The patient had weakness of the lower portion of
her face and a palpable left parotid mass. A biopsy was performed and showed high-grade
mucoepidermoid carcinoma. There were no palpable neck lymph nodes. What is the best
treatment for this patient?
A. Radiation alone
B. Concurrent chemotherapy and radiation therapy
C. Surgery alone
D. Surgery followed by adjuvant radiation therapy

Question 13.15 Which of the following statements is TRUE about locally advanced HNSCC?
(Select two correct responses)
A. The most common long-term complication of radiation therapy is xerostomia.
B. Once the patient is able to receive nutrition via a G-tube, there is no benefit from
having the patient continue to swallow during radiation therapy.
C. Intensity-modulated radiation therapy (IMRT) decreases the risk of xerostomia.
D. The severity of the acute side effects of radiation therapy is similar when chemotherapy
is given concurrently.

Question 13.16 Which of the following statements are TRUE regarding second primary
cancers in patients with HNSCC?
A. Patients with HPV-related oropharyngeal SCC have a similar risk of second primary
cancers as do patients with smoking-induced HNSCC.
B. Patients with resected HNSCC who quit smoking have a lower risk of second primary
cancers than those who continue to smoke.
C. Treatment with α-tocopherol decreases the incidence of second primary cancers.
D. Treatment with isotretinoin decreases the incidence of second primary cancers.

Question 13.17 Which of the following statements regarding induction chemotherapy in the
treatment of HNSCC is TRUE?
A. Treatment with concurrent chemotherapy and radiation therapy resulted in improved
overall survival compared to induction chemotherapy followed by radiation therapy for
locally advanced laryngeal SCC.
B. Addition of docetaxel to cisplatin and 5FU was associated with similar overall survival
when compared to cisplatin and 5FU in patients with locally advanced HNSCC
subsequently treated with radiation therapy alone or concurrent carboplatin and
radiation therapy.
C. Induction chemotherapy may facilitate organ preservation in patients with locally
advanced laryngeal or hypopharyngeal SCC.
D. Induction chemotherapy followed by concurrent cisplatin and radiation therapy results
in improved overall survival compared to concurrent cisplatin and radiation therapy.

Question 13.18 A 35-year-old nonsmoking female with a history of chronic oral lichen
planus presented with a painful right oral tongue lesion measuring 2.5 cm. Biopsy revealed
well-differentiated SCC. Ultrasound revealed that the oral tongue lesion was 0.6 cm thick. CT
showed no abnormal neck nodes. The lesion was widely excised and a right selective neck
dissection (SND) was performed. Pathology revealed a 2.6 cm SCC, negative surgical
margins, no perineural or lymphovascular invasion, and all 15 lymph nodes without SCC. The
pathologic stage was II (T2N0M0). The most appropriate next therapy is:
A. Postoperative adjuvant radiation therapy.
B. Postoperative adjuvant concurrent chemotherapy and radiation therapy.
C. Close monitoring.
D. Left selective neck dissection.

Question 13.19 A 58-year-old male with stage IVA SCC of the larynx was treated with
concurrent chemotherapy and radiation therapy 12 months ago. On routine surveillance CT,
multiple new bilateral pulmonary nodules were noted. A biopsy of one of the pulmonary
nodules showed SCC consistent with metastases from the laryngeal cancer. The patient has an
ECOG performance status of 0 and has no serious comorbidities. What is the most effective
treatment for this patient’s cancer?
A. Weekly administration of cetuximab
B. Weekly paclitaxel
C. Carboplatin, docetaxel, and cetuximab with peg-filgrastim
D. Carboplatin, 5FU, and cetuximab

Question 13.20 A 65-year-old male presented with a large hard palate mass. Biopsy showed
a salivary ductal adenocarcinoma. The mass was completely excised and pathology revealed a
4 cm tumor with positive surgical margins. Postoperative adjuvant radiation therapy was
given. Nine months later, a surveillance CT showed several pulmonary nodules. A biopsy of
one of the pulmonary nodules showed metastatic salivary ductal adenocarcinoma. The patient
was referred to medical oncology for management. The most appropriate next step in the
management of this patient is:
A. Weekly cetuximab.
B. Platin, 5FU, and cetuximab.
C. Taxane.
D. Perform immunohistochemistry stain for the androgen receptor.
ANSWERS

Question 13.1 The answer is C.


RTOG 91-11 was a three-arm randomized trial of patients with laryngeal SCC who were
candidates for total laryngectomy but wanted to preserve their larynx. These patients
were randomly assigned to receive radiation alone, induction chemotherapy followed by
radiation, or concurrent chemotherapy and radiation therapy. Concurrent chemotherapy
and radiation therapy resulted in the highest chance for laryngeal preservation and
disease control. However, overall survival was similar in the three treatment arms.

Question 13.2 The answer is C.


EGFR plays a key role in the development of HNSCC. EGFR overexpression leads to
activation of multiple signaling pathways that lead to cell growth and resistance to
apoptosis. Activating EGFR mutations are infrequent in HNSCC. Randomized trials
demonstrated improved overall survival with the addition of cetuximab to palliative
platin-based chemotherapy (EXTREME Trial) and with the addition of cetuximab to
definitive radiation therapy (Bonner et al. Trial). Analysis of EGFR expression in archived
tumor tissue from a prospective RTOG clinical trial demonstrated that EGFR
overexpression is associated with resistance to radiation therapy. In contrast to
monoclonal antibody inhibitors of EGFR such as cetuximab, no randomized trial has been
performed to show clinical benefit of an oral tyrosine kinase inhibitors (TKI) of EGFR
such as erlotinib in HNSCC.

Question 13.3 The answer is D.


Several risk factors for the development of HNSCC have been identified. The two most
important are tobacco and alcohol use. They seem to be synergistic. Other risk factors
include occupational exposure to chemicals and irritants, such as aromatic hydrocarbons
and wood dust, and viruses, such as HPV and EBV. In addition, patients with Fanconi
anemia are also at increased risk.

Question 13.4 The answer is D.


It has been recognized over the last decade that HPV is the cause of an increasing
proportion of SCC of the oropharynx. Today, most cases of oropharyngeal SCC is due to
HPV; however, smoking can also contribute to the biology of HPV-related SCC. These
tumors are often described as having a basal cell-like or poorly differentiated appearance
and are frequently nonkeratinizing. These tumors are more commonly found in never
smokers or those patients with a limited smoking history. HPV-related oropharyngeal SCC
has a much better prognosis than other oral mucosal HNSCC. The overall survival is
higher, and the risks of primary cancer relapse and secondary cancer development are
much lower in comparison to HPV-unrelated HNSCC. Patients with HPV-related HNSCC
have a better tumor response to chemotherapy and radiation therapy than patients with
smoking-associated HNSCC.
Question 13.5 The answer is C.
The patient has a tonsil mass that is between 2 and 4 cm, making the primary tumor T2
classification. He has multiple ipsilateral regional lymph nodes with the largest lymph
node being smaller than 6 cm, which is N2b classification. CT found no evidence of distant
metastasis. The patient’s disease is classified as T2N2bM0, which is within stage IVA.

Question 13.6 The answer is B.


Control of neck nodal disease is very important in the management of HNSCC. Both neck
node dissection and radiation therapy can be used. When HNSCC is treated surgically, a
neck dissection is recommended even in a clinically negative neck if the risk of occult
metastasis exceeds 10% to 15%. A radical neck dissection (RND) removes all neck nodal
groups, the internal jugular vein, the eleventh cranial nerve and the sternocleidomastoid
muscle. A RND is rarely performed today. A modified radical neck dissection (MRND)
preserves one or more of the following structures: cranial nerve XI, internal jugular vein,
and sternocleidomastoid. A MRND does not compromise neck nodal disease control. A
selective neck dissection (SND) removes selected neck nodal groups only and are the most
common type of neck dissection performed today. Most patients with neck nodes involved
by SCC should be treated with postoperative adjuvant radiation therapy to reduce the risk
of disease recurrence. Postoperative adjuvant concurrent chemotherapy and radiation
therapy should be offered to patients with positive surgical margins at the primary tumor
site and/or extracapsular nodal extension. Complications from neck dissections may
include hematoma; seroma; lymphedema; wound infections and dehiscence; damage to
the VII, X, XI, and XII cranial nerves; carotid exposure; and carotid rupture.

Question 13.7 The answer is C.


Many chemotherapeutic agents including cisplatin, carboplatin, 5FU, taxanes, and
methotrexate have activity against metastatic HNSCC. Combinations of conventional
cytotoxic chemotherapy and higher doses of chemotherapy result in higher tumor
response rates but have not shown an overall survival benefit in comparison to single
agent cytotoxic chemotherapy. The anti-EGFR antibody cetuximab has shown significant,
albeit modest, activity against HNSCC. The EXTREME trial demonstrated that the addition
of cetuximab to platin-based chemotherapy improved overall survival in patients with
incurable HNSCC. Also, cetuximab is approved for the treatment of platin-refractory
incurable HNSCC.

Question 13.8 The answer is A.


This patient has a stage IVA (T4aN1M0) SCC of the oral cavity. The primary therapy that
is most effective in most cases of oral cavity SCC is surgery. Primary radiation therapy
alone, chemotherapy alone, or concurrent chemotherapy would not be considered to be as
effective, particularly with invasion of the mandible. The patient will likely require
postoperative adjuvant radiation therapy alone or with chemotherapy, depending on the
pathologic findings.
Question 13.9 The answer is C.
After gross tumor resection, adjuvant therapy with either radiation or concurrent
chemotherapy and radiation therapy is recommended for most cases of locally advanced
HNSCC. Pathologic findings determine which adjuvant therapy is to be recommended.
Major adverse pathologic risk factors for local recurrence are positive surgical margins or
extracapsular nodal extension. Minor adverse pathologic risk factors for recurrence
include T3 or T4 primary tumors, N2 or N3 nodal disease, and perineural or
lymphovascular invasion. If the patient has any major risk factor, adjuvant concurrent
chemotherapy and radiation therapy is recommended based on the results of two
randomized trials. If the patient has any minor risk factor, adjuvant radiation therapy is
recommended. If the patient has no adverse risk factors, observation is recommended.

Question 13.10 The answer is D.


Cetuximab is an EGFR inhibitory antibody that has been used alone or combined with
chemotherapy for treatment of metastatic disease, and combined with radiation therapy
to treat locally advanced stage III to IV HNSCC. In a randomized phase III trial that
compared concurrent radiation therapy with cetuximab to radiation therapy alone for
nonmetastatic stage III to IV HNSCC of the oropharynx, larynx, and hypopharynx,
cetuximab improved both local disease control and overall survival. In patients receiving
cetuximab with radiation therapy, the incidence of acne and infusion reactions were
increased but the incidence of mucositis was similar. Concurrent cetuximab with radiation
therapy has not been compared with cisplatin and radiation therapy. RTOG 0522 showed
the lack of benefit and increased acute toxicity with the addition of cetuximab to
concurrent cisplatin and radiation therapy.

Question 13.11 The answer is B.


The treatment of larynx cancers has changed significantly over the last 30 years.
Historically, the traditional treatment approach for locally advanced SCC of the larynx
was a total laryngectomy and neck dissection followed by adjuvant radiation therapy.
This resulted in loss of speech, even if the quality of the speech was acceptable before
therapy. The Veterans Administration Laryngeal Study Group Trial was a randomized trial
that compared total laryngectomy followed by adjuvant radiation therapy to induction
chemotherapy followed by radiation therapy. In the latter treatment arm, salvage total
laryngectomy was performed in patients with persistent or recurrent local disease. This
pivotal trial established that overall survival was similar between the two treatment arms;
however, approximately two-thirds of patients had laryngeal preservation with the
nonsurgical treatment approach. Subsequently, RTOG 91-11 was a three-arm randomized
trial that compared radiation therapy alone, induction chemotherapy followed by
radiation therapy, and concurrent chemotherapy with radiation therapy for treatment of
patients with locally advanced laryngeal SCC. This trial excluded patients with large
volume primary tumors who were felt to benefit from a primary surgical approach. RTOG
91-11 demonstrated that the best chance for laryngeal preservation was when patients
were treated with concurrent chemotherapy and radiation therapy, even though overall
survival was the same in each of the three treatment arms. However, subsequent
improvements in surgical techniques including trans-oral endoscopic laser resection and
transoral robotic surgery have also provided surgical approaches to preserve the larynx.

Question 13.12 The answer is B.


Adjuvant chemotherapy has been tested in several trials of HNSCC without resulting in
survival improvement. Currently, adjuvant chemotherapy is only used in the treatment of
patients with locally advanced nasopharyngeal SCC following concurrent chemotherapy
and radiation therapy based on the result of the Intergroup 0099 trial. Even in this
setting, the benefit of adjuvant chemotherapy is unclear.

Question 13.13 The answers are A and B.


Nasopharyngeal carcinomas are unusual in the United States but are much more common
in China and Southeast Asia. On the basis of pathology, there are three subtypes of
nasopharyngeal carcinoma: WHO type I keratinizing SCC; WHO type II nonkeratinizing
SCC; and WHO type III, undifferentiated carcinoma or lymphoepithelioma. The WHO
type III is associated with EBV and has a much more favorable prognosis than type I
disease. Nasopharyngeal SCC usually presents with a painless upper neck mass, nasal
stuffiness, facial pain, and headache. Cranial nerve involvement occurs in 25% of the
patients. Cranial nerves II, III, IV, V, and VI can be affected by the tumor extending into
the cavernous sinus. Involvement of the sixth (abducens) cranial nerve is most common,
resulting in diplopia. Lateral pharyngeal space extension of the tumor affects cranial
nerves IX through XII. Approximately, 80% to 90% of the patients have neck lymph node
involvement at presentation and half of the patients having bilateral neck nodal disease.
Nasopharyngeal SCC is almost exclusively treated with radiation therapy because of the
challenges of performing effective surgery in the skull base. Concurrent chemotherapy
and radiation therapy followed by adjuvant chemotherapy is the current standard
treatment for patients with stage II to IV SCC of the nasopharynx; however, a recent
randomized trial did not show a benefit of adjuvant chemotherapy for patients with EBV-
related nasopharyngeal SCC. Definitive radiation therapy alone is effective treatment for
stage I nasopharyngeal SCC.

Question 13.14 The answer is D.


The treatment of choice for malignant tumors of the parotid gland is surgery. High grade
tumors and tumors with perineural invasion are also treated with adjuvant radiation
therapy. There is no clear role for chemotherapy in salivary gland cancers that have been
resected or in those treated with definitive or postoperative radiation therapy.

Question 13.15 The answers are A and C.


Xerostomia is the most common long-term complication of radiation therapy for
treatment of HNSCC. Xerostomia impairs swallowing function and sleeping and results in
poor dentition. Radiation to the head and neck can cause acute and late toxicities. The
degree of toxicity from radiation varies from patient to patient and depends on the dose
of radiation, the radiation port, the radiation technique, and whether chemotherapy is
given concurrently. The addition of chemotherapy to radiation therapy increases the acute
toxicity. IMRT allows varying doses of radiation to be given to different parts of a
radiation field. IMRT permits the parotid gland and other major salivary glands to be
spared from treatment, resulting in a lower risk of xerostomia. To decrease the risk of
long-term swallowing dysfunction, patients should be strongly encouraged to continue to
swallow to exercise the pharyngeal muscles even if they are receiving the majority of
nutrition via a G-tube.

Question 13.16 The answer is B.


Prospective trials on secondary chemoprevention with isotretinoin and α-tocopherol have
shown no significant benefit in preventing second primary cancers in patients with
resected HNSCC. Smoking status is an important risk factor for the development of second
primaries and never smokers have lower risk for second primaries compared to former
and current smokers. Those patients who quit smoking have a lower risk of second
primary cancers than those patients who continue to smoke. The risk of second cancers is
lower in patients with HPV-related oropharyngeal SCC than those with smoking-induced
HNSCC.

Question 13.17 The answer is C.


Induction chemotherapy followed by definitive radiation therapy has the potential for
organ preservation in patients with locally advanced SCC of the larynx or hypopharynx,
based on RTOG 91-11 and the European randomized trials, respectively. In the RTOG 91-
11 randomized trial, overall survival was similar with induction chemotherapy followed
by definitive radiation therapy compared with concurrent chemotherapy and radiation
therapy. However, there was a significant organ preservation advantage with the
concurrent chemotherapy and radiation therapy approach. The addition of docetaxel to
cisplatin and 5FU was associated with improved overall survival when compared to
cisplatin and 5FU in patients with locally advanced HNSCC subsequently treated with
radiation therapy alone (TAX 323 trial) or concurrent carboplatin and radiation therapy
(TAX 324 trial). The recent results of the PARADIGM and DeCIDE randomized trials failed
to demonstrate an overall survival benefit of induction chemotherapy added to concurrent
chemotherapy and radiation therapy; however, these trials had several limitations
complicating trial interpretation.

Question 13.18 The answer is C.


The most appropriate management of a patient with pathologic stage II (T2N0M0) oral
cavity SCC is close monitoring. Approximately 80% of these patients will be cured with
appropriate surgery alone. Appropriate surgery in such patients includes resection of the
primary tumor site to achieve wide negative margins, and a selective ipsilateral neck
dissection if the tumor thickness exceeds 4 to 6 mm. There is no established role for
adjuvant radiation therapy or concurrent chemotherapy and radiation therapy in such
patients. Also, a lateralized oral tongue SCC would not require a contralateral neck
dissection since the risk of contralateral nodal involvement by SCC would be low
(<15%). This patient should be followed closely by an experienced otolaryngologist for
the possibility of a local-regional recurrence. Also, patients with lichen planus are at risk
for development of oral mucosal SCC and must be followed to monitor for this potential
problem.

Question 13.19 The answer is D.


The EXTREME trial showed that the addition of cetuximab to platin and 5FU improved
overall survival over platin and 5FU alone in patients with incurable HNSCC. While single
agent cetuximab or paclitaxel and the three-drug regimen of carboplatin, docetaxel, and
cetuximab have antitumor activity in incurable HNSCC, neither has been compared to
platin, 5FU, and cetuximab. The current standard-of-care first-line therapy for patients
with incurable HNSCC with good performance status and adequate organ function is the
combination of platin, 5FU, and cetuximab.

Question 13.20 The answer is D.


Salivary gland cancers are rare and heterogeneous in pathology, prognosis, and treatment.
The most common histologic types include adenoid cystic carcinoma, mucoepidermoid
carcinoma, adenocarcinoma, and salivary ductal carcinoma. Salivary ductal carcinomas
are aggressive cancers that are treated with surgery and adjuvant radiation therapy.
Incurable salivary ductal carcinoma often spreads to the lungs and bone and cytotoxic
chemotherapy has limited activity. Several recent reports demonstrated frequent
expression of the androgen receptor in salivary ductal carcinomas and, in these patients,
high tumor response to androgen blockade. Androgen blockade can include GNRH
agonists (leuprolide) with or without androgen receptor blockade (bicalutamide), similar
to strategies used to treat prostate cancer. Other potentially effective agents in these
patients include enzalutamide and abiraterone.

Corresponding chapters in Cancer: Principles & Practice of Oncology, Tenth Edition: 37 (Molecular Biology of Head and Neck
Cancers), 38 (Cancer of the Head and Neck), and 39 (Rehabilitation after Treatment of Head and Neck Cancer).
14 Lung Cancer and Mesothelioma
Siddhartha Devarakonda and Saiama N. Waqar

QUESTIONS
Each of the numbered items below is followed by lettered answers. Select the ONE lettered response
that is BEST in each case.

Question 14.1 A 40-year-old never-smoker woman was diagnosed with adenocarcinoma of


the lung a year ago. Molecular testing of her tumor revealed an exon 19 deletion in EGFR.
She was started on erlotinib and had a partial response. However, her most recent scan
suggested significant progression in her disease burden with multiple new liver and lung
metastases. Her tumor was rebiopsied. Which of the following alterations is most likely to
explain the tumor’s resistance to erlotinib?
A. L858R
B. MET amplification
C. T790M
D. The biopsy is likely to reveal transdifferentiation to small cell lung cancer (SCLC).

Question 14.2 Sequencing the lung adenocarcinoma of a 54-year-old man with a 60 pack-
year smoking history is likely to reveal which of the following features?
A. EML4-ALK translocation
B. An excess of C>T transitions
C. Co-mutation of TP53 and RB1
D. An excess of G>T transversions

Question 14.3 A 55-year-old man with a 30 pack-year history of smoking, presents to the
emergency room with shortness of breath. Chest radiograph demonstrates a right upper lobe
opacity. Computed tomography (CT) scan reveals a 3.5-cm spiculated mass in the peripheral
right upper lobe, which is suspicious for malignancy, without any hilar or mediastinal
lymphadenopathy. What is the next best step in management?
A. Bronchoscopy and biopsy of the mass
B. CT-guided biopsy of the mass
C. Brain MRI
D. Refer to thoracic surgeon for resection.

Question 14.4 A 65-year-old man with a 40 pack-year history of smoking, presents to the
emergency room with shortness of breath. Chest radiograph demonstrates a left lower lobe
mass and left pleural effusion. CT reveals a 3-cm left lower lobe mass, left hilar fullness, and
a moderate left pleural effusion. Biopsy of the mass and thoracentesis are both positive for
adenocarcinoma. Staging studies do not reveal any distant metastases. Which of the following
is the next best step in his management?
A. Referral to a thoracic surgeon
B. Radiation to the chest
C. Concurrent chemotherapy and radiation
D. Platinum-based doublet therapy

Question 14.5 A 66-year-old man, with a 30 pack-year history of smoking, presents to your
office for consultation regarding chemotherapy options for metastatic non–small cell lung
cancer (NSCLC), squamous histology. He has no significant medical problems and the
performance status (PS) is 1. Laboratory studies reveal normal blood counts, liver enzymes,
and kidney function. Which of the following treatment regimens would you recommend?
A. Cisplatin and pemetrexed
B. Carboplatin, paclitaxel, and bevacizumab
C. Carboplatin and paclitaxel
D. Carboplatin and erlotinib

Question 14.6 A 55-year-old Asian woman, who is a never smoker, completed four cycles of
front-line carboplatin and paclitaxel, for metastatic NSCLC. Imaging studies done after
completion of therapy show stable disease. She is very active and has continued to work as a
nurse throughout her treatment. Her PS is 0 and she has tolerated the treatment well, other
than grade I neuropathy. Her tumor EGFR status is wild type. She wants “the best treatment
possible” and desires further treatment. What would you recommend?
A. Treatment break, erlotinib at the time of disease progression
B. Stopping carboplatin, continuing paclitaxel till disease progression
C. Continuing carboplatin and paclitaxel for four additional cycles
D. Pemetrexed maintenance therapy

Question 14.7 A 58-year-old man, with a 40 pack-year history of smoking is referred to you
by a radiation oncologist for management of limited-stage SCLC. What treatment would you
recommend?
A. Cisplatin and etoposide chemotherapy
B. Cisplatin and etoposide chemotherapy with concurrent thoracic radiation
C. Cisplatin and etoposide chemotherapy, followed by PCI if response to chemotherapy
D. Cisplatin and etoposide chemotherapy with concurrent thoracic radiation, followed by
PCI if response to treatment

Question 14.8 Which of the following statement(s) is/are CORRECT regarding the role of
adjuvant chemotherapy after resection for NSCLC?
A. Adjuvant chemotherapy benefits patients with stage IA disease.
B. Adjuvant chemotherapy benefits patients with node-positive disease.
C. Adjuvant chemotherapy may benefit patients with stage IB disease, who have primary
tumors less than 4 cm in size.
D. All of the above.

Question 14.9 Which of the following statements is CORRECT regarding pulmonary


carcinoid tumors?
A. Patients usually present with carcinoid syndrome.
B. Surgery has a curative potential for resectable localized tumors.
C. Adjuvant chemotherapy is the standard of care following complete surgical resection.
D. Most pulmonary carcinoids are atypical carcinoids.

Question 14.10 Which of the following statement is TRUE regarding first-line therapy with
EGFR tyrosine kinase inhibitors (TKIs) in patients with EGFR-mutated lung adenocarcinoma?
A. Response rates with TKIs are similar to that achieved with chemotherapy, although they
are more durable.
B. TKIs have consistently shown superior overall survival compared to chemotherapy.
C. TKIs when combined with chemotherapy yield better outcomes than treatment with
either modality alone.
D. TKIs show better response rates and progression-free survival compared to
chemotherapy.

Question 14.11 A 60-year-old man with a 45 pack-year history of smoking, presents with
chest pain. Chest x-ray reveals a right upper lobe mass. CT scan of the chest demonstrates a
4-cm right upper lobe lung mass, with right hilar and multiple ipsilateral enlarged
mediastinal lymph nodes. Bronchoscopy and biopsy of the mass reveals NSCLC. PET scan
demonstrates increased FDG-uptake in the lung mass, right hilar and mediastinal lymph
nodes, but no other site of metastatic disease. CT of the abdomen and brain MRI are
unremarkable. Mediastinoscopy and biopsy reveals NSCLC. His cancer is staged as T2aN2M0,
stage IIIA NSCLC. His PS is 1 and he is otherwise in good health. Which of the following is
the best management for this patient?
A. Definitive radiation to the chest
B. Radiation to the chest, followed by platinum-based chemotherapy
C. Concurrent radiation to the chest and platinum-based chemotherapy
D. Platinum-based chemotherapy

Question 14.12 When treating a patient with metastatic ALK rearranged adenocarcinoma of
the lung, disease progression was noted after 9 months of treatment with crizotinib. Which of
the following is/are valid treatment option(s) for this patient?
A. Treatment with a second-generation ALK inhibitor such a ceritinib
B. Treatment with chemotherapy
C. Treatment with local ablative techniques for oligo-progression
D. All of the above

Question 14.13 A 45-year-old Asian woman, who is a never smoker, presents to your office
for consultation regarding systemic therapy for metastatic adenocarcinoma of the lung. Her
tumor has an activating EGFR mutation. Which of the following is a valid first-line treatment
option for this patient?
A. Cetuximab
B. Afatinib
C. Necitumumab
D. Crizotinib

Question 14.14 A 65-year-old Caucasian man, who is a never smoker, presents to your office
for consultation regarding therapy for metastatic adenocarcinoma of the lung. Genotyping of
his tumor revealed a gene fusion involving ROS1. The patient is interested in knowing more
about this alteration and if he would qualify for treatment with any of the currently approved
TKIs. Which of the following response is CORRECT?
A. While ROS1 alterations are seen in nearly 15% of adenocarcinomas, but there are no
TKIs known to be active in this setting
B. ROS1 rearrangements are seen in 5% of adenocarcinomas, and these tumors respond
well to treatment with erlotinib.
C. ROS1 rearrangements are seen in nearly 2% of adenocarcinomas, and these tumors
respond well to treatment with crizotinib.
D. ROS1 rearrangements are seen in 15% of all adenocarcinomas, and these tumors can
respond to crizotinib.

Question 14.15 A 65-year-old man, who is a former smoker presents for consultation
regarding management of unresectable stage IIIB adenocarcinoma of the lung. He has a PS of
0 and adequate blood counts, hepatic, and renal function. Which of the following would you
recommend?
A. Concurrent definitive cisplatin and etoposide therapy, no consolidation therapy, no PCI
B. Concurrent definitive cisplatin and etoposide therapy, consolidation docetaxel therapy,
no PCI
C. Concurrent definitive cisplatin and etoposide therapy, consolidation gefitinib therapy,
no PCI
D. Concurrent definitive cisplatin and etoposide therapy, consolidation gefitinib therapy,
followed by PCI

Question 14.16 A 44-year-old woman presents with a newly diagnosed adenocarcinoma of


the lung located at the apex of the left lung and involving the brachial plexus and first rib.
She undergoes a PET scan that reveals no other nodal involvement or distant sites of FDG
uptake. The preferred treatment strategy in this situation would be?
A. Definitive treatment with concurrent chemoradiation
B. Surgical resection followed by adjuvant chemotherapy
C. Palliative chemotherapy
D. Concurrent chemoradiation followed by surgical resection

Question 14.17 Which of the following is a characteristic genomic feature of small cell lung
cancer?
A. Recurrent alterations in EGFR and ALK
B. Recurrent alterations in TP53 and RB1
C. Recurrent mutations in DDR2
D. Recurrent KRAS mutations.

Question 14.18 A 45-year-old male was diagnosed with stage II adenocarcinoma of the right
lung. Staging revealed ipsilateral hilar lymph-node involvement and no distant metastatic
disease. Patient underwent a sleeve resection of the tumor and his postoperative recovery
was uneventful. Which is the next preferred step in the management of his disease?
A. No further treatment, observation only.
B. Adjuvant chemotherapy with a doublet containing cisplatin
C. Adjuvant chemotherapy with a doublet containing carboplatin
D. Prophylactic cranial irradiation

Question 14.19 What is the most common histologic subtype of malignant mesotheliomas?
A. Epithelial
B. Sarcomatoid
C. Poorly differentiated
D. Biphasic

Question 14.20 Which of the following immunohistochemistry markers is present in


malignant mesotheliomas?
A. CEA
B. TTF-1
C. Moc-31
D. Calretinin

Question 14.21 Which of the following is an indicator of poor prognosis in malignant


mesothelioma?
A. Anemia
B. Epithelial histology
C. Female gender
D. Thrombocytopenia

Question 14.22 Which of the following finding(s) indicate an unresectable pleural


mesothelioma?
A. Direct extension into the spine
B. Extension into the internal surface of the pericardium without pleural effusion
C. Direct transdiaphragmatic extension to the peritoneum
D. All of the above

Question 14.23 Frequent alteration of which of the following novel tumor suppressor genes
is seen in mesothelioma?
A. RBL1
B. APC
C. RB1
D. BAP1

Question 14.24 Which of the following statements is applicable to peritoneal


mesotheliomas?
A. It is not associated with airborne asbestos fiber exposure.
B. Median age of presentation is 50 years.
C. Pleural plaques are observed in approximately 50% of patients.
D. It is more common in females.

Question 14.25 A 60-year-old man was recently diagnosed with metastatic pleural
mesothelioma. Assuming that this patient has no other significant comorbidities, which of the
following chemotherapy regimens would be most preferable in the first-line setting?
A. Cisplatin and gemcitabine
B. Carboplatin and gemcitabine
C. Single agent pemetrexed
D. Cisplatin and pemetrexed
ANSWERS

Question 14.1 The answer is C.


EGFR mutations occur in nearly 10% of NSCLCs among North American and European
patients. These mutations are found more frequently in Asian populations (50%), women
and never smokers. EGFR mutations predict response to EGFR–tyrosine kinase inhibitors
(TKIs). The most common mutations observed are an in-frame deletion in exon 19 and a
point mutation in exon 21 (L858R). These mutations result in prolonged EGFR activation
and downstream signaling. Patients treated with EGFR TKIs eventually develop resistance
to these drugs. Acquisition of a secondary EGFR T790M gatekeeper mutation is the most
common mechanism of resistance and is seen in about 50% of all cases. MET
amplification, activation of other bypass signaling pathways and transdifferentiation to
small cell lung cancer (SCLC) represent other, less prevalent, mechanisms of acquired
resistance.

Question 14.2 The answer is D.


Data from several studies that have characterized the genomic profile of lung cancers
through next-generation sequencing have offered a valuable insight into the molecular
heterogeneity of lung cancers. Tumors from smokers are enriched for G>T transversions
while tumors obtained from never smokers show C>T transitions. EML4-ALK
translocations are frequently seen in never or former light smokers. Co-mutation of TP53
and RB1 is predominantly seen in SCLC, not adenocarcinoma.

Question 14.3 The answer is B.


The right upper lobe mass is suspicious for malignancy, however, a tissue diagnosis is
needed. Since the location of the mass is peripheral, the best way to approach it would be
through CT-guided biopsy, rather than bronchoscopy and biopsy. Once a diagnosis of lung
cancer is established, staging studies, such as CT of the abdomen, MRI of the brain, PET,
or bone scan may be performed. If the diagnosis is NSCLC, and there is no evidence of
distant metastases on staging studies, then he should be referred to a thoracic surgeon for
discussion regarding surgical interventions.

Question 14.4 The answer is D.


The seventh edition of the TNM staging system includes several changes to the T
descriptors and reclassification of malignant pleural nodules, pleural effusions and
pericardial effusions as M1 disease. He has stage IV disease due to the presence of
malignant pleural effusion and should receive treatment with platinum-based doublet
chemotherapy for metastatic disease. Surgery would be appropriate only for early-stage
disease, while chemoradiation would be the treatment of choice for locally advanced,
stage III disease, in patients with good PS and no significant weight loss.

Question 14.5 The answer is C.


Cisplatin, pemetrexed and carboplatin, paclitaxel, bevacizumab are possible combinations
to consider for patients with advanced NSCLC with nonsquamous histology. For patients
with squamous histology, carboplatin and paclitaxel is a reasonable treatment option. The
addition of erlotinib to chemotherapy does not appear to improve survival, compared to
chemotherapy alone, in patients with advanced NSCLC.

Question 14.6 The answer is D.


Several randomized studies have demonstrated that there is no survival advantage to
continuing platinum doublet therapy beyond four to six cycles. If patients experience
significant toxicities following upfront therapy, delaying further treatment till disease
progression is a reasonable option. Agents used in the second-line setting for the
treatment of NSCLC include docetaxel, erlotinib, and pemetrexed. Patients whose tumors
have an activating mutation in the EGFR–tyrosine kinase domain, have the most durable
benefit from erlotinib. Maintenance chemotherapy is an option for patients with good PS
and stable disease following front-line chemotherapy, who do not experience significant
treatment-related toxicities. Maintenance chemotherapy (either pemetrexed or
bevacizumab) has been shown to improve overall survival in patients with NSCLC.
Pemetrexed maintenance would be an appropriate option for this patient.

Question 14.7 The answer is D.


In a patient with limited-stage SCLC, the optimal treatment would comprise concurrent
chemotherapy and radiotherapy including cisplatin and etoposide, followed by
prophylactic cranial irradiation (PCI).

Question 14.8 The answer is B.


Adjuvant platinum-based chemotherapy has been shown to improve survival in patients
with stage II and III NSCLC who undergo surgical resection. Patients with stage I disease
do not benefit from chemotherapy and a subset analysis of the JBR-10 study suggested
that patients with tumor size greater than 4 cm may benefit from adjuvant chemotherapy.

Question 14.9 The answer is B.


Only 2% of patients with pulmonary carcinoid present with carcinoid syndrome.
Approximately 30% of patients are asymptomatic at presentation. Ninety percent of
pulmonary carcinoids are typical, while 10% are atypical. Surgery is the treatment of
choice for carcinoid tumors. There is no role for adjuvant chemotherapy, following
complete resection for carcinoid tumors.

Question 14.10 The answer is D.


Several prospective randomized clinical trials have shown better response rates and
progression-free survival in patients treated with EGFR TKIs, when activating mutations
in the tyrosine kinase domain of EGFR (exon 19 deletion or exon 21 point mutation
L858R) are present in their tumors. The lack of significant overall survival benefit in these
trials has been attributed to a high rate of crossover in the control arm (chemotherapy
arm). There is no benefit in combining TKIs with chemotherapy.
Question 14.11 The answer is C.
This patient has locally advanced NSCLC. In this setting, concurrent chemoradiation is
better than radiation therapy alone or sequential chemotherapy followed by radiation.
Chemotherapy alone would be indicated for palliation of metastatic disease and is not
appropriate in this setting.

Question 14.12 The answer is D.


Acquired crizotinib resistance in adenocarcinomas of the lung can be due to secondary
mutations in ALK, ALK amplifications, or activation of bypass signaling pathways through
EGFR, KIT etc. Treatment with a second-generation TKI such as ceritinib has been shown
to be active and safe in crizotinib resistant tumors, irrespective of the mechanism of
resistance. As of early 2015, there have been no clinical trials that have compared
ceritinib to chemotherapy in this setting. Treatments with chemotherapy or local ablative
therapies are also valid treatment options in this setting, depending on the tempo of
progression.

Question 14.13 The answer is B.


In patients with EGFR mutations, initial therapy with an EGFR TKI results in better
response rates and longer time to tumor progression compared with chemotherapy.
Erlotinib and afatinib are currently approved EGFR TKIs. Cetuximab and necitumumab
are EGFR directed monoclonal antibodies that have been studied in NSCLC. However,
they have no established role in the management of patients with EGFR mutated NSCLC
at the current time. Crizotinib is a TKI that is active against ALK, ROS1, and MET altered
tumors.

Question 14.14 The answer is B.


The frequency of ROS1 translocations in all patients with NSCLC is approximately 1% to
2%. Crizotinib has shown to be active in this subset of lung cancers with a response rate
of 72%.

Question 14.15 The answer is A.


The standard of care for locally advanced unresectable NSCLC is concurrent definitive
chemoradiation with cisplatin and etoposide. The role of consolidation docetaxel was
investigated in a phase III study conducted by the Hoosier Oncology Group (HOG), where
it was associated with increased toxicity and no survival improvement compared to
concurrent chemoradiation without consolidation therapy. The Southwest Oncology Group
(SWOG) conducted another study in which patients who completed chemoradiation and
did not experience progressive disease were randomized to receive gefitinib or placebo.
The gefitinib arm had a shorter survival than the placebo arm. Prophylactic cranial
radiation decreases the incidence of brain metastases in patients with stage III disease, but
does not improve overall survival. At present, there is no sufficient evidence to
recommend PCI in patients with stage III disease.
Question 14.16 The answer is D.
This patient has a pancoast tumor of the lung. Based on data from multiple phase II
studies, the preferred management approach in these patients is trimodality therapy with
concurrent chemoradiation (typically with a platinum doublet such as cisplatin-etoposide)
followed by surgical resection. R0 (complete) resection and pathologic complete response
rates as high as 90% and 50%, respectively, have been reported with this approach
(Intergroup trial 0160). Curative intent concurrent chemoradiation alone is recommended
in patients not deemed to be surgical candidates. Radiotherapy for symptom relief is an
option in patients treated with a palliative intent.

Question 14.17 The answer is B


Genomic characterization of SCLCs has consistently revealed co-alteration of TP53 and
RB1 across several different studies. Recurrent EGFR and ALK alterations are more
prevalent among patients with adenocarcinoma of the lung, particularly never or former
smokers. KRAS mutations are present in nearly a third of all lung adenocarcinomas, and
DDR2 mutations in roughly 4% of squamous cell lung cancers.

Question 14.18 The answer is B.


The Lung Adjuvant Cisplatin Evaluation (LACE) meta-analysis indicated an absolute 5-
year overall survival benefit of 5% with adjuvant cisplatin based chemotherapy regimens.
The analysis included approximately 4,500 patients treated on five large clinical trials.
Significant survival benefit was seen in patients with stage II and III NSCLC.

Question 14.19 The answer is A.


Epithelial is the most common subtype of malignant mesotheliomas, comprising
approximately 50% to 60% of cases. Biphasic and sarcomatoid mesotheliomas are less
frequent. Uncommon tumors that cannot be categorized morphologically are considered
to be poorly differentiated.

Question 14.20 The answer is D.


Mesothelioma cells are diffusely positive for pankeratin, keratin 5/6, calretinin, and WT1.
CEA, a nonspecific marker, Moc-31, which is commonly seen in adenocarcinomas, and
TTF-1, which is seen in primary tumors of the lung and thyroid, are negative in
mesotheliomas.

Question 14.21 The answer is A.


The established poor prognostic factors in malignant mesothelioma are poor performance
status, male gender, anemia, thrombocytosis, leukocytosis, elevated LDH, and
nonepithelial histology including sarcomatoid and biphasic subtypes.

Question 14.22 The answer is D.


Locally advanced and technically unresectable tumor (T4) is defined by a tumor involving
all the ipsilateral pleural surfaces and extension of the tumor to the peritoneum,
contralateral pleura, mediastinal organs, spine, chest wall, or internal surface of the
pericardium with or without effusion. Involvement of the endothoracic fascia describes a
locally advanced but potentially resectable disease.

Question 14.23 The answer is D.


Mutations in the BRCA associated protein 1 (BAP1) tumor suppressor gene have been
described in nearly 20% of mesotheliomas. These mutations are more frequent among
smokers. Germline mutations in BAP1 have also been reported in families with a high
incidence of mesotheliomas.

Question 14.24 The answer is C.


The incidence of peritoneal mesothelioma is increasing and it currently accounts for 25%
to 33% of all mesotheliomas. This disease has a median age at presentation of 60 years
and is more common in men with a ratio of three to one. There is a clear relationship
between peritoneal mesothelioma and heavy exposure to airborne asbestos fibers, and
approximately 50% of patients have pleural plaques.

Question 14.25 The answer is D.


The standard therapy for mesothelioma is the combination of cisplatin and pemetrexed,
which is associated with increased overall survival compared to cisplatin alone (12.1 vs.
9.3 months). Patients that are unlikely to tolerate cisplatin may be treated with
carboplatin while gemcitabine regimens may be used in patients unable to be treated with
pemetrexed.

Corresponding chapters in Cancer: Principles & Practice of Oncology, Tenth Edition: 40 (Molecular Biology of Lung Cancer), 41
(Non–Small Cell Lung Cancer), 42 (Small Cell and Neuroendocrine Tumors of the Lung), and 114 (Benign and Malignant
Mesothelioma).
15 Neoplasms of the Mediastinum
Gregory P. Kalemkerian

QUESTIONS
Each of the numbered items below is followed by lettered answers. Select the ONE lettered answer
that is BEST in each case unless instructed otherwise.

Question 15.1 A 46-year-old woman presents with dull anterior chest pain. She is a lifelong
nonsmoker. Her medical history is significant for Hashimoto thyroiditis, for which she has
been taking levothyroxine for 1 year. Examination is unremarkable. Chest radiograph reveals
clear lung fields with a retrosternal density. Computed tomography (CT) scan shows a 4-cm,
smooth, anterior mediastinal mass. Laboratory studies show normal blood counts, blood
chemistry, lactate dehydrogenase (LDH), α-fetoprotein (AFP), and β-human chorionic
gonadotropin (β-hCG). What is the most likely diagnosis?
A. Small cell lung cancer (SCLC)
B. Thymoma
C. Pericardial cyst
D. Nonseminomatous germ cell tumor (NSGCT)

Question 15.2 Which of the following statements regarding thymoma is TRUE?


A. The incidence of thymoma is higher in women than in men.
B. Thymoma is most commonly seen in people under 30 years old.
C. The most common site of metastatic spread for thymoma is bone.
D. Complete surgical resection is the most important predictor of long-term survival for
patients with thymoma.

Question 15.3 Which of the following statements regarding myasthenia gravis are TRUE?
(Select three correct responses).
A. Myasthenia gravis occurs in 30% to 50% of people with thymoma.
B. Ocular symptoms are the most common initial manifestation of myasthenia gravis.
C. Myasthenia gravis is caused by autoantibodies against presynaptic muscarinic
acetylcholine receptors.
D. Thymoma is diagnosed in 15% of people with myasthenia gravis.

Question 15.4 A 32-year-old man presents with an anterior mediastinal mass that was
identified incidentally on a chest radiograph done as part of an employment examination. He
is asymptomatic and a lifelong nonsmoker, with no significant medical history. Examination
is unremarkable. CT scan shows a 4-cm, smooth, lobulated, anterior mediastinal mass without
local invasion. Resection of the mass reveals an encapsulated, lymphocyte-rich thymoma
(World Health Organization [WHO] type B1) with no capsular invasion. He recovers from
surgery without complications. What is the most appropriate next step in the management of
this patient?
A. Clinical surveillance
B. Adjuvant radiotherapy alone
C. Adjuvant chemotherapy alone
D. Adjuvant chemotherapy plus radiotherapy

Question 15.5 Which of the following are the most appropriate systemic therapy options for
the treatment of advanced, stage IV thymoma? (Select two correct responses)
A. Gemcitabine and docetaxel
B. Cyclophosphamide, doxorubicin, and cisplatin
C. Cisplatin plus etoposide
D. Carboplatin plus paclitaxel

Question 15.6 Which of the following paraneoplastic syndromes are associated with
thymoma? Select all that apply.
A. Myasthenia gravis and pure red cell aplasia
B. Subacute cerebellar degeneration and gynecomastia
C. Polymyositis and hypothyroidism
D. Opsoclonus/myoclonus syndrome and Pel–Ebstein fever

Question 15.7 Which of the following molecularly targeted therapies has meaningful clinical
activity in advanced thymoma?
A. Octreotide, a somatostatin analog
B. Erlotinib, an EGFR inhibitor
C. Imatinib, a c-kit inhibitor
D. Sorafenib, a multitargeted kinase inhibitor

Question 15.8 A previously healthy 51-year-old man presents with facial and bilateral upper
extremity edema that has progressed over the past 2 weeks. Examination reveals moderate
facial, cervical, and bilateral upper extremity edema and prominent anterior chest wall
vasculature. He is tachycardic, but his heart sounds are regular and his lungs are clear. There
is no lower extremity edema. CT scan of the chest shows a large anterior mediastinal mass
encasing and narrowing the superior vena cava, and invading the pericardium and upper lobe
of the left lung. There are numerous dilated, intrathoracic collateral vessels. An experienced
thoracic surgeon deems that the lesion is primarily unresectable. Mediastinotomy with biopsy
of the mass reveals well-differentiated thymic carcinoma (WHO type B3). PET scan shows the
large FDG-avid mediastinal mass with no evidence of metastatic disease. What is the most
appropriate management of this patient?
A. Palliative radiotherapy followed by chemotherapy
B. Definitive radiotherapy with concurrent chemotherapy
C. Neoadjuvant chemotherapy followed by surgical resection and postoperative
radiotherapy
D. Palliative chemotherapy alone

Question 15.9 Which of the following histologic subtypes is associated with a more
favorable outcome in patients with thymic carcinoma?
A. Clear cell carcinoma
B. Well-differentiated squamous cell carcinoma
C. Sarcomatoid differentiation
D. Small cell carcinoma

Question 15.10 Thymic carcinoid is associated with which genetic predisposition syndrome?
A. Multiple endocrine neoplasia type 1 (MEN 1)
B. Louis–Bar syndrome
C. Li–Fraumeni syndrome
D. Cowden syndrome

Question 15.11 A 21-year-old man presents with anterior chest pain and cough. Physical
examination is unremarkable and his performance status is excellent. Chest radiography
shows widening of the superior mediastinum. CT scan shows a 6-cm anterior mediastinal
mass encasing the trachea and great vessels and three 1-cm pulmonary nodules. CT scan of
the abdomen and pelvis and a testicular ultrasound examination are normal. Laboratory
studies reveal normal blood counts and blood chemistry, LDH of 880 IU/L (normal, 120 to
240 IU/L), AFP of 1,800 ng/mL (normal, <8 ng/mL), and β-hCG of 1.2 mIU/mL (normal,
<5 mIU/mL). What is the most likely diagnosis?
A. Hodgkin lymphoma
B. Benign teratoma
C. Nonseminomatous germ cell tumor (NSGCT)
D. Seminoma

Question 15.12 Which of the following statements is TRUE regarding mediastinal germ cell
tumors?
A. The incidence of malignant mediastinal germ cell tumors is the same in men and
women.
B. Seminoma is the most common mediastinal germ cell tumor.
C. An elevated serum AFP in a patient with biopsy-proven seminoma indicates the
presence of a nonseminomatous component.
D. Mediastinal NSGCTs are associated with better overall survival than testicular NSGCTs.

Question 15.13 A 33-year-old woman presents with a cough and dyspnea on moderate
exertion. Physical examination shows an anxious woman with a pulse of 110 beats/min, a
respiratory rate of 24 breaths/min, and mild stridor. Lungs are otherwise clear to
auscultation, and heart sounds are regular. Chest radiography shows a large anterior
mediastinal mass with narrowing of the midtrachea. CT scan shows a 10-cm, heterogeneous,
anterior mediastinal mass with foci of dense calcification that compresses the trachea and
narrows, but does not obstruct, the superior vena cava. Laboratory studies reveal normal
blood counts, blood chemistry, LDH, carcinoembryonic antigen (CEA), AFP, and β-hCG. She
undergoes complete resection of the mass. Pathologic evaluation reveals a multicystic mass
with foci of mature gland formation, respiratory epithelium, cartilage, and bone. There is no
invasion into adjacent structures, and surgical margins are negative. What is the most
appropriate management of this patient?
A. Clinical surveillance
B. Adjuvant radiotherapy
C. Cisplatin plus etoposide × 4 cycles
D. Doxorubicin plus ifosfamide × 6 cycles

Question 15.14 A 23-year-old man presents with hoarseness, cough, and anterior chest pain.
He has a 10 pack-year smoking history. Physical examination is normal. Chest radiography
shows a large left mediastinal mass and clear lung fields. CT scan of the chest shows a 5-cm
irregular left paratracheal mass. Serum CEA, β-hCG, and AFP are normal. Left
mediastinotomy with biopsy of the anterior mediastinal mass reveals a poorly differentiated
malignant neoplasm that is immunohistochemically negative for leukocyte common antigen,
vimentin, S100, TTF1 and chromogranin, but positive for low–molecular-weight cytokeratin.
Genetic studies reveal no B- or T-cell rearrangements, and karyotypic analysis shows
aneuploidy and isochromosome 12p. Which of the following is the most appropriate therapy
for this patient?
A. Cisplatin plus etoposide
B. Cisplatin, etoposide, and bleomycin (BEP)
C. Concurrent chemotherapy and radiation therapy
D. Cyclophosphamide, doxorubicin, vincristine, and prednisone (CHOP)

Question 15.15 A previously healthy 28-year-old man presents with fatigue and vague chest
discomfort. A chest radiograph shows a widened mediastinum, and a CT scan confirms a 9-cm
anterior mediastinal mass with focal hemorrhage that fills the substernal space and invades
into the upper lobe of the left lung. Laboratory studies show LDH of 850 IU/L (normal, 120
to 240 IU/L), AFP of 8,700 ng/mL (normal, <8 ng/mL), and β-hCG of 220 mIU/mL (normal,
<5 mIU/mL). Biopsy confirms embryonal carcinoma with elements of choriocarcinoma.
After four cycles of cisplatin, etoposide, and bleomycin (BEP), a CT scan shows marked
shrinkage of the mediastinal mass, now 3 cm in maximal diameter. One month after
completion of therapy, LDH is 150 IU/L, AFP is 5 ng/mL, and β-hCG is 2.4 mIU/mL. Which
of the following is the most appropriate management of this patient?
A. Clinical surveillance
B. Cisplatin, ifosfamide, and vinblastine (VIP)
C. Two additional cycles of cisplatin, etoposide, and bleomycin (BEP)
D. Resection of the residual mediastinal mass

Question 15.16 A 22-year-old man has a retrosternal mass identified on a chest radiograph
done during his enlistment into military service. He is an asymptomatic nonsmoker with no
significant medical history. CT scan shows a 2.5-cm smooth anterior mediastinal mass
without local invasion. Serum LDH, β-hCG, and AFP are normal. Testicular ultrasound is
normal. He refuses primary surgical resection. A percutaneous core biopsy shows pure
seminoma. Which of the following is the most appropriate management of this patient?
A. Surgical resection
B. Radiotherapy
C. Cisplatin, etoposide, and bleomycin (BEP)
D. Cisplatin plus etoposide followed by surgical resection

Question 15.17 Mediastinal nonseminomatous germ cell tumors are associated with which
of the following? (Select two correct responses)
A. Myasthenia gravis
B. Acute megakaryocytic leukemia
C. Klinefelter syndrome
D. Thymic carcinoid

Question 15.18 A 30-year-old man presents with chest tightness and shortness of breath. A
chest radiograph shows a large mediastinal mass. CT scan shows a 9-cm, lobulated, anterior
mediastinal mass invading the left lung and the pericardium with a small pericardial effusion,
a 3-cm right paratracheal lymph node, and a 5-cm subcarinal lymph node. Echocardiogram
shows no tamponade. Serum LDH is 440 IU/L (normal, 120 to 240 IU/L), but AFP and β-hCG
are normal. Testicular ultrasound is normal. Bronchoscopic core biopsy of the subcarinal mass
reveals seminoma. Which of the following is the most appropriate initial management of this
patient?
A. Carboplatin plus etoposide
B. Radiotherapy
C. Surgical resection
D. Cisplatin plus etoposide

Question 15.19 A previously healthy 45-year-old man presents with hoarseness and vague
chest discomfort. A chest radiograph shows a widened mediastinum, and a CT scan confirms a
large anterior mediastinal mass encasing the trachea and abutting the superior vena cava, left
pleura, and superior pericardium. Serum AFP is normal, but β-hCG is 10 mIU/mL (normal,
<5 mIU/mL). Biopsy shows seminoma. He is treated with cisplatin, etoposide, and
bleomycin (BEP). CT scan after treatment shows marked shrinkage of the mediastinal mass,
now 2 cm in maximal diameter. β-hCG is 1.2 mIU/mL. Which of the following is the most
appropriate management of this patient?
A. Clinical surveillance
B. Cisplatin, ifosfamide, and vinblastine (VIP)
C. Resection of the residual mediastinal mass
D. Involved-field radiotherapy

Question 15.20 Which of the following is TRUE for mediastinal germ cell tumors according
to the International Germ Cell Consensus Classification (IGCCC)?
A. All pure mediastinal seminomas are good risk
B. All mediastinal nonseminomatous germ cell tumors are poor risk
C. Pure mediastinal seminomas with nonpulmonary visceral metastases are poor risk
D. Mediastinal nonseminomatous germ cell tumors without visceral metastases are
intermediate risk
ANSWERS

Question 15.1 The answer is B.


This middle-aged woman has a localized anterior mediastinal mass with mild symptoms.
Thymoma is the most common tumor arising in the anterior mediastinum. Approximately
one-half of patients are symptomatic because of compression of mediastinal structures
(e.g., pain, cough, superior vena cava syndrome) or associated paraneoplastic syndromes.
Paraneoplastic syndromes can be identified in up to 70% of patients with thymoma, but
are uncommon in patients with thymic carcinoma. Patients with small cell lung cancer are
typically very symptomatic with a central lung mass and bulky mediastinal
lymphadenopathy, and nearly all have a history of tobacco use. Pericardial cysts occur
primarily in the middle mediastinum and are usually asymptomatic. NSGCTs occur
predominantly in young men, and 80% to 85% are associated with elevations of AFP
and/or β-hCG.

Question 15.2 The answer is D.


The incidence of thymoma is slightly greater in men than in women. Although thymoma
can occur in people of all ages, the incidence does increase in the elderly. The most
common site of metastasis is the pleura through direct invasion of the pleural space by the
primary anterior mediastinal mass resulting in “drop metastases.” Thymoma can
metastasize to regional lymph nodes or to distant sites, including lung nodules, but
hematogenous metastases tend to occur very late in the course of the disease. The best
predictor of long-term prognosis for people with thymoma is the completeness of surgical
resection. Other prognostic factors include histologic grade (defined by the World Health
Organization [WHO] classification system) and anatomic stage of disease (defined by the
TNM or Masaoka staging systems). The presence of a paraneoplastic syndrome, such as
myasthenia gravis, may be associated with a better prognosis due to earlier diagnosis of
thymoma.

Question 15.3 The answers are A, B, and D.


Myasthenia gravis is the most common paraneoplastic syndrome associated with
thymoma, occurring in 30% to 50% of patients. The most common initial manifestation of
myasthenia is ocular symptoms, such as diplopia, which progress on to generalized muscle
weakness in 80% of patients. Myasthenia gravis is caused by the production of
autoantibodies targeting the postsynaptic nicotinic acetylcholine receptor within the
myoneural junction. Although pathologic thymic changes, mainly lymphoid hyperplasia,
are seen in most patients with myasthenia gravis, only 15% have actual thymoma. In
patients with thymoma, thymectomy usually results in partial improvement in myasthenic
symptoms, though most continue to require treatment with immunosuppressive drugs
and/or anticholinesterase-mimetic agents (e.g., pyridostigmine).

Question 15.4 The answer is A.


This young man has undergone complete resection of a noninvasive, low-grade thymoma.
Several histologic classification schemes and staging systems have been developed for
thymic neoplasms. The WHO histologic classification is most commonly used and includes
both thymomas (types A, AB, B1, and B2) and thymic carcinomas (types B3 and C) with
varying degrees of pathologic atypia and aggressiveness. The most commonly used
staging system was developed by Masaoka: stage I, no microscopic capsular invasion
(noninvasive); stage II, microscopic capsular invasion or gross invasion into perithymic
fat; stage III, gross invasion into adjacent organs; stage IVa, pleural or pericardial
dissemination; and stage IVb, lymphogenous or hematogenous metastases. Complete
resection of Masaoka stage I, low-grade thymoma is associated with 5- and 10-year
overall survival rates of 90% to 100% and 70% to 80%, respectively. Neither adjuvant
radiotherapy nor chemotherapy has been shown to benefit patients who have undergone
complete resection of Masaoka stage I, or noninvasive, thymoma.

Question 15.5 The answers are B and C.


Systemic treatment for stage IV thymoma is given with palliative intent, with the goals of
shrinking tumor, improving symptoms, and prolonging survival. Due to the rarity of
metastatic thymoma, few randomized trials have been performed. However, data from
phase II studies suggest that the combinations of cyclophosphamide, doxorubicin, and
cisplatin (CAP) and cisplatin plus etoposide (PE) yield similar favorable outcomes, with
response rates of 50% to 64% and median overall survivals of 38 to 52 months. In a
single-arm, phase II study, the combination of carboplatin plus paclitaxel resulted in an
apparently less favorable response rate of 33%. Most other regimens, such as gemcitabine
plus docetaxel, have not been adequately studied.

Question 15.6 The answers are A and C.


A wide variety of paraneoplastic syndromes, primarily due to autoimmune dysregulation,
have been associated with thymoma. These include connective tissue disorders
(polymyositis, systemic lupus erythematosus, Sjogren syndrome, scleroderma), endocrine
disorders (Addison disease, hypothyroidism, panhypopituitarism), neuromuscular
syndromes (myasthenia gravis, Eaton–Lambert myasthenic syndrome, myotonic
dystrophy), hematologic disorders (pure red cell aplasia, hypogammaglobinemia, T-cell
deficiency syndrome, amegakaryocytic thrombocytopenia), and miscellaneous disorders
(hypertrophic pulmonary osteoarthropathy, nephrotic syndrome, paraneoplastic
pemphigus). Subacute cerebellar degeneration is associated with small cell lung cancer
(SCLC), while opsoclonus/myoclonus syndrome occurs primarily with both SCLC and
neuroblastoma. Gynecomastia is associated with HCG secretion by germ cell tumors. Pel–
Ebstein fever is a classic sign of Hodgkin disease.

Question 15.7 The answer is A.


A growing number of potentially oncogenic molecular abnormalities have been identified
in both thymoma and thymic carcinoma. The most promising clinical results for
molecularly targeted agents in thymoma come from a phase II study of the somatostatin
analog, octreotide, that reported an objective response rate of 13% when given alone and
32% when used in conjunction with prednisone in patients with tumors that were positive
on octreotide scan. Despite the expression of EGFR in many thymic tumors, trials of the
EGFR inhibitors gefitinib and erlotinib have reported minimal clinical activity. Imatinib
may have activity in the small subset of patients with thymic carcinoma whose tumors
harbor mutations of c-kit, though clinical trials have been largely disappointing. Other
multikinase inhibitors, such as sunitinib or dasatinib, are currently undergoing clinical
study in patients with thymic malignancy.

Question 15.8 The answer is C.


This middle-aged man presents with superior vena cava syndrome caused by a locally
advanced, primarily unresectable thymic carcinoma. Symptoms caused by compression or
invasion of mediastinal structures, including anterior chest pain, cough, dyspnea, and
superior vena cava syndrome, occur in up to 70% of patients with thymic carcinoma,
whereas paraneoplastic syndromes are rare. Because completeness of resection is the most
important predictor of outcome, multimodality therapy should be recommended for all
patients with thymic neoplasms, both thymoma and thymic carcinoma, whose tumors are
judged to be primarily unresectable because of the extent of local invasion. The most
commonly studied combined modality approach is neoadjuvant cisplatin-based
chemotherapy followed by surgical resection and postoperative radiotherapy. Cumulative
results of studies including patients with both thymoma and thymic carcinoma reveal that
neoadjuvant chemotherapy results in an objective response rate of 89% and a complete
response rate of 31%. Long-term survival remains the goal of therapy for patients with
locally advanced disease, with one trial noting a 7-year disease-free survival rate of 83%
after trimodality therapy. Therefore, purely palliative approaches are only justifiable in
patients whose performance status or comorbidities preclude potentially curative
combined modality therapy. Although there are some data supporting the use of definitive
chemotherapy plus radiotherapy without surgical resection, further studies are needed to
define the long-term utility of this approach.

Question 15.9 The answer is B.


In patients with thymic carcinoma, improved survival has been correlated with the
following histologic features: negative margins/complete resection, encapsulated tumor,
lobular growth pattern, low mitotic activity, and low histologic grade. Low-grade tumors
include well-differentiated squamous cell carcinoma, well-differentiated mucoepidermoid
carcinoma, and basaloid carcinoma. High-grade tumors include small cell carcinoma,
poorly differentiated lymphoepithelioma-like carcinoma, sarcomatoid carcinoma,
anaplastic carcinoma, and clear cell carcinoma.

Question 15.10 The answer is A.


Thymic carcinoid is a rare tumor that is associated with MEN I. Thymic carcinoids rarely
cause serotonin-mediated carcinoid syndrome, but can produce adrenocorticotropic
hormone (ACTH), leading to Cushing syndrome. The primary treatment of thymic
carcinoid tumors is complete surgical resection, which results in a 5-year survival rate of
60%, although late recurrences and mortality are not uncommon. Adjuvant radiotherapy
is reserved for patients with incomplete resections, and the role of adjuvant systemic
therapy is unclear.

Question 15.11 The answer is C.


This young man presents with an anterior mediastinal mass and an elevated AFP, a
combination of findings that is pathognomonic for NSGCT. Approximately 10% of germ
cell tumors originate in extragonadal sites, with 50% to 70% of these arising in the
anterior mediastinum. Mediastinal germ cell tumors account for 10% to 15% of all
mediastinal tumors in adults and 6% in children. The serum tumor markers LDH, AFP, and
β-hCG, are helpful in the diagnosis and management of germ cell tumors. Benign
teratomas, the most common mediastinal germ cell tumors (60% to 70%), are not
associated with significant tumor marker elevations. Pure seminoma can cause modest
elevations of β-hCG, but not AFP. AFP is produced by yolk sac elements present only in
NSGCTs. Although serum LDH levels are commonly elevated in Hodgkin and non-Hodgkin
lymphoma, AFP levels are not affected.

Question 15.12 The answer is C.


Although benign teratoma occurs with equal frequency in men and women, approximately
90% of malignant germ cell tumors occur in men. Benign teratoma is the most common
mediastinal germ cell tumor, accounting for 60% of such tumors in adults. Mediastinal
NSGCT is more common that primary pure mediastinal seminoma, which accounts for
about 35% of malignant mediastinal germ cell tumors. Patients with pure seminoma may
have modest elevations of serum β-hCG, but an elevated serum AFP indicates the presence
of a NSGCT component, even in a patient with biopsy-proven seminoma. The overall 5-
year survival rate of patients with mediastinal NSGCT is about 50%, which is significantly
worse than the prognosis of those with testicular NSGCT.

Question 15.13 The answer is A.


This woman has undergone complete resection of a benign, noninvasive teratoma, the
most common mediastinal germ cell tumor. Unlike malignant germ cell tumors, benign
teratoma occurs with equal incidence in both men and women, usually 20 to 40 years of
age. Benign teratoma typically presents as a large anterior mediastinal mass with normal
serum tumor markers and symptoms that are due to compression of mediastinal
structures. Although they may be adherent to adjacent structures, they are typically
noninvasive. Histologically, both benign and malignant teratomas contain elements from
all three germ cell layers: ectoderm (skin, hair, sweat glands, teeth); mesoderm (fat,
muscle, bone, cartilage); and endoderm (bronchial or intestinal epithelium). Malignant
teratoma, in which one or more of these elements have an invasive or metastatic
phenotype, is an aggressive tumor that responds poorly to therapy and is associated with
an unfavorable prognosis. In contrast, surgical resection of benign teratoma results in an
excellent long-term survival rate with a low risk of recurrence. Neither adjuvant
radiotherapy nor chemotherapy is indicated in the management of patients with benign
teratoma.

Question 15.14 The answer is B.


This young man presents with a mediastinal, poorly differentiated carcinoma of unknown
primary site that is clinically and molecularly consistent with an extragonadal germ cell
tumor. Poorly differentiated carcinoma arising in the midline structures of a young man is
highly suggestive of an unrecognized extragonadal germ cell tumor, even in the face of
normal tumor markers. In this patient, immunohistochemical analysis is consistent with
carcinoma (positive cytokeratin), and argues against lymphoma (common leukocyte
antigen), neuroendocrine tumor (chromogranin), sarcoma (vimentin), lung cancer (TTF1),
and melanoma (S100). Genetic analysis can frequently be useful in clarifying histogenesis
in such situations. Isochromosome 12p (i[12p]) is pathognomonic of germ cell tumors and
is found in approximately 30% of patients in this setting. A long-term survival rate of 16%
has been reported for patients with midline, poorly differentiated carcinoma of unknown
primary site treated with cisplatin-based chemotherapy appropriate for NSGCT. The
presence of i(12p) has been favorably associated with response and survival. Because the
International Germ Cell Cancer Collaborative Group considers mediastinal NSGCT a poor-
risk tumor, the treatment should consist of four cycles of cisplatin, etoposide, and
bleomycin (BEP). Treatment aimed at good-risk NSGCT (answer A), stage III NSCLC
(answer C), or lymphoma (answer D) is not appropriate.

Question 15.15 The answer is D.


This young man has had an excellent response to initial chemotherapy for mediastinal
NSGCT with normalization of tumor markers and a small residual mediastinal mass.
Mediastinal NSGSTs commonly arise in the anterior compartment, and most patients are
symptomatic at diagnosis. Elevations of β-hCG and AFP are found in 30% to 50% and 60%
to 80% of patients, respectively. NSGCTs frequently consist of a mixture of histologic
subtypes, including choriocarcinoma, embryonal carcinoma, endodermal sinus (yolk sac)
elements, and teratoma. Primary treatment of mediastinal NSGCTs is cisplatin, etoposide,
and bleomycin, as per the guidelines for poor-risk nonseminomatous testicular cancer.
Such treatment yields a complete response rate of 40% to 64% and a long-term survival
rate of 45%. Surgical resection is recommended for patients with residual masses after
completion of initial chemotherapy and normalization of tumor markers. If viable, NSGCT
is identified in the resected specimen, then two further cycles of cisplatin-based therapy
are indicated. If the residual mass reveals only mature teratoma or necrosis, then clinical
surveillance without further treatment is indicated. The role of surgical resection in
patients with incomplete tumor marker responses has not been clearly defined, but such
patients would typically proceed to salvage chemotherapy with a regimen such as VIP.

Question 15.16 The answer is B.


This young man presents with a localized mediastinal seminoma. Pure mediastinal
seminoma is a rare disease that occurs mainly in men 20 to 40 years of age and accounts
for 35% of malignant mediastinal germ cell tumors. Despite a slow growth rate,
mediastinal seminoma tends to present as a large tumor that causes symptoms due to
compression of mediastinal structures. Serum β-hCG and LDH levels may be modestly
elevated in patients with pure seminoma, but an elevated AFP level would be indicative
of a nonseminomatous germ cell component. Radiotherapy is the treatment of choice for
localized mediastinal seminoma, yielding long-term survival rates of 60% to 80%. The
total radiation dose should be ≥45 Gy, and the field should include the mediastinum and
bilateral supraclavicular fossae. Although surgery followed by adjuvant therapy also
yields high long-term survival rates, resection is not generally indicated in the
management of mediastinal seminoma. Because of the exquisite sensitivity of seminoma
to radiotherapy, the relatively low rates of recurrence after radiotherapy and the high
cure rates associated with salvage chemotherapy, systemic treatment is not recommended
for patients with localized disease.

Question 15.17 The answers are B and C.


A number of hematologic malignancies occur in association with mediastinal NSGCT,
including acute megakaryocytic leukemia, acute myeloid leukemia, erythroleukemia,
malignant histiocytosis, and myelodysplastic syndrome. These may be diagnosed either
before or at the same time as the NSGCT. In addition, mediastinal NSGCTs can develop
malignant, nongerm cell components, such as adenocarcinoma, squamous cell carcinoma,
or sarcoma. Klinefelter syndrome (47XXY) is found in up to 20% of patients with
mediastinal NSGSTs. Myasthenia gravis is associated with thymoma, not NSGCT. Thymic
carcinoid is associated with MEN1 syndrome.

Question 15.18 The answer is D.


This man presents with locally advanced, mediastinal seminoma that invades adjacent
structures and involves regional lymph nodes. More than half of patients with mediastinal
seminoma will have metastases at diagnosis, primarily to the intrathoracic lymph nodes,
lung, or bone. Patients with locally advanced disease are usually treated with cisplatin-
based chemotherapy with or without radiotherapy, whereas those with distant disease are
treated with chemotherapy alone. Cisplatin-based treatment results in a response rate that
approaches 95% and a complete response rate of up to 85%, with a long-term disease-free
survival rate of 80% to 85%. Carboplatin-based therapy is known to be inferior to
cisplatin-based regimens. For patients with bulky, locally advanced mediastinal
seminoma, the large radiation fields required to encompass all disease can result in
excessive pulmonary or cardiac toxicity. In addition, despite high rates of local control,
20% to 40% of patients with locally advanced disease who are treated with radiotherapy
alone will develop distant metastases, leading to the recommendation for primary
systemic chemotherapy in such patients. Surgery is not indicated in the primary treatment
of mediastinal seminoma.

Question 15.19 The answer is A.


This man has had an excellent response to initial chemotherapy for locally advanced
mediastinal seminoma with normalization of β-hCG and a small residual mediastinal mass.
The optimal management of patients with a residual mass after initial chemotherapy or
radiotherapy for seminoma remains controversial. Most studies have demonstrated that
viable seminoma is rarely found in resected residual masses and that about 85% of such
specimens consist entirely of scar tissue. Other studies have shown that up to 25% of
residual masses more than 3 cm in size contain viable seminoma. Nevertheless, resection
of a residual mass is not recommended unless the mass enlarges during clinical
surveillance. Similarly, the addition of radiotherapy after chemotherapy has not been
shown to be of significant benefit. Salvage chemotherapy with a regimen such as VIP,
with or without aggressive local therapy, is indicated for recurrent disease.

Question 15.20 The answer is B.


All patients with mediastinal NSGCT are considered poor risk per the International Germ
Cell Consensus Classification (IGCCC) regardless of serum tumor marker levels, the extent
of local/regional disease, or the presence/absence of distant metastases. As such, all
patients with mediastinal NSGCT who require primary chemotherapy should receive four
cycles of cisplatin, etoposide, and bleomycin (BEP) or vinblastine, ifosfamide, and
cisplatin (VIP). For mediastinal seminoma, patients without nonpulmonary visceral
metastases are categorized as good risk, while those with nonpulmonary visceral
metastases are intermediate risk. Patients with good-risk disease who require
chemotherapy may receive either three cycles of cisplatin, etoposide, and bleomycin
(BEP) or four cycles of cisplatin plus etoposide, whereas those with intermediate-risk
disease should receive four cycles of cisplatin, etoposide, and bleomycin (BEP).

Corresponding chapter in Cancer: Principles & Practice of Oncology, Tenth Edition: 43 (Neoplasms of the Mediastinum).
16 Esophagus and Stomach Cancer
Hassan Hatoum and Renuka Iyer

QUESTIONS
Each of the numbered items below is followed by lettered answers. Select the ONE lettered answer
that is BEST in each case unless instructed otherwise.

Question 16.1 Which of the following statements about the incidence of esophageal cancer
is TRUE?
A. Esophageal cancer is relatively uncommon in the United States, and the lifetime risk of
being diagnosed with the disease is less than 1%.
B. Incidence rates among white men continue to increase and now exceed 8 per 100,000
person-years, and reflect the marked increase in the incidence of adenocarcinoma of the
esophagus of more than 400% in the past 2 decades.
C. Although the incidence of adenocarcinoma in white women (2 per 100,000) is lower
than that in white men, rates of adenocarcinoma have increased in women by more
than 300% during the past 20 years.
D. Only A and B
E. All of the above

Question 16.2 Which of the following statements are TRUE? (Select two correct responses)
A. Blood group B has been associated with gastric cancers.
B. Most of the patients with Helicobacter pylori infection will develop gastric cancer.
C. Epstein–Barr virus infection has been noted in a certain type of gastric carcinoma.
D. WHO classifies H. pylori infection as class I carcinogen.

Question 16.3 Which of the following are risk factors for development of esophageal
adenocarcinoma? (Select two correct responses)
A. Gastroesophageal reflux disease
B. Low BMI
C. Barrett esophagus
D. H. Pylori infection

Question 16.4 Which of the following is TRUE for cancer of the esophagus?
A. Adenocarcinoma has a better outcome compared to squamous cell carcinoma of the
esophagus.
B. HER2+ overexpression is an adverse prognostic factor similar to breast cancer.
C. The incidence of esophageal cancer is higher in African-American compared to
Caucasian men.
D. The lifetime risk of being diagnosed with esophageal cancer is 1%.

Question 16.5 Which of the following statement(s) about the molecular biology of
esophageal cancer is/are TRUE?
A. EGFR overexpression correlates with poor prognosis including poor response to
chemotherapy.
B. Presence of p53 point mutation correlates with response to induction chemotherapy and
predicts survival after esophagectomy.
C. Reduced expression of E-cadherin correlates with progression from Barrett esophagus to
dysplasia and finally to adenocarcinoma.
D. All of above

Question 16.6 Which of the following is TRUE about Barrett esophagus with high-grade
dysplasia?
A. Esophageal cancer incidence per year is 30%.
B. Esophagectomy is the only available treatment option.
C. Endoscopic mucosal resection can be used as a therapeutic option in this case.
D. Surveillance with annual endoscopy

Question 16.7 A 57-year-old Caucasian woman with long-standing reflux symptoms


presented with hematemesis to a local emergency department. Endoscopy revealed a distal
esophageal malignant ulcer with involvement of the gastroesophageal (GE) junction and 5 cm
of the gastric cardia. Biopsy revealed intestinal metaplasia at the GE junction and confirmed a
diagnosis of adenocarcinoma. CT shows thickened distal esophagus and proximal stomach,
hiatal hernia, and enlarged celiac nodes. Which statement most accurately describes her
cancer?
A. She has a Siewert type I cancer: adenocarcinoma of the distal esophagus, which usually
arises from an area with specialized intestinal metaplasia of the esophagus (i.e., Barrett
esophagus) and may infiltrate the esophagogastric junction from above.
B. She has a Siewert type II cancer: adenocarcinoma of the cardia, which arises from the
epithelium of the cardia or from short segments with intestinal metaplasia at the
esophagogastric junction.
C. She has a Siewert type III cancer: adenocarcinoma of the subcardial stomach, which
may infiltrate the esophagogastric junction or distal esophagus from below.

Question 16.8 A 48-year-old man with a long-standing history of GERD presented to the
clinic with dysphagia and weight loss. The patient had EGD with biopsies that showed a
friable circumferential lesion in the lower part of the esophagus. EUS revealed a cT3 lesion
with one regional lymph node. CT and PET scan for evaluation did not show any distant sites
of metastasis. Initial treatment with preoperative chemoradiation using weekly carboplatin
and paclitaxel was suggested for the patient. Which of the following is TRUE for this
treatment modality?
A. The rate of pathologic complete response is 5%.
B. Chemoradiation followed by surgery can be used only for localized esophageal
adenocarcinoma but not squamous cell carcinoma.
C. Chemoradiation followed by surgery improves PFS but not OS compared to surgery
alone.
D. Chemoradiation followed by surgery improves both PFS and OS compared to surgery
alone.

Question 16.9 A 54-year-old male has a 10-year history of untreated GERD. He presents to
his primary care physician with melena. The patient underwent EGD which revealed a mass
in the GEJ junction that is biopsy proven to be adenocarcinoma. EUS and PET-CT scan
showed a mass that is abutting the diaphragm, but no evidence of distant metastasis. The
patient has no comorbid conditions and has very good performance status. What of the
following responses are TRUE about using perioperative chemotherapy ECF? (Select two
correct responses)
A. The lesion is not resectable and hence only chemotherapy should be offered.
B. There was a 6-month improvement in progression-free survival, a 4-month
improvement in median survival.
C. There was 25% pathologic complete response seen.
D. The use of ECF in this setting did not increase the rate of curative resection though it
improves survival.

Question 16.10 A 49-year-old female with a long-standard history of GERD presents to her
gastroenterologist with worsening abdominal pain, weight loss, and heart burn. EGD with
biopsy shows a tumor in the lower third of the esophagus. Biopsy reveals high-grade
adenocarcinoma. EUS shows cT3 and one paraesophageal lymph node. CT scan and PET scan
done for staging shows multiple liver lesions consistent with metastasis. The patient has no
comorbidities and her performance status is 0. Which of the following is the most appropriate
next step?
A. Chemotherapy with cisplatin and 5FU
B. Docetaxel, cisplatin, and 5FU
C. Epirubicin, cisplatin, and 5FU
D. Check for HER2 overexpression on the esophageal tumor tissue

Question 16.11 In the question above, HER2 was overexpressed on the tumor tissue. Which
treatment do you offer?
A. Chemotherapy
B. Chemotherapy plus cetuximab
C. Chemotherapy plus trastuzumab
D. Supportive care

Question 16.12 Which of the following cases would be sent for genetic testing? (Select two
correct responses)
A. A patient with gastric cancer histopathologically confirmed as diffuse and older than 50
years with a first-degree relative with stomach cancer at the age of 55.
B. A patient with diffuse gastric cancer, who has an uncle with stomach cancer diagnosed
at the age of 45 years with an aunt with lobular breast cancer.
C. A 37-year-old man diagnosed with diffuse gastric cancer, and no family history of
gastric cancer.
D. A 62-year-old male smoker diagnosed with gastric cancer and history of nasopharyngeal
cancer.

Question 16.13 Which of the following syndrome is usually associated with intestinal type
of gastric cancer?
A. Lynch syndrome
B. Li–Fraumeni syndrome
C. Familial adenomatous polyposis syndrome
D. Peutz–Jeghers syndrome

Question 16.14 Which of the following gene mutations is associated with the highest gastric
cancer risks?
A. BRCA1/2
B. P53
C. CDH1
D. APC

Question 16.15 A 22-year-old woman is diagnosed with metastatic gastric cancer, with
diffuse involvement of the stomach and linitis plastica. Her father died of the same cancer at
an age of 42 years. Her mother is concerned about familial gastric cancer and is asking for
information about hereditary gastric cancer and appropriate screening for her other children.
Which of the following would be the most appropriate recommendation to this family?
A. Hereditary gastric cancer is rare and unlikely. She may just have some environmental
exposures or DNA mismatch repair gene mutations that cannot be screened for.
B. She may very well have hereditary early onset diffuse gastric cancer, but no
surveillance or workup is recommended because this has yet to be confirmed in larger
studies.
C. E-cadherin mutation testing should be considered here, and, in fact, prophylactic
gastrectomy should be considered strongly for her siblings if a germline E-cadherin
mutation is confirmed and mucosal abnormality can be documented by endoscopic
examination of the stomach.
D. E-cadherin mutation testing should be considered, and, in fact, prophylactic
gastrectomy should be considered strongly for her siblings if a germline E-cadherin
mutation is confirmed even if no mucosal abnormalities are seen by endoscopic
examination of the stomach.

Question 16.16 Which of the following is TRUE about the use of diagnostic modalities in
patients with gastric cancer?
A. PET scan is less useful in assessing metastasis of signet cell subtype.
B. The accuracy rate of CT scan for staging of advanced gastric cancer was 96%.
C. There is no role for laparoscopy if staging CT scan was negative.
D. Brain MRI should be routinely done as part of the initial work up.

Question 16.17 A 48-year-old male who was diagnosed with adenocarcinoma of the stomach
underwent partial gastrectomy. PET-CT prior to surgery shows no evidence of distant
disease. He had 1 of 18 lymph nodes removed involved with cancer. Margins were negative
for cancer. The pathologic stage is IIB (T3 N1 tumor). He was referred to you to discuss
adjuvant treatment options. He has an ECOG performance status of 1. What is the best
treatment modality?
A. Chemotherapy with Carboplatin and Paclitaxel
B. There is no evidence for benefit from adjuvant treatment in this case.
C. 5FU and leucovorin, followed by 45 Gy in 25 fractions plus concurrent 5FU and
leucovorin.
D. Check for HER2 over expression and if positive, treat with cisplatin/5FU plus
trastuzumab.

Question 16.18 A 48-year-old gentleman presents to the clinic with abdominal pain and
weight loss. CT scan of the abdomen and pelvis showed diffuse gastric wall thickening as
well as liver lesions suspicious for metastasis. CT-guided biopsy of one of the lesions
confirmed metastatic adenocarcinoma of the stomach. HER2 was not overexpressed on the
tumor tissue. Which of the following are TRUE regarding combination chemotherapy for the
treatment of metastatic gastric cancer? (Select two correct responses)
A. Cisplatin is superior to oxaliplatin with a better OS.
B. 5FU is superior to capecitabine with better OS.
C. Oxaliplatin is noninferior to cisplatin in terms of OS.
D. Capecitabine is noninferior to 5FU in terms of OS.

Question 16.19 57-year-old man presented to your clinic for a second opinion on treatment
of metastatic HER2 negative gastric cancer. The patient has received first-line treatment with
epirubicin/cisplatin/5FU. On his last evaluation, his CT scan of the chest, abdomen, and
pelvis showed progression of the disease in the lungs and liver. The patient remains
asymptomatic except for fatigue. His weight is stable and his performance status is 1. Blood
work included CBC that showed mild anemia and his chemistry panel including liver function
test and bilirubin was normal. Which of the following is the best recommendation for this
patient?
A. Refer to hospice
B. Erlotinib single agent
C. Ramucirumab single agent
D. Trastuzumab/cisplatin/capecitabine

Question 16.20 Which of the following is TRUE about ramucirumab?


A. It is used only in combination with chemotherapy as a second-line for advanced gastric
cancer.
B. It prolongs only progression-free survival but not overall survival.
C. It prolongs overall survival when used as a single agent in the second line.
D. It is a murine monoclonal antibody targeting VEGF.
ANSWERS

Question 16.1 The answer is E.


Esophageal cancer is relatively uncommon in the United States and the lifetime risk of
being diagnosed with the disease remains less than 1%. Incidence rates among Caucasian
men increased up until the year 2000, reflecting the marked increase in the incidence of
adenocarcinoma of the esophagus of more than 400% in the past two decades, but now
have stabilized between 7 and 8 per 100,000 person-years. Although the incidence of
esophageal cancer in Caucasian females (1.6 per 100,000) is lower than that in Caucasian
males, rates of adenocarcinoma have increased in white women by more than 300%
during the past 20 years.

Question 16.2 The answers are C and D.


As a commensal organism, H. pylori infection is widely prevalent throughout the world.
Despite its classification by the World Health Organization as a class I carcinogen,
infection with H. Pylori does not typically lead to gastric cancer. This underscores the
importance of other factors, such as virulence, environmental factors, and host factors, as
well as genetic polymorphisms (e.g., interleukin-1β, a potent inhibitor of acid secretion).
The blood group A phenotype has been reported to be associated with gastric cancers.
Epstein–Barr virus infection has been noted in a certain type of gastric carcinoma
(lymphoepithelioid type), although the importance of this is unclear.

Question 16.3 The answers are A and C.


GERD has been implicated as one of the strongest risk factors for the development of
adenocarcinoma of the esophagus. Chronic reflux is associated with Barrett esophagus, the
premalignant precursor of esophageal adenocarcinoma. Population-based case-control
studies that examined the relationship between symptomatic reflux and risk of
adenocarcinoma of the esophagus have demonstrated that increased frequency, severity,
and chronicity of reflux symptoms are associated with a 2- to 16-fold increased risk of
adenocarcinoma of the esophagus, regardless of the presence of Barrett esophagus.
Increased body mass index (BMI) is a risk factor for adenocarcinoma of the esophagus,
and individuals with the highest BMI have up to a sevenfold greater risk of esophageal
cancer than those with a low body mass index. Barrett esophagus remains the single most
important risk factor for developing esophageal adenocarcinoma; patients with Barrett
esophagus are 11-fold more likely to develop esophageal adenocarcinoma than individuals
without Barrett esophagus. Infection with Helicobacter pylori, and particularly with cagA+
strains, is inversely associated with the risk of adenocarcinoma of the esophagus. The
mechanism of action is unclear, although an H. pylori infection can result in chronic
atrophic gastritis, leading to decreased acid production and potentially reducing the
development of Barrett esophagus.

Question 16.4 The answer is D.


In patients with localized esophageal adenocarcinoma, HER2+ disease was associated
with a significantly lower tumor grade, earlier stage of disease with fewer malignant
lymph nodes, and the presence of Barrett’s esophagus. These features of HER2+ localized
esophageal cancer led to an independent improvement in disease-specific survival (HR,
0.54; 95% CI, 0.35–0.84) and overall survival (p = 0.0022), suggesting that HER2+ is a
favorable prognostic feature in localized esophageal adenocarcinoma. Esophageal cancer
is relatively uncommon in the United States, and the lifetime risk of being diagnosed with
the disease remains less than 1%. It was estimated that 17,990 new cases would be
identified in 2013, with 15,210 patients expected to die of the disease. Age-adjusted
incidence rates are essentially equivalent among African-American and Caucasian men.

Question 16.5 The answer is D.


All of these molecular mechanisms are involved in the pathogenesis of esophageal cancer.

Question 16.6 The answer is C.


High-grade dysplasia in Barrett esophagus is the most powerful predictor of subsequent
invasive adenocarcinoma and is associated with a per-year cancer incidence of 6%,
thereby warranting therapeutic intervention. The rationale for esophagectomy is that
resection completely eradicates the mucosa at risk, which prevents progression to invasive
carcinoma. This is supported by older surgical series reporting previously unidentified
invasive cancer, which was present in up to 40% of resected specimens. The argument
against esophagectomies is that most patients with high-grade dysplasia do not develop
invasive carcinoma in their lifetimes and, in the era of endoscopic resection, that early
cancers can be effectively addressed without an esophagectomy. Those supporting
endoscopic methods, ranging from surveillance to mucosal ablative and resection
techniques, argue that this allows for the identification of patients with an early invasive
lesion that is readily amenable to cure or elimination of the mucosa at risk, thus
preventing progression. Endoscopic mucosal resection (EMR) is now considered an
essential diagnostic, staging, and therapeutic option available for patients with either
high-grade dysplasia or superficial esophageal cancers (T1a). Annual endoscopy is
recommended for those patients with low-grade dysplasia, whereas more frequent
screening (every 3 months) is recommended for those patients with high-grade dysplasia
if eradication therapy has not been instituted.

Question 16.7 The answer is B.


Adenocarcinomas of the GEJ present a unique challenge because appropriate management
of these tumors as either esophageal or gastric cancers has been uncertain. Siewert et al.
have offered a classification system based on demographics, histopathologic variables, and
patterns of lymphatic spread that provides clarity, which is well established, and has been
generally accepted worldwide. In this classification scheme, type I tumors are considered
adenocarcinomas of the distal esophagus and type II and III lesions are classified as gastric
cancers (cardia and subcardia). This classification system allows for a tailored and
consistent surgical approach to these tumors as well as consistency in reporting outcome
results associated with therapeutic interventions. However, it should be noted that in the
most recent guidelines established by the American Joint Committee on Cancer, GEJ
tumors are included under the esophageal cancer staging classification.

Question 16.8 The answer is D.


The CROSS trial, has for many practitioners established a new standard or care for
preoperative chemoradiotherapy. The rate of pathologic complete response was 25%. This
study randomized 366 patients with squamous cell carcinoma or adenocarcinoma of the
esophagus or GEJ to treatment with (1) preoperative carboplatin at an area under the
curve of 2 mg/mL per minute and paclitaxel 50 mg/m2 once weekly for 5 weeks, and
concurrent radiotherapy (1.8 Gy daily to 41.4 Gy) in 23 fractions, followed by
transthoracic esophagectomy or transhiatal esophagectomy for GEJ cancers, or (2)
immediate surgery. Unlike earlier trials, this more modern trial staged all patients by
endoscopic ultrasound and CT scan, and patients were required to have either node
positive or T2–3N any disease. The majority of patients treated had adenocarcinoma
(75%) and most tumors involved the distal third of the esophagus (58%). The majority of
patients were node positive (65%), and slightly more patients on the chemoradiotherapy
arm had T3 tumors (84%) compared to the surgery-alone arm (78%). At a median follow-
up of 45 months, the trial showed a significant survival benefit for chemoradiotherapy
added to surgery, with a median survival increased from 24 months to 49 months (HR,
0.0657; p = 0.003), and improvement in 2- and 5-year overall survival (67% and 47% vs.
50% and 34%; HR, 0.665).

Question 16.9 The answers are B and D.


Positive results of a recent trial of perioperative chemotherapy in gastric and distal
esophagus or GEJ cancer have been reported. In a trial by Cunningham et al., 503 patients
were assigned to three cycles of preoperative and three cycles of postoperative
epirubicin/cisplatin/5FU or surgery alone. Preoperative chemotherapy resulted in
significant improvement in patient survival, with a 6-month improvement in progression-
free survival, a 4-month improvement in median survival, and a 13% improvement in 5-
year overall survival (23% to 36%), all of which are statistically significant. Despite the
survival improvement with pre- and postoperative chemotherapy, there was no
improvement in the rate of curative resection in patients treated with preoperative
chemotherapy compared with surgery alone (66% to 69%), and there were no cases of
pathologic complete response to preoperative chemotherapy. Downstaging was also
observed with preoperative chemotherapy, with a shift to earlier T and N stage tumors
with preoperative chemotherapy compared to surgery alone. Because 26% of patients on
this trial had tumors in the GEJ and lower esophagus, the results may apply to locally
advanced esophageal cancer. The median follow-up was 47 months in the surgery-alone
arm, and 49 months in the chemotherapy arm.

Question 16.10 The answer is D.


HER2 testing is now recommended for all patients with metastatic EGJ adenocarcinoma at
the time of diagnosis.

Question 16.11 The answer is C.


Validation of the activity of a growth factor receptor-targeted agent, trastuzumab, was
recently achieved in esophagogastric cancer. Over 3,800 patients with gastric or GEJ
adenocarcinoma were screened for overexpression of the HER2 receptor by fluorescence
in situ hybridization (FISH) and immunohistochemistry; 22.1% tested positive. Five
hundred ninety-four patients were ultimately randomized to chemotherapy alone with (1)
capecitabine 1,000 mg/m2 twice a day for 14 days or (2) infusional 5FU 800 mg/m2 per
day for 5 days, combined with cisplatin 80 mg/m2 on day 1, cycled every 3 weeks, or to
chemotherapy plus trastuzumab 6 mg/kg once every 3 weeks. The majority of patients
received capecitabine plus cisplatin as the chemotherapy regimen. All end points were
improved with the addition of trastuzumab to chemotherapy, including antitumor
response (47.3% vs. 34.5%), progression-free survival (6.7 months vs. 5.5 months), and
overall survival (13.8 months vs. 11.1 months) (HR, 0.74; p = 0.0046). Toxicity was
comparable for the two.

Question 16.12 The answers are B and C.


At the second meeting of The International Gastric Cancer Linkage Consortium (IGCLC) in
2010, hereditary diffuse gastric cancer (HDGC) guidelines were extended to recommend
CDH1 genetic testing to families with the following:
• Cases of gastric cancer in which one case is histopathologically confirmed as diffuse and
younger than 50 years
• Families with both lobular breast cancer and diffuse gastric cancer, with one diagnosed
younger than 50 years
• Probands diagnosed with diffuse gastric cancer younger than 40 years, with no family
history of gastric cancer

Question 16.13 The answer is A.


Lynch syndrome-associated gastric cancers predominantly show intestinal histology (more
than 90% of the cases). This correlation echoes the strong association between MSI tumor
phenotype and intestinal gastric cancer.

Question 16.14 The answer is C.


Mutations in CDH1 gene are associated with hereditary diffuse gastric cancer, which
carries a more than 80% risk of developing gastric cancer.

Question 16.15 The answer is D.


Large families with an autosomal-dominant, highly penetrant inherited predisposition for
the development of gastric cancer are rare. However, early-onset diffuse gastric cancers
have been described and linked to the E-cadherin/CDH1 locus on 16q and associated with
mutations in this gene, which may be the case in this family. This seminal finding has
been confirmed in other studies with gastric cancers at a relatively high (67% to 83%)
penetrance rate. Thus, E-cadherin mutation testing should be considered in the
appropriate clinical setting. In fact, prophylactic gastrectomy should be considered
strongly in families with germline E-cadherin mutation even without gross mucosal
abnormalities on endoscopic examination of the stomach. Hereditary nonpolyposis colon
cancer (HNPCC) involves germline mutations of DNA mismatch repair genes. Gastric
adenocarcinoma may be observed in families with HNaPCC.

Question 16.16 The answer is B.


Whole-body 2-[18F]-fluoro-2-deoxyglucose (FDG) positron emission tomography (PET) is
being applied increasingly in the evaluation of gastrointestinal malignancies. In gastric
cancer, approximately half of the primary tumors are FDG-negative; the diffuse (signet
cell) subtype was most likely to be non-FDG avid, likely because of decreased expression
of the glucose transporter-1 (Glut-1). Although laparoscopic staging is thought to detect
CT-occult metastatic disease in approximately 40% of patients and spares nontherapeutic
operations in approximately one-third of patients with gastric cancer, one needs to
remember that tumor biology, not staging, will eventually guide outcomes. Clearly, not
all patients benefit from preoperative laparoscopic staging; therefore, future studies
should address the issue of selective laparoscopy based on noninvasive staging (i.e.,
patients with T1 tumors). Staging laparoscopy with or without cytology should be
considered only if therapy will be altered consequent to information obtained by
laparoscopy.

Question 16.17 The answer is C.


The Intergroup Trial (INT 0116) randomized patients to receive surgery alone or surgery
plus postoperative 5FU-based chemotherapy and radiation. The trial included patients
with stages IB–IVA nonmetastatic adenocarcinoma of the stomach or GEJ. After en bloc
resection, 556 patients were randomized to either observation alone or postoperative
combined-modality therapy consisting of one monthly 5-day cycle of 5-FU and leucovorin,
followed by 45 Gy in 25 fractions plus concurrent 5-FU and leucovorin (4 days in week 1,
3 days in week 5) followed by two monthly 5-day cycles of 5FU and leucovorin. Nodal
metastases were present in 85% of the cases. With 5 years of median follow-up, 3-year
relapse-free survival was 48% for adjuvant treatment and 31% for observation (p =
0.001); 3-year OS was 50% for treatment and 41% for observation (p = 0.005). The
median OS in the surgery-only group was 27 months, compared with 36 months in the
chemoradiotherapy group; the HR for death was 1.35 (95% CI = 1.09 to 1.66; p =
0.005). The hazard ratio for relapse in the surgery-only group as compared with the
chemo-radiotherapy group was 1.52 (95% CI = 1.23 to 1.86; p <0.001). The median
duration of relapse-free survival was 30 months in the chemoradiotherapy group and 19
months in the surgery-only group. Patterns of failure were based on the site of first
relapse only and were categorized as local, regional, or distant. Local recurrence occurred
in 29% of the patients who relapsed in the surgery-only group and 19% of those who
relapsed in the chemoradiotherapy group. Regional relapse, typically abdominal
carcinomatosis, was reported in 72% of those who relapsed in the surgery-only group and
65% of those who relapsed in the chemoradiotherapy group.

Question 16.18 The answers are C and D.


Cunningham et al. in the REAL-2 trial studied 1,002 patients who were randomized to one
of four treatment groups: a control arm of ECF and three investigational arms. The central
question in this study was: Can capecitabine be substituted for 5FU and/or oxaliplatin
substituted for cisplatin? The four arms were ECF, epirubicin–oxaliplatin–5FU, epirubicin–
cisplatin–capecitabine, and epirubicin–oxaliplatin–capecitabine (EOX). The four regimens
are shown in Table 46.9. Patients were stratified for performance status and extent of
disease. The primary end point was in OS. The study was powered to show noninferiority
for capecitabine compared with 5FU, and oxaliplatin compared with cisplatin. There were
approximately 250 patients per arm. The study design was a two-by-two comparison. A
total of 40% of patients had primary gastric cancer, and the remainder had either EGJ or
esophageal cancers, with 10% of patients having squamous cell cancer of the esophagus.
There was no difference in median OS between the arms (ECF, 9.9 months; epirubicin–
oxaliplatin–5FU, 9.3 months; epirubicin–cisplatin–capecitabine, 9.9 months; and EOX,
11.2 months). The 1-year OS was also similar and ranged from 38% to 47%, the best
outcome evident with EOX and the lowest with the control arm of ECF. The authors
concluded the oxaliplatin could be substituted for cisplatin, and capecitabine could be
substituted for 5FU in the palliative setting.

Question 16.19 The answer is C.


The REGARD study was a placebo-controlled, double blind, phase 3 international trial
conducted in the second-line setting in patients with metastatic gastric or EGJ
adenocarcinoma. Median OS was 5.2 months for ramucirumab and 3.8 months for placebo
(HR = 0.776; 95% CI = 0.603 to 0.998; p = 0.0473). The significance of this study is
that it provides a proof-of-principal that antiangiogenic therapy does have activity in
gastroesophageal malignancies.

Question 16.20 The answer is C.


Ramucirumab is a fully human IgG1 monoclonal antibody targeting VEGFR2. The
REGARD study showed that single agent ramucirumab prolonged the median OS
compared to placebo (median OS 5.2 months for ramucirumab and 3.8 months with
placebo; HR 0.776, 95% CI 0.60-0.99, p = 0.04).

Corresponding chapters in Cancer: Principles & Practice of Oncology, Tenth Edition: 44 (Molecular Biology of the Esophagus
and Stomach), 45 (Cancer of the Esophagus), 46 (Cancer of the Stomach), and 47 (Genetic Testing in Stomach Cancer).
17 Pancreatic and Hepatobiliary Cancer
Thomas Regenbogen and Andrea Wang-Gillam

QUESTIONS
Each of the numbered items below is followed by lettered answers. Select the ONE lettered answer
that is BEST in each case unless instructed otherwise.

Question 17.1 Which of the following factors are associated with an increased risk of
pancreatic cancer?
A. Cigarette smoke
B. Chronic pancreatitis
C. Prior cholecystectomy
D. African-American ethnicity
E. All of the above

Question 17.2 Which of the following statements regarding pancreatic cancer is TRUE?
A. At diagnosis, 31% have evidence of distant metastases.
B. Pancreatic cancer decreases in incidence later in life.
C. Activation of the KRAS oncogene plus inactivation of tumor suppressor genes (TP53,
DPC4, p16, and BRCA2) are associated with the development of pancreatic cancer.
D. Pancreatic intraepithelial neoplasms (PanINs) are intraductal proliferative epithelial
lesions that will not progress to pancreatic cancer.
E. Patients with advanced pancreatic cancer will have high levels of CA 19-9 if they are
Lewis antigen-a or -b negative.

Question 17.3 A 56-year-old man is evaluated for a 1-month history of gradually worsening
painless jaundice and a 10-lb weight loss. Computed tomography (CT) scan of the abdomen
and pelvis revealed a 3.3-cm pancreatic head mass adjacent to the superior mesenteric vein
with no intervening fat plane and encasing both the superior mesenteric vein and artery. The
common bile duct was dilated and the pancreatic body and tail were atrophied with dilatation
of the pancreatic duct. Portal lymphadenopathy measuring 2 cm was present. The patient
underwent endoscopic retrograde cholangiopancreatography and a biliary stent was placed.
Biopsy tissue of the mass was consistent with moderately differentiated pancreatic
adenocarcinoma. What stage is this patient’s cancer per the TNM staging system?
A. Stage I
B. Stage II
C. Stage III
D. Stage IV
Question 17.4 A 67-year-old woman presents to her local emergency department with a 2-
month history of right upper quadrant pain, jaundice, and 20-lb weight loss. CT of the
abdomen with contrast reveals a 2.5-cm, ill-defined soft tissue density within the head of the
pancreas and mild celiac axis, porta hepatis, and portacaval adenopathy, with the largest
being within the celiac axis region measuring 2.0 × 1.8 cm. Following admission endoscopic
retrograde cholangiopancreatography with biliary stent placement and biopsy is done.
Pathology reports poorly differentiated adenocarcinoma. What test should be ordered at this
point to help establish her stage?
A. Multiphase multidetector helical computerized tomography
B. Magnetic resonance imaging
C. Ultrasonography
D. Serum CA 19-9 measurement
E. Endoscopic ultrasonography

Question 17.5 A 71-year-old woman undergoes Whipple resection for a T3N1M0 pancreatic
adenocarcinoma. Postoperative recovery was uneventful, and she starts adjuvant therapy 7
weeks later with gemcitabine given intravenously weekly for 3 weeks, followed by a 1-week
break. In a follow-up visit after her first cycle, she reports a 5-lb weight loss, nausea,
decreased appetite, and diarrhea with floating stools. What should be done next?
A. Increase pancreatic enzyme supplementation
B. Admit the patient for small bowel obstruction
C. Hold chemotherapy for 1 week and follow up on symptoms
D. CT scan of the chest, abdomen, and pelvis to rule out metastatic disease

Question 17.6 Which of the following statements regarding adjuvant therapy of pancreatic
adenocarcinoma is TRUE?
A. In RTOG 9704 administering gemcitabine before and after adjuvant 5-FU based CRT for
resected pancreatic head adenocarcinoma demonstrated a trend toward improved
median OS compared with 5-FU before and after 5-FU CRT.
B. CONKO-001 demonstrated gemcitabine when compared with observation improves
disease free survival but not overall survival.
C. GITSG 9173 clearly showed no benefit with 5-FU based chemoradiation followed by
chemotherapy compared with observation.
D. ESPAC-1 showed that those who received CRT did better than those treated with
chemotherapy alone.

Question 17.7 A 64-year-old woman is diagnosed with locally advanced unresectable


pancreatic cancer. After 2 months of a gemcitabine-based chemotherapy regimen, CT scans
demonstrate a decrease in the size of the lesion and her CA19-9 decreased from 854 to 201.
What should be done now?
A. Switch to 5FU-based chemoradiation
B. Continue current gemcitabine-based chemotherapy for another 2 months
C. Add radiation to her current regimen
D. Add 5FU to her chemotherapy regimen

Question 17.8 A 52-year-old woman is evaluated for chronic right upper quadrant
abdominal pain and is diagnosed with pancreatic adenocarcinoma with metastasis to the
liver. She is distraught and wants to do everything possible to prolong her life. Which of the
following regimens has been shown to prolong overall survival compared with gemcitabine
alone?
A. Gemcitabine and oxaliplatin
B. Gemcitabine and capecitabine
C. Gemcitabine and nab-paclitaxel
D. Gemcitabine, cisplatin, and bevacizumab
E. Gemcitabine and cetuximab

Question 17.9 A 45-year-old man with unintentional weight loss is diagnosed with
metastatic adenocarcinoma of the pancreas. Which of the following chemotherapy regimens
would be appropriate first-line treatment choice in this otherwise healthy patient with normal
organ function?
A. Gemcitabine
B. FOLFIRINOX (5FU, leucovorin, irinotecan, and oxaliplatin)
C. 5FU with radiation
D. FOLFOX (5FU + leucovorin + oxaliplatin)

Question 17.10 Mutation of which tumor suppressor gene is most frequently associated with
familial pancreatic cancer?
A. BRCA2
B. PALB2
C. KRAS
D. TP53

Question 17.11 Which one of the following statements regarding risk factors for pancreatic
cancer is CORRECT?
A. People with blood types A, B, and AB are more likely than type O to develop pancreatic
cancer.
B. Testing for KRAS mutations in the pancreatic juice of patients is an effective screening
test for pancreatic cancer.
C. Patients with hereditary nonpolyposis colorectal cancer syndrome do not have an
increased risk for pancreatic cancer.
D. Hereditary pancreatitis is not a significant risk factor for pancreatic cancer.

Question 17.12 A 65-year-old male diagnosed with metastatic pancreatic cancer received
treatment with single-agent gemcitabine. He now has disease progression and is interested in
pursuing further systemic therapy. Which one of the following should be offered as a second-
line treatment choice?
A. OFF (oxaliplatin, 5FU, and leucovorin)
B. Erlotinib
C. Paclitaxel
D. Supportive care alone

Question 17.13 Which of the following is/are TRUE about pancreatic cancer?
A. Most pancreatic cancers have mutations in KRAS, TP53, SMAD4, p16/CDKN2A.
B. Telomere shortening is the earliest and prevalent genetic change identified in the
precursor lesions.
C. Underexpression of TGF-β is observed in some pancreatic cancers.
D. p16-mediated CDK inhibition is a protective mechanism against pancreatic cancer.
E. All of the above.

Question 17.14 A 52-year-old man is found to have mildly abnormal liver function and an
elevated serum α-fetoprotein (AFP). Workup reveals prior hepatitis B viral (HBV) infection.
Ultrasound reveals a 4-cm lesion in the left hepatic lobe, and a computed tomography (CT)
scan reveals no evidence of metastatic disease or vascular involvement. MRI demonstrates
features consistent with hepatocellular carcinoma (HCC). The patient undergoes partial
hepatectomy, surgical margins are clear. The pathology report confirms the diagnosis of
HCC. Which of the following approaches should be followed?
A. Adjuvant sorafenib
B. Combination chemotherapy that is doxorubicin based
C. Adjuvant external beam radiation to surgical bed
D. Routine surveillance

Question 17.15 Which of the following criteria help(s) guide selection of patients with HCC
appropriate for potential liver transplantation?
A. Patients with solitary tumors ≤5 cm, or patients with multifocal disease with ≤3
tumor nodules each ≤3 cm in size
B. Patients with Child–Pugh B or C cirrhosis
C. Tumors without evidence of macrovascular invasion and distant metastasis
D. All of the above

Question 17.16 Which of the following statements about staging systems for hepatocellular
carcinoma is TRUE?
A. The Okuda system takes into account several clinical features that include tumor size
(>50% of liver), ascites (positive or negative), hypoalbuminemia (<3 g/dL), and
hyperbilirubinemia (>3 mg/dL).
B. The Cancer of the Liver Italian Program system uses hepatic tumor morphology and
extent of liver replacement, Child–Pugh score, portal vein thrombosis, and serum AFP
levels.
C. The Barcelona Clinic Liver Cancer scoring system combines assessment of tumor stage,
liver function, and patient symptoms with a treatment algorithm and has been shown to
correlate well with patient outcomes.
D. All of the above.

Question 17.17 Which of the following abnormalities is both a paraneoplastic syndrome


associated with HCC, and may also be caused by end-stage liver failure?
A. Hypoglycemia
B. Erythrocytosis
C. Hypercalcemia
D. Hypercholesterolemia

Question 17.18 An increased risk of developing HCC is associated with which of the
following?
A. Wilson disease
B. Hereditary tyrosinemia
C. Porphyria cutanea tarda
D. Primary biliary cirrhosis
E. All of the above

Question 17.19 Which of the following statements about screening and prevention is
CORRECT?
A. The advent of vaccination for hepatitis B is unlikely to reduce HCC in endemic areas.
B. A combination of AFP and ultrasound screening is used in high-risk populations.
C. Aggressive screening programs for HCC have been shown to improve survival.
D. Detection of HCC, through surveillance of patients awaiting liver transplantation, does
not increase priority for orthotopic liver transplantation.

Question 17.20 Which of the following increase(s) the risk of developing


cholangiocarcinoma?
A. Primary sclerosing cholangitis
B. Clonorchis sinensis infestation
C. Chronic portal bacteremia and portal phlebitis
D. All of the above

Question 17.21 A 66-year-old man is noted to have painless jaundice on a routine follow-up
at his primary care physician’s office. Workup reveals a mass causing biliary obstruction at
the hilum. Endoscopic retrograde cholangiopancreatography confirms a high-grade stricture
predominantly involving the left hepatic duct; however, brushings reveal atypical cells and
no malignancy. He is seen at a tertiary care center and offered surgical management, an en
bloc resection of the left hepatic lobe and extrahepatic bile duct, and a complete periportal
lymphadenectomy. Which of the following statements about management/natural history of
hilar cholangiocarcinoma is/are TRUE?
A. Surgical resection is associated with an operative mortality rate of 30%.
B. Recurrences occur most commonly at the bed of resection, followed by retroperitoneal
lymph nodes. Distant metastases occur in one-third of cases.
C. Less than 10% of patients have resectable cancer at the time of diagnosis.
D. All of the above.

Question 17.22 A 70-year-old man presents with 16-lb weight loss and persistent right upper
quadrant pain. CT scan reveals a gall bladder stone and thickening of the anterior wall of the
gall bladder. He undergoes a laparoscopic cholecystectomy. Pathology reveals a moderately
differentiated 2-cm gallbladder adenocarcinoma invading the perimuscular connective tissue.
Margins of resection are negative for tumor. Based on the available information, what is the
stage of this cancer?
A. Stage IA
B. Stage IB
C. Stage II
D. Stage IIIA

Question 17.23 The patient in Question 17.22 recovers from surgery and seeks a second
opinion at a tertiary care center 5 weeks after his cholecystectomy. A CT scan 2 weeks after
surgery shows mild periportal fullness. What is the most appropriate next step in
management?
A. Perform en bloc resection of the gallbladder, resection of segments IVb and V of the
liver, and regional lymph node dissection.
B. No further therapy is warranted; surveillance with CT scans and laboratories done every
3 months.
C. He requires a second laparotomy to assess the extent of remaining disease to guide
further therapy.
D. Perform ultrasound-guided biopsy of the periportal nodes; if positive, then
fluoropyrimidine-based chemoradiation is indicated.

Question 17.24 Which one of the following statements about adjuvant therapy for biliary
cancers is TRUE?
A. Based on retrospective data it appears patients may benefit from adjuvant
chemotherapy.
B. Adjuvant radiation is superior to chemotherapy alone.
C. Adjuvant therapy can improve overall survival for patients with R0 resections.
D. Fluoropyrimidine-based chemoradiation is standard because it is superior to radiation
alone.

Question 17.25 Which of the following statements regarding fibrolamellar HCC is


CORRECT?
A. Fibrolamellar HCC occurs more frequently in men compared with women.
B. This variant of HCC is associated with viral hepatitis but not cirrhosis.
C. Lymph node metastases at the time of presentation is common.
D. Most patients with fibrolamellar HCC present in their sixth decade of life.
Question 17.26 A 50-year-old man with a history of chronic hepatitis B infection and Child–
Pugh A cirrhosis presents with abdominal pain. He is otherwise in good health. CT of the
abdomen reveals a cirrhotic liver with a 2.5-cm liver mass that rapidly enhances during the
arterial phase of contrast administration and “washout” during the later venous phases. There
is no involvement of the portal vein. AFP is 300 ng/mL. What is the next best step in his
management?
A. Biopsy to obtain a histologic diagnosis.
B. Refer him to a hepatobiliary surgeon.
C. Start him on sorafenib.
D. Refer him to a radiation oncologist.

Question 17.27 Which of the following statements is CORRECT regarding hepatoblastoma?


A. This is the most common primary cancer of the liver in adults.
B. Hepatoblastoma is a chemoresistant tumor.
C. Patients with this tumor have a poor outcome after liver transplantation, with a 5-year
survival rate of 20%.
D. The peak incidence of hepatoblastoma is within the first 2 years of life.

Question 17.28 Which of the following statements is CORRECT?


A. The hepatitis B x gene product has no known role in causing HCC.
B. NS5A protein product of hepatitis C viral (HCV) genome inactivates p53.
C. Level of HBV replication is inversely related to the risk of liver cancer.
D. Hepatitis B virus genotype C is associated with decreased risk of HCC.

Question 17.29 Which of the following statement(s) regarding hepatitis C infection is/are
CORRECT?
A. Sixty percent to 80% HCV infections become chronic in contrast to 10% HBV infections.
B. HBV genome integrates into hepatocyte DNA while HCV genome does not.
C. The average interval from HCV infection to HCC is 30 years in contrast to 40 to 50
years for HBV infection.
D. All of the above.

Question 17.30 A 45-year-old man with a history of alcoholic cirrhosis was found by
screening ultrasound to have two new lesions in hepatic segments 7 and 3, measuring 3 cm
and 2 cm, respectively. MRI findings were consistent with hepatocellular carcinoma. He has
been abstinent from alcohol use for the past 2 years. His total bilirubin is 1.5 mg/dL, serum
albumin 3 g/dL and INR is 1.6. He does not have ascites or encephalopathy. There is no
evidence of metastatic disease and he is referred for orthotopic liver transplantation
evaluation. He is deemed to be an acceptable candidate. While waiting for a donor organ,
what should be done next?
A. Refer to interventional radiology for ablative therapy
B. Sorafenib to prevent disease progression while waiting for donor organ
C. Cisplatin, IFNα-2b, doxorubicin, and 5FU as neoadjuvant therapy
D. Observation

Question 17.31 A 45-year-old woman presents with painless jaundice and weight loss. Her
total bilirubin is 17.6 mg/dL, alkaline phosphatase is 568 units/L and alanine transaminase
(ALT) is 138 units/L. Viral hepatitis serology is negative. CT chest abdomen and pelvis with
contrast is performed and reveals an ill-defined, infiltrative, hypoattenuating lesion at the
hepatic hilum with marked intrahepatic biliary ductal dilatation. MRI of the liver
characterizes the lesion as compatible with cholangiocarcinoma centered in the hepatic hilum
extending to the confluence of the left medial and lateral hepatic ducts, involving the cystic
duct, proximal extrahepatic duct, and abutting the undersurface of the portal vein and right
hepatic artery. ERCP is performed with common bile duct stenting. Pathology reports
atypical epithelial cells in the brushing. Hepatobiliary surgery is consulted and determines
that her tumor is unresectable and she is not a liver transplant candidate due to psychosocial
issues. What is the next step in management once hyperbilirubinemia resolves?
A. FOLFOX
B. Gemcitabine and cisplatin
C. External beam radiation therapy with concurrent 5FU
D. Palliative care alone including routine stent exchange

Question 17.32 Which of the following individuals would meet criteria to have surveillance
for HCC?
A. A 28-year-old African woman who is a hepatitis B carrier.
B. A 47-year-old Indian woman who is hepatitis B carrier and has a brother with HCC.
C. A 62-year-old Hispanic male with alcoholic cirrhosis.
D. A 42-year-old Chinese man who is a hepatitis B carrier.
E. All of the above

Question 17.33 A 52-year-old man is admitted to the hospital due to new onset of
symptomatic ascites and jaundice. He is an alcoholic and has no history of medical care prior
to this encounter. He is mildly encephalopathic. Serum total bilirubin is 5.6 mg/dL and INR is
2.1. He is diagnosed with unresectable metastatic hepatocellular carcinoma and alcoholic
cirrhosis after an extensive work up. The patient’s family arrives and would like to discuss
treatment options. Your recommendation will be:
A. Gemcitabine and oxaliplatin.
B. Sorafenib.
C. Sunitinib.
D. Hospice or supportive care alone
ANSWERS

Question 17.1 The answer is E.


Tobacco smoke exposure plays a significant role in the development of pancreatic
adenocarcinoma. It has been estimated that tobacco smoking contributes to the
development of 20% to 30% of pancreatic cancers. The strongest associations between
cigarette smoking and pancreatic cancer have been observed when the smoking occurred
within the previous 10 years. Smoking cessation can reduce this risk. Environmental
tobacco smoke contains the same toxins, irritants, and carcinogens, such as carbon
monoxide, nicotine, cyanide, ammonia, benzene, nitrosamines, vinyl chloride, arsenic, and
hydrocarbons, as primary cigarette smoke. Host etiologic factors associated with an
increased risk of pancreatic cancer include a history of diabetes mellitus (DM), chronic
cirrhosis, pancreatitis, a high-fat and high-cholesterol diet, and prior cholecystectomy.
Although not all studies have supported a relationship between DM and pancreatic cancer,
a meta-analysis of 20 epidemiologic studies confirms that the pooled relative risk of
pancreatic cancer in persons with DM for 5 years is double (relative risk, 2.0; 95%
confidence interval [CI], 1.2 to 3.2) the risk of persons without DM. There is no
association with a high-protein diet. People of African-American descent experience a
higher rate of pancreatic cancer than those of European ancestry in the United States, with
an annual incidence of 16.7 per 100,000 versus 10.9 per 100,000, respectively. Death
from pancreatic cancer is similarly elevated, with an annual rate of 14.6 per 100,000
versus 10.6 per 100,000. The diagnosis is slightly but significantly earlier in African-
Americans compared with Caucasians, with a median age of diagnosis of 68 and 73 years,
respectively. In developed countries, the incidence and mortality rates range from 7 to 9
per 100,000 for men and 4.5 to 6 per 100,000 for women.

Question 17.2 The answer is C.


CA 19-9 is a clinically useful tumor marker. The epitope of this antibody is a sialylated
lacto-N-fucopentaose II related to the Lewis-a blood group antigen. Limitations of CA 19-9
are that it is not specific for pancreatic cancer and has been found to be elevated in other
tumor types, such as biliary tract, colon, and stomach cancers. Ten percent of patients are
Lewis antigen-a or b negative and unable to synthesize this antigen, and therefore may
have undetectable levels of CA 19-9, even in the setting of advanced pancreatic cancer.
Cholestasis can falsely elevate serum CA 19-9 levels. Therefore, in patients who present
with obstructive jaundice, an elevated CA 19-9 is not specific for the presence of
pancreatic malignancy. Despite these caveats, serum CA 19-9 levels seem to have
prognostic utility, particularly when measured either preoperatively or postoperatively in
patients with resectable pancreatic cancer. Pancreatic ductal adenocarcinoma arises from
ductal epithelial cells. Neoplasia arising from these cells progresses from initial pancreatic
intraepithelial neoplasms (PanINs) to invasive carcinomas. The evidence that these lesions
are precancerous includes the observation that after segmental resection of pancreas with
PanINs in the resected tissue, pancreatic cancer can develop in the pancreatic remnant.
The strongest evidence that PanINs are precancerous lesions is the stepwise progression of
genetic mutational events that correlates with the stepwise progression of worsening
PanINs culminating in adenocarcinoma. PanIN-1 lesions show a predominance of KRAS
mutations and overexpression of HER2/neu. In PanIN-2 lesions, p16 mutations are the
typical mutational event, and in PanIN-3 lesions, TP53, DPC4, and BRCA2 mutations
predominate. The mutational pattern of PanIN-3 lesions is equivalent to mutations found
in pancreatic adenocarcinoma. These events are the basis of a current progression model
for pancreatic cancer in which point mutation of KRAS and overexpression of HER2/neu
are initiating early events, p16 inactivation is an intermediate event, and TP53, DPC4,
BRCA2 inactivation follows just before invasion outside of the duct. Approximately 95%
of all cases have KRAS mutation, 90% have p16 mutations, 75% have TP53 mutations, and
55% have DPC4 mutations. It is estimated that 5% to 10% of pancreatic cancers are
hereditary or have a familial link. Of patients with available data in the United States who
were diagnosed with pancreatic cancer from 1996 to 2002, 8% presented with local
disease, 31% presented with regional disease, and 61% had distant metastases. Therefore,
the majority of patients are metastatic at diagnosis. Five-year survival from all stages of
disease is 5%. Pancreatic cancer tends to occur later in life. Only 10% of patients in
Europe present before the age of 50 years, whereas those aged 50 to 54 years, experience
an incidence of 9.8 per 100,000, and those aged 70 to 74 years, experience an incidence of
57 per 100,000. The median age of diagnosis with pancreatic cancer in the United States is
72 years.

Question 17.3 The answer is C.


The patient has a stage III, T4N1M0 pancreatic cancer. As per the seventh edition of the
American Joint Committee on Cancer staging system, this represents unresectable disease.
The T-stage designation classifies T1 to T3 tumors as potentially resectable and T4 tumors
as locally advanced (unresectable). Tumors with any involvement of the superior
mesenteric or celiac arteries are classified as T4. However, tumors that involve the
superior mesenteric, splenic, or portal veins are classified as T3 because these veins can be
resected and reconstructed, provided that they are patent.

Question 17.4 The answer is A.


Laparoscopy and multiphase CT have evolved concurrently as methods to evaluate a
pancreatic mass. Both have emerged as highly effective in evaluating the tumors, but CT
as a noninvasive modality supplants the use of routine laparoscopy. Currently, routine use
of laparoscopy is not warranted. The cornerstone of diagnostic evaluation of a pancreatic
tumor is the multiphase CT scan, coordinating intravenous contrast administration with
subsequent rapid thin-cut CT through the pancreas during arterial, portal venous and
parenchymal phases of enhancement. With this type of CT, extension of the tumor to the
superior mesenteric artery, celiac axis, superior mesenteric vein/portal vein complex, and
contiguous structures can be clearly determined, as well as an assessment of distant
metastasis. Optimally, CT imaging should precede stent placement and biopsy because of
the possibility of postprocedure inflammation from the biopsy and artifact from the stent
that can confound interpretation of the images. Magnetic resonance imaging has not been
widely used to assess pancreatic cancer. Endoscopic ultrasound can image the primary
cancer and be a means of obtaining a fine-needle aspiration of pancreatic adenocarcinoma,
but in general, the procedure is noncontributory when CT scan characterizes the tumor.
When a mass cannot be visualized on CT scan, sonography through the wall of the
stomach or duodenum can image tumors in the body/tail and head of the pancreas,
respectively. Although preoperative ultrasonography is useful in assessing tumor
characteristics and resectability of pancreatic adenocarcinoma, it is particularly operator
dependent. Serum CA 19-9 is not a component of the TNM staging system for pancreatic
cancer.

Question 17.5 The answer is A.


Malabsorption is a frequent complication seen in patients after Whipple surgery because
of pancreatic enzyme insufficiency. Most patients require adjustment of the pancreatic
enzyme supplement for adequate control of this symptomatology. Although small bowel
obstruction and metastatic disease can be seen, the patient’s clinical picture does not go
along with these diagnoses. The aim during adjuvant therapy is to administer the
chemotherapy with minimal delays to obtain the full benefit from therapy.

Question 17.6 The answer is A.


The Radiation Therapy Oncology Group (RTOG) performed a prospective randomized
trial (RTOG 9704) comparing gemcitabine with infusional 5FU as the systemic component
of therapy with all patients also receiving 5FU-based chemoradiation. There was no
survival difference between patients randomized to gemcitabine and those who received
infusional 5FU. However, among the 380 patients with resected pancreatic head lesions,
survival was superior for patients randomized to gemcitabine compared with those who
received infusional 5FU (20.6 months vs. 15.9 months; hazard ratio [HR] for death 0.79;
95% CI, 0.63 to 0.99; p = 0.033). In CONKO-001, patients randomized to receive
gemcitabine had a median disease-free survival of 13.9 months (95% CI, 11.4 to 15.3),
whereas those patients who underwent surgery alone had a median disease-free survival
of only 6.9 months (95% CI, 6.1 to 7.8; p = 0.001). However, there was no statistically
significant difference in overall survival when the initial results of the study were
reported in 2007. The long-term survival outcomes of this trial were reported in 2013,
and the median overall survival of 22.8 months was seen in the gemcitabine group,
compared with 20.2 months in the observation group (HR, 0.76; p = 0.01). Additionally,
10-year survival rate was superior in the gemcitabine group (12.2% vs. 7.7%), compared
with the observation group. In the GITSG 9173 trial 5FU-based chemoradiation followed
by up to 2 years of weekly bolus 5FU was evaluated against observation alone. A
preliminary analysis of survival was reported after only 43 patients had completed
treatment and showed a striking survival advantage for patients receiving combined
modality therapy compared with survival of patients who underwent surgery alone
(median 21.0 months vs. 10.9 months, respectively; one-tailed p = 0.03). In ESPAC-1,
patients receiving chemoradiation did worse (median survival of 15.9 months; HR for
death 1.28; 95% CI, 0.99 to 1.66) than those not receiving chemoradiation (median
survival of 17.9 months; p = 0.05). Conversely, patients who received chemotherapy had
a median survival of 20.6 months (HR for death 0.71; 95% CI, 0.55 to 0.92) versus 15.5
months for those patients who did not receive chemotherapy, a statistically significant
result (p = 0.009).

Question 17.7 The answer is B.


The optimal treatment of locally advanced pancreatic cancer remains controversial.
Locally advanced disease is generally incurable, and all therapies have significant
limitations. Systemic therapy for 2 to 4 months followed by 5FU- or capecitabine-based
chemoradiation is a reasonable approach for some patients with locally advanced disease.
However, concurrent chemoradiotherapy approaches has not impact the survival,
compared with patients with chemotherapy alone. If patients are responding to
chemotherapy (objective radiographic response or CA 19-9 level decline) after 2 months
and tolerating therapy well, it is reasonable to continue for 2 more months.

Question 17.8 The answer is C.


The combination of nab-paclitaxel and gemcitabine is the only gemcitabine combination
that showed overall survival benefit in patients with advanced pancreatic cancer by a
large randomized study. The overall survival was superior in patients who received the
combination regimen (8.5 months vs. 6.7 months; HR 0.72; p <0.001), compared with
the gemcitabine only group.

Question 17.9 The answer is B.


Results from of the phase III randomized trial reported significant improvement in overall
survival for patients receiving FOLFIRINOX compared to single-agent gemcitabine alone
(11.6 months vs. 6.8 months; HR 0.57; p <0.0001). Therefore, FOLFIRINOX should be
considered for first-line treatment in patient with good performance status and normal
serum bilirubin level. FOLFOX has shown activity in patients with pancreatic cancer in the
second-line setting, but further evaluation of clinical benefit in the front line has not been
validated.

Question 17.10 The answer is A.


BRCA2 is the most commonly mutated tumor suppressor gene in familial pancreatic
cancer followed by PALB2. KRAS is an oncogene and not a tumor suppressor gene. TP53
gene mutation is found in more than 70% of all pancreatic cancers, but there is no
evidence to link this mutation with familial pancreatic cancer.

Question 17.11 The answer is A.


Non type O blood type has been long known to be associated with increased risk for lung
cancer. A genome-wide association study identified high-risk single nucleotide
polymorphism for pancreatic cancer in the gene encoding for the ABO blood type.
Patients with nonpolyposis colorectal cancer syndrome or Lynch syndrome are at higher
risk for gastrointestinal malignancies including pancreatic cancer. KRAS mutation testing
has so far not proven to be effective in differentiating between pancreatic cancer and
other conditions such as pancreatitis and pancreatic adenoma.
Question 17.12 The answer is A.
Data on second-line treatment of patients with pancreatic cancer who failed first-line
gemcitabine are limited. Based on the CONKO-003 study, patients with pancreatic cancer
experienced progression during the first-line gemcitabine had better overall survival in
the OFF (oxaliplatin plus FF) group compared with the FF group (5FU and folinic acid)
(5.9 months vs. 3.3 months, HR, 0.66; p = 0.01). Hence, an OFF chemotherapy regimen
would be an appropriate second-line option in patients progressed on the first-line
gemcitabine therapy. Single-agent erlotinib has not shown significant treatment benefit
for patients with metastatic pancreatic cancer. Raltitrexed is a thymidylate synthase
inhibitor currently unavailable in the United States that has shown activity in the second-
line setting in combination with irinotecan or oxaliplatin. Supportive care alone is a
reasonable choice in those with progressive metastatic disease in the absence of
evidenced-based treatment options or patients unwilling or unable to tolerate more
antineoplastic therapy.

Question 17.13 The answer is E.


Telomere shortening is thought to predispose to chromosome fusion (translocation) and
missegregation of genetic material during the mitosis and later during tumor genesis,
telomerase is reactivated. SMAD4 (DPC4) pathway mediates signals initiated on the
binding of the extracellular proteins TGF-β. The underexpression of TGF-β receptors
results in cellular resistance to the usual suppressive effects of the TGF-β ligand. The
cyclin D oncogene complexes with CDKs and phosphorylates the retinoblastoma protein
(tumor suppressive protein) that results in loss of negative regulatory effect of
retinoblastoma protein.

Question 17.14 The answer is D.


There is no treatment demonstrating a consistent survival advantage to support adjuvant
therapy for hepatocellular carcinoma. The phase III STORM trial of adjuvant sorafenib
compared with placebo failed to demonstrate a significant difference in the primary
endpoint of recurrence-free survival or the secondary endpoints of time to relapse and
overall survival.

Question 17.15 The answer is D.


The Milan criteria for selection of patients appropriate for liver transplantation include
patients with solitary tumors less than or equal to 5 cm, patients with multifocal disease
with less than or equal to three tumor nodules each less than or equal to 3 cm in size,
patients with Child–Pugh B or C cirrhosis, patients who are not candidates for primary
liver resection, and patients who have tumors with no evidence of macrovascular
invasion.

Question 17.16 The answer is D.


Multiple clinical staging systems for hepatic tumors have been described. The most widely
used is the American Joint Committee on Cancer/Tumor-Node-Metastasis (TNM). A
staging system based entirely on clinical grounds that incorporate the contribution of the
underlying liver disease was originally developed by Okuda et al. Adverse prognostic
signs are as listed. Patients with Okuda stage III (advanced), namely, with three or more
positive features, have a dire prognosis because they usually cannot be curatively resected
and the condition of their liver typically precludes chemotherapy. The American
Association for the Study of Liver Diseases has endorsed the use of the Barcelona Clinic
Liver Cancer system for staging of HCC. This has now been validated both internally and
externally in several studies.

Question 17.17 The answer is A.


Various paraneoplastic syndromes have been described to occur in patients with
hepatocellular cancer. Most of these are biochemical abnormalities without associated
clinical consequences. The most important ones include hypoglycemia (also caused by
end-stage liver failure), erythrocytosis, hypercalcemia, hypercholesterolemia,
dysfibrinogenemia, carcinoid syndrome, increased thyroxin-binding globulin, sexual
changes (gynecomastia, testicular atrophy, and precocious puberty), and porphyria
cutanea tarda. The pathogenesis of hypoglycemia is unclear but may be related to
production of insulin-like growth factor I by the tumor.

Question 17.18 The answer is E.


Chronic viral hepatitis due to both hepatitis B and C account for 75% of HCC cases
worldwide. Primary biliary cirrhosis is associated with an increased risk of
cholangiocarcinoma as well as hepatocellular carcinoma. Conditions causing cirrhosis such
as Wilson disease, chronic alcohol abuse, autoimmune hepatitis and cryptogenic cirrhosis
are risk factors for HCC. Hereditary hemochromatosis and α-1-antitrypsin deficiency also
place a patient at high risk. Noncirrhotic conditions such as hereditary tyrosinemia,
porphyria cutanea tarda, galactosemia, hereditary hemorrhagic telangiectasia, and
cigarette smoking are also associated with HCC.

Question 17.19 The answer is B.


A combination of ultrasonography and serum AFP levels is used for the screening of high-
risk populations for HCC. Although screening programs are in place in endemic areas of
HCC and in high-risk populations, it is unclear whether screening identifies patients at an
earlier stage or improves patient survival. Detection of HCC, through surveillance of
patients awaiting liver transplantation, results in increased priority for orthotopic liver
transplantation.

Question 17.20 The answer is D.


In most patients, cholangiocarcinoma is sporadic and no precipitating factor can be
identified. Risk factors that have been associated with the development of
cholangiocarcinoma can be divided into congenital (choledochal cysts, anomalous
pancreatic-biliary tree junction), autoimmune (primary biliary cirrhosis), infectious
(Clonorchis sinensis and Opisthorchis viverrini infestation, chronic portal bacteremia, and
portal phlebitis), and finally environmental exposures (the radiocontrast agent thorotrast
and possibly cigarette smoking).

Question 17.21 The answer is B.


Approximately one-third of patients presenting with the suspected diagnosis of
cholangiocarcinoma will have resectable disease. Operative mortality averages
approximately 8%, indicating the high-risk population that this tumor affects and the
complexity of the procedure. Approximately 10% to 35% of patients survive 5 years after
surgical resection. Recurrences occur most commonly at the bed of resection, followed by
retroperitoneal lymph nodes. Distant metastases occur in one-third of cases. The most
common sites of metastases are the lung and mediastinum, followed by liver and
peritoneum. Comparisons of outcome over time suggest improved outcome in more
recent series as a result of routine inclusion of liver resection. Prognostic factors for
survival include negative microscopic margin status, lymph node metastases, tumor size,
tumor grade, preoperative serum albumin, hepatic resection, and postoperative sepsis.

Question 17.22 The answer is C.


The pathologic stage is T2 NX (stage II) gallbladder cancer. Tumor invading the
perimuscular connective tissue but not extending through the serosa is classified as T2 per
the seventh edition of the AJCC Cancer Staging Manual.

Question 17.23 The answer is A.


Numerous studies have demonstrated that simple cholecystectomy is curative for stage I
disease (T1, N0). Recent studies have suggested that the prognosis is different for pT1a
and pT1b tumors after simple cholecystectomy. Invasion of the muscular layer allows
access to lymphatics and vessels, providing the rationale for extended cholecystectomy in
this population. When an extended cholecystectomy is performed for T2 disease, the 5-
year survival has been reported to be as high as 100%, but probably falls in the range of
70% to 90%. Simple cholecystectomy alone is associated with a 5-year survival rate of
20% to 40.5%. Lymph node metastases are seen in 46% of patients with T2 primary
tumors, providing another reason in favor of radical repeat resection after simple
cholecystectomy. In series of extended cholecystectomies, the operative morbidity ranges
from 5% to 46%, and the mortality ranges from 0% to 21%. The risk of resection for each
patient and each type of resection needs to be weighed against the chance of benefiting
from the procedure on the basis of the tumor stage.

Question 17.24 The answer is A.


Balachandran et al. reported on 117 patients with gallbladder cancer, of whom 80
underwent simple cholecystectomy and 37 underwent extended resections. Seventy-three
patients received adjuvant chemoradiotherapy and 44 patients did not. The median
survival of all 117 patients was 16 months. On multivariate analysis, the T stage and use
of adjuvant therapy were the only statistically significant independent predictors of
survival. Median survival was 24 months and 11 months, in patients with or without
adjuvant chemoradiotherapy (p = 0.001), respectively, and this difference was most
pronounced for patients with T3, node-positive disease, or after a simple cholecystectomy.
Ben-David et al. reported on 14 patients with gallbladder cancer treated at the University
of Michigan with resection followed by radiotherapy or chemoradiotherapy. The median
radiation dose was 54 Gy, and approximately half the patients received concurrent
chemotherapy. The median survival was 23 months. Interestingly, there was no difference
in survival between patients with R0 or R1 resection. No differences were observed in
survival or pattern of failure between patients with gallbladder cancer and bile duct
cancer (distal or hilar). The high risk of systemic spread and locoregional failure
associated with gallbladder cancer that extends beyond the mucosa has led most cancer
centers in the United States to recommend consideration of adjuvant chemotherapy and
radiotherapy. For external beam radiation therapy, the target volume should include the
gallbladder fossa and adjacent liver, as well as the regional nodal areas.

Question 17.25 The answer is C.


Fibrolamellar HCC is a rare histologic variant of HCC. Most patients present in the third
decade of life, and it affects men and women equally. This variant of HCC is uncommonly
associated with prior cirrhosis or viral hepatitis. In addition, a higher proportion of
patients with fibrolamellar HCC have lymph node metastases at presentation than the
usual HCC.

Question 17.26 The answer is B.


This man has a hepatic mass within a cirrhotic liver, which is very suspicious for HCC.
The radiologic features are also highly suggestive of HCC. If the mass is larger than 2 cm
in size with characteristic appearance of HCC on imaging and AFP is higher than 200
ng/mL, a biopsy is not essential for management. Patients with a high clinical suspicion
for HCC who are deemed appropriate surgical candidates should be taken to surgery
without a preoperative biopsy.

Question 17.27 The answer is D.


Hepatoblastoma is the most common primary cancer of the liver in children. The peak
incidence is within the first 2 years of life. Surgical resection is the first line of therapy.
However, these tumors are very chemosensitive, and neoadjuvant chemotherapy can
render unresectable tumors operable. Five-year survival after liver transplantation is
excellent (83%).

Question 17.28 The answer is B.


The NS5A protein product of HCV genome has been demonstrated to inactivate p53 by
sequestration. The hepatitis B x gene product has been implicated in causing HCC because
it is a transcriptional activator of various cellular genes associated with growth control.
The elevated serum level of HBV DNA (a marker of higher levels of HBV replication) is
associated with a higher risk of HCC. The HBV genotype C is generally thought to
increase the risk of HCC because these individuals are likely to remain seropositive for
hepatitis B e antigen and thus have higher serum levels of HBV DNA for a longer time.

Question 17.29 The answer is D.


Hepatitis B virus is a DNA virus and it may become integrated within the chromosomes of
infected hepatocyte. On the other hand, hepatitis C virus is an RNA virus without a DNA
intermediate form, and therefore cannot integrate into hepatocyte DNA.

Question 17.30 The answer is A.


This patient is an ideal candidate for locoregional therapy while waiting for liver
transplantation. Due to donor organ shortages, patients may wait for months prior to
surgery. Transarterial chemoembolization (TACE) has been shown to reduce dropout from
the wait list, reduce posttransplant recurrence, and occasionally downstage tumors that
are initially beyond transplant criteria. Radiofrequency ablation or TACE may be used for
definitive therapy in patients, not candidates, for surgical resection or transplantation.
Contraindications to TACE include portal vein thrombus, poorly compensated cirrhosis.
Ascites is a relative contraindication as it indicates elevated portal vein pressure
portending a higher risk of worsening liver function with TACE. Sorafenib would not be a
good choice for a patient on the wait list for surgery because of its VEGF inhibition.
Neoadjuvant chemotherapy with PIAF (cisplatin, IFNα-2b, doxorubicin, and 5FU) has been
shown in a phase II trial in patients with advanced HCC to have a 26% response rate and
some patients who achieved a partial response went on to have surgery. This patient is
already a candidate for surgery and the toxicity of this regimen may complicate plans for
transplantation. Observation alone would not be appropriate given the wait time for
transplantation and the available therapies for locoregional control including surgery and
intravascular procedures.

Question 17.31 The answer is B.


The UK-ABC-02 trial was a large phase III randomized control trial conducted after the
phase II portion demonstrated an improvement in progression-free survival (PFS) in
patients with locally advanced or metastatic biliary tract cancers treated with either
gemcitabine and cisplatin or gemcitabine alone. The trial redemonstrated the PFS
advantage (8 months vs. 5 months, p <0.001) and met its primary endpoint of overall
survival (OS) with a median OS of 11.7 months in the gemcitabine and cisplatin arm and
8.1 months in the gemcitabine-alone arm (HR, 0.64; 95% CI, 0.52 to 0.8; p <0.001). This
combination was tolerated without substantially increased toxicity. There is no high-
quality evidence for second-line therapy and in fact only 17% of patients in the UK-ABC-
02 trial went on to receive second-line therapy. 5FU containing regimens as second-line
therapy is supported by a small series of patients who received FOLFOX4 (four of
eighteen patients responded) but high-quality evidence is lacking to guide second line and
beyond therapy and clinical trial enrollment is encouraged. There is mixed literature for
radiation therapy used in the palliative setting for cholangiocarcinoma. The risk of
toxicity is substantial but a small phase II trial suggested the median OS may be superior
to chemotherapy with stereotactic body radiotherapy. Larger comparative trials are
needed. Given the survival benefit with gemcitabine and cisplatin, palliative care alone
would not be appropriate unless requested by the patient or the patient is medically unfit.

Question 17.32 The answer is E.


The following groups should undergo surveillance for HCC.
Hepatitis B carrier—Asian men more than 40 years old, Asian women more than 50
years old, Africans more than 20 years old, all cirrhotic hepatitis B carriers, family history
of HCC and patients with high hepatitis B virus DNA with ongoing hepatic injury.
Nonhepatitis B cirrhosis—hepatitis C, alcoholic cirrhosis, genetic hemochromatosis, and
primary biliary cirrhosis
Tests that have been used for surveillance for HCC include serologic tests
(measurement of serum AFP) and radiologic tests (such as ultrasonography). Although the
ideal surveillance interval is not known, a surveillance interval of 6 to 12 months has
been proposed based on estimates of tumor doubling time.

Question 17.33 The answer is D.


This patient has Child–Pugh C cirrhosis at presentation and unresectable HCC. He would
not have qualified for enrollment in the SHARP study as only patients with Child–Pugh A
were enrolled. In clinical practice, patients with Child–Pugh B cirrhosis are often offered
sorafenib therapy if no viable alternatives such as clinical trial enrollment are available.
However, survival is markedly shorter than patients with preserved hepatic function and
there is no evidence that sorafenib has a meaningful impact on survival in this population.
A phase III trial compared sunitinib with sorafenib in patients with Child–Pugh A cirrhosis
and was stopped early due to poorer survival in the sunitinib arm. This patient should be
offered hospice services or supportive medical care given his poor prognosis and lack of
evidence-based treatment options.

Corresponding chapters in Cancer: Principles & Practice of Oncology, Tenth Edition: 48 (Molecular Biology of Pancreas
Cancer), 49 (Cancer of the Pancreas), 51 (Molecular Biology of Liver Cancer), 52 (Cancer of the Liver), and 53 (Cancer of the
Biliary Tree).
18 Small Bowel Cancers and Gastrointestinal Stromal
Tumors (GIST)
Benjamin R. Tan, Jr.

QUESTIONS
Each of the numbered items below is followed by lettered answers. Select the BEST answer(s).

Question 18.1 A 60-year-old previously healthy woman noted abdominal distension and
discomfort for 6 months, associated with nausea and vomiting. Computed tomography (CT)
scan shows a 20 × 25-cm abdominal mass, and exploratory laparotomy demonstrated a
pedunculated mass arising from the stomach. No other metastases were found. A partial
gastrectomy was done, and pathology revealed a gastrointestinal stromal tumor (GIST) which
strongly stains for CD117 and CD34. Sixty mitoses were seen per 50 high-power field (HPF).
Which of the following is TRUE regarding GISTs?
A. The most common mutation associated with GIST involves the inactivation of a tumor
suppressor gene.
B. Both tumor size and mitotic index predict response to imatinib therapy.
C. Gastric GISTs are associated with relatively worse outcomes compared with small
intestinal GISTs.
D. Patients with metastatic GIST tumors harboring exon 9 mutations have a better
prognosis and response to imatinib compared with those with exon 11 mutation.
E. None of the above.

Question 18.2 After the patient in Question 18.1 has recovered from surgery, what would
you recommend for this patient on the basis of current data?
A. Observation with serial scans
B. Imatinib 400 mg per os (PO) daily for 1 year
C. Imatinib 400 mg PO daily for at least 3 years
D. Sunitinib 50 mg 4 weeks on/2 weeks off therapy for 5 years

Question 18.3 A 52-year-old woman with metastatic gastric GIST had an initial complete
response to daily imatinib 400 mg with resolution of her hepatic and peritoneal metastases
after 6 months of therapy. Imatinib was continued for 18 months when her CT scan showed
recurrent hepatic lesions. Imatinib was increased to 800 mg daily. However, subsequent scans
revealed progressive disease. You recommend starting sunitinib for this patient. Which of the
following statements are TRUE? (Select two correct responses)
A. Acquired resistance to imatinib therapy may be associated with the development of
secondary KIT or PDGFRA mutations.
B. Sunitinib therapy for patients with imatinib-resistant GIST improved progression-free
survival compared with placebo.
C. Patients with GIST harboring exon 11 mutation have a higher response to sunitinib than
those with exon 9 mutation.
D. Patients with the wild-type GIST are resistant to both imatinib and sunitinib therapy.

Question 18.4 The patient in Question 18.3 started sunitinib 50 mg daily for 28 days
followed by a 2-week break. After two cycles, repeated CT scans showed a decrease in the
measurable lesions. In addition to hypopigmentation of her hair, she also noted progressive
generalized fatigue. Patient denies any dyspnea on exertion, diarrhea, or pedal edema.
Physical examination reveals an erythematous rash in the hands, clear lungs, no cardiac
gallops or rubs, and no focal neurologic deficits. Pertinent laboratory tests are as follows:
White blood cell 5.6 × 103 cells/μL
Hemoglobin 11.8 g/dL
Sodium 145 mmol/L
Potassium 4.5 mmol/L
Creatinine 0.8 mg/dL
Total bilirubin 0.5 mg/dL
Alkaline phosphatase 118 μ/L
What would you order next?
A. Magnesium level
B. Magnetic resonance imaging of the brain
C. Thyroid function tests
D. 25-Hydroxycholecalciferol level

Question 18.5 After 6 months of sunitinib therapy, the patient in Question 18.4 was noted to
have progression of her liver and omental lesions. Her Karnofsky performance status is 90%
and hematologic, renal, and hepatic functions are all adequate. She is interested in further
therapy. What would be your best recommendation?
A. No further therapy and proceed with hospice because there is no standard therapy after
second-line sunitinib.
B. Sorafenib
C. Regorafenib
D. Bevacizumab

Question 18.6 Familial and genetic syndromes associated with GIST are: (Select two correct
responses)
A. Cowden syndrome
B. Li–Fraumeni syndrome
C. Carney triad
D. Neurofibromatosis

Question 18.7 What is the most common type of small bowel malignancy?
A. Adenocarcinoma
B. Lymphoma
C. Carcinoid
D. Schwannoma

Question 18.8 Which of the following are characteristic of primary intestinal mucosal-
associated lymphoid tissue (MALT) lymphoma? (Select two correct responses)
A. Association with Hashimoto thyroiditis
B. The majority of patients present with stages I and II
C. Most common in women
D. Associated with the translocation t(11;14)

Question 18.9 A 50-year-old male presented with bloating, and tarry stools. Upper
endoscopy revealed a mass in the third portion of the duodenum, with the biopsy showing
moderately differentiated adenocarcinoma. Resection demonstrated three periduodenal
lymph nodes involved with carcinoma. CT scan showed no distant metastases. Which of the
following statements regarding small bowel adenocarcinoma is TRUE?
A. The duodenum is the most common location for adenocarcinoma.
B. Jejunal and ileal tumors are associated with worse outcomes compared to duodenal
adenocarcinoma.
C. Stage III small bowel adenocarcinoma is associated with a 63% survival.
D. Adjuvant therapy with irinotecan, 5-fluorouracil, and leucovorin is standard for stage III
duodenal adenocarcinoma.

Question 18.10 Which of the following is a criterion to differentiate between primary


intestinal and secondary lymphomas?
A. No superficial adenopathy
B. No evidence of splenic involvement except through direct extension of the primary
tumor
C. No evidence of peripheral blood or bone marrow involvement
D. All the above

Question 18.11 The risk for progressive disease for a patient with small intestinal GIST
measuring less than 2 cm with more than five mitoses/50 hpf is:
A. 0%
B. 4.3%
C. 24%
D. 50%
E. 85%

Question 18.12 A 20-year-old student presented to the emergency department with a 2-day
history of right lower abdominal pain associated with fever. His abdomen was slightly
distended with diffuse tenderness but without guarding or rebound tenderness. Rectal
examination showed no masses and was negative for occult blood. Complete blood count
showed a slightly elevated white blood cell at 11,000. CT scan showed a mass in the terminal
ileum with no evidence of free peritoneal air. Colonoscopy revealed a terminal ileum mass,
the biopsy of which showed sheets of monotonous round nucleated cells with abundant
basophilic cytoplasm with numerous macrophages. Ki-67 index was 100%. What is your
diagnosis?
A. MALT lymphoma
B. Burkitt lymphoma
C. Peripheral T-cell lymphoma
D. Medullary carcinoma of the small intestine
ANSWERS

Question 18.1 The answer is E.


In the majority of malignant GISTs, a gain-of-function mutation results in the constitutive
activation of the KIT proto-oncogene. Approximately 80% of GISTs harbor KIT mutations,
whereas another 5% to 7% have activating PDGFRA mutations. The most common
mutation is identified in the KIT juxtamembrane domain, exon 11. Other mutations in
exon 9, 13, 17, and 18 have been described. In the Intergroup US–Finland study, higher
response rates and better outcomes were observed among patients with GIST with the
exon 11 mutation compared with those with exon 9 mutation. Objective responses to
imatinib therapy among patients with GIST with exon 11 and exon 9 mutations are
approximately 70% and 40%, respectively. Patients with wild-type GIST have a response
rate of approximately 30%. Median time to progression also is longer for those with exon
11 compared with other genotypes. The higher prevalence of a specific exon 9 mutation
among GISTs originating in the small intestine may explain in part the observation of
worse prognosis of these tumors compared with gastric GISTs. Although tumor size and
mitotic index carry prognostic significance for patients with GIST, neither has been
associated with response to tyrosine kinase inhibition.

Question 18.2 The answer is C.


Based on the tumor size and mitotic rate, this patient is at high risk for recurrence. Thus,
observation and serial scans will not be appropriate. Adjuvant imatinib for 3 years is
associated with increased recurrence-free and overall survival compared to 1 year. There
are no randomized studies on the use of sunitinib in the adjuvant setting.

Question 18.3 The answers are A and B.


Primary resistance to imatinib, manifested by continued tumor growth within the first 6
months of imatinib therapy, occurs in a minority of GISTs. Secondary resistance generally
occurs after a median of 24 months of continued tyrosine kinase inhibition. Acquired
mutations in KIT or PDGFRA have been implicated in the development of drug resistance
to imatinib. The emergence of KIT-independent genotypes may also result in imatinib
resistance. Sunitinib was approved by the Food and Drug Administration for the treatment
of GIST after disease progression or intolerance to imatinib. The pivotal Phase III study
demonstrated a superior time to progression of 27.3 weeks for patients treated with
sunitinib compared with 6.4 weeks for patients treated with placebo. Six-month survival
rates also favor sunitinib at 79.4% compared with 56.9% for placebo. With the crossover
design, the advantage for survival benefit on longer follow-up diminished. A subset
analysis showed improvement in response rates and outcomes for patients with GIST
harboring exon 9 mutation compared with those harboring exon 11 mutations. Responses
were also observed among patients with wild-type genotype.

Question 18.4 The answer is C.


Biochemical and clinical hypothyroidism has been associated with sunitinib therapy.
Routine monitoring of thyroid function test is warranted and thyroid hormone
replacement therapy may alleviate the symptoms. The precise mechanism for sunitinib-
associated hypothyroidism is unclear. Destructive thyroiditis through follicular apoptosis
has been postulated for sunitinib-associated thyroid dysfunction based on the observation
of transient thyroid-stimulating hormone suppression and subsequent absence of
visualized thyroid tissue. Other causes of fatigue associated with sunitinib include adrenal
insufficiency or congestive heart failure.

Question 18.5 The answer is C.


Regorafenib, a tyrosine kinase inhibitor targeting KIT, PDGFR, and VEGFR1,2 and 3, is
the standard third-line treatment for GIST refractory to imatinib and sunitinib. Although
sorafenib appears to have activity against GIST, it is not approved as a standard third-line
therapy for GIST due to lack of confirmatory Phase III studies. There are no current data
on the use of bevacizumab, an anti-VEGF monoclonal antibody for the treatment of GIST.

Question 18.6 The answers are C and D.


Both familial paraganglioma and neurofibromatosis type 1 are associated with GIST.
These syndromes, along with pediatric GIST, can be characterized by the wild-type KIT
gene. Carney triad, which encompasses multifocal GIST with pulmonary chondromas and
extra-adrenal paraganglioma is another syndrome associated with GIST. Cowden
syndrome is an autosomal dominant subtype of the PTEN hamartoma syndrome associated
with a lifetime increased risk for various cancers including breast, colorectal, endometrial,
renal cancers, and melanoma. Li–Fraumeni syndrome is an autosomal dominant hereditary
disorder characterized by TP53 mutation and the development of multiple malignancies
including sarcoma, breast cancer, colorectal and pancreatic cancers, lymphoma,
melanoma, and germ cell tumors.

Question 18.7 The answer is C.


According the National cancer database, the most common tumors of the small bowel are
carcinoids. Carcinoids arise from the neuroendocrine cells of Kulchitsky and have
increased fourfold in incidence during the last two decades. Carcinoids are followed by
adenocarcinomas and lymphomas as the most common small intestinal malignancies.
Sarcomas are less likely and schwannomas are rare.

Question 18.8 The answers are A and B.


MALT lymphomas are predominantly seen in men, with a peak incidence in the sixth
decade. These malignancies may be associated with chronic inflammatory disorders such
as Hashimoto thyroiditis and Sjögren disease. The majority of patients present with stages
I or II. The translocation t(11;14) causing overexpression of cyclin D1 is characteristic of
mantle cell lymphoma.

Question 18.9 The answer is A.


Small bowel adenocarcinomas are rare malignancies which are slightly more common in
men. The duodenum represents with most common location, accounting for
approximately half of the cases. Compared to more distal lesions, tumors of the
duodenum tend to occur in older patients and are associated with worse outcomes.
Overall survival for patients with small bowel adenocarcinomas for patients with stage I,
II, III, and IV disease are as follows: 63%, 48%, 32% and approximately 4%. Despite the
high risk of recurrence for patients with stage II and III disease, there are no standard
adjuvant therapy regimens.

Question 18.10 The answer is D.


The main criteria to distinguish between primary intestinal lymphomas and secondary
involvement include no superficial palpable lymphadenopathy, no mediastinal
adenopathy, no evidence of peripheral blood or bone marrow involvement, no
involvement of the liver or spleen unless by direct extension of the primary tumor, and
disease confined to the affected small bowel and regional draining mesenteric lymph
nodes.

Question 18.11 The answer is D.


GISTs arising from the small intestine appear to have a more malignant behavior at a
smaller size compared with those found in the stomach or in the colon. For small tumors
less than 2 cm, those with less than 5 mitoses per 50 hpf have a 0% risk of progression
according to one study, whereas those with more than 5 mitoses per 50 hpf have a 50%
risk for progression. GISTs larger than 10 cm with more than 10 mitoses per 50 hpf have
a 90% risk of progression.

Question 18.12 The answer is B.


This patient has Burkitt lymphoma arising from the terminal ileum presenting with
appendicitis-like symptoms. This type of lymphoma accounts for less than 5% of all
lymphomas in the small intestine. A “starry-eyed” pattern interspersed throughout
monomorphic cells with abundant basophilic cytoplasm is characteristic and related to
numerous macrophages with ingested apoptotic tumors within. A high proliferative index
is also characteristic of this malignancy. Treatment consists primarily of aggressive
chemotherapy.

Corresponding chapters in Cancer: Principles & Practice of Oncology, Tenth Edition: 54 (Cancer of the Small Bowel) and 55
(Gastrointestinal Stromal Tumor).
19 Colorectal and Anal Cancers
Ashley Morton and Benjamin R. Tan, Jr.

QUESTIONS
Each of the numbered items below is followed by lettered answers. Select the ONE lettered answer
that is BEST in each case unless instructed otherwise.

Question 19.1 A 30-year-old multigravid woman presents with a large abdominal mass
associated with abdominal pain. She underwent a total proctocolectomy for colon cancer
secondary to familial adenomatous polyposis (FAP) 4 years ago followed by adjuvant
chemotherapy for node-positive disease. A recent esophagogastric endoscopy revealed only a
tubulovillous adenoma in the duodenum but the computed tomography (CT) scan revealed a
10-cm mass filling the pelvis. Biopsy showed desmoid tumor. A TRUE statement regarding
desmoid tumors is:
A. Desmoid tumors represent the second most common cause of death for patients with
FAP.
B. Desmoid tumors are uniformly aggressive and locally invasive.
C. Adjuvant radiation is recommended.
D. Adjuvant chemotherapy is recommended.

Question 19.2 A 55-year-old woman underwent a right hemicolectomy for a cecal mass, 15
years ago. Four years later, she was found to have a hepatic flexure adenocarcinoma and a
total colectomy was performed. Four years ago, she presented with postmenopausal bleeding
and was found to have endometrial adenocarcinoma. She also had multiple skin malignancies,
including sebaceous adenomas and keratoacanthomas. One year ago, she underwent a
Whipple procedure for a duodenal adenocarcinoma invading into the pancreas. She has three
siblings and a paternal uncle with colorectal cancer all diagnosed in their late 30s. What is
the most probable primary genetic explanation for her inherited colorectal cancer syndrome?
A. Chromosomal instability characterized by the deletion or mutation of a tumor
suppressor gene
B. Chromosomal instability characterized by activation of an oncogene
C. Microsatellite instability (MSI) caused by germline mutations in a DNA mismatch repair
(MMR) gene
D. MSI caused by epigenetic changes associated with hypermethylation in CpG islands

Question 19.3 Other than genetic counseling, which test would you order to confirm the
diagnosis in the patient in Question 19.2?
A. TP53 mutation and loss of heterozygosity of chromosome 18q
B. APC mutation, including the I1307K allele
C. MSI testing and methylation of CpG islands
D. MSI test and MMR gene mutation including MLH1, MSH2, and MSH6

Question 19.4 The patient in Question 19.3 had a positive test result. What screening tests
and surveillance program would you recommend to her three daughters aged 24, 22, and 20
years?
A. Colonoscopy now and repeat every 1 to 2 years and transvaginal ultrasound for her
daughters starting at age 30 to 35 years
B. Colonoscopy for all and transvaginal ultrasound for her daughters starting at age 30 to
35 years
C. Colonoscopy now and repeat every 1 to 2 years with transvaginal ultrasound at age 30
to 35 years only for her children confirmed to have the same genetic mutation as the
patient; colonoscopy at age 40 to 50 years for those with no mutation
D. Colonoscopy and transvaginal ultrasound at age 30 to 35 years only for those confirmed
with the same mutation as the patient; colonoscopy at age 40 to 50 years for those with
no mutation

Question 19.5 Which gene is associated with hereditary nonpolyposis colorectal cancer
(HNPCC)?
A. APC
B. MYH
C. STK11
D. MSH2

Question 19.6 A 65-year-old woman presented with intermittent constipation and diarrhea
associated with abdominal cramping. A colonoscopy revealed a near-obstructing mass and
biopsy showed a villoglandular polyp. She underwent an exploratory laparotomy and
resection of a 5 × 5-cm circumferential necrotic and fungating mass. Pathology revealed a
moderately differentiated adenocarcinoma invading into the pericolonic fat with 2 of 30
positive lymph nodes. Margins were negative. CT scan showed no evidence of metastatic
disease. After recovery from her surgery, based on current evidence, you would recommend:
A. 5FU with leucovorin × 6 months
B. Capecitabine × 6 months
C. Irinotecan with 5FU (FOLFIRI) × 6 months
D. Oxaliplatin with 5FU (FOLFOX) × 6 months
E. Oxaliplatin with 5FU (FOLFOX) and bevacizumab × 6 months

Question 19.7 One week after the first dose of chemotherapy, the patient in Question 19.6
developed a fever of 102°F associated with chills. She also developed diarrhea, mucositis,
confusion, and ataxia. Repeat complete blood count showed a white blood cell count of 0.5 ×
103 cells/μL with an absolute neutrophil count of 100, hemoglobin of 11.7 g/dL, and platelet
count of 42,000. Which one of the following pharmacogenetic conditions would best explain
her clinical course?
A. The patient is homozygous for the thymidylate synthase (TYMS) *3/*3 polymorphism.
B. The patient is homozygous for UGT1A1*28 polymorphism.
C. The patient is heterozygous for the IVS14 + 1 G>A DPYD*2A mutation.
D. The patient is heterozygous for the ERCC2 Lys751Gln polymorphism.

Question 19.8 A 62-year-old engineer was diagnosed with metastatic cecal adenocarcinoma
to the lung and liver. He was initially treated with oxaliplatin plus infusional 5FU (FOLFOX6)
and bevacizumab. After four cycles (2 months), his CT scan showed progressive disease. You
discussed irinotecan 180 mg/m2 every 2 weeks plus weekly cetuximab based on the results of
the EPIC study. A test for UGT1A1 polymorphism was done, and results revealed him to be
homozygous for UGT1A1*28/*28. Which of the following statements is/are TRUE?
A. Patients homozygous for the UGT1A1*28 polymorphism glucuronidate SN38 more
efficiently than those with the wild-type *1 genotype.
B. Patients homozygous for the UGT1A1 *28 polymorphism glucuronidate SN38 less
efficiently than those with the wild-type *1 genotype.
C. Patients homozygous for UGT1A1*28 polymorphism are at greater risk for severe
neutropenia with irinotecan compared with those with the wild-type *1 genotype.
D. A and C are true.
E. B and C are true.

Question 19.9 A 52-year-old teacher presents with a 2-month history of rectal bleeding. A
rectal examination revealed a palpable nonobstructing mass 5 cm from the anal verge.
Subsequent colonoscopy confirmed a friable, tethered mass biopsy that showed moderately
differentiated adenocarcinoma. Transrectal ultrasound revealed a T3N1 cancer. CT scans of
the chest, abdomen, and pelvis revealed no metastatic sites. Which of the following treatment
strategies would you recommend?
A. Total mesorectal excision (TME) alone
B. TME followed by chemotherapy
C. TME followed by chemoradiation
D. Neoadjuvant 5FU-based chemoradiation followed by TME and adjuvant chemotherapy

Question 19.10 Which of the following appropriately staged patients with rectal cancer
would be the best candidate for transanal excision?
A. A 48-year-old woman with a 2.5-cm T1 moderately differentiated rectal
adenocarcinoma 4 cm from the anal verge.
B. A 65-year-old man with a 4.5-cm T2 well-differentiated circumferential rectal mass 6 cm
from the anal verge.
C. A 30-year-old man with a 2-cm well-differentiated T1 mucinous adenocarcinoma 12 cm
from the anal verge.
D. A 52-year-old woman with a 3-cm well-differentiated T1N1 adenocarcinoma 5 cm from
the anal verge.
E. None of the above.

Question 19.11 For the patient you have selected for transanal resection in Question 19.10,
pathologic review of the excised specimen showed no lymphovascular invasion and all
margins were negative. Which of the following options would you recommend?
A. No further therapy
B. Short-course (25 Gy/5 fractions) radiation
C. Intracavitary radiation
D. Adjuvant chemotherapy without radiation
E. Adjuvant chemotherapy with radiation

Question 19.12 Which of the following are TRUE statements regarding MYH-associated
polyposis? (Select two correct responses)
A. Inheritance is autosomal dominant.
B. Clinical features of MYH-related polyposis may be similar to FAP.
C. The MYH gene is a base-excision repair gene.
D. A deficiency in MYH leads to accumulation of germline mutations in the APC gene.

Question 19.13 A 49-year-old woman noted rectal bleeding for 2 months. She denied any
pain, diarrhea, constipation, or weight loss. A colonoscopy was done that revealed a 2-cm
low-lying mass 1 cm from the anal verge. A transrectal ultrasound revealed no lymph nodes.
Biopsy showed basaloid squamous cell carcinoma. CT scans of the abdomen and pelvis did
not reveal any metastatic disease. The BEST curative treatment option for this patient is:
A. APR
B. Short-course (25 Gy/5 fractions) radiation followed by APR
C. Neoadjuvant chemoradiation with 5FU followed by APR
D. Chemoradiation alone with 5FU and mitomycin

Question 19.14 A 58-year-old man presented with a 6-month history of anorexia, fatigue,
and a vague right-sided abdominal discomfort. His physical examination was unremarkable
except for mild pallor. Initial laboratory test revealed a hemoglobin level of 10.7 g/dL with a
mean corpuscular volume of 73. He also had mildly elevated alkaline phosphatase and
hepatic transaminases. Colonoscopy revealed a nonobstructing transverse colon mass. Biopsy
demonstrated moderately differentiated adenocarcinoma. CT scan revealed a 3-cm lesion in
the left lobe of the liver and two other lesions measuring 1.5 to 2 cm in the right lobe of the
liver. Positron emission tomography revealed uptake in the transverse colon and all three
known hepatic lesions. Among the following options, what would be the best option for this
patient based on current studies?
A. Curative-intent resection of the transverse colon primary and all three hepatic lesions
followed by active surveillance
B. Palliative-intent front-line chemotherapy with bevacizumab for metastatic colon cancer
until progression, followed by palliative second-line chemotherapy
C. Curative-intent resection of the transverse colon primary and all three hepatic lesions
with perioperative chemotherapy
D. Curative-intent resection of the transverse colon primary and all three hepatic lesions
with radiotherapy
Question 19.15 Which genetic change is associated with resistance to treatment with
cetuximab?
A. KRAS mutation
B. EGFR over expression
C. KRAS wild type
D. MLH1 inactivation

Question 19.16 A 54-year-old woman has completed six cycles of first-line therapy for
metastatic KRAS codon 12 mutated colon cancer with FOLFOX and bevacizumab, and was
shown to have stable disease on restaging scans. Her CEA has also decreased from 135 ng/mL
at initial diagnosis to 4.5 ng/mL. She is asking about taking a break from chemotherapy
altogether for the next few months given the stable disease. When considering observation
versus maintenance chemotherapy in metastatic colorectal cancer, you inform her that:
A. There is no benefit in maintenance therapy with capecitabine and bevacizumab and
observation is preferred.
B. She should switch to FOLFIRI–panitumumab given no response noted on imaging.
C. Maintenance therapy with capecitabine and bevacizumab will lengthen the time to
progression and is preferred
D. She needs to complete a total of 12 cycles of oxaliplatin-based regimen before she can
consider observation.

Question 19.17 Common clinical characteristics for BRAF V600E colorectal tumors include
all of the following: (Select two correct responses)
A. Male
B. Right-sided tumors
C. Low-grade features
D. MSI-H

Question 19.18 High-risk features for stage II colon cancers include which TWO of the
following characteristics?
A. T4 perforated tumor
B. Moderately differentiated
C. Sampling of 0/8 lymph nodes
D. No lymphovascular invasion
ANSWERS

Question 19.1 The answer is A.


Desmoid disease is second only to colorectal cancer as a common cause of death among
patients with FAP. There is heterogeneity in the clinical presentation of desmoid tumors
from asymptomatic abdominal mass to bowel or ureteral obstruction. Sporadic desmoid
tumors may also occur. Retrospective studies have not confirmed the role of adjuvant
radiation or chemotherapy for desmoid tumors. For unresectable tumors, chemotherapy,
hormonal therapy, and targeted agents such as imatinib have been reported in literature
as possible palliative options.

Question 19.2 The answer is C.


This patient meets the Amsterdam criteria for HNPCC with at least three affected relatives
in at least two successive-generation relatives and one first-degree relative diagnosed with
colorectal cancer before age 50 years. Unlike FAP, which is characterized by chromosomal
instability caused by mutations in the APC tumor suppressor gene, HNPCC is
characterized by MSI caused by mutations in the DNA MMR genes. HNPCC is the most
common hereditary syndrome predisposing one to colorectal cancers, accounting for
approximately 2% to 3% of all colorectal cancer cases. The lifetime risk for developing
colon cancer among patients with HNPCC approaches 80%. HNPCC is also associated with
other malignancies, including endometrial, gastric, ampullary, biliary, and urinary tract
cancers. This patient has Muir–Torre syndrome characterized by multiple colon cancers
and multiple cutaneous neoplasias, including sebaceous adenomas.

Question 19.3 The answer is D.


HNPCC can be confirmed by the demonstration of the MSI-H phenotype and germline
mutation in any of the DNA MMR genes, such as MLH1 on chromosome 3p, MSH2 on
chromosome 2p, MSH6 on chromosome 2p, PMS1 on 2q, and PMS2 on 7q. Germline
mutations involving MSH2 and MLH1 genes account for more than 60% of the known
mutations present in patients with HNPCC. The National Comprehensive Cancer Network
guidelines recommend the use of tumor screening with MSI and lack of expression of
MMR protein expression by immunohistochemistry initially, followed by MMR mutation
testing, although proceeding directly to MMR mutation testing is also acceptable. A
negative MMR mutation test result does not rule out Lynch syndrome or HNPCC.

Question 19.4 The answer is C.


Once a germline MMR mutation is identified in a patient, genetic counseling and testing
of at-risk family members are essential. The patient’s three daughters, along with all her
first-degree relatives, have a 50% probability of being a carrier of the mutant gene. In
general, for family members who do not carry the known mutation, there is no need for
intensive surveillance, and routine colorectal screening according to national guidelines is
recommended. For family members who carry the germline MMR mutation, the National
Comprehensive Cancer Network recommends colonoscopies every 1 to 2 years beginning
at age 20 to 25 years or 5 to 10 years younger than the earliest age of diagnosis in the
family, whichever comes first. For women with the MMR mutation, screening for
endometrial cancer with transvaginal ultrasound with or without CA 125 for ovarian
cancer screening is recommended at age 30 to 35 years. Women who have completed
childbearing could opt for prophylactic hysterectomy and bilateral salpingo-oophorectomy
to reduce their risks for developing endometrial and ovarian cancers.

Question 19.5 The answer is D.


Germline mutations in the MMR genes MSH2, MLH1, and MSH6 account for 90% of all
patients with HNPCC. APC gene mutation is commonly associated with FAP. MYH-
associated polyposis is a rare inherited syndrome with increased for colorectal cancer.

Question 19.6 The answer is D.


The standard adjuvant therapy for patients with stage III colon cancer involves the
combination of oxaliplatin and 5FU based on the MOSAIC and the NSABP-C07 studies.
Recent updates of the MOSAIC study demonstrated a significant improvement in 6-year
overall survival among patients with stage III colorectal cancer treated with FOLFOX
compared with those treated with infusional 5FU and leucovorin. Irinotecan-based
regimens are not recommended for adjuvant therapy for patients with stage III according
to the negative results of three large randomized studies. Current studies on adding
bevacizumab and cetuximab to oxaliplatin regimens are ongoing.

Question 19.7 The answer is C.


More than 80% of the administered dose of 5FU is eliminated via the dihydropyrimidine
dehydrogenase (DPD) enzyme. Patients with DPD deficiency are at high risk for 5FU
toxicities, such as severe neutropenia, neutropenic fever, diarrhea, mucositis, cerebellar
ataxia, neurotoxicity, and even death. The most common mutation associated with DPD
deficiency is the IVS14 + 1 G>A DPYD*2A mutation. Complete DPD deficiency has been
reported in patients homozygous for this allele, whereas partial DPD deficiency occurs in
patients heterozygous for this allele. More than 40 other mutations and polymorphism
have been identified in the DPD gene, although their functional significance is not clear.
Moreover, instances of low DPD activity have been reported in patients without any
identified DPD mutation or polymorphism. The TSER*3/ *3 genotype is associated with
higher levels of thymidine synthase and lower tumor response to 5FU therapy. Although it
is possible that this patient is homozygous for the UGT1A1*28 gene resulting in severe
neutropenia and diarrhea, the constellation of the patient’s symptoms, including cerebellar
ataxia and mucositis, is more consistent with 5FU-related toxicities. Furthermore,
irinotecan therapy would not have been given to this patient with stage III colon cancer.

Question 19.8 The answer is E.


SN-38 is the active metabolite of irinotecan and is 100- to 1,000-fold more potent than
irinotecan as a topoisomerase I inhibitor. Glucuronidation is the principal elimination
pathway for SN-38. UDP-glucuronosyltransferase 1 family polypeptide A1 mediates this
glucuronidation of irinotecan encoded by the UGT1A1 gene. Patients who are
homozygous for the UGT1A1*28 allele glucuronidate SN-38 less efficiently than patients
who have one or two wild-type alleles; therefore, homozygous patients are exposed to
higher plasma concentrations of SN-38 and are thus at a greater risk for severe
neutropenia.

Question 19.9 The answer is D.


The results of the German Rectal Cancer Study confirmed the benefit of neoadjuvant
prolonged course RT with 5FU-based chemotherapy for patients with T3+/N0–2 rectal
cancer. Although overall survival was similar for patients treated with neoadjuvant
chemoradiotherapy compared with those treated with postoperative chemoradiotherapy,
local recurrence rates and toxicities were more favorable with the neoadjuvant approach.

Question 19.10 The answer is A.


Transanal excision should only be considered for select patients with early stage rectal
cancer with the following characteristics: small rectal cancers less than 3 cm; well-to-
moderately differentiated T1 tumors within 8 cm from the anal verge and limited to less
than 30% of the rectal circumference with no evidence of nodal metastases. Only patient
A meets all of these criteria.

Question 19.11 The answer is A.


After transanal resection of appropriately selected patients with early stage rectal cancer,
no further therapy is recommended.

Question 19.12 The answers are B and C.


MYH-associated polyposis is an autosomal-recessive disease affecting the MYH gene, a
base-excision repair gene. A deficiency in MYH leads to somatic, not germline, mutations
in the APC gene. Thus, clinical features of patients with MYH-associated polyposis are
similar to FAP.

Question 19.13 The answer is D.


The standard therapy for early localized squamous carcinoma of the anus is
chemoradiation with 5FU and mitomycin, which is associated with excellent cure rates.
Salvage APR is reserved for recurrent anal cancer.

Question 19.14 The answer is C.


This patient is potentially curable with an aggressive multidisciplinary approach.
Resection of all tumor sites with close surveillance certainly is an option for this patient.
However, the results of the recently published study, EORTC 40983, showed an
improvement in progression-free survival for patients treated with perioperative
chemotherapy with resection. Historically, cure rates of up to 35% have been observed
even in patients with metastatic disease, as long as all tumor sites are resected. Thus,
palliative chemotherapy would not be the best option for this patient. There is currently
no role for radiotherapy for this patient.

Question 19.15 The answer is A.


Patients with colorectal cancer who test positive for the KRAS, NRAS, or BRAF V600E
mutations are resistant to treatment with cetuximab.

Question 19.16 The answer is C.


Based on the phase III CAIRO3 data, after completion of 6 cycles of capecitabine +
oxaliplatin (CAPOX) and bevacizumab, the patients who transitioned to maintenance
therapy with capecitabine and bevacizumab had a significantly longer time for
progression than those on observation (8.5 months vs. 4.1 months). This study also
reported the patients placed back on CAPOX/bevacizumab after progression on
maintenance therapy were noted to have continued improved time to second progression
of 19.8 months versus 15.0 months. Complete treatment break at this point would not be
preferred, but the patient could make an informed decision based on this information.
Changing to second-line therapy with FOLFIRI plus panitumumab at this point is an
incorrect treatment option since anti-EGFR therapy is not beneficial for patients with
KRAS-mutated colorectal cancer. There is no need to go through all 12 cycles of
oxaliplatin at this time (if the patient is able to tolerate the side effects) based on the
OPTIMOX-1 data, since the original chemotherapy may be restarted at the time of tumor
progression.

Question 19.17 The answers are B and D.


BRAF V600E is the most frequent BRAF mutation in colorectal cancers. It is most
commonly associated in older women, right-side location, high-grade features and
presence of microsatellite instability (MSI-high). Nevertheless, this mutation is
uncommon, occurring in approximately 5% to 10% of the colon cancer population. The
prognosis is worse than in those with wild-type BRAF.

Question 19.18 The answers are A and C.


High-risk features for stage II colon cancers include: bowel obstruction, perforated T4
tumors, poorly differentiated, signet ring or mucinous tumors, inadequate lymph node
sampling (at least 10 is considered adequate), and positive lymphovascular invasion.

Corresponding chapters in Cancer: Principles & Practice of Oncology, Tenth Edition: 56 (Molecular Biology of Colorectal
Cancer), 57 (Cancer of the Colon), 60 (Cancer of the Rectum), and 61 (Cancer of the Anal Region).
20 Prostate, Bladder, and Kidney Cancer
Russell K. Pachynski

QUESTIONS
Each of the numbered items below is followed by lettered answers. Select the ONE lettered answer
that is BEST in each case unless instructed otherwise.

Question 20.1 Activating mutations in which of the following genes is seen in patients with
hereditary papillary renal cell carcinoma?
A. VHL
B. MET
C. FLCN
D. SDHB

Question 20.2 A 56-year-old moderately obese woman with a medical history of chronic
hypertension well controlled on a thiazide diuretic presents to the emergency department
with a 1-day history of abdominal pain, diarrhea, nausea, and fever. General physical
examination is significant only for some mild abdominal tenderness; negative stool guaiac; a
white blood cell count of 14.2/μL; hemoglobin of 14.5 g/dL; a normal platelet count; normal
electrolytes, amylase, lipase, and transaminases; and a creatinine of 0.9 mg/dL. Workup
includes an abdominal computed tomography (CT) scan, which is remarkable for a 1.5-cm
enhancing left renal mass in the left lower pole that is interpreted by the radiologist as a
“probable renal cell carcinoma” without evidence of other metastases. The patient undergoes
a laparoscopic partial nephrectomy. There are no postoperative complications, and she is
back to work 3 weeks later. Postoperative creatinine is 1.3 and pathology reveals a 2.5-cm
renal cell carcinoma, granular cell type, that is confined to the renal parenchyma. No lymph
nodes were recovered. The next appropriate step is:
A. Open retroperitoneal lymph node dissection.
B. Adjuvant sunitinib.
C. Adjuvant local radiotherapy.
D. Submission of pathology specimen for second review.

Question 20.3 A 55-year-old woman undergoes partial nephrectomy for clear cell carcinoma
of the kidney, Fuhrman grade IV. The patient does well for 10 years, at which time she
develops a pathologic intratrochanteric fracture of her left hip. CT scanning of the chest,
abdomen, pelvis, and brain and bone scan reveals no other sites of disease. The most
appropriate next step is:
A. Radiation alone.
B. High-dose interleukin-2 alone.
C. Orthopedic resection of the tumor with reconstruction followed by radiation.
D. Temsirolimus.

Questions 20.4–5 A 76-year-old man with chronic obstructive pulmonary disorder and
diabetes mellitus presents with back pain and confusion. Workup reveals extensive metastatic
disease in the lungs, bones, and liver and a 6-cm tumor in the kidney. There is no evidence of
cord compression. Laboratory studies reveal a calcium level of 11.5 mg/dL, lactate
dehydrogenase (LDH) of 600, and a creatinine level of 2.0 mg/dL. After hydration and
zoledronate, his calcium normalizes, hemoglobin is 9.5, creatinine decreases to 1.7, and
Eastern Cooperative Oncology Group performance status is 2.

Question 20.4 Based on MSKCC risk factors, this patient would be considered:
A. No risk.
B. Low risk.
C. Intermediate risk.
D. Poor risk.

Question 20.5 The most appropriate therapy in this case is:


A. Interferon-α.
B. Temsirolimus.
C. Sunitinib.
D. Sorafenib.

Question 20.6 A 25-year-old Caucasian man without a medical history presents with
hematuria. Workup reveals bilateral renal cysts, at least one of them suspicious for
malignancy. Family history is significant for a pheochromocytoma in his father and a
pancreatic islet cell tumor and early death from kidney cancer in a paternal aunt. The most
likely familial cancer syndrome is:
A. Von Hippel–Lindau disease.
B. Birt–Hogg–Dubé syndrome.
C. Hereditary papillary renal cancer.
D. Hereditary leiomyomatosis and renal cancer.

Question 20.7 Which of the following associated with loss of VHL function is most likely to
lead to tumor angiogenesis?
A. Modulation of NF-kB activity and downregulation of JUNB
B. Increase in matrix metalloproteinases
C. Increase in hypoxia inducible factor (HIF)
D. Destabilization of microtubule formation

Question 20.8 Inactivation or alteration in which of the following tumor suppressor genes is
implicated in the pathogenesis of invasive bladder cancer?
A. TP53
B. RB1
C. PTEN
D. All of the above

Question 20.9 A patient with hematuria is taken to the operating room, where an
examination under general anesthesia reveals a mobile bladder. Resection of the papillary
lesion reveals grade III urothelial papillary carcinoma, and multiple biopsies of the
erythematous areas of the bladder all reveal diffuse carcinoma in situ. Muscle is present in
the pathologic specimens, and there is no evidence for invasive tumor. The appropriate
therapy is:
A. Intravesical Bacillus Calmette–Guérin (BCG) vaccine.
B. Intravesical cyclophosphamide.
C. Radiation.
D. Cystectomy.

Question 20.10 The patient receives definitive treatment, as well as with a follow-up
maintenance program; however, 4 months after initiating the maintenance BCG program, the
urologist notes multiple recurrent papillary lesions. Repeat biopsy reveals urothelial cancer
invasive into muscle. CT scans of the chest, abdomen, and pelvis are unremarkable,
creatinine remains normal at 1.2 mg/dL, and his performance status is excellent. Appropriate
initial therapy at this point is:
A. Reinduction with intravesical BCG.
B. Intravesical chemotherapy with mitomycin-C.
C. Partial cystectomy.
D. Cisplatin-based multiagent chemotherapy.

Question 20.11 Which of the following is the most common molecular abnormality seen in
patients with prostate cancer?
A. KRAS mutation
B. BRAF mutation
C. p53 mutation
D. Chromosomal translocations involving TMPRSS2

Question 20.12 Which of the following are risk factors for cancer of the male urethra?
A. HPV-16
B. Chronic irritation
C. Infection
D. Caucasian race

Questions 20.13–14 A 62-year-old woman is newly diagnosed with muscle-invasive bladder


cancer. She quit smoking 10 years ago, had a non–ST-elevated myocardial infarction 4 years
ago, underwent coronary artery bypass surgery and has had no residual cardiac symptoms,
takes only a beta-blocker and a thiazide for hypertension, and has normal laboratory test
results, including a creatinine of 0.9. Cystoscopic biopsy revealed a muscle-invasive bladder
cancer without associated carcinoma in situ (Tcis), and CT of the chest, abdomen, and pelvis
is unremarkable.

Question 20.13 Which of the following statements about radical cystectomy is most
CORRECT?
A. An orthotopic neobladder is less effective in women than in men.
B. An abdominal wall diversion will require a urostomy bag.
C. An orthotopic neobladder will require the patient to be willing and able to perform self-
catheterization.
D. Metabolic acidosis is not a significant problem with continent diversions.

Question 20.14 Which of the following statements about combined radiation and
chemotherapy is most CORRECT?
A. Toxicity profile and tolerability of combined radiation and chemotherapy are
significantly better than that of radical cystectomy.
B. Long-term cancer outcome is similar to cystectomy.
C. It is preferred over cystectomy because of her cardiac history.
D. It will obviate the need for cystectomy.

Question 20.15 Which of the following is CORRECT about neoadjuvant chemotherapy?


A. Three cycles of methotrexate, vinblastine, doxorubicin, and cisplatin before cystectomy
are a standard of care.
B. Gemcitabine/carboplatin should be considered to decrease the risk of renal failure with
cystectomy.
C. It increases the risk of surgical complications of cystectomy.
D. It should always be used with an organ preservation approach but is optional if
cystectomy is chosen.

Question 20.16 Which of the following genes is most commonly found to be abnormally
altered in invasive (≥T2) urothelial carcinoma of the bladder?
A. ERBB2
B. TP53
C. MDM2
D. FGFR3

Question 20.17 A 71-year-old man without significant medical history presents with
hematuria and flank pain. CT scan reveals a mass at the pelvic–ureteral junction with
associated hydronephrosis, but no associated lymphadenopathy. Cystoscopy and ureteroscopy
reveal an obstructing mass at the pelvic–ureteral junction, and cytology is diagnostic for
urothelial cancer. Which of the following is the most appropriate therapy?
A. Open nephroureterectomy and bladder cuff resection
B. Open radical nephrectomy with retroperitoneal lymph node dissection
C. Laparoscopic radical nephrectomy without retroperitoneal lymph node dissection
D. Definitive radiation and combined chemotherapy

Question 20.18 A 51-year-old man with a strong family history of prostate cancer, a normal
digital rectal examination, no significant comorbid medical problems, and a PSA of 2.9 seeks
advice on prostate cancer prevention. He is sexually active in a monogamous relationship and
denies any urinary or rectal symptoms. Which is the most likely to reduce his risk of
developing prostate cancer?
A. Reducing his alcohol intake.
B. Taking supplemental high dose vitamin E.
C. Avoiding high fat intake, reducing his BMI (avoiding obesity), and increasing his
physical activity.
D. Increasing his calcium/vitamin D intake.

Questions 20.19–20 A 71-year-old white man with a history of hypertension,


hyperlipidemia, coronary artery disease, and prior angioplasty with stent placement, but no
prior myocardial infarction, is noted to have an increase in his PSA from 3.0 to 3.9 ng/mL
and then to 4.6 ng/mL over 19 months. He is a semiretired accountant, swims actively three
times per week, and helps care for his mildly demented 95-year-old father. General physical
examination is unremarkable; a rectal examination reveals a mildly enlarged prostate gland
without any palpable nodules.

Question 20.19 The most CORRECT statement about this case is:
A. Biopsy should be discussed because the PSA increase is >0.75 ng/ mL/year.
B. Biopsy should not be discussed because PSA is normal for his age.
C. Biopsy should not be discussed because his expected survival makes treatment not
worthwhile, even if prostate cancer is discovered.
D. Biopsy should be discussed because the PSA is >4 ng/mL.
E. The free-to-total PSA ratio will determine the need for biopsy.

Question 20.20 Biopsy reveals Gleason 8 prostate cancer in six of six cores. CT scan of the
abdomen and pelvis and bone scan are unremarkable. The most appropriate therapy is:
A. Radical retropubic prostatectomy.
B. Three-dimensional conformal radiotherapy with concomitant androgen ablation.
C. Interstitial radiotherapy with 125I.
D. All of the above.

Question 20.21 After discussion with a radiation oncologist and a urologist, the patient
elects to undergo combined androgen ablation and external beam radiation therapy. The
androgen ablation is administered before the radiation therapy and continued for 3 months
thereafter. Radiotherapy is complicated only by a mild diarrhea that resolves once the
radiation therapy is complete. The PSA nadirs at 1.2 ng/mL; however, 9 months after his last
dose of the luteinizing hormone–releasing hormone (LHRH) agonist, the PSA increases to 2.4
ng/mL and then to 3.6 ng/mL 1 month later. Testosterone level is normal at 350 ng/mL. The
most appropriate next therapeutic and/or diagnostic maneuver is:
A. Perform MRI of the pelvis to assess for local recurrence.
B. Reinitiate androgen ablation.
C. Refer the patient to a urologist for salvage prostatectomy.
D. Initiate docetaxel-based chemotherapy.

Question 20.22 The patient is treated with an LHRH agonist along with the antiandrogen
bicalutamide. PSA declines to 0.8 ng/mL, but after 10 months, the PSA begins to slowly
increase to a value of 3.7. He continues to feel well and has minimal urinary symptoms, no
bone pain, and no weight loss. The most appropriate therapy at this point is:
A. Docetaxel-based chemotherapy.
B. Discontinuing the antiandrogen bicalutamide.
C. Hospice care.
D. Radionuclide therapy with strontium-98 (Metastron).

Question 20.23 Infestation with which of the following parasites is a risk factor for
developing bladder cancer?
A. Clonorchis sinensis
B. Opisthorchis viverrini
C. Schistosoma haematobium
D. None of the above

Question 24–25 A 51-year-old black male executive with no medical history undergoes a
routine PSA screening evaluation and is found to have a PSA of 5.5 ng/mL. Biopsy reveals a
Gleason 3 + 3 prostate cancer in two of six biopsy cores. After discussion with a radiation
oncologist and urologist, he elects to receive treatment with a radical retropubic
prostatectomy.

Question 20.24 Which of the following statements about the surgery is TRUE?
A. Robotic laparoscopic prostatectomy is associated with a lower incidence of impotence
than open retropubic prostatectomy.
B. The incidence of impotence under the assumption that a bilateral nerve sparing
procedure can be performed is <10%.
C. Problems with incontinence persist in approximately 20% of patients.
D. Surgical experience has only a minimal impact on the positive margin rate.

Question 20.25 Surgical pathology confirms a Gleason score 6 tumor in both lobes of the
prostate. There is a focal surgical positive margin. There is no evidence of seminal vesicle or
lymph node invasion. His postoperative PSA is undetectable, and he has good continence. The
most appropriate next step is:
A. Adjuvant radiotherapy.
B. Repeat surgical exploration with possible reexcision of the prostatic bed.
C. Pelvic CT scan.
D. Prostascint scan.

Question 20.26 The patient maintains an undetectable PSA until 8 years later (at the age of
59 years), recurrent biochemical disease is noted. After appropriate discussion, androgen
ablation with an LHRH agonist alone is initiated, and the PSA once again becomes
undetectable. The patient maintains an undetectable PSA while on androgen ablation for 3
years, when he develops sudden midback pain after lifting his grandson. There are no
associated neurologic signs or symptoms. Bone scan shows marked uptake at the T8 vertebra
and PSA remains undetectable. The most appropriate therapeutic or diagnostic maneuver is:
A. Immediate radiotherapy to T8.
B. Therapy with ketoconazole, 400 mg three times daily with hydrocortisone replacement.
C. Spinal MRI to rule out cord compression.
D. Bone densitometry to assess for osteoporosis.

Question 20.27 A 75-year-old man with diabetes, hypertension, and coronary artery disease
who is receiving atorvastatin, glyburide, and an angiotensin-converting enzyme inhibitor is
under surveillance after external beam radiotherapy for a Gleason 3 + 3 prostate cancer that
was diagnosed and treated 10 years earlier when he was found to have a PSA of 4.3 on
routine screening. His PSA level, which had been 0.2 ng/mL, has increased to 0.3, 0.35, and
then 0.40 over the period of 18 months. The most appropriate therapy at this time is:
A. Androgen ablation with an LHRH agonist.
B. Continued active surveillance.
C. High-intensity focused ultrasound to his prostate.
D. High-dose (150 mg) bicalutamide.

Question 20.28 A 65-year-old man has been receiving combined androgen ablation with
leuprolide and bicalutamide for 4 years for biochemical recurrence after radical
prostatectomy. His PSA has increased from an undetectable nadir to 1.1 ng/mL on serial
measurement over the period of 6 months. The PSA then continues to increase 2 months later
to 2.5, with serum testosterone of 10 ng/mL. Bone scan and CT of the abdomen/pelvis do not
reveal any metastatic disease, and he remains asymptomatic. The most appropriate therapy
option is:
A. Continue current treatment regimen and active surveillance.
B. Docetaxel-based chemotherapy.
C. Start enzalutamide.
D. Discontinue bicalutamide.

Question 20.29 Which of the following has the highest risk of developing prostate cancer?
A. A 60-year-old Caucasian male with no family history of cancer.
B. A 60-year-old African-American male with a father with prostate cancer.
C. A 40-year-old Asian male.
D. A 40-year-old obese Caucasian male.

Question 20.30 Genetic alterations in which of the following is most common in prostate
adenocarcinoma?
A. NRIP1
B. FOXA1
C. PTEN
D. CDK2

Question 20.31 Which of the following statements about the androgen receptor is most
CORRECT?
A. The majority of its activity in prostate cancer is due to its cytoplasmic effects.
B. Upregulation of androgen receptor expression has been linked to prostate cancer
development.
C. Castration leads to complete inactivation of all androgen receptor-mediated pathways.
D. Castrate-resistant prostate cancer (CRPC) is associated with upregulation of androgen
receptor expression.

Question 20.32 A 63-year-old uncircumcised man without any significant medical history
presents to his physician with an inability to retract the foreskin. Examination reveals
phimosis, with an underlying painless ulcerated mass of 1 × 2 cm. A 2.5-cm hard node is
palpated in the left inguinal region. Biopsy of the penile lesion reveals squamous cell cancer.
In addition to wide surgical resection of the primary lesion, other appropriate therapeutic
and/or diagnostic maneuvers at this time include:
A. Four-week course of a broad-spectrum antibiotic.
B. Left inguinal lymph node dissection.
C. Bilateral inguinal radiotherapy.
D. Taxane-based chemotherapy.
ANSWERS

Question 20.1 The answer is B.


Hereditary papillary renal cell carcinoma is an autosomal-dominant hereditary cancer
syndrome characterized by mutations in the MET protooncogene, which encodes the
hepatocyte growth factor/scatter factor receptor tyrosine kinase. Although MET
overexpression has been demonstrated in a number of epithelial cancers, HRPC is the first
cancer syndrome for which germline MET mutations have been identified. Individuals
with this syndrome are at risk for developing multifocal, bilateral papillary type 1 kidney
cancer in their fifth and sixth decades of life, with age-dependent penetrance of 67% by
60 years of age.

Question 20.2 The answer is D.


Renal cell cancer is currently divided into clear cell and non–clear cell subtypes. The non–
clear cell subtypes have been further divided into papillary, chromophobe, and collecting
duct tumors. Several renal cancers cannot be accurately subtyped, often because of poor
histologic differentiation such that the originating subtype cannot be easily recognized.
Such tumors are often classified as “sarcomatoid.” Granular cell carcinoma has been used
in the past but is not currently an accepted pathologic classification. Thus, a second
opinion and pathologic review should be requested. The risk of recurrence in this case is
low, and adjuvant sunitinib has not been shown to decrease the risk of recurrence.
Patients who do have recurrence after nephrectomy almost always recur systemically, and
thus additional local radiotherapy is not indicated. Likewise, the role of extensive lymph
node dissection is highly controversial and generally not recommended. In any case,
formal retroperitoneal lymph node dissection as is carried out in testes cancer would
certainly not be standard in patients with localized renal cell carcinoma.

Question 20.3 The answer is C.


Patients with renal cancer can develop recurrent disease more than 5 years after initial
definitive therapy. Development of a single metastatic site is not unusual. Patients with a
single site of metastatic disease and a long interval between initial diagnosis and recurrent
metastatic disease typically have a good prognosis. Thus, aggressive surgical resection
including orthopedic stabilization is indicated. Because orthopedic surgery such as this
often leaves microscopic tumors behind in the surgical field, additional radiotherapy to
the site is appropriate. Radiation alone is insufficient to completely eradicate metastatic
renal cancer and does not repair the underlying structural bony abnormality. High-dose
interleukin-2 is approved for metastatic renal cancer but is less effective than aggressive
local surgery for complete eradication of disease when possible. Temsirolimus improves
survival in patients with poor prognosis metastatic disease, which does not apply here. In
addition, it is not curative.

Question 20.4 The answer is D.


Several prognostic factors have been identified that can help explain the highly
heterogeneous natural history of metastatic renal cancer. Hypercalcemia, poor
performance status, elevated LDH, and anemia are significant prognostic factors in most
studies, and the most widely used prognostic factor model is from the Memorial Sloan
Kettering Cancer Center (MSKCC). Age, in and of itself, is not a prognostic factor. The
presence of one to two factors is considered intermediate risk, while three or more factors
is considered poor risk.

Question 20.5 The answer is B.


Of the choices, phase III data supportive of a survival benefit in a patient with poor
prognosis such as this are available only for temsirolimus. The available data suggest that
sunitinib or sorafenib could be considered. However, data suggest that interferon is not
effective in patients with poor prognosis, and phase III data show that sunitinib is more
effective than interferon for metastatic disease in patients with a good and intermediate
prognosis.

Question 20.6 The answer is A.


Von Hippel–Lindau disease, caused by mutation of the VHL gene, is an autosomal-
dominant cancer syndrome characterized by multiple renal cysts, early onset and multiple
renal tumors, retinal angiomas and central nervous system hemangioblastomas,
pheochromocytomas, and pancreatic islet cell tumors. The Birt–Hogg–Dubé syndrome,
caused by mutations of the BHD gene, is characterized by early onset chromophobe
cancers, benign hair follicle tumors (fibrofolliculomas), and pulmonary cysts. Hereditary
papillary renal cancer, caused by mutations of the MET gene, is characterized by multiple
type I papillary cancers. The hereditary leiomyomatosis and renal cancer syndrome,
caused by mutation of the fumarate hydratase gene, is characterized by cutaneous and
uterine leiomyomas and aggressive type II papillary cancers.

Question 20.7 The answer is C.


VHL is part of the normal oxygen sensing and response system in all cells and is mutated
or altered in greater than 60% of sporadic clear cell carcinomas of the kidney. Under
normoxic conditions, VHL is part of a multiprotein complex that recognizes the
hydroxylated HIF transcription factor, acts as a ubiquitin ligase, and thus targets it for
proteasomal degradation. Under hypoxic conditions, HIF is not hydroxylated, not targeted
to VHL and thus not degraded leading to upregulation of multiple genes critical for the
hypoxic response including vascular endothelial growth factor and ultimately increased
angiogenesis. The VHL gene is expressed constitutively.

Question 20.8 The answer is D.


The tumor suppressor genes TP53, RB1 CDKN2A, and PTEN are implicated in the
pathogenesis of invasive bladder cancer.

Question 20.9 The answer is A.


Indications for intravesical therapy are multiple recurrent Ta lesions, especially if they are
high grade, or Tcis. It is critical that the pathologic specimen in patients diagnosed with
superficial bladder cancer contains muscle to determine whether there is an noninvasive
component. Most, but not all, studies have suggested that intravesical BCG is more
effective than intravesical chemotherapy. Although other chemotherapeutic agents are
being investigated, cyclophosphamide needs to be activated to 4-hydroxy-
cyclosphosphamide in the liver to act as an alkylating agent, and thus bladder instillation
of the parent compound is not expected to be effective. Radiation or cystectomy, although
potentially useful for invasive carcinoma or refractory bladder cancer, is not appropriate
as initial therapy in this patient.

Question 20.10 The answer is D.


Patients with muscle-invasive bladder cancer require definitive local therapy. Additional
intravesical therapy is inappropriate. Partial cystectomies are only rarely indicated and
should be performed in only a very highly select group, which does not include patients
with a history of Tcis or patients with multiple tumors. The standard definitive therapy
for bladder cancer is cystectomy. Multiagent cisplatin-based neoadjuvant chemotherapy
improves survival for patients with invasive urothelial bladder cancer and is considered a
standard of care.

Question 20.11 The answer is D.


Chromosomal translocations fusing the TMPRSS2 androgen-responsive gene to various
ETS family transcription factors, of which ERG is the most common, occur in
approximately 60% of patients. Prostate cancer is characterized by a relatively low rate of
mutations in KRAS, BRAF, and p53, unlike most other tumors.

Question 20.12 The answer is A, B, and C.


Risk factors for male urethral cancers include HPV-16, chronic irritation, and infection.
The incidence of urethral cancer in men with urethral strictures ranges from 24% to 76%.
No racial predisposition has been noted for urethral cancers.

Question 20.13 The answer is C.


There are several options for urinary diversion after cystectomy, and no clear evidence
has emerged that any is more or less desirable or effective based on patient gender.
Diversion to the abdominal wall with a urostomy can be performed as a conduit, requiring
a urine collection device, or as a continent diversion, requiring regular catheterization.
Metabolic complications, including metabolic acidosis, can occur with both conduits and
continent diversions. Complications of orthotopic continent diversions, or neobladders,
include intermittent urethral obstruction and frequent requirement for intermittent or
temporary self-catheterization.

Question 20.14 The answer is B.


Organ preservation with combined radiation and chemotherapy has been investigated in
numerous trials, and long-term outcomes are similar to those observed in cystectomy
series. The therapy is, however, an aggressive and potentially toxic approach in which
approximately 30% of patients will require cystectomy for incomplete response. It is thus
not more tolerable than cystectomy. The patient’s cardiac history does not preclude major
surgery, and in fact willingness to accept a cystectomy has been an inclusion criterion for
most organ preservation trials.

Question 20.15 The answer is A.


Randomized studies have demonstrated that cisplatin-based combination chemotherapy
provides a survival advantage. The largest phase III studies used methotrexate,
vinblastine, doxorubicin, and cisplatin or a similar regimen in which the doxorubicin was
eliminated (MVC). The strongest data supporting neoadjuvant chemotherapy are in the
surgical setting, with no survival advantage demonstrated in modest-sized trials using
combined chemotherapy and radiation for primary tumor treatment. The randomized data
further demonstrate that there is no increase in surgical morbidity or complications after
neoadjuvant chemotherapy. There are no data on the neoadjuvant use of carboplatin-
based regimens, and data in the metastatic setting strongly suggest that carboplatin is an
inferior agent in comparison with cisplatin for this disease.

Question 20.16 The answer is B.


Alterations (deletion/mutation) in the tumor suppressor gene p53 are most common
(∼50% to 70%) in invasive disease and are markers of poor prognosis. Activating
mutations of FGFR3 are common in papillary noninvasive cancers, though less commonly
found in T2 tumors (0% to 20%). MDM2 and ERBB2 are also less commonly amplified
(∼10%), although ERBB2 overexpression can be 10% to 50%.

Question 20.17 The answer is A.


The standard definitive treatment for renal pelvis urothelial cancers is radical
nephroureterectomy with resection of the bladder cuff at the ureteral insertion site.
Radical nephrectomy without ureter resection is inappropriate because of the high
incidence of undiagnosed or recurrent disease in the ureter. Definitive radiotherapy would
also be inappropriate because this would be expected to lead to kidney necrosis.

Question 20.18 The answer is C.


High-fat diet has been associated with increased prostate cancer incidence and mortality,
and obesity and increased BMI have an increased risk of death from prostate cancer.
There is evidence to suggest that diet and exercise may slow the course of the disease
once diagnosed. Although dietary studies have suggested that vitamin E correlates with a
decreased incidence of prostate cancer, data from the phase III intervention study
(SELECT) of selenium and vitamin E to prevent prostate cancer demonstrates that vitamin
E use is actually associated with an increased risk of developing prostate cancer (JAMA
2011;306:1549). Alcohol use and calcium/vitamin D intake are not associated with an
increased risk of prostate cancer.

Question 20.19 The answer is A.


The decision to screen for and diagnose prostate cancer always needs to be accompanied
by a discussion between the physician and the patient regarding risks and benefits of
screening. To this end, it should be recognized that the median survival of an average 70-
year-old man without significant comorbidities is more than 10 years, and that such a
patient could benefit from the treatment of localized disease. Nonetheless, there is a risk
of treating disease that will never become clinically significant within the patient’s
lifetime. It also needs to be recalled that the sensitivity and specificity of all PSA-based
screening methods are limited, and there are no absolute “normal” criteria. Therefore, no
single test can determine the absolute need for a biopsy. Predictors of cancer on biopsy
include PSA greater than 4.0 ng/mL, PSA greater than age-adjusted PSA norms (based on
normal increasing PSA with age), free-to-total PSA ratio, and PSA velocity. Of these, and
in this case, the rapid increase in the PSA is most predictive of malignancy, and this is also
predictive of aggressive disease.

Question 20.20 The answer is D.


On the basis of the high Gleason score, the aforementioned rapid PSA doubling time, and
the tumor in all cores, this patient is at a high risk for locally advanced disease and
systemic recurrence. As such, staging CT scan and bone scan are reasonable, although not
generally necessary in patients with lower-risk disease. Because of the presence of high-
risk features and the general good health of the patient, watchful waiting is probably not
an appropriate option in this case. Radiotherapy, with external beam radiotherapy or
interstitial brachytherapy, or surgical prostatectomy is equally appropriate for patients
with clinically localized disease. The choice is dependent on a discussion of expected risks
and benefits.

Question 20.21 The answer is B.


Rapidly increasing PSA before diagnosis, high Gleason score, PSA nadir of greater than
1.0 after radiotherapy, short interval between definitive local therapy and biochemical
recurrence, and rapid PSA increase once recurrence is identified are all associated with a
poor prognosis. The probability of locally recurrent disease alone is extremely low, and
there is not much value to pelvic MRI for assessing local recurrence. Despite the poor
prognosis, the role of chemotherapy in patients whose testosterone axis is intact is
controversial. Although timing of androgen ablation is not clearly defined, some
retrospective data suggest that “early” ablation in high-risk patients such as this provides
a long-term survival advantage over “later” ablation.

Question 20.22 The answer is B.


Approximately 10% to 15% of patients receiving treatment with an antiandrogen will
experience an antiandrogen withdrawal response. Although the majority of these
responses are of brief duration, it is obviously the easiest therapeutic maneuver.
Docetaxel-based chemotherapy leads to improved survival in castrate-resistant prostate
cancer (CRPC), which is generally defined as a progressive disease after androgen
ablation, an antiandrogen, and antiandrogen withdrawal. Radioactive nuclides are
occasionally useful for the palliation of diffuse bone pain but have not been shown to
have an impact on survival.

Question 20.23 The answer is C.


Over 90% of bladder cancers in Western countries are urothelial carcinomas. In areas
where the parasite Schistosoma haematobium is endemic, squamous cell bladder carcinomas
are more common. Opisthorchis viverrini and Clonorchis sinensis are associated with
cholangiocarcinoma, and not bladder cancer.

Question 20.24 The answer is C.


Although robotic laparoscopic prostatectomy has been touted as a less-invasive and
potentially more effective approach, there is no reliable evidence that the risk of long-
term complications, including impotence, is different than that observed in patients
undergoing an open procedure with an experienced surgeon. In both groups, the incidence
of impotence is at least 25%, even in young men with good erectile function before
surgery. The risk of incontinence, when assessed by anonymized, validated quality of life
instruments, is on the order of 20%. One of the most important variables for both
pathologic outcome and complications is the surgeon’s experience.

Question 20.25 The answer is A.


Adjuvant radiotherapy has been shown to decrease the risk of recurrence in patients with
T3 disease. However, overall survival and clinical metastases-free survival have not been
affected by this approach. Some have thus argued for close monitoring and salvage
radiotherapy on biochemical recurrence in patients such as this. Imaging tests have not
been shown to be helpful in identifying patients most likely to benefit from adjuvant
therapy, and surgical reexploration is not indicated either.

Question 20.26 The answer is D.


Patients on long-term androgen ablation are at high risk for the development of
osteoporosis and its complications, and an osteoporotic fracture must be considered in this
patient before initiating any therapy for progressive metastatic disease. If progressive
disease were to be diagnosed, ketoconazole is not an unreasonable second-line hormonal
therapy, but the addition of an antiandrogen would likely be more appropriate. In the
absence of neurologic signs or symptoms, an MRI is not necessary.

Question 20.27 The answer is B.


Patients with a slowly increasing PSA after definitive local therapy may have a very long
natural history, with only a minority of patients having cancer-related mortality within 10
years. Androgen ablation, especially in elderly patients, is associated with osteoporosis,
muscle loss, and changes in lipid metabolism that may have significant impact on
cardiovascular comorbidities. High-intensity focused ultrasound for treatment of possible
localized recurrence is still investigational. Single-agent antiandrogen therapy is not
approved and may lead to higher mortality in these patients.
Question 20.28 The answer is D.
The patient has nonmetastatic CRPC. The initial diagnostic test to perform is to confirm
the patient is truly at castrate levels of testosterone (<50 ng/mL), as a very small
percentage of patients (∼1%) do not achieve complete testosterone suppression with
LHRH agonists. The patient’s imaging is negative for metastatic disease, so he would not
be an appropriate candidate for therapies approved for metastatic CRPC (e.g.,
enzalutamide, abiraterone, docetaxel, sipuleucel-T). His PSA doubling time (PSADT) is
short (<2 months), indicating aggressive disease kinetics, so continued surveillance
would be less desirable. Antiandrogen withdrawal (AAW) – for example, discontinuation
of bicalutamide – is based on the observation of antiandrogen conversion to an agonist,
thus stimulating tumor growth. Upon discontinuation of the antiandrogen, AAW can occur
in up to 30% of patients, although the timing of response is variable and can take up to 8
weeks before response is seen, which is dependent on the half-life of the drug (earlier for
flutamide, later for bicalutamide). There are several other treatment options for
nonmetastatic CRPC (e.g., high dose ketoconazole, estrogens, flutamide, nilutamide),
with varying PSA responses, but none has shown an improvement in overall survival.

Question 20.29 The answer is B.


Prostate cancer is rare before the age of 40, and thus additional risk factors would likely
be involved in the etiology of prostate cancer in younger males. Incidence and mortality
rates are significantly lower for Asian-American descent, and somewhat lower for Mexican
Americans and other Latinos. African-American males have the highest incidence of
prostate cancer, with almost twofold compared with American men of European descent.
African-American men are diagnosed at a younger age with more aggressive disease and
worsened clinical outcomes. There is now also confirmation of a genetic risk component
because genetic polymorphisms, including several on 8q24, have been found to be closely
associated with prostate cancer. In fact, the large difference in prostate cancer mortality
between African Americans and Caucasian Americans cannot be explained by
socioeconomic factors alone. Men with a first-degree relative with prostate cancer have
approximately two- to threefold increased risk of developing prostate cancer.

Question 20.30 The answer is C.


PTEN is a tumor suppressor that deactivates PI3K signaling and its deletion is seen in
∼30% of primary prostate cancers, and more commonly in advanced disease (∼50% to
55%). PTEN plays a prominent role in tumorigenesis and is used in preclinical models of
prostate cancer. FOXA1 is a transcription factor that is involved in AR and steroid
receptor function, and can promote progression to CRPC, but is altered in ∼5% of cases.
CDK2 is a cyclin dependent kinase involved in the RB pathway, and NRIP1 is an AR
corepressor; these are also less commonly altered.

Question 20.31 The answer is D.


Although the androgen receptor does have cytoplasmic effects, some of which may be
important to prostate cancer oncogenesis, the majority of its effects occur in the nucleus
where it modifies expression of genes with an androgen-responsive promoter.
Upregulation of androgen receptor expression has been reported to be both necessary and
sufficient for the castrate-resistant state, which is likely an adaptation to prolonged
exposure to a low androgen environment. Such upregulation is not generally observed de
novo and has not been linked to the development of prostate cancer. Castration
dramatically lowers testosterone, but the levels of other androgenic steroids are sufficient
to cause partial activation of androgen receptor-mediated pathways.

Question 20.32 The answer is A.


Penile carcinoma is a rare malignancy that presents most commonly with phimosis in an
uncircumcised patient. Metastatic spread of penile carcinoma is via superficial inguinal
lymphatics, followed by deep inguinal lymphatics, and then to the pelvic lymphatics.
Systemic metastatic disease almost never develops in the absence of pelvic
lymphadenopathy. Therefore, surgery and occasionally radiotherapy form the cornerstone
of treatment for lymph node-positive patients; however, half of the patients with
clinically palpable nodes will have inflammatory lesions only. Thus, a course of antibiotics
before any further therapy is indicated. If the nodes persist after a course of antibiotics,
biopsy and surgical dissection are indicated. Management of patients with clinical N0 is
more controversial, with some authors recommending immediate lymph node dissection
and others recommending a watchful waiting approach. Pathologic risk factors as assessed
in the primary lesion may assist in decision making. Lymphangiography before a course of
antibiotics is generally not considered helpful.

Corresponding chapters in Cancer: Principles & Practice of Oncology, Tenth Edition: 62 (Molecular Biology of Kidney Cancer),
63 (Cancer of the Kidney), 64 (Molecular Biology of Bladder Cancer), 65 (Cancer of the Bladder, Ureter, and Renal Pelvis), 66
(Genetic Testing in Urinary Tract Cancers), 67 (Molecular Biology of Prostate Cancer), and 68 (Cancer of the Prostate).
21 Testicular Cancer
Pasquale Benedetto

QUESTIONS
Each of the numbered items below is followed by lettered answers. Select the ONE lettered answer
that is BEST in each case unless instructed otherwise.

Question 21.1 A 24-year-old man has, for the past week, experienced dyspnea on climbing
two flights of stairs. In the ED arterial blood gas on room air demonstrates PO2 66 mm Hg,
PCO2 34 mm Hg, and pH 7.42. On examination he is in no acute distress at rest. Lungs are
clear to auscultation, there is a no organomegaly but a firm enlargement of the right testis.
Chest x-ray reveals multiple bilateral pulmonary nodules. Pathology from the orchiectomy
specimen demonstrates embryonal carcinoma. hCG level is 1,024 mIU/mL, alpha-fetoprotein
(AFP) is 248 ng/mL. While in the hospital he develops headaches and a CAT scan of his brain
shows multiple small hemispheral metastases. Which of the following is the most appropriate
next step in management?
A. Begin whole brain radiotherapy, and after its completion in 2 weeks, start
chemotherapy.
B. Begin chemotherapy immediately.
C. Begin whole brain radiotherapy and whole lung radiotherapy to be followed in 3 weeks
by chemotherapy.
D. Begin concomitant whole brain radiotherapy and chemotherapy.
E. Use CCNU in high dose because it will cross the blood brain barrier and thus be
effective against both lung and brain metastases.

Question 21.2 23-year-old male presents to his physician with a 3-week history of back pain.
Physical examination demonstrates a 3-cm hard mass in the left testis and an easily palpable
mass is noted in the left upper quadrant. Chest x-ray is negative. Abdominal imaging reveals
a 10-cm retroperitoneal mass only. AFP is within normal limits, beta-hCG is 3,000 mIU/mL.
Which one of the following statements is most accurate?
A. Initial diagnostic procedure should include percutaneous biopsy of the testis.
B. Histology is likely to be nonseminomatous germ cell cancer.
C. The patient should undergo debulking retroperitoneal lymphadenectomy.
D. The probability of cure with combination chemotherapy is approximately 60%.
E. The patient should receive radiation therapy to the abdomen.

Question 21.3 A 30-year-old man has multiple pulmonary lesions detected at the time of
diagnosis of an embryonal carcinoma of the right testis. After orchiectomy and three cycles of
BEP chemotherapy, chest x-ray demonstrates a persistent 2-cm nodule in the right mid lung
field; the other lesions have disappeared. Computed tomography confirms this finding; the
mediastinum and retroperitoneum appear normal. The serum level of beta-human chorionic
gonadotropin, previously elevated, is now undetectable. The most appropriate management
of this patient is:
A. Needle biopsy of the nodule
B. Surgical resection of the nodule
C. Continuation of chemotherapy, without bleomycin
D. Radiotherapy to the residual nodule
E. Obtain PET scan

Question 21.4 A 26-year-old man presents with cough and a left testicular mass. Work-up
reveals multiple retroperitoneal lymph nodes and several lung nodules. Serum hCG is 80
mIU/mL, AFP is normal. Pathologic diagnosis of the testicular mass is pure seminoma. What
is the best management decision?
A. Two cycles of BEP (bleomycin, etoposide, cisplatin)
B. Three cycles of BEP
C. Four cycles of BEP
D. Three cycles of EP

Question 21.5 A 28-year-old man with a history of Klinefelter syndrome presents to his
physician with a complaint of cough and hoarseness of 3 months duration. A chest x-ray
reveals a widened mediastinum. CT confirms the presence of an 8-cm superior mediastinal
mass, Serum AFP is 1,500 ng/mL, beta-hCG 20,000 mIU/mL. A diagnosis of germ cell tumor
of mediastinal origin is made. Which is the most appropriate next step in management?
A. Surgical resection of the mass
B. Three cycles of BEP
C. Four cycles of BEP
D. Radiation therapy
E. Testicular ultrasound

Question 21.6 A 23-year-old male presents with painless enlargement of the testis and
cough. Ultrasound reveals an intratesticular mass. CT scan of the chest, abdomen, and pelvis
reveals multiple retroperitoneal nodes, liver metastases, and multiple pulmonary nodules,
AFP is 16,000 ng/mL, beta-hCG is 200 mIU/mL. At the start of the third cycle of BEP, the
patient is asymptomatic. AFP has decreased to 200, beta-hCG is undetectable. What is the
next step in management?
A. Complete four cycles of BEP
B. Change treatment to salvage therapy with TIP
C. Repeat CT scans for assessment of disease response
D. Consider tandem high-dose therapy with stem cell rescue
E. Obtain PET scan
Question 21.7 A 29-year-old male presents to his physician with left supraclavicular
adenopathy. Examination reveals a hard left testicle. Further workup includes CT imaging,
which demonstrates a 7-cm retroperitoneal mass without parenchymal pulmonary nodules.
AFP is 16,534 ng/mL, beta-hCG 333 mIU/mL. Pathology of the orchiectomy specimen
confirms a mixed germ cell tumor with elements of seminoma, embryonal carcinoma, and
yolk sac tumor. At the completion of the fourth cycle of chemotherapy, repeat CAT scans
reveal a large residual retroperitoneal mass with both solid and cystic components. Tumor
markers have normalized. The patient undergoes retroperitoneal exploration and removal of
the large mass. Pathology reveals mature teratoma. Three years after surgery, the AFP is
now elevated to 33.5. Serial assays confirm increase to 243.6. Repeat scans reveal only a new
retroperitoneal density at the site of previous disease. What is the next best step in the
management of this patient?
A. Initiate salvage chemotherapy with ifosfamide-based chemotherapy
B. Refer for resection of the abdominal mass
C. Prepare for high-dose chemotherapy and stem cell rescue.
D. Any of the above

Question 21.8 A 26-year-old man presents with cough and a left testicular mass. Work-up
reveals a palpable abdominal mass and several lung nodules, Serum AFP is 11,000 ng/mL,
beta-hCG is 450 mIU/mL. Pathologic diagnosis of the testicular mass is pure seminoma. What
is the next best step in the optimal management of this patient?
A. Two cycles of BEP
B. Three cycles of BEP
C. Four cycles of BEP
D. Three cycles of EP

Question 21.9 A 32-year-old man previously treated for testicular cancer 3 years ago with
four cycles of BEP now presents with pancytopenia. Evaluation reveals acute myelogenous
leukemia (AML). What is the likely chromosome abnormality associated with this leukemia?
A. 47XXY
B. t(9:22)
C. 5q-
D. 13q deletion
E. 11q23 translocation

Question 21.10 A 24-year-old man presents with a left testicular mass and undergoes a
radical orchiectomy. Pathologic evaluation of the resected specimen reveals embryonal
carcinoma with vascular invasion. The preoperative level of human chorionic gonadotropin
(hCG) is 950 mIU/mL. The AFP level is within normal limits. CT imaging demonstrates no
evidence of metastatic disease. The marker normalizes postsurgery, but 3 months later on
surveillance the hCG is elevated to 110 mIU/mL, confirmed on repeat. Repeat imaging
demonstrates no pulmonary, liver, or retroperitoneal abnormalities. Which one of the
following would you advise?
A. Four cycles of BEP (bleomycin, etoposide, and cisplatin)
B. Three cycles of BEP
C. Three cycles of etoposide and cisplatin
D. Radiation therapy to the abdomen
E. Observation

Question 21.11 A 27-year-old man is hospitalized with respiratory symptoms. His physical
examination is noncontributory. Plane x-ray of the chest shows several 2-cm nodular
pulmonary lesions bilaterally and a large mediastinal mass. A computed tomographic (CT)
scan of the chest, abdomen, and pelvis confirms a large anterior mediastinal mass and
pulmonary nodules. Fine-needle biopsy of the lung reveals anaplastic carcinoma.
Laboratory studies reveal:

The most appropriate therapeutic option is:


A. Systemic chemotherapy with a cisplatin-based combination
B. Irradiation of the thorax
C. Bilateral thoracotomy and wedge resections of tumors
D. Combination chemotherapy with ABVD (Adriamycin, bleomycin, vinblastine,
dacarbazine)
E. Surgical resection of gross disease followed by systemic chemotherapy

Question 21.12 Risk factors for germ cell tumors includes which one(s) of the following?
(Select three correct responses)
A. Cryptorchidism
B. Intratubular germ cell neoplasia (ITGCN)
C. Father with history of testicular cancer
D. Turcot syndrome

Question 21.13 Germ cell tumors may express which of the following immunohistochemical
proteins? (Select four correct responses)
A. Cytokeratin
B. Placental alkaline phosphatase (PLAP)
C. CD 20
D. CD 30
E. CD 10
F. OCT3/4

Question 21.14 For patients with clinical stage I testis cancer which factor(s) predict for risk
of recurrence after orchiectomy? (Select two correct responses)
A. Presence of lymphovascular invasion and embryonal carcinoma histology
B. Presence of choriocarcinoma as an element in mixed germ cell tumors
C. Seminoma ≥4 cm
D. Degree of elevation of the preorchiectomy markers
E. Seminoma with presence of syncytiotrophoblasts

Question 21.15 Adjuvant chemotherapy for patients with pathologic stage II disease after
RPLND is recommended for patients with:
A. Three positive lymph nodes.
B. Extranodal extension of tumor.
C. Pure embryonal carcinoma histology.
D. Microscopic disease with no lymph node greater than 2 cm.

Question 21.16 According to the International Germ Cell Consensus Classification, which
patient vignette is consistent with poor risk seminoma?
A. Multiple pulmonary metastases with hCG 10,000
B. Disseminated disease with a solitary brain metastasis
C. Multiple liver metastases associated with enlarged retroperitoneal nodes
D. None of the above

Question 21.17 Which of the following is a TRUE statement regarding hCG in germ cell
tumors?
A. The presence of elevated hCG is indicative of nonseminomatous histology
B. The half-life of hCG is 5 to 7 days
C. Very high levels of hCG can produce symptoms of hyperthyroidism
D. Very high levels of hCG are common in patients with disseminated yolk sac tumor

Question 21.18 Which one of the following statements is TRUE regarding NSGCT in first
relapse?
A. Such patients are incurable with standard-dose chemotherapy.
B. EP chemotherapy can be repeated if the patient achieved a complete remission with
first-line BEP therapy.
C. High-dose chemotherapy and autologous stem cell rescue should be considered in the
treatment paradigm.
D. Three cycles of ifosfamide-based chemotherapy should be administered.

Question 21.19 Long-term sequelae of cisplatin-based chemotherapy treatment of germ cell


tumors include:
A. A one- to twofold risk of cardiovascular events.
B. A greater than 50% risk of infertility after primary therapy.
C. Peripheral neuropathy in more than half of the patients treated.
D. Single-digit risk of acute leukemia proportional to the dose of etoposide.
E. Risk of contralateral testis cancer of 5%.

Question 21.20 A 55-year-old male patient presents to his physician because of scrotal
swelling. An ultrasound demonstrates an intratesticular mass. What is the most likely
histology of the lesion?
A. Leydig cell tumor
B. Liposarcoma of the spermatic cord
C. Large B-cell lymphoma
D. Granulosa cell tumor
E. Metastatic carcinoma
ANSWERS

Question 21.1 The answer is B.


The diagnosis of poor risk nonseminomatous germ cell tumor has been established. The
presence of CNS metastases is increased in patients with extensive pulmonary
involvement. The patient has significant tumor burden as indicated by hypoxemia and
immediate therapy for pulmonary disease is warranted. CNS disease can respond rapidly
to chemotherapy. Thus there is no need for prechemotherapy radiotherapy. The
appropriate chemotherapy is the combination of bleomycin, etoposide, and cisplatin
(BEP).

Question 21.2 The answer is B.


The patient has a tumor of the testis with large volume metastatic disease to the
retroperitoneum at presentation. The appropriate diagnostic procedure would include
orchiectomy, not biopsy of the testis which could lead to the incorrect diagnosis due to
sampling and would not remove the primary site which would be necessary in the course
of the disease. 70% of patients with seminoma, while a much smaller percentage of
nonseminoma patients, present with localized, stage I disease; thus on the basis of
probability, the patient should have nonseminomatous disease since metastases at
presentation for seminoma are much less common than for NSGCT. As described the
patient has bulky stage IIC disease and should be treated with chemotherapy. There is no
role in germ cell tumors for debulking. The probability of cure with chemotherapy for
good risk NSGCT is >90%, the outcome is higher for seminoma. Even if the diagnosis
were seminoma, the large volume retroperitoneal disease would best be treated with
chemotherapy not radiation therapy.

Question 21.3 The answer is B.


The presence of residual x-ray findings after chemotherapy for disseminated NSGCT
requires decision regarding intervention versus observation. The likelihood of persistent
cancer after the response described is small, but since the initial path identified
nonseminomatous disease there is a risk of residual teratoma even though this histology
was not described in the original tumor. Needle biopsy of the lesion is insufficient to
identify tumor versus fibrosis based on sampling error. Chemotherapy without
documentation of persistent disease is not warranted and radiotherapy is not indicated for
NSGCT. Likewise PET scanning in nonseminomatous disease does not have a high positive
or negative predictive value to discriminate between viable tumor, fibrosis, or teratoma.
The best answer is to proceed to surgery to remove the lesion to confirm complete
remission and remove a possible site of residual teratoma.

Question 21.4 The answer is B.


The patient has good risk metastatic seminoma (no visceral disease). Thus the standard of
care would include three cycles of BEP. In an attempt to avoid bleomycin exposure,
potential equivalent therapy would include four cycles of EP, not three. Two cycles of
BEP would be insufficient and four, excessive for good risk disease.

Question 21.5 The answer is C.


Klinefelter syndrome is a known risk factor for germ cell tumor involving the
mediastinum. The elevation of AFP identifies the tumor as nonseminomatous. NSGCT of
the mediastinum by definition is poor risk. Therefore the appropriate treatment is four
cycles of BEP. Surgical resection should not be undertaken as primary therapy, radiation
therapy is inadequate for nonseminomatous histology, and testicular pathology is
unlikely. These tumors originate in the mediastinum.

Question 21.6 The answer is A.


The degree of elevation of the AFP at initiation of treatment for NSGCT defines poor risk
disease. After 6 weeks of therapy the marker although lower remains abnormal. The
interpretation of this requires knowledge of the half-life of AFP. At 5 to 7 days, six to
eight half-lives will have transpired in 6 weeks (42 days). Thus the marker should be
between (1/2)6 = 1/64 and (1/2)8 = 1/256 of the original value or between 62.5 and
250. At 200 the value is as predicted and there is no need to change therapy for lack of
response. Likewise there is no indication for transplant strategy. Assessment of response
by imaging in the middle of treatment when the patient demonstrates evidence of
response by markers should not change management. PET scanning is not useful in
nonseminomatous histology. Since this is poor risk disease four cycles of therapy are
indicated.

Question 21.7 The answer is B.


The recurrence of nonseminomatous disease 2 years after complete remission is the
definition of late relapse. This situation is almost always heralded by AFP elevation. Late
relapse is considered relatively chemoresistant. Thus surgical resection should be
considered in those situations in which the tumor appears to be resectable. High-dose
chemotherapy and stem cell rescue is not indicated as primary treatment but consolidation
of patients identified to have chemosensitive disease at relapse.

Question 21.8 The answer is C.


While the pathology of the surgical specimen identifies seminoma only, this histology is
inconsistent with the marked elevation of the AFP. Patients with seminoma and elevated
AFP who do not have another cause for elevation such as chronic liver disease or
congenital neural tube defects should be treated along the algorithm of NSGCT. Thus this
patient has poor risk disease by definition since the AFP is >10,000 (S3) and the best
treatment includes four cycles of chemotherapy.

Question 21.9 The anwser is E.


The long-term risk of leukemia in patients treated with BEP is small, related to the
cumulative dose of etoposide, and is less than 1% for doses included in three or four
cycles of BEP. The characteristic genetic alteration is 11q23 three cycles of BEP.
Question 21.10 The answer is B.
The elevation of a tumor marker while on surveillance is unequivocal evidence of
recurrence. Thus observation at this point is inappropriate. The treatment paradigm for
non seminomatous germ cell tumor depends on the presence of visceral (liver, bone, CNS)
versus nonvisceral disease and the level of the markers. The elevation of the hCG qualifies
as S1; there is no evidence of visceral disease. Thus the patient has good risk disease and
should be treated with three cycles of BEP.

Question 21.11 The answer is A.


Although the histology of the biopsy material does not specifically identify a germ cell
element, the presence of both AFP and beta-hCG elevation is consistent with a germ cell
neoplasm, nonseminomatous type. Thus the treatment for Hodgkin disease with ABVD is
inappropriate. There is no indication for surgery with multiple sites of disease. Irradiation
is not an option for nonseminomatous disease. The best answer is to begin chemotherapy
immediately for poor risk germ cell tumor.

Question 21.12 The answers are A, B, and C.


Cryptorchidism, ITGCN, and first-degree relatives with testis cancer are all risk factors for
the development of testicular cancer. Turcot syndrome is a genetic disorder associated
with polyposis, colorectal cancer, glioblastoma, and medulloblastoma, but not testicular
cancer. Klinefelter’s, a rare genetic disorder, is associated with increased risk of germ cell
neoplasm, usually mediastinal presentation.

Question 21.13 The answers are A, B, D, and F.


Embryonal carcinoma may express CD30 and cytokeratin. Seminoma may express PLAP,
while cytokeratin expression may be weak or negative. Yolk sac tumor, embryonal
carcinoma, and seminomas may express PLAP. OCT3/4 may be expressed in classic
seminoma, yolk sac, and embryonal carcinomas. CD 20 is a marker for mature B cells,
while CD10 is expressed in early B-cell development.

Question 21.14 The answers are A and C.


The major risk factor for recurrence of nonseminomatous testis cancer in stage I patients
is evidence of lymphovascular invasion which predicts a risk of relapse of 50+% for
patients on surveillance. For seminoma the size of the lesion ≥4 cm is a risk factor for
relapse. While pure choriocarcinoma is associated with poor outcome because of the likely
presence of metastatic disease at presentation, the presence of choriocarcinoma as a
component of a mixed germ cell tumor does not of itself predict for a poor outcome.
Elevated markers preop which normalize postorchiectomy, which is required for stage I
classification, do not have predictive value. Syncytiotrophoblasts seen on the histologic
tumor specimen in patients with seminoma may be associated with the measurement of
hCG, but have no predictive or prognostic significance.

Question 21.15 The answer is B.


While primary RPLND is infrequently performed because of the overall excellent outcome
of patients with stage I disease on surveillance, compliance is a major factor in
observation strategies. Those patients who do undergo surgical intervention are at higher
risk for recurrence if there are more than six involved lymph nodes, nodes larger than 2
cm or of any size with extranodal extension. These patients should be considered for
adjuvant chemotherapy. Patients with embryonal histology can be effectively managed
with surveillance and treatment at the time of relapse.

Question 21.16 The answer is D.


While each of these patient vignettes would constitute poor risk disease in a patient with
nonseminomatous histology, there is no category of poor risk disease for seminoma.
These clinical situations would all be classified as intermediate risk.

Question 21.17 The answer is C.


While elevated hCG is more commonly seen in patients with nonseminomatous histology,
a small proportion of patients with seminoma can express hCG. The half-life of hCG is 2 to
3 days, whereas the half-life of AFP is 5 to 7 days. Yolk sac tumors express AFP, not hCG.
Hyperthyroidism can be associated with very high levels of hCG in patients with NSGCT
especially disseminated choriocarcinoma since there is homology between hCG and TSH
which results in excess thyroid hormone production.

Question 21.18 The answer is C.


Patients who relapse after initial therapy for germ cell malignancy can achieve a second
durable remission with ifosfamide-based chemotherapy. However these patients are
generally considered “poor risk” and are given four cycles of therapy. There is no
precedent for repeating EP after BEP. In general patients who relapse should be
considered for high-dose chemotherapy and stem cell rescue strategies.

Question 21.19 The answer is D.


Long-term sequelae of patients treated with chemotherapy include increased risk of
cardiovascular events, peripheral neurotoxicity, ototoxicity, second malignancies,
infertility, and contralateral testis cancer among others. The risk of cardiovascular events
for survivors is estimated at two to seven times that of an age adjusted cohort. Paternity
occurs in >60% patients treated with first-line chemotherapy. 20% to 40% of patients
have chemotherapy-induced neurotoxicity. The risk of contralateral testis cancer is
approximately 2% of survivors. Leukemia can occur within a few years after exposure to
etoposide with incidence proportional to the dose received. For patients receiving 1,500
to 2,000 mg/m2, the risk is less than 1%. The highest risk is noted for patients with
>3,000 mg/m2 which is 6%.

Question 21.20 The answer is C.


The most common tumor of the testis in men older than 50 is lymphoma. Spermatic cord
liposarcomas are usually identified by an extratesticular mass on ultrasound. Granulosa
cell tumors in general are rare and seen in very young patients. The testicle is a very rare
site for a metastatic carcinoma as well. Leydig cell tumors can occur in this age
population but are less frequent in incidence than lymphomas.

Corresponding chapter in Cancer: Principles & Practice of Oncology, Tenth Edition: 70 (Cancer of the Testis).
22 Vulvar, Vaginal, Uterine, and Cervical Cancer
Matthew A. Powell and Laura Divine

QUESTIONS
Each of the numbered items below is followed by lettered answers. Select the ONE lettered answer
that is BEST in each case unless instructed otherwise.

Question 22.1 Which of the following feature is most characteristic of type I endometrial
cancers?
A. Most of these cancers have serous or clear cell histology.
B. Risk factors include unopposed estrogen, anovulation, and obesity.
C. They are rarely (less than 5% of cases) associated with microsatellite instability.
D. The precursor lesion is preneoplastic atrophic endometrium.

Question 22.2 Type II endometrial cancers have which of the following features?
A. The precursor lesion is atypical hyperplasia.
B. The majority of tumors are slow growing.
C. These cancers are unrelated to estrogen exposure.
D. All of the above.

Question 22.3 Genetic changes commonly seen in type I endometrial cancers include:
A. KRAS mutation
B. PTEN mutation
C. β-Catenin mutation
D. All of the above

Question 22.4 Which of the following genetic change(s) is/are most characteristic of type II
endometrial cancers (select three correct answers)?
A. Microsatellite instability (MSI)
B. HER2/neu amplification
C. BCL2 overexpression
D. p53 mutations

Question 22.5 Risk factors for endometrial cancer include: (Select two correct responses)
A. Increasing age.
B. Black race.
C. Family history of endometrial cancer.
D. Combination oral contraceptives (contain estrogen and progestogen).

Question 22.6 The majority of cervical, vaginal, and vulvar cancers appear to have a
common cause, which is:
A. Increased exposure to exogenous estrogen.
B. Chronic bacterial and parasitic infections.
C. Multiple prior herpes simplex virus (HSV) infections.
D. Human papillomavirus (HPV) infection with high-risk types.

Question 22.7 In patients with gestational trophoblastic disease (GTD) with a complete
mole, molecular features include which of the following?
A. Mutations in p53.
B. Most are diploid with duplication of a haploid maternal genome.
C. Predominance of maternal chromosomes is common.
D. Several genes, including CMYC, ERBB2, CFMS, and BCL2, have been implicated in the
pathogenesis of complete moles.

Question 22.8 A 39-year-old married woman is seeing you in the office after recently having
a cervical biopsy that demonstrated cervical cancer. She has no prior medical problems other
than anemia and one prior uncomplicated child birth. You tell her which of the following
epidemiologic factors are TRUE?
A. During the past 80 years, death rates from cervical cancer have decreased primarily
because of improved treatment of STDs such as gonorrhea, chlamydia, and syphillis.
B. Delayed childbearing is associated with an increased risk of cervical cancer.
C. HSV is thought to be the causative agent for the majority of patients.
D. The incidence of cervical cancer is 30% and 100% higher in Black and Hispanic women,
respectively, compared with Whites in the United States.

Question 22.9 The patient asks about prevention and treatment of HPV infections, and you
advise her:
A. The viral infections are rare.
B. Treatments for HPV infections with antiviral therapy are effective if taken within 4 days
of exposure.
C. HPV vaccines have been approved by the Food and Drug Administration (FDA) to help
prevent infection from the most common HPV types.
D. She should receive the HPV vaccine now to help treat her cancer.

Question 22.10 You counsel her about HPV and human immunodeficiency virus (HIV). After
extensive discussion, you decide to proceed with testing for HIV with which of the following
justifications? (Select three correct responses)
A. HIV immunosuppression is correlated with an increased risk of cervical HPV infections.
B. Patients with HIV appear to have a faster rate of progression to high-grade dysplasia
(cervical intraepithelial neoplasia [CIN]).
C. Antiretroviral therapy to manage HIV has demonstrated direct activity against HPV.
D. Cervical cancers in HIV-positive women may be more aggressive than in HIV-negative
women.

Question 22.11 She has done extensive reading about cervical cancer and asks you about
factors that are associated with metastatic disease. You advise her that the following are
associated with extracervical disease:
A. Tumor size and histologic subtype
B. Presence of microinvasion and lymphovascular space invasion
C. Depth of invasion and presence of microinvasion
D. Presence of lymphovascular space invasion and history of pelvic surgery

Question 22.12 If her initial biopsy returned with invasive squamous cell carcinoma with 2
mm of invasion and no lymphovascular space involvement (LVSI), you recommend:
A. Simple hysterectomy.
B. Radiation therapy with possible chemotherapy.
C. Chemotherapy with cisplatin 40 mg/m2 weekly for six cycles.
D. Cervical conization.

Question 22.13 The patient has a 3-cm visible tumor found on her cervix that is biopsied
and is frankly invasive squamous cell carcinoma. She has several questions regarding staging
of her cancer. You advise the following:
A. If enlarged lymph nodes are seen on her computed tomography (CT) scan, her cancer
would be staged appropriately as at least stage III.
B. Staging for cervical cancer is clinical, involving pelvic examination.
C. If hydronephrosis is demonstrated on her CT scan, her cancer would be staged
appropriately as at least stage II.
D. Positron emission tomography (PET) scanning is used in staging to determine nodal
involvement.

Question 22.14 The patient’s 3-cm visible tumor appears confined to the cervix and is staged
as IB1. Which of the following therapies is most appropriate?
A. Radiation and chemotherapy with weekly cisplatin dosed at 40 mg/m2
B. Simple hysterectomy with removal of fallopian tubes and ovaries
C. Radiation therapy or radical hysterectomy with lymphadenectomy
D. Brachytherapy radiation with a dose of 75 Gy

Question 22.15 A 40-year-old woman is diagnosed with invasive cervical cancer. She has a
5-cm cervical tumor with parametrial involvement, and evidence of hydronephrosis on
imaging. Her cancer is stage IIIB. You elect to treat her with combined chemotherapy and
radiation. Which of the following applies to the treatment of locoregionally advanced
cervical cancer?
A. Randomized trials involving patients with locally advanced cervical cancer have failed
to demonstrate a benefit of chemotherapy in addition to standard radiation therapy.
B. Carboplatin appears to be the most appropriate agent to combine with radiation therapy
for cervical cancer and this should be followed by hysterectomy.
C. Paclitaxel with cisplatin is an acceptable regimen to combine with radiation therapy and
has demonstrated improved survival.
D. Weekly cisplatin with radiation therapy appears as active as other regimen with
manageable toxicity.

Question 22.16 After receiving definitive concurrent chemoradiotherapy for her stage IIIB
cervical cancer, the patient develops a recurrence in the cervix, 18 months from the
completion of therapy. Imaging studies suggest no extrapelvic disease. You recommend the
following:
A. Referral back to the radiation oncologist for consideration of further radiation
B. Chemotherapy with combined cisplatin and topotecan
C. Chemotherapy with combined cisplatin and paclitaxel
D. Referral for consideration of further surgery

Question 22.17 A 67-year-old woman presents for evaluation of a cancer found in the
vagina. Before any examination or evaluation, you review with her that which of the
following are TRUE? (Select two correct responses)
A. Prior in utero exposure to the synthetic estrogen diethylstilbestrol (DES) places a
woman at an increased risk for development of vaginal clear cell adenocarcinoma.
B. Most cancers found in the vagina are actually metastases or direct extensions from other
gynecologic tumors.
C. Vaginal squamous cell carcinoma is thought to be unrelated to the HPV.
D. Staging for primary vaginal cancers includes surgical exploration.

Question 22.18 The patient is found to have had a prior hysterectomy for mild dysplasia.
Her apparent primary vaginal cancer extends to the subvaginal tissues and is appropriately
staged as a stage II cancer. You recommend:
A. Referral for total radical vaginectomy
B. Chemotherapy with a cisplatin-based regimen
C. Referral for radiation therapy
D. Local excision followed by close observation

Question 22.19 Which of the following statement(s) applies to invasive vulvar cancer?
(Select two correct responses)
A. There are three distinct types of invasive squamous vulvar cancer.
B. HPV-associated vulvar cancer tends to occur in younger women (age <55 years) and is
associated with prior cervical precancerous abnormalities.
C. Melanoma of the vulva is caused by exposure to HPV.
D. Non–HPV-associated invasive squamous vulvar cancer is associated with lichen
sclerosis.
Question 22.20 A 55-year-old patient with a history of abnormal Pap smears presents for
evaluation of a 3-cm vulvar mass. The mass is located 1 cm lateral to the clitoris on the right
and has an ulcerated appearance. She states she noticed it 2 years ago and has tried many
different ointments to help control the itching and irritation. This was recently biopsied to
reveal invasive squamous cell carcinoma. What treatment would you recommend?
A. Neoadjuvant chemoradiation followed by local resection
B. Concurrent chemoradiation
C. Radical vulvectomy with bilateral inguinal lymphadenectomy
D. Local resection followed by radiation and/or chemotherapy

Question 22.21 Which of the following histologic feature(s) is/are most significant in
predicting the outcome for a patient with vulvar cancer? (Select two correct responses)
A. Presence of lymphovascular invasion
B. Tumor grade
C. Depth of invasion and tumor diameter
D. Presence and number of positive lymph nodes

Question 22.22 The final pathology reveals a 3.3-cm invasive squamous cell carcinoma with
2 involved (positive) lymph nodes larger than 1.0 cm, without extracapsular spread, in the
right inguinal/femoral lymph node dissection out of 12 removed. The contralateral node
dissection was negative. Margins around the primary tumor were negative and greater than 1
cm. Her appropriate International Federation of Gynecology and Obstetrics (FIGO) and TNM
stage are:
A. II and T2N2M0.
B. IIIA and T3N1M0.
C. IIIB and T2N1M0.
D. IVA and T2N1M0.

Question 22.23 The most appropriate therapy for this patient after she recovers from
surgery is:
A. Close observation.
B. Chemotherapy with cisplatin plus 5FU.
C. Referral for consideration of radiation therapy.
D. Exploration with dissection of the pelvic lymph nodes.

Question 22.24 Unfortunately, despite the use of adjuvant therapy, this patient’s cancer
recurs locally on the vulva with a 2-cm lesion. Which of the following would you advise?
A. Repeat resection
B. Chemotherapy with bleomycin
C. Best supportive care
D. Chemotherapy with cisplatin plus 5FU

Question 22.25 A 62-year-old woman has been diagnosed with a uterine corpus (body)
cancer. You have not received her records for review, but she is seen in your office for
consultation and asks many general questions. Which of the following would you tell her?
A. Approximately 90% of these cancers arise from the endometrial lining and are typically
managed with hysterectomy and staging.
B. Most of these cancers are caused by exogenous estrogen use.
C. Uterine sarcomas are more common than endometrial cancers and represent
approximately 90% of corpus cancers.
D. Endometrial cancer typically presents at late stage with patients having abnormal
uterine bleeding.

Question 22.26 This patient’s records arrive in the office, and the pathology verifies the
diagnosis of a grade 2 endometrioid-type endometrial cancer from an office endometrial
biopsy. She asks what has caused her cancer to develop. Which of the following are
considered to be independent risk factors for the development of endometrial cancer? (Select
three correct responses)
A. Obesity
B. Diabetes mellitus
C. Premature ovarian failure with early menopause
D. Presence of an estrogen-producing tumor

Question 22.27 For this patient with an office biopsy demonstrating grade 2 endometrioid
endometrial cancer who is of apparent good heath with no other medical comorbidities, you
recommend:
A. Further evaluation with CT scan and PET scan to evaluate for metastatic disease.
B. A formal dilation and curettage (D&C) to obtain a more accurate pathologic diagnosis.
C. Hysterectomy with removal of the tubes and ovaries with consideration of pelvic and
para-aortic lymphadenectomy.
D. Referral for radiation therapy with possible chemoradiation.

Question 22.28 During the patient’s initial visit, she reports that her mother was diagnosed
with endometrial cancer at age 42, her maternal aunt with colon cancer at age 51, and her
sister with endometrial cancer at age 38. You are concerned her cancer may be hereditary.
You advise that:
A. She undergoes colon assessment if she is not up to date with age-appropriate screening
and considers counseling and possible testing for hereditary nonpolyposis colorectal
cancer (Lynch II syndrome).
B. She undergo breast MRI and be tested for BRCA1 and BRCA2.
C. Colon and endometrial cancers are common, and no further evaluation is necessary at
this time.
D. The maternal side of her family very likely has familial adenomatous polyposis and
should be further evaluated.

Question 22.29 The patient undergoes hysterectomy with full staging, including pelvic
washing for cytology, removal of both tubes and ovaries as well as pelvic and para-aortic
lymph nodes. She is noted to have cancer involving three pelvic lymph nodes, but not the
para-aortic lymph nodes. Her stage is designated:
A. IIIB.
B. IIIC1.
C. IIIC2.
D. IVA.

Question 22.30 For this patient with three positive pelvic lymph nodes and a grade 2
endometrioid endometrial cancer, you recognize adjuvant therapy is controversial but
ultimately recommend:
A. Hormone therapy with a progestational agent
B. Referral for pelvic radiation therapy
C. Chemotherapy with combined paclitaxel, cisplatin, and doxorubicin
D. Combination of radiation and chemotherapy

Question 22.31 The most active (improved disease-free and overall survival) chemotherapy
regimen as determined in randomized clinical trials in women with advanced or recurrent
endometrial cancer with measurable disease is:
A. Doxorubicin plus paclitaxel
B. Cisplatin/doxorubicin
C. Cisplatin/doxorubicin/paclitaxel
D. Ifosfamide plus paclitaxel

Question 22.32 A 17-year-old female patient presents to your office with pelvic ultrasound
suggesting intrauterine gestational trophoblastic neoplasia (GTN). The most common of the
distinct clinicopathologic entities of GTN is:
A. Complete hydatidiform mole.
B. Partial hydatidiform mole.
C. Choriocarcinoma.
D. Placental site trophoblastic tumor.

Question 22.33 This patient has a beta-human chorionic gonadotropin (hCG) of 122,300 and
undergoes therapy with a suction D&C. Final pathology confirms the diagnosis of complete
hydatidiform mole. The patient is followed with the following:
A. CT scan every 3 months.
B. Ultrasound of the pelvis every 3 months.
C. Pelvic examination with Pap smear every 6 weeks.
D. Beta-hCG weekly.

Question 22.34 After a complete metastatic workup, the patient is determined to have low-
risk disease. You recommend the following chemotherapy:
A. Etoposide
B. Vincristine
C. Methotrexate
D. Cyclophosphamide
ANSWERS

Question 22.1 The answer is B.


The Type I endometrioid adenocarcinomas account for 80% of endometrial cancer.
Estrogen acts as a growth factor for the endometrium by promoting glandular
proliferation; thus, unopposed estrogen, anovulation, obesity, early age at menarche, and
nulliparity are risk factors for endometrioid adenocarcinomas. They demonstrate a large
number of genetic changes including microsatellite instability (approximately 20%) or
specific mutations of PTEN, KRAS, and β-catenin genes. The precursor lesion is atypical
hyperplasia and these tumors are usually superficial and low grade.

Question 22.2 The answer is C.


Type II endometrial cancers have serous or clear cell histology. Most tumors are
aggressive and progress rapidly. They are not associated with estrogen exposure. The
usual precursor lesion is endometrial intraepithelial neoplasia.

Question 22.3 The answer is D.


Genetic changes that are characteristic of type I endometrial cancer include KRAS, PTEN,
and β-catenin mutations and MSI.

Question 22.4 The answers are B, C, and D.


MSI is seen in type I endometrial cancers. Genetic changes characteristic of type II
endometrial cancers include p53 mutations, HER2/neu and BCL2
amplification/overexpression.

Question 22.5 The answers are A and C.


Risk factors for endometrial cancer include increasing age, white race, obesity, family
history of endometrial cancer, prior pelvic radiation, non–insulin-dependent diabetes
mellitus, estrogen-only hormone-replacement therapy, estrogen-producing tumors,
nuliparity, early menarche, and late menopause. Combined oral contraceptive
preparations actually have a protective effect.

Question 22.6 The answer is D.


Infection with high-risk HPV appears to be the common cause for these cancers.

Question 22.7 The answer is D.


Several genes, including CMYC, ERBB2, CFMS, and BCL2, have been implicated in the
pathogenesis of complete moles though p53 mutations have not been identified. Most
complete moles are diploid with duplication of the haploid paternal genome and have a
predominance of paternal chromosomes.

Question 22.8 The answer is D.


The increased incidence of cervical cancer in Blacks and Hispanics in the United States is
thought to be secondary to barriers in screening because of the lack of insurance, low
income, and cultural differences. Increased risk of cervical cancer exists in women who
are of young age at time of first coitus, have multiple sexual partners, have partners with
multiple partners, or who bear children at a young age. Cervical cancer is caused by
exposure to high-risk strains of HPV. The presence of HPV DNA has been identified in
more than 99% of cervical carcinomas.

Question 22.9 The answer is C.


HPV infections are not rare. There is no currently effective antiviral therapy that is active
against HPV. Treatments of the precancerous and cancerous changes that develop from
HPV are usually indicated, but the virus can still be often detected even after therapy.
Multiple HPV vaccines have been FDA approved as a preventative vaccine. It appears that
primarily those patients who are naive to the virus benefit from vaccination. Young
women between the ages of 9 and 26 years are currently approved for vaccination.
Ideally, vaccination should occur before the onset of sexual activity.

Question 22.10 The answers are A, B, and D.


All of the factors are correct except that current antiretroviral therapy has not
demonstrated benefit against HPV.

Question 22.11 The answer is A.


The incidence of nodal (extracervical) involvement is directly correlated with tumor
stage, size, histology, depth of invasion, and lymphovascular space invasion, but not
presence/absence of microinvasion or history of pelvic surgery. For patients with stage I
disease treated with radical hysterectomy, 15% to 20% and 1% to 5% of pelvic and para-
aortic lymph nodes, respectively, will be involved. Cervical cancer usually follows an
orderly pattern of spread from the cervix to the pelvic nodes, then to the common iliacs,
and finally to the para-aortic lymph nodes before systemic spread. Lung, extrapelvic
lymph nodes, liver, and bone as the most common sites of distant metastases.

Question 22.12 The answer is D.


Patient with stage IA1 cancers have been safely managed with conization only or simple
hysterectomy. These patients typically are not treated with radiation or chemotherapy.
The difficulty and confusion that arise result from practitioners attempting to make the
diagnosis from small biopsies. To make the diagnosis, the patient should have less than 3
mm of invasion and no more than 7 mm of lateral spread, along with negative margins.
The management of patients with LVSI remains controversial, but its presence is typically
treated as frankly invasive disease. Therefore, patients with superficial invasion on a
small biopsy and without visible tumor on the cervix, should be further evaluated with
cervical conization to allow a proper stage prior to deciding the most appropriate therapy.

Question 22.13 The answer is B.


Staging of cervical cancer is clinical, which is based on careful clinical examination and
routine radiographic studies (e.g., chest radiograph, intravenous pyelogram, lower
gastrointestinal) and procedures (cystoscopy, proctoscopy). Because cervical cancer is
primarily a disease of the developing world, staging procedures have been kept simple to
allow consistency staging throughout the world. CT and PET are often used for treatment
planning in the United States but should not be the basis for changing the stage.

Question 22.14 The answer is C.


For patients with small stage IB1 tumors, either radiation or radical hysterectomy with
lymphadenectomy are appropriate. Factors such as patient preference, anesthetic, and
surgical risks should be considered. Surgical treatment tends to be preferred for young
women with small tumors because it permits preservation of the ovaries and likely better
vaginal function. Radiation invariably leads to menopause with loss of ovarian hormone
production and possible vaginal stenosis. Simple hysterectomy would be considered
inadequate therapy for this size and stage of cancer. Chemoradiotherapy would typically
be indicated for patients with more advanced cancers. Brachytherapy alone would also be
inadequate because the patient is at risk for nodal involvement, and external beam
therapy would also be indicated to treat the nodal area at risk.

Question 22.15 The answer is D.


Several randomized trials have demonstrated a benefit to the addition of chemotherapy,
to standard radiation therapy, for treating women with locoregionally advanced cervical
cancer. Based on these trials, regimens that are commonly used in practice include 5FU
with cisplatin and concurrent radiation, and weekly cisplatin with radiation. None of the
combined chemotherapy and radiation trials included the use of carboplatin.
Hysterectomy following radiation and chemotherapy is no longer considered routine.

Question 22.16 The answer is D.


This patient has developed an apparently isolated central pelvic recurrence after combined
chemoradiotherapy. Her only real curative option is surgery, which likely involve pelvic
exenteration. Patients typically receive the maximum tolerated dose of radiation during
their primary therapy, and further meaningful doses of radiation are usually not possible.
Chemotherapy is typically palliative in nature only. Recent randomized trials have
suggested improved survival with combination chemotherapy over single-agent cisplatin,
but this approach remains palliative.

Question 22.17 The answers are A and B.


DES is a synthetic estrogen previously given to women during pregnancy primarily in
women with a history of miscarriage or premature birth. First reported in 1971, an
association between DES exposure in utero and clear cell vaginal cancer was made. Few
cases of cancer in DES-exposed women have been reported in those aged more than 40
years, and these women continued to be followed closely. If a cancer in the vagina is in
contact with the cervix or the vulva, then it is not classified as a vaginal cancer and the
primary is consider either the cervix or the vulva, respectively. Vaginal cancers are
thought to be related to HPV in at least 60% to 65% of cases. Staging for primary vaginal
cancer is clinical, similar to cervical cancer.

Question 22.18 The answer is C.


Stage II vaginal cancer involves the subvaginal tissues and is typically not amendable to
local resection without injury to the bladder or rectum. The most appropriate therapy is
radiation, likely with both brachytherapy and external beam radiotherapy to treat the
regional nodal areas. Chemotherapy is not indicated for primary therapy. Extrapolating
from randomized studies in patients with cervical cancer, the combination
chemoradiotherapy could be considered for patients with vaginal cancer.

Question 22.19 The answers are B and D.


There appears to be only two types of invasive squamous vulvar cancers. HPV-related
cancers typically occur in younger women and can be multifocal and associated with
vulvar intraepithelial neoplasia (VIN), whereas non-HPV cancers occur in older women
and are often associated with chronic vulvar inflammation or lichen sclerosis. Vulvar
melanoma is not thought to be related to HPV infection.

Question 22.20 The answer is C.


Although all options are used to treat patients with invasive vulvar cancer, the apparent
best option is typically radical vulvectomy with inguinal lymphadenectomy. Staging is
then determined on the basis of the surgical findings, which also dictate adjuvant therapy.
Typically, neoadjuvant and definitive chemoradiation are often necessary with very large
cancers involving the urethra or anus and can help achieve organ preservation, but would
not be typically used with a 3-cm tumor.

Question 22.21 The answers are C and D.


Depth of invasion, tumor diameter, and tumor thickness are all directly related to lymph
node involvement and the patient’s ultimate survival. Tumor grade has been inconsistent
and has not typically been found to be an independent risk factor. LVSI is a very strong
predictor of positive inguinal lymph nodes.

Question 22.22 The answer is C.


Staging for carcinomas of the vulva has been established by the FIGO. In 1983, FIGO
initially adopted a clinical TNM staging system. The reliability of clinical assessment of
regional lymph node status was found to be poor. In 1988, FIGO adopted a surgical
staging system, and this was updated in 1994 to include an additional substage for
minimally invasive cancers (IA) and again in 2009 to expand Stage III. IIIA includes one
lymph node metastasis ≥5 mm or one or two lymph node metastasis(es) <5 mm, IIIB
disease involves two or more lymph node metastases ≥5 mm, or three or more lymph
node metastases <5 mm, and IIIC involves positive nodes with extracapsular spread.
Typically, both TNM and FIGO stages are provided.
Question 22.23 The answer is C.
Radiation is indicated for patients with more than one positive lymph node and should be
considered with any positive lymph nodes or if resection margins are positive and further
resection is not possible. Chemotherapy has not been demonstrated to be of benefit in the
adjuvant setting. A randomized trial evaluating the role of pelvic lymphadenectomy for
patients with positive groin lymph nodes was completed, suggesting there is no role for
routine pelvic lymphadenectomy for these patients.

Question 22.24 The answer is A.


Local recurrence of vulvar cancer is not uncommon. These patients can often be
successfully managed with repeat resection. Large surgical defects from further resections
in the vulva can be managed with the use of flaps and skin grafting. Chemotherapy is
usually of limited efficacy and not needed if the lesion is resectable. Radical surgical
resection, including pelvic exenterations, has been used successfully for large recurrent
cancers.

Question 22.25 The answer is A.


The majority (90%) of uterine corpus cancers arise from the endometrium with sarcomas
representing less than 10% of cases. Exogenous estrogen use appears to cause a minority
of endometrial cancers. In the past, when estrogens were given frequently to women
without progestins to protect the endometrial lining, this was a more frequent cause of
endometrial cancer. Abnormal uterine bleeding typically leads to early presentation and
evaluation of patients with endometrial cancer.

Question 22.26 The answers are A, B, and D.


Risk factors for the development of endometrial cancer are typically related to chronic
estrogenic stimulation. Estrogen therapy, obesity, early age of menarche, late age of
menopause, anovulation, and estrogen-producing tumors have all been associated with the
development of endometrial cancers. Interestingly, diabetes mellitus imparts an increased
relative risk of 3, even when controlling for other known factors.

Question 22.27 The answer is C.


Hysterectomy, whether performed through a large open incision, laparoscopically, or
vaginally, is indicated. Ovarian preservation has been performed in rare circumstances but
is typically discouraged. Preoperative imaging, beyond a chest radiograph for patients
with disease apparently confined to the uterus on clinical examination, is rarely helpful.
Thus, CT, MRI, and PET scans are not typically recommended as a preoperative
evaluation for these patients. The office endometrial biopsy is sufficiently accurate in this
setting, and a D&C would not offer any further benefit to the preoperative evaluation.
Radiation can be used as primary therapy for the treatment of endometrial cancer but is
typically reserved for patients with severe medical comorbidities in whom surgery is
thought to be too high a risk. These patients will still typically require general anesthesia
to allow for brachytherapy implantation into the uterine fundus.
Question 22.28 The answer is A.
The family history presented is worrisome for Lynch II syndrome. Approximately 5% of
endometrial cancers are hereditary, with the majority being Lynch II (hereditary
nonpolyposis colorectal cancer syndrome). Lynch syndrome is caused by a defect in
mismatch repair genes. Women who carry one of these abnormal genes have a 22% to
60% chance of developing endometrial cancer, which is similar to their risk of developing
colorectal cancer. Patients with the diagnosis of endometrial cancer with a strong family
history should be counseled and offered genetic testing. Families with BRCA1 or BRCA2
mutations confer an increased risk in breast and ovarian cancer. Endometrial cancer is not
related to familial adenomatous polyposis syndrome.

Question 22.29 The answer is B.


There are both clinical and surgical staging systems for endometrial cancer. A minority of
patients with major medical comorbidities would typically not undergo surgical
management and require staging via the older clinical staging system. This patient is
staged as IIIC1 due to the involvement of pelvic nodes but not para-aortic nodes.

Question 22.30 The answer is D.


Adjuvant therapy for patients with stage IIIC disease remains controversial. GOG-122
compared whole abdomen/pelvic radiation therapy with chemotherapy (cisplatin plus
doxorubicin for eight cycles) and demonstrated a progression-free and overall survival
benefit to chemotherapy. The GOG-184 evaluated “volume-directed” radiation therapy
that included pelvic and para-aortic lymph nodes when indicated, followed by
chemotherapy. Patients were randomized to two differing chemotherapy arms (cisplatin
plus doxorubicin or cisplatin plus doxorubicin, and paclitaxel). There was no significant
benefit to the addition of paclitaxel for these patients. Methods to reduce toxicity of
radiation therapy including intensity-modulated radiotherapy (IMRT) are being
investigated. Lower toxicity with potentially sparing the pelvic bone marrow should allow
for combination of radiation and chemotherapy to maximize efficacy.

Question 22.31 The answer is C.


The Gynecologic Oncology Group (GOG) has conducted several randomized trials in
patients with advanced or recurrent endometrial cancer. GOG-177 demonstrated the
combination of cisplatin, doxorubicin, and paclitaxel to have an improved progression-free
and overall survival benefit that was statistically significant. This regimen had a response
rate of 57% compared with 34% in the cohort treated with cisplatin and doxorubicin
alone. Ifosfamide plus paclitaxel is a regimen that has been developed and demonstrated
superiority in uterine carcinosarcomas.

Question 22.32 The answer is A.


GTN includes rare gynecologic tumors that represent less than 1% of gynecologic
malignancies. These tumors are potentially life-threatening but usually should be highly
curable when treated appropriately. The most common pathologic entity in GTN is
complete hydatidiform mole.

Question 22.33 The answer is D.


Beta-hCG is an extremely sensitive and specific marker used in the management of these
patients. Patients are typically followed post evacuation with weekly beta-hCG
determinations. The diagnosis of persistent GTN is based on the following: a plateau in
the level for at least 3 weeks, a 10% or greater increase in the level for three or more
values during a 2-week time period, persistence of beta-hCG levels greater than 6 months
after evacuation, or histologic evidence of choriocarcinoma.

Question 22.34 The answer is C.


Treatment for GTN is based on risk assessment either by the WHO or the FIGO guidelines.
Low-risk disease is typically treated with methotrexate or actinomycin-D. Treatment is
continued until the hCG levels are normal for 3 consecutive weeks. The patient is then
followed monthly for 12 months. Hysterectomy can be offered for patients who have
completed childbearing.

Corresponding chapters in Cancer: Principles & Practice of Oncology, Tenth Edition: 71 (Molecular Biology of Gynecologic
Cancers), 72 (Cancer of the Cervix, Vagina, and Vulva), 73 (Cancers of the Uterine Body), 74 (Genetic Testing in Uterine
Cancer), and 75 (Gestational Trophoblastic Neoplasms).
23 Ovarian Cancers
David G. Mutch

QUESTIONS
Each of the numbered items below is followed by lettered answers. Select the ONE lettered answer
that is BEST in each case unless instructed otherwise.

Question 23.1 A 35-year-old premenopausal woman of Ashkenazi-Jewish ancestry presents


to you because her 60-year-old paternal aunt was recently diagnosed with ovarian cancer,
and she is concerned about her own risk for ovarian cancer. She has no siblings, and there is
no other family history of cancer. She has two living children and is in good health, and her
pelvic examination is unremarkable. You should advise her that:
A. Ovarian cancer cannot be inherited through the paternal side and she does not need any
particular screening.
B. A single second-degree relative with cancer at age more than 50 years does not confer a
significantly increased risk for her.
C. She should visit a genetic counselor.
D. Annual transvaginal ultrasound and CA-125 screening can reduce her risk of mortality.
E. She should have a prophylactic total abdominal hysterectomy/ bilateral salpingo-
oophorectomy (TAH/BSO) if/when she does not want to have any more children.

Question 23.2 A 35-year-old woman presents to you for recommendations regarding therapy
of her newly diagnosed mucinous ovarian cancer. This was a 5-cm, grade 1, left-sided mass
that was incidentally found at the time of surgery for endometriosis as part of an infertility
workup. The ovary was removed, and the operative note states that there was no evidence of
tumor on the external surface of the ovary or elsewhere in the abdomen, but full surgical
staging was not performed. A postoperative computed tomography (CT) scan of the abdomen
and the pelvis is unremarkable, and CA-125 is within normal limits. Pelvic examination is
unremarkable. The patient would like to have children but does not want to compromise her
survival. You should advise her that:
A. She is unlikely to have any residual cancer or a recurrence, and further surgery or
chemotherapy is not needed.
B. She should have a positron emission tomography (PET) scan, and if there is no uptake,
she does not need further surgery or chemotherapy.
C. Because her CT scan and CA-125 are normal, she is unlikely to have any residual
disease, and further surgery is not needed. However, because the mucinous subtype of
ovarian cancer has a very poor prognosis, she will require three to six cycles of
carboplatin/paclitaxel chemotherapy.
D. She should have complete surgical staging, if possible, via laparoscopy, with the option
of preserving her uterus and contralateral ovary if there is no further evidence of
tumor, which is associated with a >90% chance of 5-year survival and does not require
chemotherapy.
E. She should have complete surgical staging, including TAH/BSO. If no further disease is
found, she will need only three cycles of carboplatin/paclitaxel chemotherapy, but if
there is disease outside the ovary, she will need six cycles.

Question 23.3 In what aspect of ovarian cancer management is CA-125 most useful?
A. Screening
B. Diagnosis
C. Monitoring treatment
D. Equally useful in all of the above

Question 23.4 Which of the following increases the risk of a woman developing ovarian
cancer?
A. Use of oral contraceptives for >5 consecutive years
B. Nulliparity
C. Breastfeeding
D. Tubal ligation

Question 23.5 A 50-year-old woman presents with a pelvic mass. She is found to have a
tumor of low malignant potential. With TAH/BSO and staging, there is no gross disease but
the washings were positive on final pathology and there was one positive lymph node,
making her disease stage IIIC. What is the most appropriate therapy postoperatively?
A. Intraperitoneal cisplatin and intravenous (IV) taxane
B. IV platinum and a taxane
C. Hormonal therapy with tamoxifen or an aromatase inhibitor
D. No further therapy

Question 23.6 A 50-year-old woman was diagnosed with stage III ovarian cancer and
underwent primary resection followed by chemotherapy. She is asymptomatic but has an
elevated CA-125. Imaging studies do not identify recurrent disease. Which of the following is
the best treatment option?
A. Hormonal therapy
B. Platinum-based chemotherapy
C. Single-agent doxorubicin
D. Single-agent paclitaxel

Question 23.7 Which of the following is/are a founder BRCA mutation(s) associated with
individuals of Ashkenazi descent?
A. 185delAG
B. 5382insC
C. 617delT
D. All of the above

Question 23.8 What percentage decrease in the incidence of ovarian cancer is associated
with oral contraceptive use for more than 5 years?
A. 5%
B. 10%
C. 20%
D. 50%
E. 80%

Question 23.9 A 25-year-old woman has her left ovary removed because of an ovarian mass.
The final pathology shows that this is a moderately differentiated papillary serous cancer.
There was no other evidence of disease in the abdominal cavity. What is the likelihood that
this patient has a positive pelvic or para-aortic lymph node metastasis?
A. 5%
B. 15%
C. 30%
D. 50%

Question 23.10 A 55-year-old woman who has just achieved a complete clinical remission
(normal CT scan, pelvic examination, and CA-125) after six cycles of chemotherapy for stage
IIIC optimally debulked serous ovarian cancer presents to you for a second opinion regarding
her prognosis and treatment options at this point. She is in excellent general health and
tolerated chemotherapy well except for some numbness in her fingers and toes, which caused
her treating oncologist to switch her treatment from paclitaxel/carboplatin to
docetaxel/carboplatin after cycle three. You should advise her that:
A. The risk of eventual relapse for an optimally debulked patient with a complete clinical
remission is approximately 30%. No therapy is proven to be of any further survival
benefit at this point.
B. She should have second-look surgery. If residual disease is found, she should have four
to six cycles of intraperitoneal platinum-based therapy because this can improve
survival in patients with platinum-sensitive, minimal residual disease.
C. She should have a PET scan. If residual disease is found, she should have four to six
cycles of a non–cross-resistant drug, such as topotecan.
D. Her risk of eventual relapse is approximately 70%. She should be offered consolidation
therapy with paclitaxel 175 mg/m2 every 3 weeks for 12 months with the expectation
of a 30% improvement in survival.
E. Her risk of relapse is approximately 70%. No therapy is proven to be of any survival
benefit at this point.

Question 23.11 Which of the following statement(s) is/are TRUE about granulosa cell
tumors of the ovary?
A. They usually occur in premenopausal women
B. They are usually stage III/IV
C. Survival of patients with stage I disease is generally good, but they may relapse later
D. Survival of patients with stage III/IV disease is poor, and they should consider
chemotherapy
E. They may be associated with endometrial cancer
F. C, D, and E

Question 23.12 Which of the following statements is TRUE about germ cell tumors of the
ovary?
A. They occur more often in younger women.
B. They are usually stage III/IV.
C. Appropriate therapy includes TAH/BSO/full surgical staging and chemotherapy
regimens similar to those used in male testicular cancer.
D. Survival of patients with stage III/IV disease is poor.
E. The chemotherapy will usually result in infertility.

Question 23.13 A 51-year-old woman presents to you for recommendations regarding the
treatment of her recurrent ovarian cancer. She was optimally debulked for stage IIIC serous
ovarian carcinoma and completed six cycles of carboplatin/paclitaxel 36 months ago with a
clinical complete remission. She now has recurrent ascites, which is histologically positive for
tumor compatible with her original primary. CT scan shows peritoneal carcinomatosis and a
pelvic mass. You should advise her that:
A. Prognosis of recurrent ovarian cancer is poor. She may achieve short-term benefit from
chemotherapy although hospice is a reasonable option.
B. Tamoxifen has a 40% chance of response.
C. She has a very high likelihood of disease shrinkage and symptom palliation with further
platinum-based chemotherapy.
D. She has a chance of cure with autologous stem cell transplant.
E. Liver metastases and liver failure will probably be her ultimate cause of death.

Question 23.14 A 45-year-old woman has undergone surgical resection followed by adjuvant
chemotherapy for her stage IIIC ovarian cancer. She is now in complete remission and is
interested in further treatment to reduce the risk of recurrent disease. Which of the following
would you recommend?
A. Maintenance treatment with single-agent paclitaxel for 12 months
B. Clinical surveillance
C. Maintenance treatment with single-agent topotecan
D. Maintenance treatment with bevacizumab

Question 23.15 Three randomized controlled trials utilizing intraperitoneal chemotherapy in


optimally cytoreduced advanced ovarian cancer have shown:
A. All showed that intraperitoneal therapy has a progression-free survival (PFS) benefit
over intravenous therapy.
B. All showed that intraperitoneal therapy has an overall survival (OS) benefit over
intravenous therapy.
C. Intraperitoneal and intravenous therapy are equivalent in this patient population.
D. Dose-dense intravenous therapy is equivalent to intraperitoneal therapy.

Question 23.16 A 53-year-old patient presents with ascites, omental caking, and peritoneal
nodularity. Her ovaries are normal. She undergoes surgery and all gross tumors are removed.
The final diagnosis is papillary serous primary peritoneal cancer. How should this patient be
treated?
A. As if she had a standard ovarian cancer
B. As if she had a peritoneal mesothelioma
C. As if she had breast cancer
D. As a germ cell cancer
ANSWERS

Question 23.1 The answer is C.


This woman should be referred for genetic counseling and possible genetic testing. Some
40% to 60% of patients of Jewish descent who have epithelial ovarian cancer carry one of
the three founder BRCA1 or BRCA2 mutations (irrespective of family history), which are
inherited in an autosomal dominant manner, through either the paternal or maternal side.
If this woman carries a BRCA1 mutation, her lifetime risk of ovarian cancer is 20% to
60%, and her risk of breast cancer is even higher. Although transvaginal ultrasound and
CA-125 screening are generally recommended for mutation carriers who have their
ovaries in place, there is no good evidence that such screening will decrease mortality.
Oral contraceptives have been suggested to decrease the risk of ovarian cancer by up to
50%. Removal of the ovaries likely confers the best method for reducing the risk of
ovarian cancer. It remains controversial whether the uterus should be removed as well.

Question 23.2 The answer is D.


Understaging is common, particularly when the preoperative diagnosis is that of a benign
process. Earlier laparoscopic surgical staging series suggested that up to 30% to 40% of
patients thought to have FIGO stage I or II disease actually had disease in the upper
abdomen. The incidence of extraovarian spread is lower with a grade 1 tumor.
Nonetheless, complete surgical staging, if possible, is advised in this case, because the
recommendation for a stage IA grade 1 mucinous tumor is not chemotherapy. Although
some data have suggested that advanced-stage mucinous tumors may lower response to
chemotherapy compared to serous tumors, a low-grade early stage mucinous tumor does
not have a poor prognosis. However, chemotherapy would be recommended in the case of
any extraovarian spread. In the hands of an experienced surgeon, laparoscopic staging,
including omentectomy and para-aortic lymph node examination, is an option.
Preservation of the uterus and contralateral ovary is reasonable if no further disease is
found at the time of staging. CA-125 is frequently normal in women with mucinous
ovarian tumors, even when of advanced stage. A PET scan will not detect microscopic
disease, may not be positive in low-grade malignancies, and is not likely to be helpful.

Question 23.3 The answer is C.


CA-125 is not useful for screening because early stage ovarian cancer has an elevated CA-
125 approximately 50% of the time. Furthermore, the positive predictive value of an
elevated CA-125 is very low because many other disorders also cause an elevated CA-125
and the prevalence of the disease is quite low. Although CA-125 is not particularly helpful
in the diagnosis of ovarian cancer, it is useful in monitoring treatment.

Question 23.4 The answer is B.


Any factor that decreases ovulation is also associated with a decreased the risk of ovarian
cancer. Thus, oral contraceptives, multiparity, and breastfeeding decrease the likelihood
that an individual will develop ovarian cancer. Several population-based studies show that
tubal ligation decreases the likelihood of developing ovarian cancer.

Question 23.5 The answer is D.


There are no data that suggest any adjuvant therapy will improve the survival in patients
with tumors of low malignant potential.

Question 23.6 The answer is A.


Elevated CA-125 indicates disease recurrence and in patients who are asymptomatic with
no other objective evidence of disease recurrence, the appropriate treatment would be
hormonal therapy or observation. Chemotherapy is considered when patients have
symptomatic disease or there is objective evidence for disease recurrence.

Question 23.7 The answer is D.


The majority of inherited disease in the Jewish population is due to one of the three
founder mutations in BRCA1 and BRCA2.

Question 23.8 The answer is D.


Oral contraceptives used for more than 5 years will decrease the risk of ovarian cancer by
more than 50%.

Question 23.9 The answer is B.


Up to 15% of patients with gross stage I diseases have nodal metastases. This is why it is
very important for these patients to undergo comprehensive staging.

Question 23.10 The answer is E.


Although the majority (almost 80%) of patients with advanced-stage (III to IV) ovarian
cancer will achieve a clinically complete remission with taxane/platinum combination
therapy, approximately 70% will ultimately relapse from a clinical complete remission.
Even patients with a surgically confirmed complete remission (negative second look) have
an eventual relapse rate of approximately 50%. Although patients with optimally
debulked stage III disease are a relatively favorable subgroup, those optimally debulked
patients who have a larger presurgical tumor burden (IIIC) fare worse than those with a
lesser presurgical tumor burden (IIIA). One randomized trial has demonstrated that
maintenance paclitaxel improved progression-free survival (PFS) of women with a clinical
complete response from 21 to 28 months in a randomized trial, although no survival
benefit was, and there was a significant incidence of neuropathy. Little data exist on PET
scans in this situation, and consolidation topotecan has shown no benefit in two
randomized trials. Although intraperitoneal therapy is a theoretically attractive option,
cisplatin, the most widely used drug, carries a significant risk of neurotoxicity, and there
are no data showing any survival benefit to its use in the setting of minimal residual
disease.

Question 23.11 The answer is F.


Sex cord–stromal tumors, of which granulosa cell tumors are the most common subtype,
comprise only approximately 5% of all ovarian neoplasms. The peak incidence is in
women aged more than 50 years, although a significant proportion occurs in
premenopausal women. Granulosa cell tumors may secrete estrogen and be associated
with endometrial hyperplasia and endometrial carcinoma. The majority of patients are
diagnosed in stage I, with 10-year survivals of 75% to 95%. However, late relapses may
be observed. Patients with advanced-stage disease have poor outcomes. Although the
rarity of the tumor precludes randomized trial data, most such patients will be offered
chemotherapy, traditionally with bleomycin, etoposide, cisplatin (BEP), or other regimens
used in germ cell tumors. Recent data suggest that a platinum /taxane regimen may be as
effective.

Question 23.12 The answer is A.


Germ cell tumors almost always occur in young women, with a peak incidence in their
early 20s. Some 60% to 70% are stage I at diagnosis. With the use of platinum-based
chemotherapy regimens similar to those used for men with testicular cancer, even patients
with an advanced stage have a good prognosis. Because most of these tumors occur in
young women, often before they have had children, and because the type of
chemotherapy used will not cause infertility in most young female patients, the surgical
approach is critical. In many patients, the contralateral ovary and uterus can and should
be spared. Pre-treatment infertility consultation should be considered as egg harvesting
and storage is effective and of reasonable cost.

Question 23.13 The answer is C.


Recurrent ovarian cancer is not generally curable with transplant or any other modality,
but patients whose disease recurs with a disease-free interval of more than 1 year have
more than a 50% chance of responding to platinum-based combination therapy and should
usually be offered chemotherapy. Secondary debulking surgery may also be of benefit.
Hormonal therapy in ovarian cancer generally produces response rates of only 10% to
15% and is usually reserved for patients who cannot tolerate other therapy. It has also
been recommended as the initial salvage therapy in patients who have an increasing CA-
125 as their only manifestation of recurrent disease.

Question 23.14 The answer is B.


Current evidence does not support maintenance chemotherapy with paclitaxel or
topotecan after adjuvant chemotherapy for patients with advanced disease. The role of
maintenance bevacizumab is being investigated in ongoing clinical trials.

Question 23.15 The answer is A.


All three studies showed that intraperitoneal therapy offered a PFS benefit over
intravenous therapy. Two studies (GOG 104 and 172) showed an overall survival benefit
but GOG 114 showed a trend only. Dose-dense therapy has not been compared directly to
intraperitoneal therapy. However, it appears to improve survival over standard
intravenous therapy.

Question 23.16 The answer is A.


Since primary peritoneal cancers, fallopian tube cancer, and ovarian cancers are
considered to be histologically and molecularly identical, the treatment recommendation
for all three malignancies is the same.

Corresponding chapters in Cancer: Principles & Practice of Oncology, Tenth Edition: 76 (Ovarian Cancer, Fallopian Tube
Carcinoma, and Peritoneal Carcinoma) and 77 (Genetic Testing in Ovarian Cancer).
24 Breast Cancer
Foluso C. Ademuyiwa, Ron Bose, and Cynthia X. Ma

QUESTIONS
Each of the numbered items below is followed by lettered answers. Select the ONE lettered answer
that is BEST in each case unless instructed otherwise.

Question 24.1 What percentage of breast cancers is caused by germline mutations?


A. 20%
B. 10%
C. 15%
D. 2%

Question 24.2 BRCA1-associated breast cancers are characterized by: (Select two correct
responses)
A. Aggressive features.
B. ER positive.
C. Younger age at presentation.
D. All of the above.

Question 24.3 Li–Fraumeni syndrome is characterized by which clinical features?


A. Lobular breast cancer, gastric cancer
B. Breast cancer, soft tissue sarcoma, central nervous system tumors, adrenocortical
cancer, leukemia, prostate cancer
C. Breast cancer, hamartoma, thyroid cancer, oral mucosa cancer, endometrial cancer,
brain tumors
D. Male breast cancer, pancreas cancer, gall bladder cancer, pharynx cancer, gastric
cancer, melanoma, prostate cancer

Question 24.4 Which of the following statements about Luminal A subtype is TRUE?
A. It is more common in premenopausal black women
B. Luminal A breast cancers frequently carry TP53 mutations
C. It is characterized by high-expression levels of ER-related genes and low expression of
the HER2 cluster and proliferation-associated genes.
D. It has a worse prognosis than other molecular subtypes of breast cancer.

Question 24.5 NCCN, ASCO, and/or St. Gallen clinical practice guidelines support Oncotype
DX testing in which of the following scenarios?
A. ER positive HER2-negative patients with T1–3 and N0 or N1mi disease
B. As a marker of recurrence risk in untreated HER2 positive, node-negative patients
C. Predicts 10-year distant relapse risk in patients with early stage ER positive node-
negative breast cancer who will receive adjuvant hormonal therapy
D. A and C

Question 24.6 Which of the following factors is a risk factor for developing breast cancer?
A. Cowden syndrome
B. Early age at first full term pregnancy
C. Breast density <10%
D. Simple breast cysts

Question 24.7 American Cancer Society guidelines for magnetic resonance imaging (MRI)
screening is supported for which of the following patients?
A. A 47-year-old woman with a deleterious mutation in BRCA1
B. A 60-year-old woman with biopsy proven lobular carcinoma in situ (LCIS)
C. A 55-year-old woman with a personal history of ER positive breast cancer treated 8
years ago
D. A 65-year-old woman who underwent chest radiation at age 60 years

Question 24.8 Which of the following scenarios are considered a contraindication to the use
of tamoxifen?
A. Major surgical procedure within the previous 6 months
B. A history of deep vein thrombosis, stroke, pulmonary embolism, or transient ischemic
attack
C. A 60-year-old woman with bilateral asymptomatic cataracts
D. A 65-year-old woman currently on an SSRI for hot flashes

Question 24.9 A 44-year-old woman presents to your office with a new palpable breast
mass. A biopsy has been recommended based on the results of mammogram and ultrasound.
In general, the most appropriate way to diagnose suspected invasive carcinoma is:
A. Either a core or fine-needle biopsy.
B. An excisional biopsy.
C. Diagnosis by core needle biopsy.
D. A fine-needle aspiration biopsy.

Question 24.10 You are advising a 42-year-old premenopausal woman with a history of
atypical hyperplasia on prevention strategies. Which of the following is CORRECT?
A. Tamoxifen reduces her risk by #x223C;50%.
B. Oophorectomy reduces her risk by ∼50% to 65%.
C. Bilateral mastectomy completely eliminates her risk of breast cancer.
D. Raloxifene would provide similar benefit as tamoxifen.
Question 24.11 The Gail model includes which of the following risk factors?
A. Second-degree relatives with breast cancer
B. Previous breast biopsies
C. Age at first pregnancy conception
D. Age at menopause

Question 24.12 Which of the following statement about DCIS is TRUE?


A. DCIS accounts for 15% to 30% of mammographically detected cancers.
B. DCIS is most common in women aged 40 to 59 years.
C. Younger women have a lower rate of local recurrence after local therapy
D. Sentinel lymph node biopsy should be performed routinely during breast-conservation
surgery for nonpalpable DCIS.

Question 24.13 The use of radiation therapy (RT) after lumpectomy in patients with DCIS:
A. Improves overall survival (OS).
B. Reduces the risk of recurrent DCIS, but not invasive disease, in the treated breast.
C. Does not decrease local recurrence when tamoxifen therapy is administered.
D. None of the above.

Question 24.14 What stage describes a patient with a 2-cm primary breast tumor metastatic
to a movable ipsilateral lymph node?
A. T1N1M0 (IIA)
B. T2N0M0 (IIA)
C. T2N1M0 (IIB)
D. T3N1M0 (IIIA)

Question 24.15 Which of the following statements concerning local recurrences after breast-
conserving therapy is TRUE? (Select two correct responses)
A. The underlying molecular subtype is the most significant determinant of the likelihood
of local recurrence after breast-conserving therapy and mastectomy, particularly when
negative margins are achieved.
B. Most local recurrence events appear to be residual disease as opposed to second primary
cancers following BCS and RT.
C. Widely clear margins of at least 2 cm decrease the risk of a local recurrence in young
patients under 40 years.
D. Adjuvant chemotherapy affects the risk of both systemic and local recurrences.

Question 24.16 Which of the following is an absolute contraindication to breast-conserving


surgery (BCS) requiring radiation therapy?
A. Radiation therapy during pregnancy
B. Diffuse suspicious or malignant-appearing microcalcifications
C. Widespread disease that cannot be incorporated by local excision through a single
incision that achieves negative margins with a satisfactory cosmetic result
D. All of the above

Question 24.17 Identify the CORRECT statement.


A. Patients age 70 and above with stage 1, ER+ breast cancer treated by lumpectomy and
tamoxifen alone have no difference in locoregional recurrence rates relative to those
who receive whole breast radiation in addition.
B. Preoperative chemotherapy has a long-term survival advantage relative to adjuvant
chemotherapy.
C. Concurrent use of pertuzumab with trastuzumab and chemotherapy is appropriate in
women receiving preoperative chemotherapy for HER2+ breast cancer.
D. All of the above.

Question 24.18 The gains associated with adjuvant tamoxifen:


A. Are independent of patient age.
B. Are less in patients who also receive adjuvant chemotherapy.
C. Are dependent on patient menopausal status.
D. Decline 10 years after diagnosis.

Question 24.19 Identify the CORRECT statement.


A. A maximum duration of 5 years is the only duration for which safety and efficacy data
exists for an aromatase inhibitor.
B. Ovarian suppression in combination with tamoxifen has no benefit relative to tamoxifen
alone in high-risk premenopausal women with breast cancer.
C. Concurrent use of an aromatase inhibitor and tamoxifen is appropriate therapy for high-
risk node-positive breast cancer patients.
D. All patients treated with aromatase inhibitors should undergo monitoring of bone
mineral density and treatment with denosumab when osteoporosis is detected.

Question 24.20 Identify the CORRECT statement regarding the use of taxanes in the
adjuvant setting:
A. Concurrent administration of docetaxel/doxorubicin/cyclophosphamide chemotherapy
is better tolerated than sequential dose-dense AC followed by paclitaxel due to lower
rates of neutropenia.
B. Weekly paclitaxel is associated with increased grade 3 and 4 neutropenia relative to
dose-dense paclitaxel.
C. Weekly paclitaxel is associated with increased grade 3 and 4 neuropathy relative to
dose-dense paclitaxel.
D. Incorporation of gemcitabine into adjuvant anthracyclines- and taxanes-based
chemotherapy improves efficacy.

Question 24.21 Risk factors for cardiac dysfunction with trastuzumab include:
A. Preexisting cardiac disease, age >65 years.
B. Non–anthracycline-based chemotherapy, age >60 years.
C. Age >65 years, current aspirin use for cardio protection, hyperthyroidism.
D. None of the above.

Question 24.22 A 57-year-old woman has been taking anastrozole as adjuvant therapy for a
moderately-differentiated T1N1 stage II breast cancer for 3 years. During a routine follow up
visit you recommend:
A. History and physical examination, complete blood count and liver function tests.
B. History and physical examination, complete blood count and liver function tests, cancer
antigen 15-3.
C. History and physical examination.
D. History and physical examination, complete blood count and liver function tests, cancer
antigen 15-3, and yearly computed tomography scans.

Question 24.23 Regarding inflammatory breast cancer (IBC):


A. Patients presenting with clinical features of IBC but no discrete palpable breast mass
may forego systemic staging
B. The presence of dermal lymphatic involvement indicates IBC regardless of the clinical
features
C. Adjuvant radiation is not necessary in IBC patients who achieve a pathologic complete
response to neoadjuvant systemic therapy.
D. None of the above

Question 24.24 With regard to the clinical and pathologic characteristics of male breast
cancer, which of the following is TRUE?
A. Male breast cancer is found, more often than female breast cancer, to be estrogen-
receptor negative, and the older a man is with breast cancer, the more likely the cancer
is estrogen-receptor negative.
B. Liver cirrhosis and mumps orchitis are associated with a decreased risk of male breast
cancer
C. The median age of onset is 10 years younger than the median age of onset for females.
D. Sentinel node biopsy is the preferred treatment for clinically node-negative patients.

Question 24.25 Regarding bone metastases, which of the following is TRUE?


A. The optimal duration of RANK-ligand inhibition is 2 years.
B. Intravenous bisphosphonates and RANK-ligand inhibitors lessen the pain associated with
bone metastases.
C. Bone directed therapies in patients with widespread bone metastases improve overall
survival.
D. Only patients with asymptomatic sclerotic bone metastases benefit from bone-directed
therapies.
ANSWERS

Question 24.1 The answer is B.


Approximately 10% of all breast cancers are associated with germline mutations, while
the rest occur sporadically.

Question 24.2 The answer are A and C.


BRCA1-associated breast cancers usually occur in younger women, have aggressive
features, and are characterized by a “triple negative” phenotype (ER, PR, and HER2
negative).

Question 24.3 The answer is B.


Li–Fraumeni syndrome is characterized by TP53 mutations leading to breast cancer, soft
tissue sarcoma, central nervous system tumors, adrenocortical cancer, leukemia, and
prostate cancer

Question 24.4 The answer is C.


Luminal A breast cancer is characterized by high-expression levels of ER-related genes and
low expression of the HER2 cluster and proliferation-associated genes. It rarely has
mutations in TP53 and has an overall better prognosis than other forms of breast cancer.

Question 24.5 The answer is D.


Clinical practice guidelines support consideration for multigene testing to guide the
addition of combination chemotherapy to standard hormone therapy for hormone
receptor positive patients with T1–3 tumors who are node negative or have pN1mi (less
or equal to 2 mm axillary node metastasis) involved.

Question 24.6 The answer is A.


Cowden syndrome confers a 25% to 50% lifetime risk of breast cancer. Other risk factors
include late age (after age 30) at first full term pregnancy, >75% breast density,
proliferative breast lesions.

Question 24.7 The answer is A.


ACS guidelines for MRI screening include individuals with BRCA mutations, untested first-
degree relative of BRCA carrier, those with lifetime risk of breast cancer 20% to 25%,
radiation to the chest between ages 10 to 30 years, Li–Fraumeni, Cowden, or Bannayan–
Riley–Ruvalcaba syndromes and first-degree relatives.

Question 24.8 The answer is B.


Histories of deep vein thrombosis, stroke, pulmonary embolism, or transient ischemic
attacks are considered contraindications to the use of tamoxifen and raloxifen.
Question 24.9 The answer is C.
Diagnosis by core needle biopsy is the most appropriate way to diagnose suspected
invasive disease. Fine-needle aspiration does not reliably distinguish invasive cancer from
DCIS. Excisional biopsy as a diagnostic technique should be reserved for patients with
imaging abnormalities that cannot be targeted by core biopsy.

Question 24.10 The answer is B.


In the P1 trial, tamoxifen reduced the risk of breast cancer by 84% in patients with
atypical hyperplasia. Although raloxifene provided similar benefits with a more favorable
safety profile than tamoxifen in the STAR trial, this trial only included postmenopausal
women. Oophorectomy before menopause decreases the risk of breast cancer by 50% to
65% depending on age at the time of surgery. Although bilateral mastectomy decreases
the risk by more than 90%, the risk is not completely eliminated.

Question 24.11 The answer is B.


The Gail model includes first-degree relatives with breast cancer, previous breast biopsies,
age at menarche, age at first live birth, race/ethnicity.

Question 24.12 The answer is A.


DCIS accounts for 15% to 30% of mammographically detected cancers and is most
common among women ages 49 to 69 years. Several studies have reported an increased
risk of local recurrence in younger women. Initial studies suggested that MRI can both
over- and underestimate the extent of DCIS and does not improve surgical planning.

Question 24.13 The answer is D.


The use of RT after BCS for DCIS reduces both invasive and noninvasive recurrences but
does not alter OS.

Question 24.14 The answer is A.


The patient has T1 (tumor size ≤2 cm), N1 (movable ipsilateral level 1 lymph node), and
M0. T1N1M0 represents stage IIA.

Question 24.15 The answers are A and D.


The underlying molecular subtype is the most significant determinant of the likelihood of
local recurrence after breast-conserving therapy or mastectomy, particularly when
negative margin is achieved. Most local recurrence events appear to be second primary
cancers following BCS and RT. Although RT is effective in eliminating subclinical multi-
centric foci of breast cancer present at the time of diagnosis, it does not prevent
subsequent development of new cancers. Widely clear margins do not appear to decrease
the risk of a local recurrence in young patients. Adjuvant chemotherapy affects the risk of
both systemic and local recurrences.

Question 24.16 The answer is D.


All of the above are absolute contraindications according to NCCN 2014.

Question 24.17 The answer is C.


In 2013, the FDA approved concurrent use of pertuzumab with trastuzumab and
chemotherapy in women receiving preoperative chemotherapy for HER2+ breast cancer.
Patients age 70 and above with stage I, ER+ breast cancer treated by lumpectomy and
tamoxifen alone have no difference in overall survival relative to those who receive
whole breast radiation in addition, but have an improvement in locoregional recurrence of
10% versus 2%. Preoperative chemotherapy has no long-term survival advantage relative
to adjuvant chemotherapy.

Question 24.18 The answer is A.


Adjuvant tamoxifen results in an improvement in OS for at least 15 years. The benefits are
independent of age, menopausal status, and the use of chemotherapy.

Question 24.19 The answer is A.


A maximum duration of 5 years is the only duration for which safety and efficacy data
exists for an aromatase inhibitor. Ovarian suppression in combination with tamoxifen has
proven benefit relative to tamoxifen alone in high-risk premenopausal women with breast
cancer. Concurrent use of an aromatase inhibitor and tamoxifen is not standard therapy
for breast cancer patients. All patients treated with aromatase inhibitors should undergo
monitoring of bone mineral density and be considered for denosumab to prevent bone loss
and fracture.

Question 24.20 The answer is B.


Weekly paclitaxel is associated with increased grade 3 and 4 neutropenia relative to dose-
dense paclitaxel as pegfilgrastim support is not administered with the weekly paclitaxel,
whereas dose-dense paclitaxel is associated with increased grade 3 and 4 neuropathy.
Incorporation of gemcitabine into adjuvant anthracyclines- and taxanes-based
chemotherapy does not improve efficacy.

Question 24.21 The answer is A.


Preexisting cardiac disease, age >65 years, anthracycline-based chemotherapy, borderline
left ventricular ejection fraction, and hypertension are all risk factors for cardiac
dysfunction with adjuvant trastuzumab.

Question 24.22 The answer is C.


Complete blood counts and liver function tests, cancer antigen 15-3, and yearly computed
tomography scans are not indicated in routine surveillance for early stage breast cancer
patients.

Question 24.23 The answer is D.


All patients presenting with clinical features of IBC should undergo full-staging work up
due to the substantial risk of metastatic disease. Involvement of dermal lymphatics in the
absence of clinical findings does not indicate IBC as it is a clinical diagnosis. Due to the
high risk of local recurrence, all patients with IBC should undergo postmastectomy
radiation regardless of the response to neoadjuvant therapy.

Question 24.24 The answer is D.


Male breast cancer is found, more often than female breast cancer, to be estrogen-
receptor positive. Chronic liver disorders, Klinefelter syndrome (XXY), BRCA 1 or 2
mutations, mumps orchitis, undescended testis, testicular injury, and feminization are all
associated with an increased risk of male breast cancer. The median age of onset is later
than in females. The same considerations regarding nodal surgery pertain for men as for
women.

Question 24.25 The answer is B.


The optimal duration of intravenous bisphosphonates or RANK-ligand inhibitors is not
well characterized. Intravenous bisphosphonates and RANK-ligand inhibitors lessen the
pain associated with bone metastases and prevent skeletal-related complications, such as
hypercalcemia and fractures. Bone-directed therapies in patients with widespread bone
metastases do not impact overall survival.

Corresponding chapters in Cancer: Principles & Practice of Oncology, Tenth Edition: 78 (Molecular Biology of Breast Cancer),
79 (Malignant Tumors of the Breast), and 80 (Genetic Testing in Breast Cancer).
25 Cancers of the Central Nervous System
Tanner M. Johanns, Jian Li Campian and George Ansstas

QUESTIONS
Each of the numbered items below is followed by lettered answers. Select the ONE lettered answer
that is BEST in each case unless instructed otherwise.

Question 25.1 Linkage studies have identified genes associated with neurofibromatosis type
2 (NF2), Turcot syndrome, and Li–Fraumeni syndrome in which of the following
chromosomes?
A. 10q, 22q, and 17q
B. 22q, 5q, and 17p
C. 5q, 10q, and 17p
D. 5q, 10q, and 17q

Question 25.2 Of the following, the most common primary brain tumor in adults in the
United States is:
A. Glioblastoma.
B. Meningioma.
C. Astrocytoma.
D. Oligodendroglioma.

Question 25.3 The gene that is frequently altered and plays a key role in the development of
diffuse fibrillary astrocytoma is:
A. p53.
B. KRAS.
C. MDM2.
D. MDM4.

Question 25.4 Primary (de novo) glioblastoma multiforme (GBM) is commonly associated
with which of the following?
A. PTEN inactivation
B. Loss of heterozygosity of chromosomes 1p and 19q
C. IDH1 mutation
D. Median age <40 years

Question 25.5 Which gene mutation identified in GBM is frequently seen in younger
patients, and is associated with better prognosis:
A. EGFR.
B. p53.
C. IDH1.
D. Gain in chromosome 4.

Question 25.6 Loss of heterozygosity of chromosomes 1p and 19q is common in:


A. GBM.
B. Ependymoma.
C. Oligodendroglioma.
D. Meningioma.

Question 25.7 The central nervous system (CNS) tumor that is commonly associated with
NF2 mutations is:
A. GBM.
B. Spinal ependymoma.
C. Cerebral ependymoma.
D. Oligodendroglioma.

Question 25.8 Patients with GBM have a higher likelihood of responding to therapy with
epidermal growth factor receptor tyrosine kinase (EGFR-TK) inhibitors if which of the
following biomarkers is present?
A. Methyl guanine methyl transferase (MGMT) gene methylation
B. Activated EGFRvIII
C. Retained PTEN function
D. B and C

Question 25.9 The primary CNS neoplasm that is associated with Epstein–Barr virus (EBV)
is:
A. Primary CNS lymphoma.
B. Ependymoma.
C. Oligodendroglioma.
D. GBM.

Question 25.10 Which of the following tumors exhibit contrast enhancement on magnetic
resonance imaging (MRI) scan?
A. Pilocytic astrocytoma
B. Grade 2 oligodendroglioma
C. Grade 2 astrocytoma
D. Craniopharyngioma

Question 25.11 Which of the following is associated with good prognosis in patients with
low-grade gliomas?
A. Age ≥40 years
B. Tumor diameter ≥6 cm
C. Tumor crossing midline
D. Oligodendroglioma

Question 25.12 A 45-year-old man presents with generalized seizures, and MRI of the brain
reveals a nonenhancing mass measuring 7 cm. Biopsy is done, and the tumor histology is
reported as grade II astrocytoma. The patient undergoes surgery, and a partial (85% to 90%)
tumor resection is achieved. Further treatment should include:
A. Watchful waiting.
B. Chemotherapy.
C. Radiotherapy with 50.4 Gy in 1.8 Gy fractions.
D. Radiotherapy with chemotherapy (procarbazine, CCNU, and vincristine).

Question 25.13 In patients with GBM, who have disease recurrence following initial
treatment with temozolomide and radiation, the second-line treatment of choice is:
A. Erlotinib.
B. Imatinib.
C. Topotecan.
D. Bevacizumab.

Question 25.14 A 55-year-old man presents with headache and mental status changes. MRI
scan of the brain reveals a 5-cm contrast-enhancing mass with surrounding edema. A
stereotactic biopsy reveals GBM. The patient undergoes gross total resection of the tumor.
Further treatment should include:
A. Focal external beam irradiation with 45 Gy in 30 fractions plus concurrent
temozolomide.
B. Focal external beam irradiation with 60 Gy plus a 10 Gy boost plus concurrent
temozolomide.
C. Focal external beam irradiation with 60 Gy in 30 fractions plus concurrent
temozolomide.
D. Hyperfractionated dose of 72 Gy in 1.2 Gy fractions plus concurrent temozolomide.

Question 25.15 A 62-year-old man presents with nausea, vomiting, and severe headache.
MRI scan of the brain reveals a 5-cm mass in the posterior fossa. The patient undergoes
surgical resection of the mass, and pathology is reported as ependymoma. Further treatment
should include:
A. Radiation therapy.
B. Concurrent chemoradiation with temozolomide.
C. Chemotherapy alone with temozolomide if MGMT gene methylated.
D. Chemotherapy alone with cisplatin plus etoposide.

Question 25.16 A 53-year-old woman presents with mental status changes; brain MRI
identifies a left frontal tumor with dural marginal thickening. She undergoes a gross total
resection of the tumor and the involved dural attachments. The histopathology is WHO grade
I meningioma. Further treatment should include:
A. Watchful waiting.
B. Adjuvant radiation therapy.
C. Chemotherapy.
D. Radiosurgery.

Question 25.17 In immunocompetent patients with primary CNS lymphoma, with good
performance status and adequate renal function, appropriate first-line therapy comprises of:
A. High-dose methotrexate.
B. R-CHOP.
C. Whole brain radiation.
D. None of the above.

Question 25.18 Long-term follow-up data showed improved survival for patients with GBM,
receiving concurrent brain radiation with temozolomide compared to radiation alone, with
the exception of patients with:
A. Age >50 years.
B. Unmethylated MGMT.
C. EGFR mutation positive.
D. None of the above.

Question 25.19 Which of the following is most commonly associated with


craniopharyngioma?
A. Sexual dysfunction
B. Visual dysfunction
C. Hypothyroidism
D. Diabetes insipidus

Question 25.20 In adult patients diagnosed with ependymoma, adjuvant radiation therapy
after resection is associated with improved overall survival in patients with:
A. Supratentorial location.
B. High-grade tumors.
C. Incomplete resection.
D. Poor performance status.

Question 25.21 In patients with anaplastic oligodendrogliomas, which gene mutation is an


independent prognostic factor for improved overall survival?
A. IDH1
B. EGFR
C. p53
D. NF2
Question 25.22 Which GBM gene expression profile is most commonly associated with focal
deletions or mutations in NF1?
A. Proneural
B. Neural
C. Classical
D. Mesenchymal

Question 25.23 Which three core pathways are commonly mutated in primary GBMs?
A. SHH, NF2, TRAF7
B. WNT, SHH, MAPK
C. PI3K, p53, Rb
D. NF-kB, p53, STAT

Question 25.24 Which somatic mutation is most often associated with Grade II
astrocytomas?
A. CIC
B. ATRX
C. FUBP1
D. TERT promoter
ANSWERS

Question 25.1 The answer is B.


Linkage studies have revealed the NF1 gene to reside in chromosome 17q and the NF2
gene to reside in chromosome 22q. For Turcot syndrome, the APC gene has been
identified in chromosome 5q. For Li–Fraumeni syndrome, the cancer disposition has been
shown to be associated with chromosomes 17p and 22q.

Question 25.2 The answer is B.


According to data from the Central Brain Tumor Registry of the United States (1998 to
2002), the proportionate distribution of incidence is 30.1% for meningioma, 20.3% for
glioblastoma, 9.8% for astrocytoma, and 2.3% for ependymoma.

Question 25.3 The answer is A.


Alterations of p53 gene are found in 30% of all three grades of astrocytoma. Other
reported gene alterations include the amplification of MDM2 or MDM4 gene.
Chromosome 9p deletions resulting in loss of the p14 product of the CDKN2A gene have
also been reported.

Question 25.4 The answer is A.


Primary (de novo) GBM is associated with age greater than 62 years, amplified EGFR
(polysomy chromosome 7), dysregulated p53, PTEN inactivation, homozygous loss of
CDKN2A, and decreased survival.

Question 25.5 The answer is C.


A comprehensive effort to sequence the tumor genome of GBM involved the sequencing
of 20,661 genes for somatic mutations from 22 GBM samples. This study also integrated
results from copy number and expression analysis of tumor tissue. In addition, to identify
previously known dysregulated gene pathways such as p53, EGFR, and PTEN, this study
identified mutations involving the isocitrate dehydrogenase 1 (IDH1) gene in 11% of GBM
samples. Patients with mutated IDH1 gene were more likely to be younger and had better
prognosis than patients with wild-type IDH1, the median survival was 3.7 years versus 1.1
years, respectively (p <001).

Question 25.6 The answer is C.


Allelic loss of chromosome 1p and 19q occurs in 40% to 80% of grade 2 and 3
oligodendrogliomas. The loss of tumor suppressor genes from these chromosomes is
believed to play an important role in the early stage of oligodendroglial tumorigenesis.

Question 25.7 The answer is B.


Chromosome 22q loss is common in ependymomas. In spinal ependymomas, this loss is
associated with mutations of the NF2 gene. In cerebral ependymomas, loss of
chromosome 22q is not associated with NF2 mutations.

Question 25.8 The answer is D.


Patients with GBM had improved survival when treated with EGFR-TK inhibitors when
the tumors expressed the common form of mutated EGFR (activated EGFRvIII) and
retained PTEN function compared with patients not expressing both of these biomarkers.
Patients with MGMT gene methylation were shown in a randomized phase III trial to have
significantly improved survival when treated with concurrent temozolomide and radiation
when compared with patients who were either negative for MGMT gene methylation or
received radiation only.

Question 25.9 The answer is A.


Primary CNS neoplasms are not usually associated with viral infections in humans, except
for primary CNS lymphoma, which is strongly associated with EBV. The majority of
primary CNS lymphomas are large B-cell histology, and EBV DNA is detectable in most of
these tumors. In addition, the incidence of primary CNS lymphoma is higher in patients
with HIV infection.

Question 25.10 The answer is A.


Contrast-enhanced MRI images help differentiate between different types of CNS tumors.
MRI also helps to differentiate between high- and low-grade tumors. Low-grade CNS
tumors generally do not enhance, except for pilocytic astrocytoma and pleomorphic
xanthoastrocytoma.

Question 25.11 The answer is D.


Oligodendroglioma, specifically in patients with 1p/19q codeletion, has a high response
rate to chemotherapy and is associated with an improved overall survival. Conversely,
multivariate analysis from two large European phase III trials have reported age greater
than or equal to 40 years, tumor diameter greater than or equal to 6 cm, tumor crossing
the midline, astrocytoma histology, and neurologic deficits to be associated with adverse
prognosis.

Question 25.12 The answer is C.


Radiotherapy is not routinely recommended in patients undergoing resection for low-
grade gliomas. In the European Organization for Research and Treatment of Cancer
(EORTC) 22,845 trial, patients receiving immediate postoperative radiotherapy had a
progression-free survival advantage compared with those receiving radiation therapy on
tumor progression (5.3 vs. 3.4 years, p <0001). There was no significant difference in
overall survival (7.4 years vs. 7.2 years). In another study, age greater than or equal to
40 years, tumor diameter greater than or equal to 6 cm, astrocytoma histology, and
incomplete resection were identified as adverse prognostic factors, with the presence of
three or more of these factors conferring poor prognosis. In such patients, immediate
postoperative radiotherapy is recommended.
Question 25.13 The answer is D.
The prognosis for recurrent GBM is poor with a median survival of 3 to 9 months when
using traditional chemotherapeutic agents. However, several recent publications have
demonstrated a significant improvement in quality of life among patients with recurrent
GBM using the angiogenesis inhibitor, bevacizumab, though no difference in overall
survival has been demonstrated.

Question 25.14 The answer is C.


For patients with GBM, standard radiation therapy is 60 Gy in 30 or 33 fractions. The
Medical Research Council study reported a survival advantage in patients receiving a 60
Gy dose versus a 45 Gy dose (12 months vs. 9 months, p = .007). In an RTOG/ECOG
study, patients receiving 60 Gy followed by a 10 Gy boost did not have a survival
advantage over patients receiving the standard 60 Gy dose. Hyperfractionated and
accelerated regimens have likewise not shown survival advantage. In the phase I/II RTOG
8302 study, hyperfractionated doses with 64.8 Gy, 72 Gy, 76.8 Gy, and 81.6 Gy in 1.2 Gy
twice daily fractions did not show a survival benefit between the different treatment
arms. This study included treatment arms with accelerated hyperfractionated regimens of
48 Gy and 54.4 Gy in 1.6 Gy twice daily fractions, which also did not lead to survival
benefit. Adjuvant temozolomide with radiation was shown to improve overall survival
compared with radiation alone in a randomized, phase III trial for patients with GBM
(14.6 months vs. 12.1 months).

Question 25.15 The answer is A.


Surgical resection is the primary treatment of patients with ependymoma. There are no
definitive recommendations for the addition of radiotherapy to the treatment. However,
tumors in the posterior fossa cannot always be fully resected, and patients undergoing
resection alone have a very high incidence of local recurrence. Therefore, adjuvant focal
radiotherapy to the site of the tumor may be added in the treatment of patients with
ependymoma. In one study, patients receiving focal radiotherapy of more than 45 Gy to
the tumor bed had 5- and 10-year survival rates of 67% and 57%, respectively. The
addition of focal radiotherapy has been shown to improve 10-year survival from 67% in
patients treated with gross total resection (GTR) alone to 83% in patients treated with
GTR and radiotherapy. While MGMT gene methylation is associated with better survival
in patients with GBM, there has been no reported association between MGMT gene
methylation and survival in ependymomas. Likewise, there is no evidence to date
supporting the use of chemotherapy for treating ependymoma in general.

Question 25.16 The answer is A.


Surgical resection is the mainstay for the treatment of meningioma, and it is classified
into Simpson grades based on the extent of surgery. Grade I is gross total resection (GTR)
of the tumor, with removal of the dural attachments and hyperostotic bone. The relapse
rate after this procedure is 9%. Grade II does not include removal of hyperostotic bone
(relapse rate 19%). Grade III is GTR of the tumor alone (relapse rate 29%). Grade IV is
partial resection (relapse rate 44%). Grade V refers to biopsy alone without resection.
Adjuvant radiotherapy is not recommended for patients with low-grade meningioma who
undergo Simpson grade I to II, and sometimes grade III, resection. The patient in this case
has undergone grade II resection for a low-grade meningioma and therefore does not
require further radiotherapy. For patients with resected anaplastic and malignant
meningioma, adjuvant radiotherapy is recommended. Radiosurgery can be used in
patients with unresectable disease, but there is no evidence to support the use of
chemotherapy (Tamoxifen, mifepristone, and hydroxyurea) in patients with meningioma.

Question 25.17 The answer is A.


High-dose methotrexate is the treatment of choice for immunocompetent patients with
primary CNS lymphoma. The 5-year survival with high-dose methotrexate treatment is
approximately 20%. Whereas whole brain radiation therapy is effective, it is associated
with serious neurotoxicity and 5-year survival is only 5%. Whole brain radiation is used
upfront in patients who are not candidates for methotrexate, due to poor renal function
and poor PS. A phase III, noninferiority trial comparing high-dose methotrexate with or
without whole brain radiation therapy showed no significant overall survival benefit to
combining whole brain radiation with high-dose methotrexate. R-CHOP is not an
appropriate choice for frontline therapy, though it is used to treat many types of non-
Hodgkin lymphoma.

Question 25.18 The answer is D.


Long-term follow-up data from the phase III EORTC-NCIC trial showed improved survival
in all patients treated with combined temozolomide and radiation, followed by
temozolomide compared to patients treated with radiation alone.

Question 25.19 The answer is A.


Some degree of sexual dysfunction occurs in approximately 90% of all adult patients
presenting with craniopharyngioma. Visual dysfunction occurs in 40% to 70% of these
patients. Hypothyroidism occurs in 40% of patients and diabetes insipidus occurs in 10%
to 20% of patients. However, the incidence of diabetes insipidus after surgical resection
increases substantially. In one study, it increased from 16.1% in the preoperative setting
to 59.4% after surgery.

Question 25.20 The answer is C.


Several retrospective case series have shown that incomplete tumor resection, high tumor
grade, supratentorial location, and poor performance status to be associated with poor
outcomes in patients with intracranial ependymoma. However, improvement in overall
survival with adjuvant radiation is limited to patients with incomplete tumor resection.

Question 25.21 The answer is A.


In a retrospective analysis of tumor tissue samples from patients with oligodendrogliomas
who received treatment on the EORTC study 26951, both IDH1 and IDH2 mutations were
found to be a prognostic marker for overall survival.

Question 25.22 The answer is D.


Gene expression profiling has classified GBM into four clusters—proneural, neural,
classical, and mesenchymal. While there is some overlap among the groups, the
mesenchymal group is most often associated with loss of NF1 expression. In contrast, the
proneural group is often associated with IDH1 mutation, PDGFRA amplification, and G-
CIMP. While the neural group has a high frequency of EGFR amplification without
polysomy of chromosome 7, classical GBMs are generally found to have amplification of
the entire chromosome 7 as well as loss of chromosome 10, and homozygous deletion of
CDKN2A.

Question 25.23 The answer is C.


A large gene sequencing study by The Cancer Genome Atlas (TCGA) found that the three
most commonly dysregulated core pathways in primary GBMs were p53 (85.3%), Rb
(78.9%), and PI3K (67.3%). Increased expression of the WNT (10%) and SHH (30%)
pathways have been observed in pediatric medulloblastoma, while activation of the MAPK
pathway frequently drives tumorigenesis of pilocytic astrocytomas. In pediatric
meningiomas, loss of NF2/22q occurs in 50% of sporadic meningiomas (up to 75% of
WHO grade II), along with mutations in the TRAF7 (25%) and SHH (3%) pathways.

Question 25.24 The answer is B.


ATRX is often comutated with p53 and IDH1 in grade II astrocytomas whereas grade II
oligodendrogliomas frequently possess somatic mutations in CIC (chr 19) and FUBP1 (chr
1) that often co-occur with codeletion of chromosomes 1p/19q. Mutations in the TERT
promoter is often found in oligodendrogliomas as well and are mutually exclusive with
ATRX mutations.

Corresponding chapters in Cancer: Principles & Practice of Oncology, Tenth Edition: 96 (Molecular Biology of Central Nervous
System Tumors) and 97 (Neoplasms of the Central Nervous System).
26 Sarcomas
Brian A. Van Tine and Angela C. Hirbe

QUESTIONS
Each of the numbered items below is followed by lettered answers. Select the ONE lettered answer
that is BEST in each case unless instructed otherwise.

Question 26.1 Which of the following inherited syndromes represent a predisposing factor
for the development of soft tissue sarcoma?
A. Retinoblastoma
B. Li–Fraumeni syndrome
C. Neurofibromatosis type I
D. All of the above

Question 26.2 Which of the following are TRUE about radiation-induced sarcomas?
A. Cancer history usually includes breast cancer, lymphoma, and cervical cancer.
B. They usually occur 10 to 30 years after radiation exposure.
C. Osteogenic sarcoma, undifferentiated pleomorphic sarcoma (UPS), angiosarcoma, and
lymphangiosarcoma are the usual histologic subtypes.
D. All of the Above.

Question 26.3 Which clonal cytogenetic abnormality is associated with the CORRECT
sarcoma subtype?
A. Ewing sarcoma and t(11;22) (q24;q12)
B. Synovial sarcoma and t(12;16) (q13;p11)
C. Myxoid liposarcoma and t(X;18) (p11;q11)
D. Alveolar rhabdomyosarcoma and t(17;22) (q22;q13)

Question 26.4. A 20-year-old man presents with right knee pain. X-ray reveals a “sun-burst”
appearance in the distal femur. Biopsy reveals high-grade osteosarcoma. No distant
metastases are identified. Which of the following is the most appropriate treatment?
A. Limb-sparing resection
B. Limb-sparing resection and adjuvant chemotherapy
C. Definitive radiation
D. Neoadjuvant chemotherapy, limb-sparing resection, and adjuvant chemotherapy

Question 26.5 In addition to site, which of the following variables is used to estimate the
risk of sarcoma-specific death for a given patient?
A. Tumor grade and histology
B. Tumor size and depth
C. Age
D. All of the above

Question 26.6 Which of the following is TRUE regarding the staging of soft tissue sarcomas?
A. Staging involves a four grade system.
B. Stage IV includes N1 disease.
C. High-grade tumors can be classified as stage I depending on the size of the tumor.
D. Histologic grade, size, depth, and presence or absence of nodal and distant metastases
are variables used to determine tumor stage.

Question 26.7 A 52-year-old man underwent resection of a 3-cm mass from the lateral left
thigh. Pathology revealed a high-grade leiomyosarcoma, and the lateral surgical margin was
positive. The most appropriate next step in the treatment of this patient’s cancer would be:
A. Radiation.
B. Adjuvant chemotherapy.
C. Reresection.
D. Observation.

Question 26.8 A 68-year-old woman presented with a purplish nodular lesion in the occipital
scalp. Resection revealed an angiosarcoma measuring 3 cm. Surgical margins were negative.
What is the most appropriate next step in the treatment of this patient’s cancer?
A. Radiologic imaging to look for nodal metastases and referral for adjuvant radiation
B. Monitoring
C. Adjuvant chemotherapy with an anthracycline
D. Adjuvant chemotherapy with paclitaxel

Question 26.9 A 65-year-old woman presented with abdominal pain and iron-deficiency
anemia. Workup revealed a gastric mass and multiple large intra-abdominal masses and liver
hypodensities. Biopsy of the gastric mass revealed a spindle cell neoplasm thought to be a
leiomyosarcoma. After three cycles of doxorubicin and ifosfamide, imaging showed disease
progression. The appropriate next step in the management of this patient’s cancer would be:
A. Docetaxel and gemcitabine.
B. Dacarbazine.
C. Request the pathologist to perform a CD117 (c-Kit) stain.
D. Palliative radiation.

Question 26.10 A 55-year-old man presented with a 10-cm mass in the medial left thigh.
Biopsy revealed a high-grade liposarcoma. Imaging revealed no evidence of distant
metastases. The most appropriate treatment of this patient’s cancer would be:
A. Definitive radiation.
B. Definitive radiation and concurrent doxorubicin.
C. Limb-sparing resection followed by adjuvant radiation.
D. Preoperative chemotherapy followed by resection.

Question 26.11 A 52-year-old man with metastatic unresectable gastrointestinal stromal


tumor (GIST) was treated with imatinib (400 mg/d). Imaging showed initial disease
response; however, the disease progressed after 28 months on therapy. The most appropriate
treatment would be:
A. Erlotinib.
B. Sunitinib.
C. Doxorubicin.
D. Imatinib 800 mg/d.

Question 26.12 Which of the following factors are independent predictors of poorer disease-
specific survival in patients with nonmetastatic soft tissue sarcoma?
A. Large tumor size (>10 cm)
B. High-grade histology
C. Older age (>60 years)
D. All of the above

Question 26.13 A 48-year-old woman underwent complete resection of a 9-cm high-grade


leiomyosarcoma arising in the lower extremity. Postoperative adjuvant radiation was
administered. Two years later, a chest computed tomography (CT) scan revealed a new single
3-cm, round, noncalcified pulmonary nodule. What is the most appropriate next treatment?
A. Complete resection of the lung nodule
B. Radiation
C. Ifosfamide and doxorubicin
D. Docetaxel and gemcitabine

Question 26.14 Which of the following is TRUE about patients with metastatic or locally
recurrent soft tissue sarcoma?
A. Median survival is 12 months, although 20% to 25% of patients are alive 2 years after
diagnosis.
B. Complete resection of oligometastases to the lung never results in long-term survival.
C. Radiation is the preferred treatment of a locally recurrent sarcoma.
D. Combination chemotherapy improves overall survival compared with single-agent
chemotherapy.

Question 26.15 Which of the following is TRUE about chemotherapy treatment of


metastatic soft tissue sarcoma?
A. Escalating doses of doxorubicin or ifosfamide do not improve tumor response rates over
standard doses of these agents.
B. Leiomyosarcoma is uniquely sensitive to ifosfamide, whereas synovial sarcoma is not.
C. Paclitaxel shows broad-spectrum activity.
D. Dacarbazine has modest activity.

Question 26.16 A 30-year-old man presents with a permeative bone tumor in the distal
femur. Open biopsy reveals a MFH. Radiologic imaging does not find distant metastases.
What is the most appropriate treatment?
A. Limb-sparing resection with wide margins
B. Definitive radiation
C. Chemotherapy
D. Preoperative chemotherapy, limb-sparing resection, and adjuvant chemotherapy

Question 26.17 Which of the following is TRUE regarding osteosarcoma?


A. Approximately 20% of patients with localized high-grade disease treated with resection
and chemotherapy remain disease free 5 years later.
B. Approximately 60% to 80% of patients with localized high-grade disease treated with
resection and chemotherapy remain disease free 5 years later.
C. Parosteal (low-grade cortical) osteosarcoma is best treated with resection and
chemotherapy.
D. Periosteal osteosarcoma is best treated with resection.

Question 26.18 A 45-year-old man presents with a left-sided pelvic pain. CT reveals a 5 cm
mass with appearance of chondroid matrix, arising from the left side of the pelvic girdle.
Bone biopsy reveals chondrosarcoma. Which of the following is TRUE regarding
chondrosarcomas?
A. Most are low-grade tumors.
B. Most are treated with resection and radiation therapy.
C. Children have a better prognosis than adults.
D. Adjuvant chemotherapy has no role in the management of nonmetastatic disease.

Question 26.19 A 22-year-old woman presents with a giant cell tumor (GCT) of the distal
femur. Appropriate treatment would be:
A. Curettage and debridement.
B. Amputation.
C. Radiation.
D. Preoperative chemotherapy, resection, and adjuvant chemotherapy.

Question 26.20 A 16-year-old female patient presents with a painful rapidly growing
scapular mass. Core needle biopsy reveals a Ewing sarcoma. Staging evaluation shows no
evidence of metastatic disease. The most appropriate therapy is:
A. Resection.
B. Preoperative chemotherapy (with vincristine, doxorubicin, and cyclophosphamide
alternating with ifosfamide and etoposide, IE), resection, and adjuvant chemotherapy.
C. Resection and radiation.
D. Preoperative chemotherapy (with vincristine, doxorubicin, and cyclophosphamide),
resection, and adjuvant chemotherapy.

Question 26.21 A 25-year-old woman presents with a painless right thigh mass, which has
been slowly growing over a span of 8 years. Core needle biopsy reveals alveolar soft part
sarcoma. What cytogenetic abnormality would you expect to see in this tumor?
A. t(11;22) (q24;q12)
B. t(12;16) (q13;p11)
C. der (17) t(X;17) (p11;q25)
D. t(X;18) (p11;q11)

Question 26.22 Which of the following targeted agents is used to treat alveolar soft part
tissue sarcoma?
A. Imatinib
B. Sunitinib
C. Dasatinib
D. Nilotinib

Question 26.23 Which of the following sites of soft tissue sarcomas carries the best
prognosis?
A. Head and neck
B. Extremity
C. Visceral
D. Retroperitoneal

Question 26.24 A 60-year-old woman with advanced soft tissue sarcoma is on third-line
therapy with pazopanib and develops jaundice. Which of the following should be done next?
A. Abdominal imaging
B. Laboratory testing
C. Dose reduction
D. Immediately discontinue pazopanib

Question 26.25 Which of the following criteria are predictive of the behavior of GIST
tumors and are useful for stratification of risk of recurrence and metastasis? (Select two
correct responses)
A. Mitotic rate
B. Nuclear atypia
C. Tumor site
D. Mucosal invasion

Question 26.26 What is the mechanism of action of mesna?


A. Binds and detoxifies ifosfamide metabolites in the kidney and bladder
B. Coats the bladder
C. Inhibition of monoamine oxidase (reducing further chloroacetaldehyde formation)
D. Rescuing impaired hepatic metabolic pathways by acting as an electron acceptor
ANSWERS

Question 26.1 The answer is D.


Patients with Li–Fraumeni syndrome have an increased risk for developing soft tissue
sarcomas. Neurofibromatosis type I increases the risk for the development of malignant
peripheral nerve sheath tumors. Patients with retinoblastoma also have an increased risk
for developing soft tissue and bone sarcomas.

Question 26.2 The answer is D.


Radiation-induced sarcomas usually occur 10 to 30 years after exposure to radiation. The
most common cancers for which radiation was administered include breast and cervical
cancer, and lymphomas. Most radiation-induced sarcomas are osteogenic sarcoma or UPS,
and the prognosis is poor.

Question 26.3 The answer is A.


Ewing sarcoma is associated with five known cytogenetic abnormalities, including
t(11;22) (q24;q12). Synovial sarcoma is associated with t(X;18) (p11;q11), whereas
myxoid liposarcoma is associated with t(12;16) (q13;p11). Alveolar rhabdomyosarcoma is
associated with t(2;13) (q35;q14).

Question 26.4 The answer is D.


The current standard of care for the treatment of osteosarcomas is neoadjuvant
chemotherapy, followed by limb-sparing resection and adjuvant chemotherapy. The
percentage of tumor cell necrosis on pathology from the resection gives an idea of
prognosis and helps determine the adjuvant chemotherapy regimen. Surgery alone, or
definitive radiation, offers lower chances of success.

Question 26.5 The answer is D.


The Memorial Sloan Kettering Cancer Center (MSKCC) nomogram for sarcoma-specific
mortality incorporates the following variables: age, tumor size, depth, histology, grade of
tumor, and site. Zero points are assigned for the low-risk features (age 16, size ≤5 cm,
superficial depth, fibrosarcoma histology, and low-grade and upper-extremity site). A sum
of the points assigned for each variable helps predict the risk of sarcoma-specific death.

Question 26.6 The answer is B.


N1 disease is considered Stage III disease. Stage I disease includes only low-grade tumors.
High-grade tumors are never stage I. Staging of soft tissue sarcoma requires knowledge of
histologic grade, size, depth, and presence of metastatic disease and is based on a three-
grade system.

Question 26.7 The answer is C.


Positive surgical margin predicts a higher risk of local recurrence. Reresection in an
attempt to achieve negative surgical margins would be the next most appropriate step in
the management of this patient. If negative surgical margins are achieved after
reresection, adjuvant radiation would not be required. Adjuvant chemotherapy would
have no clear benefit in this case.

Question 26.8 The answer is A.


Soft tissue sarcoma of the head and neck is associated with a poor prognosis.
Angiosarcoma has a higher risk of nodal metastases and local recurrence after resection
alone. Adjuvant radiation reduces the risk of local and regional disease recurrence.
Although angiosarcomas tend to be sensitive to anthracyclines and taxanes, there would
be no known benefit of adjuvant chemotherapy in this case.

Question 26.9 The answer is C.


Intra-abdominal leiomyosarcomas and GISTs have similar characteristics by light
microscopy; however, GISTs stain positive for the CD117 (c-Kit) protein, whereas
leiomyosarcomas usually do not. Leiomyosarcomas frequently respond to chemotherapy,
whereas GISTs do not. However, GISTs are uniquely responsive to c-Kit inhibitors. In this
case, a c-Kit immunostain should be requested to determine whether the sarcoma is a
GIST and not a leiomyosarcoma. Because this patient has metastatic disease, systemic
therapy would be favored over palliative radiation.

Question 26.10 The answer is C.


Resection of the primary tumor with intent to achieve negative surgical margins followed
by adjuvant radiation would be the most appropriate treatment of a large high-grade
extremity liposarcoma. Definitive radiation with or without concurrent chemotherapy
would be less effective treatment. Preoperative chemotherapy may be used when the
tumor is marginally resectable, but its role in this situation is controversial.

Question 26.11 The answer is D.


Studies have demonstrated that approximately one-third of patients with GIST resistant to
imatinib 400 mg/d will benefit with increasing the dose of imatinib of 800 mg/d. Should
that step fail, sunitinib would be the next most appropriate therapy. GIST is generally
resistant to chemotherapy drugs, such as doxorubicin. Epidermal growth factor receptor
inhibitors, such as erlotinib, would not be expected to benefit patients with GIST.

Question 26.12 The answer is D.


Independent predictors of prognosis of patients with nonmetastatic soft tissue sarcoma
include age, histologic subtype, histologic grade, tumor site, and tumor size. Factors
predictive of better disease-specific survival include younger age, selected histologic
subtypes (fibrosarcoma), low histologic grade, smaller (<5 cm) tumor size, and extremity
location. Nomograms are available that accurately predict the prognosis of patients with
nonmetastatic soft tissue sarcoma treated with surgery with or without adjuvant
radiation.
Question 26.13 The answer is A.
In this patient, the most likely cause of the new lung nodule is metastases of the
leiomyosarcoma. Retrospective series have observed that 20% to 30% of such patients
who undergo complete resection of the lung metastases were alive 5 years later. Factors
that predict for a lower success rate with resection in such patients include more than four
lung nodules, bilateral lung nodules, and a disease-free interval of less than 12 months.
Chemotherapy and radiation provide only palliative intent in this case.

Question 26.14 The answer is A.


For locally recurrent sarcoma without distant metastases, reresection is the preferred
treatment. For patients with metastatic sarcoma, median survival is 1 year; however, a
subset of these patients live for more than 2 years. It is important to consider complete
resection of oligometastases to the lung because approximately 25% of those patients will
be alive 5 years later. Nearly all randomized trials have shown no overall survival benefit
with combination (vs. single agent) chemotherapy, although tumor response rates were
consistently higher. A single, recent randomized trial demonstrated a modest survival
benefit with the addition of docetaxel to gemcitabine in patients with metastatic sarcoma.

Question 26.15 The answer is D.


Several studies have shown higher tumor response rates (but not improved survival) with
larger doses of doxorubicin and ifosfamide; however, this does result in greater toxicity.
Ifosfamide is an active agent in synovial sarcoma but has little activity in
leiomyosarcoma. Paclitaxel appears to only be effective in the treatment of angiosarcoma
and Kaposi sarcoma. Perhaps the first drug identified to be active in adult soft tissue
sarcoma, dacarbazine resulted in tumor response rates of 10% to 30%.

Question 26.16 The answer is D.


In contrast with UPS of soft tissue, which is primarily treated with resection, MFH of bone
is treated like high-grade osteosarcoma with preoperative chemotherapy, limb-sparing
resection, and adjuvant chemotherapy. Outcomes of patients with MFH of bone and with
osteosarcoma are similar when treated this way. Limb-sparing resection alone or
definitive radiation would offer lower chances of success. Chemotherapy alone would
provide palliation, but the chance of cure is unclear.

Question 26.17 The answer is B.


Parosteal osteosarcoma is usually a low-grade tumor that involves the bone cortex and is
associated with a better prognosis than classic osteosarcoma. Resection is the preferred
treatment, and there is no clear role for chemotherapy. In contrast, periosteal
osteosarcoma is an aggressive, high-grade cortical sarcoma that is best treated like classic
osteosarcoma with resection and chemotherapy. For classic high-grade osteosarcoma,
resection alone is curative in 20%, but the addition of chemotherapy to resection
improves cure rates to 60% to 80%.
Question 26.18 The answer is A.
The preferred therapy for chondrosarcoma is resection. Although chondrosarcomas as a
group are known to be resistant to chemotherapy, mesenchymal and dedifferentiated
chondrosarcomas have a high metastatic potential and appears to benefit from adjuvant
chemotherapy. Most chondrosarcomas are low grade and usually occur in patients aged
more than 40 years. Children have a worse prognosis than adults.

Question 26.19 The answer is A.


GCT of bone is a tumor with a high risk for local recurrence but a low risk for distal
metastases. Appropriate treatment is resection, preferably by curettage and debridement.
Amputation is reserved for massive local recurrences. Radiation is reserved for lesions of
the spine that can lead to cord compression. There is no benefit of preoperative or
adjuvant chemotherapy for GCT of bone. There is a role for denosumab for patients with
unresectable GCT.

Question 26.20 The answer is B.


A randomized trial of patients with Ewing sarcoma treated with either vincristine,
doxorubicin, and cyclophosphamide (VAC) alone or alternating with IE given as
preoperative and adjuvant chemotherapy clearly showed an overall survival and
progression-free survival advantage with the alternating regimen of VAC and IE versus
VAC alone in patients with nonmetastatic (but not in patients with metastatic) disease.
Resection alone or with radiation would be significantly less effective than when used in
combination with chemotherapy.

Question 26.21 The answer is C.


Alveolar soft part sarcoma is associated with der (17) t(X;17) (p11;q25). Ewing sarcoma is
associated with five known cytogenetic abnormalities, including t(11;22) (q24;q12).
Synovial sarcoma is associated with t(X;18) (p11;q11), whereas myxoid liposarcoma is
associated with t(12;16) (q13;p11).

Question 26.22 The answer is B.


Sunitinib is an oral small molecule, multitargeted receptor tyrosine kinase inhibitor that is
an active agent in the treatment of alveolar soft part sarcoma. Imatinib, dasatinib, and
nilotinib are tyrosine kinase inhibitors that have activity against GIST. Sunitinib also is
used to treat GIST.

Question 26.23 The answer is B.


Extremity soft tissue sarcomas have the best prognosis, followed by visceral and
retroperitoneal. Head and neck soft tissue sarcomas carry the worst prognosis.

Question 26.24 The answer is D.


Pazobanib has a black box warning for severe and fatal hepatotoxicity. This can occur at
any time during treatment. CMPs should be followed on regular bases (LFTS should be
checked on weeks 3, 5, 7, and 9, then at months 3 and 4 and as clinically indicated.
Patients should be counseled to immediately contact their physician and stop the
medication should they notice jaundice.

Question 26.25 The answers are A and D.


Tumor size, mitotic rate, and tumor site (jejunal and ileal sites having more malignant
behavior) are criteria that can be used to predict the behavior of GIST. Other
histopathologic criteria including cellularity, nuclear atypia, mucosal invasion, and
ulceration have not been shown to correlate well with prognosis.

Question 26.26 The answer is A.


Mesna functions to reduce the risk of hemorrhagic cystitis by binds and detoxifies
ifosfamide metabolites in the kidney and bladder. Two proposed mechanisms of action for
methylene blue treatment of ifosfamide induced neurotoxicity include inhibition of
monoamine oxidase (reducing further chloroacetaldehyde formation) and rescuing
impaired hepatic metabolic pathways by acting as an electron acceptor.

Corresponding chapters in Cancer: Principles & Practice of Oncology, Tenth Edition: 89 (Molecular Biology of Sarcomas), 90
(Soft Tissue Sarcoma), and 91 (Sarcomas of Bone).
27 Cancer of the Skin and Melanoma
Gerald P. Linette

QUESTIONS
Each of the numbered items below is followed by lettered answers. Select the ONE lettered answer
that is BEST in each case unless instructed otherwise.

Question 27.1 What is the most prevalent significantly mutated gene in cutaneous
melanoma?
A. BRAF V600K
B. BRAF V600R
C. BRAF V600E
D. NRAS Q61R
E. NRAS Q61K

Question 27.2 Which of the following is included in referral criteria for hereditary
melanoma genetic counseling?
A. Parent with multiple skin cancers.
B. Pancreatic cancer and melanoma on the same side of the family.
C. Three or more siblings with actinic keratosis.
D. Two relatives on the same side of the family with melanoma and prostate cancer.

Question 27.3 Which of the following genes act as founding mutations implicated in the
early transformation of cutaneous melanoma (Select two correct responses)?
A. NRAS
B. BRAF
C. TP53
D. MEK1

Question 27.4 Which of the following statements best explains the increased incidence of
melanoma over the past 25 years?
A. Decreasing use of sunscreen among middle age persons.
B. High penetrance rate of CDKN2A mutations.
C. Increasing exposure to ultraviolet light.
D. Increasing use of birth control pills.

Question 27.5 What is the most common site for primary skin melanoma in females?
A. Back
B. Face
C. Lower extremities
D. Scalp

Question 27.6 Which one of the following features of primary melanoma is the most
important prognostic factor?
A. Clark level
B. Breslow thickness
C. Mitotic rate
D. Anatomical location

Question 27.7 Basal cell nevus syndrome has been associated with mutations in which one
of the following genes?
A. Patched gene 1
B. Sonic hedgehog
C. MEK1
D. NF1

Question 27.8 A 48-year-old woman was recently diagnosed with a nonulcerated 1.6-mm
depth primary superficial-spreading melanoma excised from her back. The melanoma is
nonmitogenic and the margins are negative. She is referred to you for further evaluation and
no adenopathy is noted on examination. Which of the following is the most appropriate next
step in clinical management?
A. PET-CT to complete the staging evaluation.
B. Referral back to the Dermatologist for annual skin checks.
C. Wide excision and sentinel lymph node mapping.
D. Wide excision followed by interferon for 1 year.

Question 27.9 A 55-year-old woman in excellent health presents to the Emergency


Department with a 7-day history of headache and ataxia. She denies any recent illness and
states that acetaminophen is no longer relieving her symptoms. You evaluate the patient and
order a head CT examination, which reveals a 3-cm solitary lesion with surrounding
vasogenic edema, present in the posterior fossa. The patient denies any past history of
malignancy; however, your physical examination detects a cutaneous nonpigmented lesion on
her ankle. You order dexamethasone 10 mg intravenously and the patient’s symptoms
improve after 90 minutes. She wants to go home. You suspect a solitary brain metastasis.
What is the next step in management?
A. Discharge the patient to home and make a referral to the patient’s primary care
physician.
B. Discharge the patient to home and make a referral to the radiation and dermatology
clinics.
C. Admit the patient and request a brain MRI followed by a neurosurgical consultation.
D. Admit the patient and request a palliative care consultation.

Question 27.10 A 29-year-old man is recently diagnosed with primary cutaneous melanoma.
The pathology report confirms a T3b N2b M0 primary nodular melanoma excised from the
left shoulder. The primary lesion was ulcerated and mitogenic (4/mm2). The patient is in
excellent health and has no significant past medical history. The patient is referred to you for
adjuvant treatment. What is the final stage of this patient’s melanoma?
A. Stage IIIA
B. Stage IIIB
C. Stage IIIC
D. Stage IV, M1A

Question 27.11 A 40-year-old woman with a history of ulcerative colitis, mitral valve
prolapse, eczema, and stage 1 cutaneous melanoma presents with right inguinal adenopathy.
Physical examination confirms a 4-cm node and core biopsy confirms malignant melanoma.
BRAF mutational analysis is requested and BRAF V600E mutation is detected. Brain MRI is
unremarkable; however, CT examination reveals numerous (>10) bilateral pulmonary
metastases with the largest lesion measuring 1.2 cm in the left upper lobe. No other sites of
distant metastases are evident and you conclude that this patient has stage IV, M1b disease.
The patient does not want to participate in a clinical trial and wants to begin treatment as
soon as possible. What is the best initial treatment for this individual?
A. Ipilimumab
B. BRAF inhibitor
C. MEK inhibitor
D. BRAF and MEK inhibitor combination

Question 27.12 A 73-year-old man with a past history of stage IIC nodular melanoma
excised from his back 3 years ago presents to your office with lower back pain and fatigue
for the past 2 weeks. Your physical examination is unrevealing. However, several
laboratories are elevated, including serum alkaline phosphatase, and serum LDH (two times
upper limit of normal). PET-CT examination the following day documents metastatic disease
involving lungs, liver, peritoneum, and multiple osseous areas, including the lumbar spine.
What is the most appropriate next step in management?
A. Request a brain MRI and a biopsy of a lung lesion.
B. Request a palliative care consultation.
C. Request a second opinion review of the primary cutaneous lesion excised 3 years ago.
D. Begin systemic therapy as soon as possible.

Question 27.13 A 36-year-old woman with a past history of cutaneous melanoma (stage IIB)
has been followed by her dermatologist. She has active rheumatoid arthritis and is receiving
methotrexate under the care of a local rheumatologist. She recently presented to the
emergency department with complaints of headaches and was found to have a 3.5-cm solitary
right parietal metastasis. The metastasis was surgically resected and confirmed as metastatic
melanoma, BRAF V600E mutated. A postoperative body PET-CT examination was negative
for additional disease and the hospital physicians told her that she has no additional evidence
of disease. She is referred to you for treatment recommendations. What is the next best step
in the management of her care?
A. Request radiation oncology consultation for stereotactic radiosurgery.
B. Request radiation oncology consultation for whole brain radiation.
C. Administer adjuvant chemotherapy.
D. Administer adjuvant interferon.

Question 27.14 A 27-year-old woman has metastatic melanoma, BRAF wild type (no
mutation), stage IV, M1B. You recommended initial systemic treatment with ipilimumab and
she completed four doses with no side effect or toxicity. Two weeks after the four dose a
restaging CT examination is performed which reveals the appearance of a new 6-mm left
upper lobe lesion as well as an increase of a baseline right lower lung lesion from 17 mm to
23 mm. The other lung metastases remain unchanged. You see the patient in your office the
following day. She feels fine and has no complaints. Her laboratories are all within normal
limits. However, the patient is anxious after you discuss the CT imaging results with her. She
and her husband have many questions regarding other treatment options. What is your
recommendation to this patient?
A. Despite the fact that her tumor is BRAF wild type, there is some evidence that
treatment with a MEK inhibitor would be beneficial.
B. You recommend a repeat CT scan in 4 weeks based upon reports that 10% to 15% of
patients treated with ipilimumab can have evidence of progression prior to documented
response.
C. You recommend retreatment with four additional doses of ipilimumab.
D. You recommend immediate surgical consultation.

Question 27.15 A 66-year-old man with metastatic melanoma is receiving treatment with
combination ipilimumab and nivolumab on a clinical trial. Fourteen days after the second
dose, he calls your office to tell you that he has been having watery loose stools up to ten
times a day for the past week. However, you are out of town and your nurse practitioner
called in sick for the day. What is your recommendation to this patient?
A. Call your primary physician immediately.
B. Keep your scheduled appointment next week for the third dose but call sooner if the
diarrhea gets worse.
C. Take loperamide as directed and keep hydrated.
D. Go to the local emergency department immediately for evaluation.

Question 27.16 A 70-year-old man from out of town presents to the emergency department
with complaints of profound fatigue and headaches for the past 3 days. His vital signs are
normal except for a blood pressure of 90/50. He denies fevers and diarrhea. His medical
history is significant for gout, type 2 diabetes, and metastatic melanoma. He is receiving
treatment with ipilimumab along with an experimental immunotherapy drug but he can’t
remember the name of the investigational drug. He has received four cycles of treatment and
is due for a restaging evaluation when he returns home in a week. Unfortunately, it’s a
holiday weekend and no medical records are available. You request a CBC, CMP, and EKG
along with a head CT and CT examination of the chest, abdomen, and pelvis. The chest CT
confirms the presence of innumerable (>20) pulmonary metastases but none greater than 2
cm. The head CT is unremarkable. You are perplexed and unsure of the cause of his
symptoms. You decide to order additional blood work. What is the most appropriate test to
order?
A. Blood cultures
B. Serum LDH
C. Thyroid function tests
D. C-reactive protein
E. Discharge to home with instructions to call his oncologist

Question 27.17 A 75-year-old woman with a past history of recurrent basal cell carcinoma
of the skin was recently found to have bilateral lung lesions. CT examination confirms 10
pulmonary lesions as well as several liver lesions. The 2.5-cm right upper lobe lesion is
biopsied and confirmed as basal cell carcinoma. Her primary care physician tells her that she
will need chemotherapy. She arrives in your office along with several additional family
members, including her daughter who is a physician. They are distraught over the recent
diagnosis and have solicited the opinions of various physicians. They understand her
condition is incurable but desire the best treatment. What is your treatment
recommendation?
A. Platinum-based chemotherapy for six cycles
B. Vismodegib
C. Trametinib
D. Immediate radiation oncology consultation
E. Immediate surgical consultation
ANSWERS

Question 27.1 Answer is C.


BRAF is the most common mutation in patients with melanoma, occurring in 40% to 50%
of cases. Among patients with BRAF mutations, the vast majority has the V600E subtype.
NRAS mutations are present in 15% to 20% of cases.

Question 27.2 Answer is B.


Approximately 5% of melanomas occur in high-risk families. The melanoma suppressor
gene that has the highest frequency and penetrance is a germline mutation in CDKN2A,
which is present in approximately 25% of melanoma-prone families and also associated
with pancreatic cancer.

Question 27.3 Answers are A and B.


Both NRAS and BRAF are founding mutations frequently present in dysplastic nevi and
early thin-cutaneous melanoma.

Question 27.4 Answer is C.


Ultraviolet light is considered the primary risk factor for cutaneous melanoma. Many
social factors influence UV light exposure including increased time spent outside for
recreational activities, less clothing, and environmental changes.

Question 27.5 Answer is C.


The most common anatomical sites for melanoma are lower extremities in women and
trunk and head/neck for men.

Question 27.6 Answer is B.


The Breslow thickness remains the most important pathologic variable. The presence or
absence of ulceration is also an important variable. Clark level is no longer recommended
as a staging criterion.

Question 27.7 Answer is A.


Approximately 75% of BCC harbor somatic mutations in the PTCH1 gene.

Question 27.8 Answer is C.


The current guidelines are clear and recommend wide excision with 1 to 2 cm margins
with sentinel lymphatic mapping for cutaneous primary melanoma >1 mm depth.

Question 27.9 Answer is C.


Patients with a suspected solitary intracranial metastasis (from an unknown primary,
including suspected melanoma) should have a contrast-enhanced brain MRI along with a
formal neurosurgical consultation to determine if the lesion is resectable. This will aid in a
tissue diagnosis, and provide symptomatic relief.

Question 27.10 Answer is C.


This patient has stage IIIC according to the current seventh edition AJCC staging criteria.

Question 27.11 Answer is D.


Ulcerative colitis is a relative contraindication to the use of ipilimumab and the risks of
clinically significant/severe autoimmune colitis must be carefully weighed against the
potential benefits in this situation. Most experts would select BRAF and MEK inhibitor
combination based on the superiority of the combination (improved PFS and OS) over
either single agent inhibitor.

Question 27.12 Answer is A.


The patient needs a tissue biopsy for a suspected malignancy, including melanoma. The
tumor biopsy will also allow for BRAF mutational analysis. A brain MRI examination will
complete the staging evaluation regardless of the final pathologic diagnosis.

Question 27.13 Answer is A.


There is no role for adjuvant cytotoxic chemotherapy or interferon alpha-2b in this
setting. The role of adjuvant ipilimumab (and BRAF inhibitors) is currently under
evaluation in clinical trials. The best answer is adjuvant stereotactic radiosurgery to
reduce the risk of a recurrence in/near the resection cavity.

Question 27.14 Answer is B.


Lessons from the initial ipilimumab clinical trials in metastatic melanoma suggest that
about 10% of treated patients develop an unconventional response (such as a response
after an increase in total tumor burden), which led to the development of the immune-
related response criteria that have been incorporated into immunotherapy protocols in the
past several years. Repeat imaging in such situations in 4 to 6 weeks is recommended.

Question 27.15 Answer is D.


The patient has grade III autoimmune colitis due to the checkpoint blockade until proven
otherwise. Clinically this is considered an urgent situation that must be recognized and
treated in a timely manner. Delaying treatment poses the threat of worsening colitis and a
risk of bowel perforation and death.

Question 27.16 Answer is C.


This patient has ipilimumab-related endocrinopathy. Thyroid function tests, initially with
TSH, should be monitored at regular intervals and prior to each dose of ipilimumab. If
endocrinopathy is suspected, additional laboratories such as serum cortisol, ACTH, free
T4, total T4, LH, FSH, prolactin should be drawn and brain MRI with pituitary cuts is
suggested.
Question 27.17 Answer is B.
Vismodegib is an approved treatment option for patients with locally advanced or
metastatic BCC.

Corresponding chapters in Cancer: Principles & Practice of Oncology, Tenth Edition: 92 (Cancer of the Skin), 93 (Molecular
Biology of Cutaneous Melanomas), 94 (Cutaneous Melanomas), and 95 (Genetic Testing in Skin Cancer).
28 Lymphomas
Nina Wagner-Johnston and Dilan Patel

QUESTIONS
Each of the numbered items below is followed by lettered answers. Select the ONE lettered answer
that is BEST in each case unless instructed otherwise.

Question 28.1 An 8-year-old child is brought to clinic by his mother due to a jaw lump that
has been increasing in size over the past month. The mother states that the lump is
preventing her child from eating. Physical examination reveals a child in moderate distress
with a left jaw mass that displaces teeth and is impinging on the trachea. Biopsy reveals
sheets of atypical lymphocytes with areas of necrosis and hemorrhage. Ki-67 is 99%. Flow
cytometry shows B-cell markers as well as CD10 and BCL-6. CD5, BCL-2, and TdT are absent.
Which of the following is the most common translocation in this malignancy?
A. t(1;14)
B. t(2;8)
C. t(8;14)
D. t(8;22)

Question 28.2 Which of the following characterize/s a difference between endemic and
sporadic Burkitt lymphoma in children?
A. Endemic disease occurs more commonly in Africa and is associated with EBV infection.
B. Common sites of endemic disease are the mediastinal and hilar lymph nodes, which
often leads to rapid respiratory compromise.
C. Dysregulation of both c-myc and BCL-2 are required for tumor cell growth.
D. Sporadic Burkitt lymphoma occurs in nonmalaria endemic areas, with common areas of
development including the abdomen.

Question 28.3 Which one of the following syndromes is associated with a higher risk of
adult lymphoma as opposed to childhood lymphoma?
A. Ataxia-telangiectasia
B. HIV/AIDS
C. Wiskott–Aldrich syndrome
D. X-linked lymphoproliferative syndrome

Question 28.4 A 35-year-old Caucasian male presents to his primary doctor due to 2 months
of gradually worsening fevers, chills, and night sweats that soak through multiple pillows. He
recalls having lost some weight over the past months, involuntarily, but is most bothered by
an itchy rash on his abdomen that has recently started to ulcerate. Physical examination
reveals painless lymphadenopathy. Biopsy shows pleomorphic large cells with prominent
nucleoli. Immunophenotyping shows cells positive for CD30 and ALK, and negative for B-cell
markers. Which of the following is TRUE regarding this condition?
A. ALK-positive cases are more common in adults.
B. ALK protein is detected in a minority of cases.
C. Cells are often derived from mature, activated T cells.
D. Prognosis in ALK-positive cases is worse than ALK-negative cases.

Question 28.5 A 55-year-old African-American man presents to his PCP with abdominal pain
that has been gradually increasing over 3 months. He notes that he has been urinating less
than normal. Furthermore, he complains of persistent fevers and drenching night sweats. His
wife notes that he looks thinner than before. The patient is less active than his baseline but is
able to perform light housework without difficulty. Physical examination reveals nontender
adenopathy in the bilateral cervical and left axillary regions. PET/CT shows FDG-avid
bilateral cervical, left axillary, and mediastinal lymphadenopathy as well as a 7-cm
retroperitoneal mass resulting in bilateral hydronephrosis. CBC is within normal limits. LDH
is 210 IU/L (upper limit of normal 250 IU/L). Bone marrow biopsy demonstrates no evidence
of disease. Which of the following best describes the patient’s International Prognostic Index
(IPI) score?
A. 0
B. 1
C. 2
D. 3

Question 28.6 Human herpesvirus-8 is implicated in which of the following diseases?


A. Burkitt lymphoma
B. GCB DLBCL
C. Monocytoid B-cell lymphoma
D. Primary effusion lymphoma

Question 28.7 Which of the following is/are mature T-cell neoplasms?


A. Anaplastic large-cell lymphoma
B. Hairy cell leukemia
C. Mycosis fungoides
D. Primary effusion lymphoma

Question 28.8 Which of the following B-cell lymphomas may have CD10 positivity and the
t(14;18) translocation? (Select two correct responses)
A. Diffuse large B-cell lymphoma
B. Follicular lymphoma
C. Lymphoplasmacytic lymphoma
D. Mantle cell lymphoma
Question 28.9 A 66-year-old Caucasian male with HIV and CD4 count of 53 is being worked
up by his oncologist for a new diagnosis of diffuse large B-cell lymphoma, which involves
diffuse lymphadenopathy, the paranasal sinuses, testes, and bone marrow based on initial
studies. What is the next best step in management?
A. Initiation of testicular radiation
B. Lumbar puncture
C. Orchiectomy
D. Upper and lower endoscopy

Question 28.10 A 71-year-old man presents to clinic with abdominal pain. Review of
systems is also positive for fatigue and easy bleeding from the nose and gums. Physical
examination reveals splenomegaly. Pertinent labs include hemoglobin of 12 g/dL, platelets of
16,000, LDH of 400 IU/L (upper limit of normal 250 IU/L), and albumin of 3.6 g/dL.
Peripheral smear reveals “villous lymphocytes.” Flow cytometry is positive for CD19, CD20,
and CD22 and negative for CD5, CD25, and CD103. Which of the following are possible steps
in initial management? (Select three correct responses)
A. Autologous stem cell transplantation
B. Chemotherapy with alkylating agents
C. Splenectomy
D. Single agent rituximab

Question 28.11 Which of the following findings portend a shorter survival/worse prognosis
for patients with splenic marginal zone lymphoma? (Select three correct responses)
A. Albumin <3.5 g/dL
B. Hemoglobin <12 g/dL
C. LDH >normal
D. Presence of villous lymphocytes

Question 28.12 A 69-year-old Caucasian man presents to his primary care doctor for
persistent nausea associated with nonbloody, nonbilious emesis, loss of appetite, and 30-lb
weight loss over approximately 1 year. He also recalls having occasional dark stools. Physical
examination reveals splenomegaly. PET/CT reveals FDG avidity of intra-abdominal lymph
nodes as well as uptake in the stomach and small and large bowel. Upper and lower
endoscopy reveals a nonbleeding gastric ulcer and diffuse polyps in the colon. Biopsies are
performed. Cells express surface IgM and IgD along with CD5, CD19, and CD20.
Cytogenetics reveals t(11;14). Which of the following statement(s) is/are accurate regarding
the next step in management?
A. Consolidation with autologous stem cell transplantation in first remission is an approach
often considered for fit patients.
B. Treatment with ibrutinib in the upfront setting is limited because of its association with
peripheral neuropathy.
C. Front-line therapy for elderly patients with good performance status includes
bendamustine and rituximab.
D. Hyper-CVAD with rituximab may be administered to patients with more aggressive
disease.

Question 28.13 A 46-year-old Caucasian man presents to clinic due to drenching night
sweats associated with persistent, high-grade fevers and 40-lb weight loss over the past 6
months. Physical examination reveals lymphadenopathy of the cervical and axillary nodes as
well as hepatospenomegaly. PET/CT reveals avid uptake throughout the mediastinum,
stomach, and small bowel, with multiple nodes measuring up to 2 cm. FISH reveals BCL-
1/IgH fusion gene product. Additional studies show decreased expression of p21, p27, and
p53. Which of the following statement(s) regarding the illness is/are CORRECT?
A. Cells arise from antigen-naive B-cells of the inner mantle zone.
B. Most patients present in early-stage disease.
C. The condition is more common in females.
D. Tumor cells strongly express surface IgG and IgD as well as CD19 and CD20.

Question 28.14 A 26-year-old Caucasian man with no past medical history presents to the
emergency department with right knee pain that occurred suddenly while playing volleyball.
He is found to have a pathologic fracture of the right distal femur. Further imaging with
PET/CT reveals localized uptake of the right femur and tibia. Flow cytometry shows that
cells are CD19, CD20, and CD22 positive. Immunohistochemistry also reveals that cells are
CD10 positive and negative for BCL-6 and MUM-1. Which of the following statement(s)
regarding disease subtype is/are accurate?
A. Ig gene shows intraclonal homogeneity.
B. Nuclear factor kappa beta signaling is the primary driver of proliferation and
metastases.
C. Survival is worse compared to other subtypes.
D. The subtype is often associated with t(14;18).

Question 28.15 A 63-year-old African-American woman with a past medical history of HIV
presents to the emergency room with shortness of breath. She notes that it has been
worsening over the past 3 to 4 months, but became unbearable over the past week. Review
of systems is positive for involuntary weight loss of 30 lb over 4 months and pedal edema.
Physical examination reveals pitting edema to the knees bilaterally. PET/CT scan shows
diffuse adenopathy of the mediastinum and the pelvis, with the largest nodes measuring 2.5
cm. No extranodal disease is appreciated. Excisional biopsy reveals proliferating large and
small lymphocytes. Ki-67 is 85%. Flow cytometry reveals cells that are CD19, CD20, and
CD22 positive and negative for CD10 and BCL-6. Which of the following statements
regarding management is accurate?
A. R-CHOP chemotherapy is appropriate.
B. ABVD chemotherapy is preferred as first-line management.
C. Allogeneic stem cell transplant is the treatment of choice if the patient were to relapse
with chemosensitive disease.
D. Rituximab with CHOP confers greater benefits in patients who lack BCL-6 expression,
based on the GELA R-CHOP trial.
Question 28.16 A 55-year-old Chinese man presents to his primary care physician due to
chronic sinus pain that has been present for 5 months. He also complains of epistaxis, fevers,
drenching night sweats, and 30-lb weight loss. Physical examination is pertinent for temporal
wasting and tenderness to palpation around the paranasal sinuses as well as cervical
adenopathy. Labs reveal LDH of 425 IU/L (upper limit of normal 250 IU/L). CT of the
sinuses reveals an infiltrating mass in the midline nasal sinus. Immunophenotype reveals
atypical cells that are positive for CD2, CD56, and cytoplasmic CD3, and negative for CD4,
CD8, and surface CD3. EBV is detected. The cytoplasmic granule proteins, granzyme B and
TIA-1, are present. Cytogenetics reveals del(6)(q21;q25). Which of the following
statement(s) regarding treatment is/are CORRECT?
A. Allogeneic stem cell transplantation is a standard first-line therapy in fit patients.
B. EBV-related hemophagocytic syndrome is often fatal and treated with front-line
radiation.
C. Localized disease is initially managed with concurrent chemotherapy and radiation
D. Patients with disseminated disease that is chemotherapy sensitive are effectively treated
with a combination of R-CHOP and radiotherapy

Question 28.17 A 43-year-old woman presents to a dermatologist with a diffuse skin rash,
described as red and itchy with associated thickening of the palms and soles that has been
slowly progressing for the past 4 years. Physical examination reveals erythematous plaques
on 20% of the body surface area. FDG PET reveals absence of extracutaneous disease. Skin
biopsy reveals Pautrier microabscesses. Immunophenotyping reveals expression of CD2, CD3,
CD5, and CD7. What is the next best step in management?
A. EPOCH chemotherapy combined with pentostatin and fludarabine with interferon alpha
B. Extracorporeal photochemotherapy
C. Locally applied denileukin diftitox with or without vorinostat
D. Topical carmustine or mechlorethamine, bexarotene, or ultraviolet B with or without
interferon alpha

Question 28.18 Which of the following accurately describes an aspect of primary CNS
lymphoma (PCNSL)?
A. AIDS patients tend to have large, invasive unifocal masses that rapidly progress without
HAART therapy.
B. Incidence has decreased fivefold from 1985 to 1997 due to advances in treatment of
immunosuppressed patients.
C. Many masses are periventricular, allowing tumor cells access to cerebrospinal fluid.
D. Presents most commonly in the occipital lobe, manifesting as vision changes and gait
abnormalities.

Question 28.19 Which of the following statement(s) regarding the presentation of primary
CNS lymphoma is/are CORRECT?
A. Approximately 20% of patients have ocular involvement at the time of presentation,
particularly of the optic nerve, vitreous, retina, and choroid.
B. Most patients present in stage I, explaining the good prognosis of the condition.
C. Primary leptomeningeal lymphoma in the absence of a brain mass accounts for 40% of
cases, with symptoms including leg weakness and incontinence/retention.
D. Studies have shown that 10% of patients can have spread of disease outside of the CNS,
mostly to the bone marrow and lymph nodes.

Question 28.20 A 71-year-old Caucasian man is brought to his primary care physician by his
wife, who says that he has been exhibiting increasingly aggressive behavior over the past 5
months, manifesting as irritability and outbursts of anger, both of which are outside of his
norm. She also says that he has had two motor vehicle accidents in the same time period, due
to not noticing traffic lights. Neurologic examination reveals decreased ability to follow
commands, 20/200 vision in both eyes, and left-sided weakness. Brain MRI shows a 4 × 5
cm solitary, nonhemorrhagic mass in the right frontal lobe. Lumbar puncture reveals
predominance of clonal lymphocytes. Biopsy indicates cells positive for BCL-2, BCL-6, and
MUM1 as well as CD19, CD20, and CD22. What is the next best step in management?
A. High-dose methotrexate and rituximab, with or without Ara-C and temozolomide.
B. R-CHOP chemotherapy with intrathecal methotrexate.
C. Six weeks of concurrent chemotherapy with methotrexate and dexamethasone combined
with focal radiation therapy.
D. Systemic adriamycin and etoposide with high-dose dexamethasone.

Question 28.21 Which of the following statements best characterizes nodular lymphocyte-
predominant Hodgkin lymphoma (NLPHL) in contrast to classic Hodgkin lymphoma (cHL)?
A. Approximately 80% of patients with NLPHL have stage I to II disease at the time of
diagnosis.
B. Bone marrow involvement is frequent with stage III to IV disease.
C. Disease presentation manifests predominantly in thoracic nodes before spreading to the
mediastinum.
D. NLPHL cells express CD30 and CD45 as well as B-cell antigens.

Question 28.22 A 24-year-old G1P0 woman presents to her physician with fever, neck pain,
fatigue, and 10-lb weight loss during the first trimester of pregnancy. She is concerned about
the health of her fetus. Physical examination reveals nontender cervical adenopathy, leading
to a biopsy, which reveals cells that are positive for CD15 and CD30 and negative for CD3,
CD7, CD20, and CD45. Morphology is pertinent for Reed–Sternberg cells in a background of
inflammatory cells. Which of the following accurately describe(s) an aspect of care for such
patients?
A. BEACOPP chemotherapy is preferred to ABVD for treatment during the first trimester
due to the lack of antimetabolites in the regimen.
B. MRI scanning is preferred for staging because it is nonteratogenic.
C. Long-term survival of treated pregnant women is inferior to that of nonpregnant
women with the disease.
D. Radiation therapy with abdominal shielding can be safely used for bulky disease above
the diaphragm.
ANSWERS

Question 28.1 The answer is C.


This patient has Burkitt lymphoma. The classic translocations are t(2;8), t(8;14), and t(8;
22) causing dysregulation of the c-myc oncogene. The most common translocation is
t(8;14). The t(1;14) occurs in MALT lymphoma.

Question 28.2 The answers are A and D.


Endemic disease, commonly seen in areas of chronic malaria, occurs in children and in the
jaw region. In contrast, sporadic disease occurs more commonly in the abdomen. C-myc,
rather than BCL-2, is required for pathogenesis.

Question 28.3 The answer is B.


HIV/AIDS increases the risk of adult lymphomas, compared to the other choices, which
are associated with an increased risk of childhood lymphomas.

Question 28.4 The answer is C.


This patient has anaplastic large-cell lymphoma. ALK is detected in more than half of
patients, being more common in children and associated with better prognoses.

Question 28.5 The answer is B.


The International Prognostic Index (IPI) score for lymphomas is based on five factors
including age greater than 60, performance status (PS) greater than or equal to 2, LDH >
normal, stage III/IV, and greater than or equal to two extranodal sites. Scores of 0 to 1, 2,
3 and 4 to 5 are classified as low-risk, low-intermediate-risk, high-intermediate-risk, and
high-risk respectively. The patient has stage III/IV disease, but is younger than 60 with an
ECOG PS of 0, normal LDH, and no extranodal sites. Therefore, his IPI score is 1 (low-
risk).

Question 28.6 The answer is D.


Endemic Burkitt lymphoma is associated with EBV, while hepatitis C viruses are
associated with monocytoid B-cell lymphoma. There is no established viral etiology for
diffuse large B-cell lymphoma. Primary effusion lymphoma is associated with HHV-8.

Question 28.7 The answers are A and C.


Both mycosis fungoides and anaplastic large-cell lymphoma are T-cell neoplasms, while
hairy cell leukemia and primary effusion lymphomas are B-cell malignancies.

Question 28.8 The answers are A and B.


The two malignancies associated with CD10 positivity and t(14;18) that result in
increased expression of the antiapoptotic protein, BCL-2, are follicular lymphoma and
diffuse large B-cell lymphoma (DLBCL).
Question 28.9 The answer is B.
Patients with large-cell lymphoma and involvement of the paranasal sinuses, testes, and
bone marrow are more prone to meningeal spread, thus necessitating a lumbar puncture.
The presence of CNS involvement will impact the treatment plan. While testicular
radiation may be administered following chemotherapy, it is not the next best step. An
orchiectomy is not indicated. Similarly upper and lower endoscopies are rarely indicated
in DLBCL unless needed to obtain the initial diagnosis.

Question 28.10 The answers are B, C and D.


The patient has splenic marginal zone lymphoma (SMZL), which is characterized by
circulating villous lymphocytes that express CD19, CD20, CD22, and BCL-2. The lack of
CD103 and CD25 differentiates it from hairy cell leukemia. Hemoglobin less than 12
g/dL, high LDH, and albumin less than 3.5 g/dL predict a shorter survival of 50%.
Possible treatment options include chemotherapy with alkylating agents, rituximab, and
splenectomy. Upfront autologous stem cell transplantation would not be a routine
approach for this indolent lymphoma that often responds well to single-agent rituximab.

Question 28.11 The answers are A, B and C.


Villous lymphocytes do not portend a worse prognosis, while the aforementioned three
answer choices are associated with poorer outcomes.

Question 28.12 The answers are A, C and D.


The patient has mantle cell lymphoma. Bortezomib, not ibrutinib, is characterized by its
tendency to cause peripheral neuropathy. Ibrutinib is only approved in the relapsed
setting. The optimal front-line regimen for mantle cell lymphoma is highly debated.
Treatment options include intensive regimens such as hyper-CVAD and upfront autologous
stem cell transplant. The less intensive regimen of bendamustine and rituximab is another
option with demonstrated high overall response rates.

Question 28.13 The answer is A.


Mantle cell lymphoma is more common in males and typically presents as stage IV
disease. The tumor cells express IgM and IgD, as well as B-cell associated antigens. The
cells arise from antigen-naive B cells of the inner mantle zone. Nuclear cyclin D1 is
present in all cases and is the gold standard for diagnosis.

Question 28.14 The answer is D.


The patient has germinal center B-cell (GCB) DLBCL, based on the Hans criteria. GCB is
associated with a better prognosis than the activated B-cell (ABC) subtype DLBCL, which
is associated with NF kappa beta signaling. The GCB subtype is associated with t(14;18),
which indicates a follicle center cell origin.

Question 28.15 The answer is A.


For patients with advanced-stage DLBCL, the standard-of-care treatment is with R-CHOP
chemotherapy. Abbreviated R-CHOP chemotherapy combined with involved field
radiotherapy is an appropriate treatment option for low-risk, nonbulky, early stage
DLBCL. ABVD is the standard regimen for patients with Hodgkin lymphoma. Autologous
stem cell transplant is reserved for patient with relapsed chemosensitive disease.
Rituximab has a greater benefit for patients that express BCL-2 on IHC.

Question 28.16 The answer is C.


The patient has extranodal NK/T-cell lymphoma. Localized disease can be treated with a
combination of chemotherapy and radiotherapy. Treatment failure is common with RT
alone. Rituximab is not indicated in this T-cell lymphoma. Patients with disseminated
disease have very poor prognoses and should be enrolled in clinical trials. Early use of
high-dose chemotherapy combined with autologous transplantation may be beneficial in
some patients.

Question 28.17 The answer is D.


The patient has mycosis fungoides. For localized disease, treatment options include
carmustine, mechlorethamine, bexarotene, ultraviolet B, near ultraviolet light, interferon
alpha, and electron beam radiation. Options for systemic disease, which the patient does
not have, include EPOCH, extracorporeal photochemotherapy, denileukin diftitox, and
monoclonal antibodies.

Question 28.18 The answer is C.


The incidence of PCNSL has increased in immunocompetent individuals. Frontal lobe
presentation is the most common, leading to changes in personality, level of alertness,
headaches, and signs/symptoms of increased intracranial pressure. AIDS patients also
universally have multifocal lesions, compared to patients without HIV, which have a 40%
incidence of multiple foci.

Question 28.19 The answer is A.


Primary leptomeningeal lymphoma occurs in 7% of patients. Primary CNS lymphoma
(PCNSL) very rarely spreads outside of the CNS. Despite the confinement of the disease to
only one location, the prognosis is typically worse compared with other stage I diffuse
large B-cell lymphomas. This is an area where Ann Arbor staging is not particularly
helpful and many authors classify PCNSL as stage IV, rather than stage IE (extranodal).

Question 28.20 The answer is A.


The patient has primary CNS lymphoma for which the backbone of therapy is high-dose
methotrexate combined with rituximab. In patients older than 60 years of age, HD-MTX
combined with whole-brain radiotherapy causes significant encephalomalacia with
cognitive decline and should be avoided.

Question 28.21 The answer is A.


The atypical cells in NLPHL are CD45 positive and express B-cell antigens such as CD19,
CD20, CD22, CD79a, and PAX5. The cells do not express CD30. The condition can
transform into DLBCL in 2% to 6% of cases. Most patients present with early-stage
disease. Bone marrow involvement is rare even with more advanced disease. Unlike the
contiguous spread seen in classical Hodgkin lymphoma, NLPHL often involves peripheral
nodes in a noncontiguous fashion. Inguinal node involvement is common. Overall, the
response to therapy is more favorable, since more than 90% of patients have a clinical
remission and are alive at 10 years.

Question 28.22 The answer is B.


Hodgkin lymphoma is the fourth most common cancer diagnosed during pregnancy. ABVD
chemotherapy is the preferred regimen during pregnancy since it does not include
antimetabolites. MRI is preferred to CT scanning, in order to avoid exposing the fetus to
ionizing radiation. Radiation therapy can be used above the diaphragm in the second or
third trimester if the patient has rapid progression of lymphadenopathy.

Corresponding chapters in Cancer: Principles & Practice of Oncology, Tenth Edition: 101 (Molecular Biology of Lymphomas),
102 (Hodgkin Lymphoma), 103 (Non-Hodgkin Lymphoma), 104 (Cutaneous Lymphomas), and 105 (Primary Central Nervous
System Lymphomas)
29 Acute Leukemias
Amanda F. Cashen

QUESTIONS
Each of the numbered items below is followed by lettered answers. Select the ONE lettered answer
that is BEST in each case unless instructed otherwise.

Question 29.1 The (8;21) cytogenetic rearrange, which is associated with favorable risk in
acute myeloid leukemia (AML), juxtaposes which genes?
A. AML1-ETO
B. RUNX1-RUNX1T1
C. A core-binding factor and a zinc finger protein
D. All of the above

Question 29.2 You are caring for a 25-year-old woman with AML-M2 and normal
cytogenetics. She receives 7 + 3 induction therapy, and her day-14 bone marrow biopsy is
ablated. She returns to clinic and her day-45 bone marrow shows normal hematopoiesis.
However, she relapses after 18 months. Which of the following likely contributed to her
relapse?
A. DNMT3A mutation
B. CEBPA mutation
C. NPM mutation without FLT3-ITD
D. Her age

Question 29.3 Your well-read 30-year-old patient with AML asks you how you will follow
her disease once she is in remission. She asks which is the most sensitive test for minimal
residual disease. In which order are these assays most sensitive (least to most sensitive)?
A. Cytopathology < fluorescence in situ hybridization (FISH) < polymerase chain
reaction (PCR)
B. FISH < cytogenetics < PCR
C. PCR < cytopathology < FISH
D. Cytogenetics < PCR < flow cytometry

Question 29.4 Mutation of which of the following genes is associated with a worse
prognosis in core-binding factor AML?
A. ABL
B. CKIT
C. NPM1
D. GATA-1

Question 29.5 A 52-year-old man presents with AML. On day 2 of induction therapy, he
develops diffuse alveolar hemorrhage. An expected physical finding on examination would
be:
A. Diffuse ecchymosis.
B. Diffuse intravascular coagulopathy.
C. Swollen gums.
D. Cardiac rub.

Questions 29.6–7 You have been following a 75-year-old woman in clinic with mild renal
insufficiency and progressive anemia. She has good performance status and presents to clinic
with a 3-day history of progressive fatigue and new headaches. Her white blood cell (WBC)
count is 53,000 with 50% blasts.

Question 29.6 Given her history, what is the expected finding in the cytogenetics?
A. t(15;17)
B. –7
C. inv(16)
D. t(8;21)

Question 29.7 Which of the following is associated with improved prognosis in this patient?
A. Probable leukostasis
B. Her age
C. Prior myelodysplastic syndrome (MDS)
D. Good performance status

Question 29.8 Which of the following cytogenetic profiles from an AML patient is an
example of monosomal karyotype?
A. −7
B. +8, inv(3), del(5q)
C. inv(16)
D. −7, −5, t(3;21)

Question 29.9 Which of the following is a poor prognostic factor in adult ALL?
A. Age <55 years
B. WBC <10,000
C. Diploid chromosomes on karyotyping
D. t(4;11)

Question 29.10 A 70-year-old man presents with newly diagnosed AML-M4 with
eosinophilia. Which of the following factors will have the greatest impact on the choice of
therapy?
A. His age
B. His performance status
C. His cytogenetics
D. Excellent response to 7 + 3 induction therapy in the elderly

Question 29.11 You have been following a 54-year-old woman with acute promyelocytic
leukemia (APL) in clinic. After 2 years she returns to clinic with fatigue, an elevated WBC,
and increased promyelocytes. Peripheral blood PCR confirms recurrence of her t(15;17)
translocation. Before starting arsenic salvage therapy, which test you obtain?
A. Liver function test
B. Erythrocyte sedimentation rate
C. D-dimer
D. Electrocardiogram

Question 29.12 You have been caring for a 65-year-old woman with a distant history of
breast cancer treated with adjuvant cyclophosphamide and adriamycin. During the last year,
she developed progressive anemia and thrombocytopenia. She also recently developed
leukopenia. Her bone marrow biopsy shows decreased cellularity with dysplastic features and
25% blasts. Which of the following cytogenetic changes might you expect to find?
A. t(15;17)
B. t(9;21)
C. Complex cytogenetics
D. Trisomy 21

Question 29.13 Which of the following statements is TRUE regarding elderly patients (age
>65 years) with AML? (Select two correct responses)
A. In general, older patients with AML have poor outcomes when compared to younger
patients.
B. Treatment with hypomethylating agents leads to the same response rate as more
intensive chemotherapy.
C. Older patients are more likely to have favorable risk cytogenetics than younger
patients.
D. Treatment with decitabine leads to better response rates but similar survival compared
to low-dose cytarabine.

Questions 29.14–18 You are consulted to see a 20-year-old Hispanic woman who presented
with progressive fatigue during the last week and then significant epistaxis. Her WBC is
12,000/μL with 40% promyelocytes and a platelet count of 15,000/μL. Her international
normalized ratio (INR) is 2.7 with a prothrombin time of 45 and partial thromboplastin time
of 65. Her fibrinogen is 82. On review of her peripheral smear, you observe many
promyelocytes with large granules and multiple Auer rods.

Question 29.14 After review of her peripheral smear, you suspect that she has APL. While
awaiting confirmation of the diagnosis, your initial therapy should include which of the
following?
A. Steroids
B. Cytarabine
C. Fresh-frozen plasma
D. Arsenic trioxide

Question 29.15 Three days after starting idarubicin and ATRA, her coagulopathy has
improved. She has shortness of breath in the morning and rapidly becomes hypoxic over the
course of the day. Which of the following would prove most helpful in treating her hypoxia?
A. High-flow facemask oxygen
B. Lasix
C. Methylprednisolone
D. Albuterol

Question 29.16 Fifteen days into treatment she develops a severe headache. Her neurologic
and fundoscopic evaluations are normal. Review of her morning laboratory tests reveal a
WBC of 0.6/μL, hematocrit of 9.8, platelet count of 25,000/μL, INR of 1.4, and fibrinogen of
190. You obtain a noncontrast head computed tomography scan but are more worried that
this is a result of:
A. Relapse.
B. ATRA.
C. Idarubicin.
D. Transfusion reaction.

Question 29.17 Six months later you are reviewing her chart. Her CBC has normalized, and
she has tolerated consolidation therapy. Her most recent PCR showed no sign of residual
disease. You are most concerned about relapse because of:
A. Her microgranular variant presentation.
B. Her presenting coagulopathy.
C. Her presenting WBC.
D. Her ethnicity.

Question 29.18 Three years later she presents to clinic with an elevated leukocyte count.
PCR of peripheral blood confirms the presence of her initial t(15;17) translocation. Treatment
options at this point include which of the following?
A. Arsenic trioxide
B. Gemtuzumab ozogamicin
C. Autologous transplant after achieving CR
D. All of the above

Question 29.19 A 30-year-old, otherwise healthy woman is diagnosed with AML.


Cytogenetics reveal inv(16). She undergoes induction therapy. Day-14 bone marrow biopsy
shows an ablated marrow. Her day-45 marrow shows restored cellularity without evidence of
disease. Repeat cytogenetics do not reveal the inv(16) rearrangement. Appropriate
consolidation therapy would be:
A. Allogeneic transplant if a matched sibling donor is available.
B. High-dose cytarabine (HIDAC) 3 g/m2 every 12 hours on days 1, 3, and 5 for four 28-
day cycles.
C. Intermediate-dose cytarabine (IDAC) 300 mg/m2 every 12 hours on days 1, 3, and 5 for
four 28-day cycles.
D. Arsenic 0.15 mg/kg on days 1 to 5 for four 28-day cycles.

Question 29.20 Which of the following regimens require graft-versus-tumor effects to treat
residual AML blasts?
A. Busulfan-cyclophosphamide
B. Cyclophosphamide-TBI
C. Busulfan-fludarabine
D. Busulfan-VP16

Question 29.21 Central nervous system (CNS) prophylaxis should be considered in which of
the following patients?
A. A 25-year-old Hispanic woman with APL who presents with a WBC of 2500/μL, an INR
of 2.5, and fibrinogen of 100
B. A 78-year-old man with AML evolved from MDS
C. A 30-year-old woman with AML who develops headaches while receiving ondansetron
for nausea on day 9 of induction therapy
D. A 20-year-old man with Down syndrome and ALL who presents with leukocytosis (WBC
120,000/μL)

Question 29.22 Dexamethasone has replaced prednisone in ALL induction therapy because
of improved penetration in which tissue?
A. Testes
B. Spleen
C. Bone marrow
D. Brain

Question 29.23 How long should maintenance therapy for ALL with daily 6-mercaptopurine,
weekly methotrexate, and monthly vincristine and prednisone be continued?
A. 6 months
B. 12 months
C. 24 to 36 months
D. Until relapse

Question 29.24 Which of the following targeted agents have been shown to be beneficial in
adult ALL?
A. Imatinib
B. Alemtuzumab
C. Gemtuzumab ozogamicin
D. Sunitinib

Question 29.25 For which of the following patients would you consider myeloablative stem
cell transplantation in CR1?
A. A 30-year-old woman with inv(16) AML-M4 with eosinophilia
B. A 50-year-old man with complex cytogenetics AML-M1
C. A 65-year-old man with complex cytogenetics AML-M1
D. A 50-year-old woman with t(15;17) AML-M3

Question 29.26 Which of the following statements is TRUE regarding the nucleophosmin 1
(NPM1) mutation in AML? (Select two correct responses)
A. In normal karyotype AML, NPM1 mutation without FLT3-ITD confers a prognosis
similar to good-risk cytogenetics.
B. NPM1 mutations are found mostly in patients with AML with normal karyotype.
C. NPM1 mutation combined with FLT3-ITD is associated with good prognosis.
D. All the above
ANSWERS

Question 29.1 The answer is D.


The favorable t(8;21) translocation results in the juxtaposition of the initially named
AML1 gene with the ETO gene (the AML gene and the eight-twenty-one gene). These two
have been renamed RUNX1 and RUNX1T1, respectively. RUNX1 is a member of the core-
binding factor family of transcription factors, which regulate differentiation of normal
blasts. This fusion protein creates a dominant negative protein, which results in a
differentiation block. Inv(16) and the t(16;16) rearrangements likewise result in dominant
negative effects on core-binding factors and are associated with a favorable prognosis.

Question 29.2 The answer is A.


The DNMT3A gene encodes the DNA (cytosine-5)-methyltransferase 3A enzyme that
catalyzes the addition of a methyl group to the cytosine residue of CpG dinucleotides. In
patients with AML with normal cytogenetics, the presence of DNMT3A mutations is
independently associated with poor outcomes. CEBP/α is a transcription factor involved
in myeloid differentiation thought to be activated by ATRA treatment in APL. Mutations
in the CEBPA gene have been noted in other FAB-AML classifications and have been
associated with a superior outcome compared with the wild-type transcript. NPM
mutations, in the absence of the FLT3-ITD, have also been associated with improved
prognosis. Age is a strong prognostic factor, with patients older than 65 having decreased
survival. Her young age would be a marker of a good prognosis.

Question 29.3 The answer is A.


Cytopathology requires greater than 5% blasts on a bone marrow biopsy to diagnose AML
relapse. This results in a sensitivity of 1 in 20. A cytogenetic review of 30 metaphases
results in a 1 in 30 sensitivity. FISH has a sensitivity of approximately 1 in 500. PCR
techniques allow sensitivity of approximately 1 in 104. However, both FISH and PCR
require screening for defined cytogenetic changes. Because initial cytogenetic changes
typically recur during AML relapse, this is generally not a significant barrier, unless
patients present with uncommon cytogenetic changes for which FISH and PCR probes are
not commercially available.

Question 29.4 The answer is B.


Core-binding factor (CBF) AML includes AML with t(8;21) and inv(16). These leukemias
are generally associated with good prognosis. However, patients whose CBF AML has a
mutation in CKIT have a worse prognosis than other CBF AML patients. They may benefit
from treatment with a kinase inhibitor or allogeneic stem cell transplant.

Question 29.5 The answer is C.


Diffuse alveolar hemorrhage is a rare complication of induction therapy in AML, which
carries significant mortality risk. It is most commonly reported in patients with AML and
myelomonocytic or monocytic features (M4 or M5). These myelomonoblasts and
monoblasts display a predilection for tissue invasion, such as in the gums, as well as into
the lungs. This is thought to result from their high expression of adhesion molecules:
intercellular adhesion molecule and vascular cell adhesion molecule. During initial
treatment, activation and death of circulating blasts may lead to pulmonary inflammation
and alveolar hemorrhage. Because of the rarity of such events, treatment options remain
poorly studied. In small series, high-dose steroids have shown benefit.

Question 29.6 The answer is B.


AML which has evolved from MDS is especially recalcitrant to therapy. It may be
associated with –5 or –7 cytogenetic abnormalities but is most commonly associated with
complex cytogenetics. This patient is presenting with many poor-risk features (age >65
years, symptoms suggestive of leukostasis, and underlying organ insufficiency), which
will decrease the probability of response and survival.

Question 29.7 The answer is D.


There are many prognostic factors for AML. Of particular importance are age,
cytogenetics, and performance status. Age greater than 65 years has been associated with
worse outcomes independently of cytogenetics. AML evolving from prior MDS also
confers a worse prognosis. Leukostasis at presentation presents significant early morbidity
and mortality, especially in an older patient. An elevated peripheral blast count, which
may be associated with leukostasis, is also associated with a poor long-term prognosis.

Question 29.8 The answer is D.


Monosomal karyotype is defined by the presence of two or more autosomal monosomies
or by the presence of one autosomal monosomy and a structural change (translocation).
Monosomal karyotype is associated with a particularly poor prognosis.

Question 29.9 The answer is D.


Age greater than 50 years, poor performance status, African-American ethnicity, and
leukocytosis (>30,000/μL B-lineage or >100,000/μL T-lineage) are associated with high-
risk ALL. Immunophenotypes including early and mature-T ALL and pro-B ALL are also
considered high risk. Cytogenetics including the Philadelphia chromosome t(9;22) and
t(4;11) are both high-risk prognostic factors. Normal diploid chromosomes on karyotyping
are not associated with poor prognosis.

Question 29.10 The answer is B.


Treatment of AML in the elderly (>65 years) is difficult because of the heterogeneous
nature of response and the generally high rate of treatment-related mortality. Older
patients at MD Anderson with a performance status >2, bilirubin greater than 1.9 mg/dL,
or serum creatinine greater than 1.9 mg/dL had a treatment-related mortality of 62% and
only a 27% complete remission rate. Thus, decision to undergo standard induction therapy
must be based on a careful assessment of the patient’s ability to tolerate the therapy and
his/her prognosis.
Question 29.11 The answer is D.
Arsenic trioxide has demonstrated considerable activity in both initial and salvage
treatment of APL. Arsenic trioxide has been associated with prolonged QTc syndrome.
Therefore, electrocardiograms should be performed before and during therapy.
Electrolytes should also be monitored and corrected. Additional medications that may
prolong the QTc should be eliminated if possible.

Question 29.12 The answer is C.


Prior alkylator therapy increases the risk of MDS and treatment-related AML. Her recent
history of progressive anemia and thrombocytopenia, with dysplastic features seen in her
marrow, are all suggestive of an underlying MDS. Her blast count greater than 20% meets
criterion for AML under the current World Health Organization standards. AML evolved
from MDS could carry a loss of chromosome 5 or 7 associated with the prior MDS, but
most often is associated with complex cytogenetics. It also carries an unfavorable
prognosis in multiple studies. t(15;17) is associated with APL but is not associated with
prior breast cancer or alkylator therapy. t(9;21) is associated with the Philadelphia
chromosome and seen in chronic myelocytic leukemia or acute lymphocytic leukemia, but
is rarely associated with AML or MDS. Trisomy 21 is associated with AML-M7 with
increased megakaryocytes.

Question 29.13 The answers are A and D.


Older patients with AML have worse outcomes when compared to younger patients
irrespective of their performance status or comorbidities. Treatment with the
hypomethylating agents, azacitidine and decitabine, is associated with lower response
rates but similar overall survival compared to more intensive chemotherapy regimens.
Favorable cytogenetics is less common in older AML patients. Treatment with decitabine
is associated with better response rate, but not improved survival, compared to low-dose
cytarabine.

Question 29.14 The answer is C.


Patients with APL are at risk of early morbidity and mortality because of disseminated
intravascular coagulation and bleeding complications. Aggressive management of
coagulopathy is mandatory, with the goal of maintaining the fibrinogen greater than 150,
INR less than 1.5, and platelet count greater than 30,000.

Question 29.15 The answer is C.


Differentiation syndrome can be a life-threatening complication of APL and ATRA
treatment. As the granule-laden promyelocytes differentiate, the granular products
stimulate pulmonary edema and fluid retention. Fevers and weight gain are common
symptoms of differentiation syndrome. Treatment includes steroids and management of
coagulopathy. Differentiation syndrome may affect as many as 10% to 25% of patients
with APL.
Question 29.16 The answer is B.
Headache is a common side effect from ATRA. Treatment options include dose reduction
and symptom management with analgesics. Unfortunately, patients occasionally require
drug discontinuation because of this side effect.

Question 29.17 The answer is C.


The greatest risk of relapse in APL is associated with an elevated WBC greater than
10,000/μL on presentation. Patients with presenting WBC less than 10,000/μL achieve CR
approximately 90% of the time.

Question 29.18 The answer is D.


Arsenic trioxide has demonstrated significant efficacy in relapsed APL and in consolidation
protocols. APL promyelocytes commonly express high levels of CD33, and gemtuzumab
ozogamicin (anti-CD33) has demonstrated efficacy in relapsed APL. APL is one of the few
cases in AML in which autologous transplantation has demonstrated significant benefit,
and it is often used in a salvage consolidation protocol.

Question 29.19 The answer is B.


Although most patients with AML will relapse without consolidation therapy, the
appropriate consolidation therapy remains unclear for the majority of these patients.
Young patients with good-risk cytogenetics represent one of the few cohorts with AML in
whom specific consolidation recommendations are supported by clinical data. A study
sponsored by the Cancer and Leukemia Group B (CALGB) found that young patients with
inv(16) had a 78% CR rate at 5 years after four cycles of HIDAC therapy, whereas 57%
remained in CR in the IDAC arm and only 16% remained in CR with conventional 100
mg/m2 dosing. Patients with normal cytogenetics fared less well as a group with
equivalent outcomes (47% vs. 37% CR at 5 years) in the HIDAC and IDAC arms and
worse outcomes with conventional dosing (20%). Patients with unfavorable cytogenetics
should be considered for allogeneic transplant in first CR. Arsenic has shown benefit in a
consolidation protocol for APL, but it has not been evaluated in general AML
consolidation protocols.

Question 29.20 The answer is C.


Nonmyeloablative conditioning regimens require a graft-versus-leukemia effect to
improve the clearance of residual AML blasts not eliminated by the conditioning regimen.

Question 29.21 The answer is D.


CNS prophylaxis may consist of intrathecal chemotherapy (methotrexate, cytarabine,
corticosteroids), high-dose chemotherapy (methotrexate, cytarabine, L-asparaginase), or
CNS irradiation. Patients with ALL with increased risk for CNS disease include those with
an elevated WBC count, an elevated lactate dehydrogenase, a traumatic lumbar puncture,
and T-lineage ALL. Although APL may relapse in the CNS, relapse is uncommon in
patients with a presenting WBC less than 10,000/μL. CNS evaluation and prophylaxis are
not routinely recommended in patients with APL. There is no role for CNS prophylaxis in
elderly patients with AML without neurologic deficits. Although CNS disease must be
considered in a patient with AML and headaches, a new headache caused by CNS leukemia
on day 9 of induction therapy would be unusual and treatment could be delayed until
proper evaluation is completed.

Question 29.22 The answer is D.


Dexamethasone has improved penetration in the CNS. Given the high risk of CNS disease
in patients with ALL, dexamethasone is preferred over prednisone.

Question 29.23 The answer is C.


Maintenance therapy for ALL should be administered for 2 to 3 years since attempts to
shorten to 12 to 18 months have been associated with decreased survival.

Question 29.24 The answer is A.


Philadelphia chromosome-positive ALL carries a high-risk prognosis, but this has
improved with the ABL tyrosine kinase inhibitor imatinib. Second-generation inhibitors
such as dasatinib and nilotinib may also prove of value in the future. Alemtuzumab is an
anti-CD52 monoclonal antibody that has demonstrated significant efficacy in the
treatment of CLL. It has not been evaluated in ALL, which typically does not express
CD52. Gemtuzumab ozogamicin is an anti-CD33 monoclonal antibody cross-linked to a
cytotoxin, calicheamicin. It has been shown to improve outcomes in relapsed AML.
Sunitinib is a nonspecific tyrosine kinase inhibitor. It has significant activity against FLT3
and may prove of value in the treatment of AML with FLT3-ITD.

Question 29.25 The answer is B.


Stem cell transplantation for AML remains a developing field. Current studies and meta-
analysis have not found benefit in CR1 for patients with favorable cytogenetics (e.g.,
inv[16] and t[15;17]). Benefit is uncertain in patients with normal cytogenetics. Future
trials may be able to assess the benefit of transplant in CR1 in higher-risk subpopulations
of normal cytogenetics (e.g., FLT3-ITD or DNMT3A mutations). Patients aged less than 55
years who have unfavorable cytogenetics (e.g., –5, –7, complex cytogenetics) appear to
benefit from myeloablative transplantation in CR1. However, older patients tolerate this
approach less well, with increased treatment-related morbidity and mortality. Decision
making in older patients must be individualized, and nonmyeloablative transplant
approaches are usually preferred.

Question 29.26 The answers are A and B.


NPM1 gene encodes a protein that functions as a molecular chaperone between the
nucleus and cytoplasm. Mutations involving this gene are found in 50% to 60% of all AML
patients with normal karyotype, and less frequently are seen in patients with abnormal
cytogenetics. Aberrant cytoplasmic localization of NPM1 is associated with point
mutations in the gene. In the absence of FLT3-ITD, the NPM1 mutation is associated with
favorable prognosis.

Corresponding chapters in Cancer: Principles & Practice of Oncology, Tenth Edition: 106 (Molecular Biology of Acute
Leukemias) and 107 (Management of Acute Leukemias).
30 Chronic Leukemias
Pavan Bhamidipati and Keith Stockerl-Goldstein

QUESTIONS
Each of the numbered items below is followed by lettered answers. Select the ONE lettered answer
that is BEST in each case unless instructed otherwise.

Question 30.1 The Sokal score was developed to predict the probability of disease
progression in CML. Which of the following variables are used to calculate this score?
A. Percentage of blasts in peripheral blood
B. Platelet count
C. Spleen size (centimeters below the costal margin)
D. Age of the patient
E. All of the above

Question 30.2 Which of the following cytogenetic response is CORRECT?


1. Partial cytogenetic response is ≥35% of Ph-positive metaphases on bone marrow biopsy
2. Minor cytogenetic response is ≥35% of Ph-positive metaphases on bone marrow biopsy
3. Major cytogenetic response is 0% to 35% of Ph-positive metaphases on bone marrow
biopsy
4. Partial cytogenetic response is 1% to 35% of Ph-positive metaphases on bone marrow
biopsy
5. Complete cytogenetic response is 0% of Ph-positive metaphases on bone marrow biopsy
6. Major cytogenetic response is 35% to 99% of Ph-positive metaphases on bone marrow
biopsy
A. 1, 3, 5 only
B. 1,2,3,5 only
C. 2,3,4,5 only
D. 2,4,5,6 only

Question 30.3 Real-time quantitative polymerase chain reaction (RQ-PCR) has emerged as
the most effective and sensitive way to assess response to therapy in CML and has the added
flexibility of utilizing peripheral blood instead of bone marrow for the measurement of
cytogenetic response. How does molecular response correspond to complete cytogenetic
response (CCyR)?
A. 2-log reduction in transcript levels or 1% International scale (IS) or MR2
B. 3-log reduction in transcript levels or 0.1% IS or MR3
C. 4-log reduction in transcript levels or 0.01% IS or MR4
D. 3-log reduction in transcript levels or 1% IS or MR2

Question 30.4 A 59-year-old male with a history of coronary artery disease and diabetes
mellitus was diagnosed with chronic-phase chronic myeloid leukemia (CML). What is the
most appropriate therapeutic intervention?
A. Imatinib 400 mg QD
B. Dasatinib 100 mg QD
C. Nilotinib 300 mg BID
D. Ponatinib 30 mg QD

Question 30.5 A 60-year-old female has been under your care for chronic-phase CML for the
past 2 years. She achieved complete response on nilotinib. She subsequently developed
thrombocytopenia with elevated RQ-PCR and a bone marrow biopsy showed 42% Ph-positive
metaphases. Mutational analysis showed T315I mutation in the ABL domain. What is the
appropriate treatment for this patient?
A. Imatinib
B. Ponatinib
C. Dasatinib
D. Nilotinib

Question 30.6 A 56-year-old female was diagnosed with chronic-phase CML 6 months ago
and was started on imatinib at 400 mg QD. Treatment however, was discontinued due to skin
rash and diarrhea and replaced by dasatinib at 100 mg QD. She came to the ER 2 weeks later
with new onset of dyspnea and found to have moderate bilateral pleural effusions. What is
the appropriate management?
A. Diagnostic thoracentesis
B. Temporary dasatinib discontinuation
C. Prednisone 0.5 mg/kg for 1 or 2 weeks
D. Start diuresis with furosemide
E. B, C, and D

Question 30.7 What is the appropriate frequency of testing for responses in CML patients
who are currently on imatinib therapy?
A. Bone marrow biopsy every 6 months, RQ-PCR every 3 months for first 2 years, then
q4–6 months
B. Bone marrow biopsy every 3 months, RQ-PCR every 6 months for first 2 years, then
q4–6 months
C. Bone marrow biopsy every 12 months, RQ-PCR every 3 months for first 2 years, then
q4–6 months
D. None of the above

Question 30.8 What constitutes a relapse in a patient with CML who achieved complete
hematologic response while on tyrosine kinase inhibitor therapy? (Select two correct
responses)
A. Rise of RQ-PCR from 0.01% to 0.05%
B. Drop in platelets from 180,000 to 80,000 ten months into therapy
C. Rise in the Ph-positive metaphase levels from 0% to 20% on bone marrow biopsy ten
months into therapy
D. None of the above

Question 30.9 What are the expected landmarks that are known to be associated with better
outcomes while on imatinib therapy?
A. Complete hematologic response (CHR) at 3 months
B. Major cytogenetic response (MCyR) at 6 months
C. Complete cytogenetic response (CCyR) at 12 months
D. 3-log reduction in BCR-ABL transcripts (MR3) measured by RQ-PCR at 12 months
E. All of the above

Question 30.10 With the advent of tyrosine kinase inhibitors (TKIs), allogeneic stem cell
transplantation (allo-HCT), once a primary therapy for CML, is now reserved for specific
CML patients. What are the indications for allo-HCT in CML? (Select two correct responses)
A. Progression to CML accelerated/blast phase
B. Resistance to multiple lines of TKIs but continues to be in chronic phase
C. Availability of a matched sibling donor
D. Young patients (<50 years of age)

Question 30.11 What are the current accepted criteria to define accelerated phase in CML?
A. Progressive splenomegaly and/or myelofibrosis
B. Bone marrow or peripheral blood blasts ≥15% but <30%
C. Platelet count <100 × 109/L unrelated to therapy
D. Clonal evolution in a Ph-chromosome–positive clone
E. All of the above

Question 30.12 BCR-ABL translocation leads to the synthesis of the fusion oncoprotein that
has constitutive tyrosine kinase activity seen in CML and Ph-chromosome–positive ALL. What
are the resultant BCR-ABL fusion proteins that are commonly found in these patients?
1. p210BCR-ABL
2. p230BCR-ABL
3. p190BCR-ABL
4. p250BCR-ABL
A. 1, 3 only
B. 1, 2, 3 only
C. 1, 2, 4 only
D. 2, 3 only

Question 30.13 What are the mutations that are commonly observed in CML blast-phase
transformation? (Select three correct responses)
A. Homozygous deletion of p16 tumor suppressor gene
B. Lengthened telomeres
C. Shortened telomeres
D. Loss of BCR-ABL
E. Acquired additional cytogenetic abnormalities.

Question 30.14 Important factors associated with poor prognosis in CLL include the
following:
A. CD38 positivity
B. IGHV-unmutated status
C. ZAP-70 expression
D. Short lymphocyte doubling time (<6 months)
E. All of the above

Question 30.15 A 49-year-old business man was diagnosed with CML 6 months ago and was
started on imatinib therapy. He initially achieved CHR and MCyR at 3 and 6 months,
respectively. He does travel frequently and missed a follow-up appointment and saw you at
12 months. A repeat bone marrow biopsy showed 45% Ph-positive metaphases. What is the
best next approach?
A. Change imatinib to dasatinib/nilotinib
B. Send blood for ABL kinase domain sequencing for possible mutations
C. Address the importance of therapy adherence
D. Refer to a transplant physician for allo-HCT

Question 30.16 What is TRUE about autoimmune complications in CLL?


A. The autoantibodies are often polyclonal and are produced by nonleukemic clone
B. The severity of the autoimmune phenomenon does not necessarily correlate with the
severity of CLL
C. Can lead to hemolytic anemia, thrombocytopenia, granulocytopenia, or pure red cell
aplasia
D. All of the above

Question 30.17 A 67-year-old male presented with a 3-month history of progressive weight
loss, weakness, and night sweats. Initial examination showed massive splenomegaly. Initial
CBC revealed WBC of 2.1 cells/uL (differential 67% lymphocytes, 15% Neutrophils, 12%
monocytes), platelets are 45,000/uL. Peripheral smear showed few large lymphocytes with
open chromatin and cytoplasmic projections. Bone marrow biopsy showed that these cells are
positive for CD19, CD20, CD22, CD25, CD103, FMC7, and surface immunoglobulins but
negative for CD5 and CD23. What is the recommended treatment? (Select two correct
responses)
A. Cladribine at 0.1 mg/kg/d—continuous infusion for 7 days
B. Cladribine at 0.14 mg/kg/d—2-hour infusion for 5 days
C. Interferon-α
D. Alemtuzumab

Question 30.18 A 56-year-old man was recently diagnosed with Rai stage 0 CLL. He read
about newer therapies and wanted to know when he can start treatment. Which of the
following findings would be an indication for initiation of systemic therapy?
A. Availability of newer treatment modalities such as ibrutinib, idelalisib, and CAR-T cells
B. An increase in his absolute lymphocyte count from 25 × 109/L to 40 × 109/L over the
last 2 years
C. Recent diagnosis of hypogammaglobulinemia
D. Fever of 38°C for the last 2 weeks without evidence of infection

Question 30.19 Which of the following statements are TRUE about rituximab in the
treatment of CLL?
A. By targeting CD 20 on normal B-cells, it can lead to hypogammaglobulinemia requiring
immunoglobulin replacements.
B. Maintenance rituximab has shown to improve overall survival in CLL.
C. Rituximab monotherapy with 375mg/m2 IV weekly in frontline setting can achieve 60%
CR rates.
D. Along with fludarabine and cyclophosphamide, chemoimmunotherapy with rituximab
can achieve excellent remission rates.

Question 30.20 A 61-year-old female with a history of CLL and 17p deletion was treated
with six cycles of FCR and subsequently achieved complete remission. One year later, she
started noticing increased diffuse lymph node swelling and progressive unintentional weight
loss. Initial workup demonstrated WBC count of 1.2 × 105 cu mL and a platelet count of
65,000 cu mL. A repeat lymph node biopsy showed CLL. What is the next best treatment?
A. Ibrutinib 420 mg PO Q daily
B. Ibrutinib 840 mg PO Q daily
C. Ofatumumab
D. Fludarabine + cyclophosphamide + rituximab
E. Allo-HCT
F. None of the above

Question 30.21 Which actionable mutation is present in almost all patients with hairy cell
leukemia?
A. EGFR
B. FLT-3
C. BRAF
D. No actionable mutation

Question 30.22 Which of the following statements regarding monoclonal B lymphocytosis


(MBL) are TRUE?
A. Rate of progression of MBL to CLL is 1% to 2% per year
B. Can be an incidental finding in 3.5% of otherwise normal individuals aged >40
C. Early initiation of therapy is associated with improved survival
D. It is a benign finding and is of no significance
ANSWERS

Question 30.1 The correct answer is E.


The Sokal risk score was developed during the pre-tyrosine kinase inhibitor (TKI) era of
CML. It utilizes a complex equation that takes four patient characteristics into account.
These are patient age, percentage of blasts in peripheral blood, platelet count, and spleen
size (centimeters below the costal margin). Based on the score, patients were divided into
three risk groups and as a predictor of response to non-TKI therapy. It was found to be
still valid in predicting the probability of achieving an optimal response to imatinib.

Question 30.2 The correct answer is C.


Cytogenetic response in CML is based on the conventional bone marrow cytogenetic
analysis and is defined as follows:

One of the most prognostically significant landmarks is achieving CCyR at 12 months and
many other parameters are based on the likelihood of obtaining this response. For
example, for patients who do not achieve a MCyR by 6 months, the probability of
obtaining a CCyR at 12 months is 50% or lower.

Question 30.3 The correct answer is A.


Peripheral blood RQ-PCR is a sensitive test that can be used in monitoring for minimal
residual disease (MRD) in CML. It is usually expressed as an absolute ratio of BCR-ABL1
transcripts to a control gene such as ABL1 or as log reductions from the 100% value. This
value is adjusted to a standardized reference IS. It is generally accepted that CCyR
corresponds to an approximately 2-log reduction in transcript levels (MR2) or 1% IS.
Major molecular response (MR3) is defined as a 3-log reduction in transcript levels or
0.1% IS. A complete molecular response (CMR) is generally defined as the absence of
detectable BCR-ABL transcripts, a status that depends on the technology and criteria for
negativity employed, with the limits of technology being somewhere between MR4 and
MR5.

Question 30.4 The correct answer is A.


Currently, imatinib, dasatinib, and nilotinib are the FDA-approved TKIs for frontline use.
The choice of therapy depends on patient comorbidities and the individual TKI side-effect
profile. Nilotinib associated with hyperglycemia is peripheral vascular disease and is
probably not the best choice for this patient. The newer, more potent TKIs induce faster
cytogenetic and molecular responses, but in only the imatinib versus nilotinib study this
has translated into a significant difference in progression-free survival (PFS) at 3 years.
Dasatinib is associated with pleural effusions, pulmonary hypertension and can be
associated with significant morbidity in patients with pre-existing pulmonary disease.
Imatinib does not have specific contraindications. Until a solid body of evidence that
newer TKIs carry a superior survival benefit, imatinib remains a solid first-line choice.
The approved doses in the frontline setting are imatinib 400 mg QD, nilotinib 300 mg
BID, and dasatinib 100 mg QD. Second-line approved doses are nilotinib 400 mg BID,
dasatinib 100 mg QD, bosutinib 500 mg QD, and ponatinib at 30 mg Po QD.

Question 30.5 The correct answer is B.


A loss of disease control in the first 5 years of therapy occurs in approximately 15% of
chronic-phase patients treated with imatinib. The majority (40%) of the patients who
relapse have point mutations in the ABL kinase domain that render the kinase less
sensitive to imatinib. Substitution of threonine by isoleucine in position 315 of the kinase
domain (T315I) is the most common mutation that emerges in patients with resistance to
newer TKIs. Ponatinib is the only TKI that has activity against T315I mutation. The
optimal dose is not defined in this setting although 45 mg daily was used in the PACE
clinical trial. When this dose was used in the frontline setting, it led to several cardiac,
cerebral, and peripheral vascular thrombotic events in up to 27% of patients and have
included fatal myocardial infarction and stroke that led to the early termination of phase
III study and further led to a recommended initial dose of 30 mg daily.

Question 30.6 The correct answer is E.


Pleural effusions are one of the common side effects of dasatinib and reported at an
incidence of 14% to 54%. They can occur at any time during the therapy and can recur
even after a dose reduction. Routine surveillance chest radiographs and diagnostic
thoracentesis are not required and temporary discontinuation of the drug is often enough
for spontaneous resolution. In a symptomatic patient such as the current woman, this
process can be hastened through diuresis, corticosteroids (often as 0.5 mg/kg of
prednisone for 1 or 2 weeks). Pulmonary hypertension is also a rare side effect of
dasatinib.

Question 30.7 The correct answer is A.


The optimal monitoring for the achievement of landmarks for someone being treated with
imatinib is as follows: Bone marrow biopsy every 6 months until CCyR is achieved, RQ-
PCR every 3 months for first 2 years, then every 4 to 6 months. With the advent of RQ-
PCR standardized to international scale (IS), peripheral blood can be used for molecular
surveillance and the bone marrow biopsies may be avoided in the future.

Question 30.8 The correct answers are B and C.


Relapse on therapy in CML is defined as a loss of a previously obtained response, such as
a loss of CHR, loss of CCyR, or increase in RQ-PCR transcript level. Minor changes in RQ-
PCR are common and a 2–3 factor increase on international scale is considered normal as
accuracy of the test varies substantially depending on the depth of response. It is
important to confirm the rise through a repeat testing before changing the therapy.
Decrease in the previously normal platelet count, constitute a loss of CHR and the rise of
Ph-positive metaphase levels from 0% to 20% represents loss of CCyR. Both are
considered relapse and warrant for change in therapy.

Question 30.9 The correct answer is E.


Important landmarks that are considered to be associated with good prognosis are CHR at
3 months, MCyR at 6 months, CCyR at 12 months, and 3-log reduction in BCR-ABL
transcripts (MR3) measured by RQ-PCR at 12 months. Of all these parameters, CCyR at
12 months is considered the most important prognostic landmark and the other
parameters predict the likelihood of achieving this response. Of note, patients who
achieve a CCyR at 12 months but did not achieve MR3 tend to achieve this eventually
without any further intervention. Patients who do not achieve these landmarks on first-
line therapy constitute primary cytogenetic resistance, an event that occurs rarely, is
associated with persistent cytopenias and considered to be a poor risk category.

Question 30.10 The correct answers are A and B.


With the advent of imatinib and newer TKIs, hematopoietic cell transplantation (HCT) for
chronic-phase CML has decreased significantly. But HCT still remains an effective therapy
for patients who are resistant to TKIs and remain in chronic phase and for patients with
advanced phase disease whose leukemia can be restored to the chronic phase. Reduced-
intensity conditioning regimens (AKA nonmyeloablative regimens) have shown to be
superior to myeloablative HCT in patients >60 years of age with disease-free survival
ranging from 40% to 85% at 3 to 5 years. However, infections and acute and chronic graft
versus host disease remain significant problems.

Question 30.11 The correct answer is E.


The current accepted criteria for the diagnosis of accelerated phase include (1)
progressive splenomegaly and myelofibrosis; (2) bone marrow or peripheral blood blasts
≥15% but <30%; (3) bone marrow or peripheral blood blasts plus promyelocytes ≥30%;
(4) bone marrow or peripheral blood basophils ≥20%; (5) platelet count <100 × 109/L
unrelated to therapy; and (6) clonal evolution in a Ph-chromosome–positive clone. The
blastic phase is defined as bone marrow or peripheral blood blast ≥30% or the presence
of extramedullary blast infiltrates.

Question 30.12 The correct answer is B.


Breakpoints within BCR localize to one of three breakpoint cluster regions that lead to
three different sized BCR-ABL1 transcripts in CML. More than 90% of CML patients and
one-third of Ph-positive ALL patients express the 210 kDa due to a breakpoint in major
breakpoint cluster region (M-bcr), which spans exons e12–e16 (formerly exons b1–b5).
The remainder of Ph-positive ALL patients and rare CML cases harbor breakpoints in
minor breakpoint cluster region (m-bcr), generating an e1a2 transcript that is translated
into p190BCR-ABL1. A third breakpoint down-stream of exon 19 (e19a2) in the micro
breakpoint cluster region (μ-bcr) gives rise to p230BCR-ABL1 and is associated with
neutrophilia.

Question 30.13 The correct answers are A, C, and E.


Disease progression is believed to be due to the accumulation of molecular abnormalities
that lead to a loss of terminal differentiation capacity of the leukemic clone, which
continues to depend on BCR-ABL1 activity. The molecular basis is poorly defined, but
point mutations or deletions in the p53 tumor suppressor gene have been observed in up
to 25% of patients with myeloid blast crisis, and as many as 50% of patients with
lymphoid transformation show a homozygous deletion in the p16 tumor suppressor gene.
Studies have demonstrated that telomere length decreases with disease progression from
chronic to blast phase. Clonal evolution in a Ph-chromosome–positive clone is one of the
causative and accepted criteria for progression of CML into advanced phase.

Question 30.14 The correct answer is E.


CD38 positivity, IGHV-unmutated status, ZAP-70 expression, and Short lymphocyte
doubling time (<6 months) are associated with shorter survival. ZAP-70 is normally
expressed on T-cells. Its expression on CLL cells (B cells) is associated with activation of
these malignant cells. Zap-70 expression is associated with IGHV-unmutated state,
increased disease progression, and worse overall survival. Other factors associated with
shortened survival in CLL are diffuse pattern of bone marrow infiltration, advanced age
and male gender, abnormal karyotype, high serum levels of β2-microglobulin and soluble
CD23, and a CLL-PLL category (11% to 54% prolymphocytes in the blood).

Question 30.15 The correct answer is C.


This patient developed relapsed disease based on the loss of MCyR and that he did not
attain CCyR at 12 months probably secondary to noncompliance. Before considering the
change of therapy, patient compliance and development of resistance to TKIs need to be
addressed. Noncompliance is one of the important reasons for loss of therapy and
adherence to therapy needs to be addressed in every visit. In a compliant patient,
resistance to therapy should be considered and investigated by testing for ABL kinase
domain mutations (particularly for T315I mutation).

Question 30.16 The correct answer is D.


Autoantibodies in CLL are often polyclonal and are Ig G, indicating that they are not
produced by the leukemic clone. They are usually targeted against hematopoietic cells and
can lead to cytopenias of any of the three lineages (RBC, WBC, and platelets) or can lead
to hemolytic anemia. The severity of the autoimmune phenomenon does not necessarily
correlate with the severity of CLL, and such events may develop in patients whose disease
is responding to therapy.

Question 30.17 The correct answers are A and B.


This patient has hairy cell leukemia (HCL), a rare B-cell lymphoproliferative disorder that
comprises 2% of all leukemias. Classic “hairy cells” are seen on the peripheral smear
albeit less frequently. These cells are positive for B-cell markers CD19, CD20, CD22, and
CD25. CD103 positivity obviated the need for tartrate-resistant acid phosphatase (TRAP)
testing. Pentostatin and cladribine are the major treatment options for HCL. Cladribine at
0.1 mg/kg/d given either as continuous infusion for 7 days or at 0.14 mg/kg/d as 2-hour
infusion for 5 days is the preferred frontline therapy. Interferon-α is reserved for relapsed
HCL. Prolymphocytic leukemia (PLL), an important differential diagnosis for HCL, is
often associated with lymphocytosis, the prolymphocytes have prominent nucleoli and are
negative for CD25 and CD103. Nucleoside analogues and alemtuzumab are treatment
options for PLL.

Question 30.18 The correct answer is D.


The IWCLL revised criteria for active disease, an indication to initiate treatment include
constitutional symptoms attributable to CLL: weight loss (>10% of baseline weight
within the preceding 6 months), extreme fatigue (ECOG performance status ≥2), fever
(temperature higher than 38°C or 100.5°F for at least 2 weeks) or night sweats without
evidence of infection; evidence of progressive bone marrow failure characterized by the
development of or worsening of anemia and/or thrombocytopenia; autoimmune
hemolytic anemia and/or autoimmune thrombocytopenia, poorly responsive to
corticosteroid therapy; massive (>6 cm below the left costal margin) or progressive
splenomegaly; bulky (>10 cm in longest diameter) or progressive lymphadenopathy; and
progressive lymphocytosis defined as an increase in the absolute lymphocyte count by
>50% over a 2-month period, or a doubling time predicted to be <6 months.
Hypogammaglobulinemia or monoclonal gammopathy alone is not a sufficient criterion to
initiate therapy. Disease course in low- or intermediate-risk groups is not uniform and can
be indolent or rapidly progressive. Of note, approximately one-third of patients with
early-stage never require therapy. It is known that early treatment of low-grade patients
does not translate into better survival. Newer better therapies such as ibrutinib, idelalisib,
chimeric antigen receptor–bearing T-cell (CAR-T cells), BCL-2 inhibitors, and lenalidomide
have been shown to be effective in relapsed and refractory CLL. However, it is unknown
whether treating the low-risk patients with these novel agents improves survival.

Question 30.19 The correct answer is D.


Rituximab is a chimeric immunoglobulin IgG1 CD20 monoclonal antibody (mAb) that
became an important adjunct in the treatment of CLL. It does not have significant
activities in CLL when used as monotherapy but when used in combination with
fludarabine and cyclophosphamide it demonstrated superior remission rates (95% in FCR
arm vs. 88% with FC), improved PFS (52 months vs. 33 months with FC) and OS (84% vs.
79% at 38 months with FC). Maintenance rituximab has not been routinely used in
patients with CLL. Although rituximab targets normal B cells, studies have shown that
there is no significant effect on immunoglobulin levels.

Question 30.20 The correct answer is A.


Del(17p) is associated with poor prognosis and shortened survival in patients with CLL.
Ibrutinib is a Bruton tyrosine kinase (BTK) inhibitor that has shown promising overall
response rate in the relapsed and fludarabine-refractory setting. Similar responses were
noted across risk categories, including for high-risk del(17p), heavily pretreated, and
advanced stage disease. The 2-year PFS rate was 75% and OS rate was 83% with durable
responses. Ibrutinib is approved at a dose of 420 mg PO once daily in the
relapsed/refractory setting and as such is the preferred choice of treatment for this patient
population. While allo-SCT was associated with long-term remissions and possible cure,
with the availability of newer therapies, it may be reserved for patients with high-risk and
relapsed CLL.

Question 30.21 The correct answer is C.


BRAF V600E mutation is present in almost all cases of hairy cell leukemia (HCL). It is
currently being explored as a potential therapeutic target using vemurafenib in relapsed
HCL patients.

Question 30.22 The correct answer is A and B.


Monoclonal B lymphocytosis (MBL) is an often incidental diagnosis of defined as a
population of clonal CD5+/19+/23+ B cells in up to 3.5% of otherwise normal
individuals over the age of 40. These individuals are often asymptomatic with no
adenopathy or splenomegaly. Furthermore, as many as 13% of family members of
patients with familial CLL harbor a population of cells with an immunophenotype
consistent with CLL, but do not fulfill CLL diagnostic criteria. It is estimated that the rate
of progression from MBL to CLL is 1% to 2% per year. Currently, there is no indication to
perform screening for MBL.

Corresponding chapters in Cancer: Principles & Practice of Oncology, Tenth Edition: 108 (Molecular Biology of Chronic
Leukemias), 109 (Chronic Myelogenous Leukemia), and 110 (Chronic Lymphocytic Leukemias).
31 Plasma Cell Neoplasms
Ravi Vij and Jesse Keller

QUESTIONS
Each of the numbered items below is followed by lettered answers. Select the ONE lettered answer
that is BEST in each case unless instructed otherwise.

Question 31.1 A healthy and active 62-year-old man presents to his primary care physician
for a routine annual physical. A complete metabolic panel is significant for an elevated total
protein of 10 mg/dL, an albumin of 4 mg/dL, a creatinine of 0.6 mg/dL, and a calcium of 8.8
mg/dL. A complete blood count shows a white blood cell count of 5.2, a hemoglobin of 12
g/dL, and a platelet count of 242. Subsequent serum protein electrophoresis and
immunofixation shows an IgG kappa monoclonal peak of 0.9 g/dL. Serum-free light chains
show a total kappa-free light chain concentration of 242 mg/dL and a lambda-free light chain
concentration of 1.2 mg/dL. A skeletal survey shows no evidence of lytic lesions and a bone
marrow biopsy reveals a monoclonal plasma cell population comprising 70% of the core
biopsy sample. Which of the following management options should be pursued next?
A. Initiation of antimyeloma therapy
B. Repeat SPEP in 12 months
C. Obtain a HeavyLite Assay
D. Fusion PET/MRI scanning to assess need for therapy

Question 31.2 A 65-year-old man presents to a local emergency room with fatigue,
myalgias, and lightheadedness. Laboratory evaluation reveals a white blood cell count of 7.5,
a hemoglobin of 6.3 g/dL and a platelet count of 135. Chemistry panel shows a creatinine of
1.56 mg/dL and a calcium of 14.0 mg/dL. Workup of his anemia includes a serum protein
electrophoresis that shows a monoclonal protein at a concentration of 4.0 g/dL. Serum
immunofixation reveals an IgA kappa monoclonal protein. Bone marrow biopsy shows a
kappa-restricted plasma cell population comprising 30% of the core biopsy. Cytogenetics
reveal hypodiploidy and FISH studies show t(4;14). Which of the following best characterizes
the risk category of this patient’s multiple myeloma?
A. High Risk due to FISH findings of t(4;14)
B. High Risk due to cytogenetic findings of hypodiploidy
C. Intermediate Risk by FISH and cytogenetics
D. Standard Risk by FISH and cytogenetics

Question 31.3 A 52-year-old woman presents to her Oncologist 1 year following an


autologous stem cell transplantation for multiple myeloma. She has been continued on
maintenance therapy following her transplant and has tolerated this well. She reports that she
watched a television documentary about genomics a week prior to her appointment and asks
her physician what genomic sequencing has revealed in multiple myeloma. Which of the
following is the best response?
A. Genomic sequencing has revealed a conserved set of consistent mutations in TP53 and
CCND1 for patients with multiple myeloma.
B. Mutations in VHL are common.
C. Deep sequencing has revealed that there is a consistent pattern of clonal evolution in
multiple myeloma.
D. There have been few consistently mutated genes, but the most common include KRAS
and NRAS.

Question 31.4 A 69-year-old woman is diagnosed with an IgG lambda multiple myeloma.
Her initial labs reveal a monoclonal peak of 4.5 g/dL, with lambda-free light chains of 120
mg/dL and kappa-free light chains of 1.1 mg/dL. A bone marrow biopsy showed 50%
involvement by a lambda-restricted plasma cell population. She began induction with
lenalidomide and dexamethasone and attained a very good partial response after four cycles
of therapy. She undergoes autologous stem cell transplant and on workup 100 days post-
transplant, a bone marrow biopsy shows no evidence of persistent plasma cells. Serum
protein electrophoresis and immunofixation are negative for any evidence of monoclonal
protein. Serum-free light chains are repeated and are normal. She has minimal residual
disease testing performed as part of a clinical trial, which shows her to have persistent
disease by multiparameter flow cytometry (MFC). What would be the appropriate
information to pass onto the patient regarding this finding?
A. She needs to have the testing repeated using next-generation–based sequencing
technology.
B. The patient should undergo repeat ASCT with a goal to obtain minimal residual disease
(MRD) negativity.
C. MRD positive patients have inferior PFS when compared to patients achieving stringent
CR who are MRD negative.
D. There are no differences in outcomes among patients achieving a CR regardless of MRD
status.

Question 31.5 A 59-year-old man has a new diagnosis of multiple myeloma made after he
presented with a monoclonal peak of 6.0 g/dL, IgG-Lambda by serum immunofixation. CBC
showed a hemoglobin of 9.5 g/dL and a metabolic panel revealed no abnormalities. Bone
marrow biopsy reveals evidence of 65% involvement by a lambda-restricted monoclonal
plasma cell population. He is deemed to be an appropriate candidate for autologous
transplantation. He starts therapy with bortezomib, lenalidomide, and dexamethasone which
he tolerates well. After 3 cycles of therapy, his M-spike has improved to 4.8 g/dL, and after 6
cycles of therapy his M-spike is 4.0 g/dL. Which of the following is the next best step in
management?
A. Continue bortezomib, lenalidomide, and dexamethasone until nadir value achieved and
then proceed to autologous transplantation
B. Proceed to autologous stem cell transplantation
C. Carfilzomib-based salvage therapy for 4 to 6 cycles and then proceed to autologous
transplantation
D. Switch to carfilzomib-based regimen and abandon plans for autologous transplantation

Question 31.6 A patient with a history of multiple myeloma presents to your clinic with a
newspaper clipping describing some recent successes with monoclonal antibodies in this
disease. The patient is curious about the role of these potential new drugs in therapy and
inquires about their mechanism of action. Which of the following is accurate regarding the
new medications in this class?
A. In a phase III trial, the PFS difference noted at 1 year was maintained at 2 years on
therapy with elotuzumab, lenalidomide, and dexamethasone versus lenalidomide and
dexamethasone.
B. Daratumumab had a high rate of discontinuation due to adverse events.
C. Daratumumab has no activity in patients with extra-medullary disease.
D. In a phase III trial, elotuzumab in combination with lenalidomide and dexamethasone
produced responses even in lenalidomide refractory patients.

Question 31.7 A 63-year-old man with a history of multiple myeloma presents to clinic for
ongoing evaluation. He was diagnosed 3 years ago, and received induction treatment with
bortezomib, lenalidomide, and dexamethasone followed by autologous stem cell
transplantation. On workup today, he has evidence of progressive disease with a rising
monoclonal protein from 1.0 g/dL to 3.2 g/dL. His performance status is ECOG 0, and the
treating physician makes the decision to initiate panibinostat, bortezomib, and
dexamethasone. Which of the following regarding panibinostat should the physician inform
the patient prior to initiation?
A. Potential for serious and possibly fatal arrhythmia
B. Risk of Stevens–Johnson syndrome
C. Diarrhea is self-limiting and transient
D. Risk for hemolytic uremic syndrome

Question 31.8 A 65-year-old man with a history of multiple myeloma presents for initial
evaluation. He was diagnosed 1 year ago and underwent initial induction therapy with
bortezomib, cytoxan, and dexamethasone, followed by autologous stem cell transplantation
approximately 6 months ago. He was not treated with maintenance therapy following
transplant due to his personal preference. He presents now with worsening anemia
(hemoglobin 9 g/dL) and a rising M-protein level (0.8 g/dL to 4.5 g/dL). What is the most
appropriate statement regarding the role of carfilzomib therapy in this setting?
A. In a phase III trial, patients treated with lenalidomide and dexamethasone had improved
progression-free survival compared to those treated with carfilzomib, lenalidomide, and
dexamethasone.
B. In a phase III trial, patients treated with the combination of carfilzomib, lenalidomide,
and dexamethasone had inferior health-related quality of life compared to those treated
with lenalidomide and dexamethasone alone.
C. In a phase III trial, the benefit of carfilzomib (20/56 mg/m2) and dexamethasone was
limited to patients who had no prior exposure to bortezomib.
D. In a phase III trial, carfilzomib (20/56 mg/m2) and dexamethasone was associated with
a doubling of PFS when compared to bortezomib (1.3 mg/m2) and dexamethasone.

Question 31.9 A 57-year-old previously healthy woman presents to her primary care
physician with fatigue and weight loss over the preceding 3 months. Physical examination
reveals palpable lymphadenopathy in the axilla bilaterally. Workup is significant for a WBC
count of 7.3, a hematocrit of 21%, and a platelet count of 42. A metabolic panel is within
normal limits. SPEP and immunofixation reveal an IgM monoclonal protein and a bone
marrow biopsy shows a lymphoplasmacytic infiltrate comprising 20% of the marrow. Serum
viscosity is within normal limits. She is initially treated with bortezomib, rituximab, and
dexamethasone with a partial response. One year later she now has a recurrence of
symptoms. A tumor sample is sent for exome sequencing studies. Which of the following is
most likely to predict for best response to BTK inhibitor therapy?
A. MYD88 L365P mutation positive and CXCR4 mutation negative
B. MYD88 L365P mutation negative and CXCR4 mutation negative
C. MYD88 L365P mutation positive and CXCR4 mutation positive
D. Presence of CXCR4 WHIM mutation irregardless of MYD88 status

Question 31.10 A 52-year-old man presents to your clinic for an initial consultation. He was
referred after he was noted to have lower extremity edema and further workup revealed the
presence of a nephrotic syndrome. He underwent a renal biopsy which showed evidence of
amyloid deposition, positive by Congo red staining. His labs revealed a kappa-free serum
light chain concentration of 140 mg/L and a lambda-free light chain concentration of 2 mg/L.
His NT-proBNP is 10,000 ng/L and his troponin is 0.09 ng/mL. His performance status is
rated as an ECOG 1. What should be the next step in management?
A. Start cyclophosphamide, bortezomib, and dexamethasone
B. Autologous stem cell transplantation with melphalan 140 g/m2
C. Autologous stem cell transplantation with melphalan 200 g/m2
D. Start therapy with ixazomib-based therapy
ANSWERS

Question 31.1 The correct answer is A.


The International Myeloma Working Group released an update to the criteria for
diagnosis of multiple myeloma in 2014 [Rajkumar et al. Lancet Oncol. 2014;15:e538–
e548]. Historically defined by the presence of the CRAB (hypercalcemia, renal failure,
anemia, or bony disease) criteria, denoting the presence of end-organ damage, the newly
updated criteria include a number of radiographic or biochemical criteria. These take into
account the potential benefits available to patients from expanding and improved
treatment options and data that show early intervention in high-risk asymptomatic
patients improves survival [Mateos et al. N Engl J Med. 2013;369:438–447]. The defining
criteria for multiple myeloma now include a clonal bone marrow plasma cell population
≥10% or biopsy-proven bony or extramedullary plasmacytoma and one or more of the
following defining elements:
• Evidence of end-organ damage attributed to the plasma cell disorder
• Hypercalcemia (>1 mg/dL above ULN or >11 mg/dL)
• Renal insufficiency (CrCl <40 or sCr >2 mg/dL)
• Anemia (Hbg >2 g/L below LLN or an Hbg <10 g/L
• Bone lesions (one or more osteolytic lesions on radiograph, CT or PET-CT)
• Any of the following biomarkers of malignancy:
• Clonal bone marrow population ≥60%
• Involved:uninvolved serum-free light chain ratio ≥100 (The involved free light chain
must be ≥100 mg/L)
• >1 focal lesions on MRI
Since this patient’s bone marrow biopsy reveals a monoclonal plasma cell population
encompassing 70% of the bone marrow, he meets the diagnosis of multiple myeloma
based on the newly established criteria. Additionally, this patient’s ratio of
involved:uninvolved serum-free light chains is >100, with an involved chain
concentration >100 mg/L. By current definition the patient has overt multiple myeloma
and will benefit from treatment with an active antimyeloma regimen. Under prior
definitions of multiple myeloma the patient would have been classified as smoldering
multiple myeloma, and repeating the SPEP and labs would be a reasonable approach. The
HevyLite assay is a novel technique of quantifying intact light and heavy chain
monoclonal proteins. It measures intact immunoglobulin proteins by measuring unique
junctional epitopes between heavy and light chain constant regions. This is particularly
useful for IgA monoclonal proteins which often have limited detection by serum protein
electrophoresis, but other roles in multiple myeloma are currently under investigation.
Fusion PET-MRI is an evolving imaging strategy in multiple myeloma and its role is
currently being evaluated.

Question 31.2 The correct answer is C.


Prognosis in multiple myeloma is dependent on several factors, including age,
comorbidities, stage, and disease biology. FISH and cytogenetic studies are ideal to assess
for the presence of genetic alterations and are crucial to pursue for newly diagnosed
patients. Integrating these factors, a risk classification system for newly diagnosed
multiple myeloma has been developed by the Mayo clinic, the Mayo Stratification for
Myeloma and Risk-adapted Therapy (mSMART) criteria [Mikhael et al. Mayo Clin Proc
2013;88:360–376]. This divides newly diagnosed patients into risk categories based on
cytogenetics, FISH, gene expression profiling (GEP) and plasma cell labeling index
studies. High-Risk patients are defined by a high-risk GEP signature, or FISH studies
showing deletion of 17p, t(14;16), or t(14;20). Intermediate-risk disease is denoted by a
high plasma cell S-phase labeling study, FISH showing t(4;14), 1q gain, cytogenetics with
a complex karyotype, deletion 13 or hypodiploidy. Standard risk disease encompasses all
other abnormalities. Mikhael et al. report overall survival and incidence rates for
mSMART risk classes as follows:
• High-risk: incidence of 20% with a median OS of 3 years
• Intermediate-risk: incidence of 20% with a median OS of 4 to 5 years
• Standard-risk: incidence of 60% with a median OS of 8 to 10 years
Risk-adapted strategies to improve these outcomes have been investigated in multiple
myeloma, particularly for patients with high-risk multiple myeloma. In the HOVON-
65/GMMG-HD4 trial [Sonneveld et al. J Clin Oncol. 2012;30(24):2946] newly diagnosed
multiple myeloma patients were randomized to bortezomib-based induction followed by
bortezomib post-transplant maintenance therapy versus thalidomide post-transplant
maintenance. Significant improvement in PFS and median OS was seen in 17p-deleted
patients who were randomized to bortezomib-based therapy. A further phase II study
[Nooka et al. Leukemia. 2014;28(3):690–693] evaluated the efficacy of lenalidomide,
bortezomib, and dexamethasone maintenance therapy following autologous
transplantation. Given for up to 3 years, followed by lenalidomide maintenance, the study
had a median PFS of 32 months and a 3-year OS of 93%. Further research is necessary to
determine the most appropriate strategy to mitigate adverse outcomes in patients with
high-risk disease.

Question 31.3 The correct answer is D.


The initial study evaluating the genomic architecture of multiple myeloma was published
in 2011 [Chapman et al. Nature. 2011;471:467–472]. These findings, which evaluated
sequencing of 38 tumor genomes revealed a lack of consistently conserved mutations
among individual multiple myeloma genomes. The most frequent mutations included
those in NRAS (9/38) and KRAS (10/38). Frequent mutations were also seen in genes
involved in RNA processing and protein homeostasis, including DIS3, XBP1, LRRK2, and
FAM46C. The NF-kB pathway was the most frequently affected pathway, with mutations
and rearrangements noted in 11 NF-kB pathway genes. Additional mutations were seen in
histone-modifying enzymes, including HOXA9. One genome contained a possibly clinically
actionable mutation in BRAF. An additional analysis genotyping 161 multiple myeloma
patients found 7 BRAF mutations, which raised the possibility of benefit from BRAF
inhibitors. Overall, there are few consistently mutated genes in the multiple myeloma
genome, with the most commonly mutated being NRAS and KRAS.
Two additional studies have investigated an additional 203 patients [Lohr et al. Cancer
cell. 2014. 25:91–101] and 67 patients [Bolli et al. Nat Commun. 2014;5] using whole-
genome and whole-exome sequencing to identify frequent mutations in multiple
myeloma. These studies supported the frequent mutations in KRAS, NRAS, FAM46C, and
BRAF. Subclonal structure was investigated in these studies, revealing complex structures
with clusters of subclonal variants and driver mutations, including frequent mutations in
the same pathway within single patients. Serial sampling revealed diverse patterns of
clonal evolution, with significant heterogeneity across samples.

Question 31.4 The correct answer is C.


There is a clear link between depth of response and prolonged survival in multiple
myeloma. Autologous stem cell transplantation has been successful in producing deep
responses among eligible patients, and deeper responses have been found to be associated
with improved clinical outcomes [Lahuerta et al. J Clin Oncol. 2008;26(35):5775–5782].
Likewise, novel agents have allowed an increasing number of transplant ineligible
patients to achieve a deep response and overall depth of response has been associated
with improved outcomes in this population as well [Gay et al. Blood. 2011;117(11):3025–
3031]. With modern regimens capable of producing high rates of CR, it is felt that one
needs to define even greater depths of response to distinguish therapies and better
prognosticate among individual therapies. The definition of a stringent CR (including
normalization of serum-free light chains and absence of clonal PCs in Bone Marrow
Biopsy) was meant to address this issue, and achievement of a stringent CR has been
shown to be predictive of improved overall survival [Kapoor et al. J Clin Oncol.
2013;31(36):4529–4535].
Modern technology permits us to define yet deeper levels of disease response. Several
technologies have been utilized to detect MRD, MFC, next-generation sequencing (NGS),
and allele-specific oligonucleotide-based quantitative polymerase chain reaction (ASO-
PCR). Levels of detection of these technologies enable the identification of as low as 1 cell
in 100,000 for MFC and 1 in 1,000,000 for NGS and ASO-PCR. ASO-PCR analysis is
largely a research tool requiring patient-specific probes to be generated for MRD
assessment. The clonoSEQ MRD assessment is a commercially available tool, which
utilizes NGS of the IgH V(D)J regions to monitor MRD in multiple myeloma patients.
The utility of MRD assessment has been investigated in several clinical trials. In the
PETHEMA/GEM 2000 study, patients who were negative by multiparameter flow
cytometry for MRD after ASCT had significantly improved PFS and OS rates. Likewise in
the MRC Myeloma IX study, MRD negativity after ASCT correlated with improved PFS
and OS. This has also been observed in the transplant ineligible population with patients
showing molecular CR after induction therapy having a statistically significant
improvement in PFS over those with MRD-positivity [Puig et al. Leukemia.
2014;28(2):391–397].
Martinez-Lopez et al. evaluated the clonoSEQ method of monitoring MRD assessment in
a study assessing 133 multiple myeloma patients who had achieved at least a very good
partial response (VGPR) following frontline therapy [Martinez-Lopez et al. Blood.
2014;123(20):3073–3079]. MRD was assessed by the IgH-V(D)J sequencing technique
utilized by the clonoSEQ commercial assay and compared and contrasted with those of the
MFC and ASO-PCR techniques. A total of 91% of patients were effectively sequenced
using the novel NGS technique, and concordance with the MFC and ASO-PCR techniques
was 83% and 85%, supporting the widespread applicability and accuracy of the method.
Patients who were MRD negative by sequencing had a significantly longer time to
progression (median 80 vs. 31 months, P < 0.001) and overall survival (median not
reached vs. 81 months, P = 0.02). A total of 92% of VGPR patients were MRD positive.
Time to progression improved continuously with decreasing levels of residual disease
detected. Among patients achieving a complete response (CR) the time to progression was
longer for MRD negative patients versus those who were MRD positive (131 months vs.
35 months, P = 0.0009).
At this time, MRD assessment remains a research tool. Though several publications now
seem to suggest that patients who are MRD negative after a finite period of therapy do
better than those who are MRD positive, there is no data to support giving additional
therapy to patients who are not MRD negative to improve outcomes. Additionally, there
is no current consensus on the definition of MRD negativity with both flow cytometry and
deep sequencing–based technology having their advantages and disadvantages.

Question 31.5 The correct answer is B.


The optimal depth of disease response necessary to proceed with autologous stem cell
transplantation has been a matter of controversy. Previously, data have suggested that
patients with a better pretransplant paraprotein nadir have had improved outcomes
[Wang et al. Bone Marrow Transplant. 2010;45:498–504.]. However, some centers had
experienced benefit for patients even with disease progression after initial induction
therapy [Kumar et al. Bone Marrow Transplant. 2004;34:161–167]. A recently published
study evaluated the outcomes of patients who had achieved less than a partial response to
initial treatment for their multiple myeloma [Vij et al. Biol Blood Marrow Transplant.
2015;21(2):335–341]. A total of 539 patients were identified who had an ASCT after less
than a PR to initial chemotherapy. While additional pretransplant chemotherapy resulted
in deepening responses in 68% of patients, there was no impact of pretransplant salvage
chemotherapy on treatment-related mortality, progression-free survival, or overall
survival. Additional pretransplant therapy for those who achieved less than a PR to initial
chemotherapy was not found to be beneficial in this large study. Based on their analysis,
switching to carfilzomib-based therapy would likely have little impact. Transplant still
offers the best option for prolonged PFS.

Question 31.6 The correct answer is A.


Currently, there are several monoclonal antibodies under investigation in multiple
myeloma. Daratumumab targets CD38, and elotuzumab is a CS-1/SLAMF7 targeting
antibody. Elotuzumab is postulated to function solely via immune-mediated processes,
while daratumumab has dual effects including direct action on multiple myeloma cells as
well as immune stimulation targeting tumor cells. The ELOQUENT-2 trial was a phase III
trial comparison of elotuzumab in combination with lenalidomide and dexamethasone to
lenalidomide and dexamethasone alone [Lonial et al. N Engl J Med. 2015;373:621–631].
The median PFS was 19.4 months for elotuzumab compared with 14.9 months with
lenalidomide and dexamethasone. The absolute PFS differences at 1 and 2 years were 11%
(68% vs. 57%) and 14% (41% vs. 27%). The hazard ratio for progression of disease at 2
years was 0.70 [0.57–0.85] which implies a 30% overall reduction in risk for disease
progression. This long-term response to therapy raises the possibility that a subset of
patients may have prolonged benefit with the drug and increases the likelihood that an
overall survival benefit may be appreciated with longer-term follow-up. Notably, this trial
excluded patients who were refractory to lenalidomide and it is not known if the addition
of elotuzumab will prove effective in patients whose disease has progressed on
lenalidomide alone.
The SIRIUS trial was a phase II study of single-agent daratumumab in patients who had
received >3 lines of prior therapy or were double refractory to a proteasome inhibitor
and an immunomodulatory drug [Lonial et al. J Clin Oncol. 2015;33(suppl; abstr
LBA8512)]. Among 106 patients with relapsed and refractory multiple myeloma who had
been treated with a median of 5 prior lines of therapy (range 2 to 14), single-agent
daratumumab produced an overall response rate (ORR) of 29% in this population. Among
these responses, 3% were stringent CRs and 12% were a VGPR, despite the late-stage
disease population that was enrolled. Responses were seen regardless of age, number, and
type of prior therapy, or the presence or absence of extramedullary disease. The median
duration of response was 7.4 months with a progression-free survival of 3.7 months (95%
CI [2.8, 4.6]) and a 1-year overall survival (OS) rate of 65% (95% CI [51.2, 75.5]).
Median OS data are not yet available (not reached). Among enrolled patients, 95% were
refractory to a proteasome inhibitor and an immunomodulatory drug. A total of 42% were
refractory to bortezomib, lenalidomide, carfilzomib, pomalidomide, and/or thalidomide.
The SIRIUS study also showed that daratumumab had a response rate of 20% in patients
with extramedullary disease. Overall, daratumumab was well tolerated with no
discontinuations due to adverse events attributable to the drug.

Question 31.7 The correct answer is A.


Panibinostat is a histone deacetylase (HDAC) inhibitor that was approved by the FDA in
combination with bortezomib and dexamethasone for use in patients with multiple
myeloma who had received at least two lines of prior therapy, including bortezomib and
an immunomodulatory drug. The efficacy of panibinostat has been investigated in a large
phase III study, the PANORAMA-1 trial [San-Miguel et al. Lancet. 2014;15:1195–1206]. In
this study, 768 patients with relapsed/refractory multiple myeloma who had received
between 1 and 3 prior lines of therapy were randomized to receive either panibinostat,
bortezomib, and dexamethasone or bortezomib and dexamethasone. With a median
follow-up of 6.5 months, median progression-free survival was longer in the panibinostat
group (12 months) versus patients treated with a doublet (8.1 months). The overall
response rate was not different between groups (61% for the panibinostat arm versus
54.6%). Survival data continue to mature, but were not different between groups. Serious
adverse events were reported in 60% of the panibinostat patients and 42% of the placebo
group. Diarrhea was common (68%) in panibinostat patients and was severe (requiring IV
fluids or hospitalization) in 25% of patients. Cardiac deaths were increased on the
panibinostat arm, prompting a Boxed Warning over the risk of ischemic events or
arrhythmia. Excessive diarrhea is also included in this warning due to its high incidence in
the panibinostat-treated patients. Recommendations for management of diarrhea for
patients on panibinostat include initiation of antidiarrheal medications at the first sign of
symptoms and interruption of dose until symptom resolution for those with at least 4
stools per day. Panibinostat may be resumed at its prior dose for those with 6 or less
stools per day, and dose adjustments are necessary for those with ≥7 stools daily. There
was no increase in the rates of hemolytic uremic syndrome or Stevens–Johnson syndrome
among patients taking panibinostat. Thrombocytopenia was a common toxicity, occurring
in 67% of panibinostat patients and 37% of the bortezomib and dexamethasone arm.
Platelet counts <50 should be monitored weekly, while a drop below 25 or any evidence
of bleeding should result in holding panibinostat until improvement in counts (>50) and
a resumption of panibinostat at a lower dose.

Question 31.8 The correct answer is D.


The ASPIRE trial was a randomized, open-label, multicenter phase III study of carfilzomib,
lenalidomide, and dexamethasone (KRd) versus lenalidomide and dexamethasone (Rd) in
patients with relapsed multiple myeloma [Stewart et al. N Engl J Med. 2015;372:142–
152]. The patients all had relapsed multiple myeloma with 1 to 3 prior lines of treatment,
and patients with lenalidomide were eligible so long as they had not progressed on
lenalidomide and dexamethasone. With a median follow-up of 32 months, the primary
endpoint, progression-free survival, was superior for the three-drug combination (26.3
months vs. 17.6 months; P = 0.0001), and median OS was not reached for either cohort.
The Kaplan–Meier 24-month OS rates were 73.3% in the three-drug regimen and 65.0% in
the two-drug regimen. Grade 3 or higher adverse events were reported in 83.7% of the
three-drug group and 80.7% of the two-drug regimen. There were similar rates of trial
discontinuation due to adverse events in the three- and two-drug regimens (15.3% vs.
17.7%). Other side effects occurred at similar rates in the two arms: dyspnea (2.8% in
KRd vs. 1.8% in Rd), cardiac toxicity (3.8% in KRd and 1.8% in Rd), and peripheral
neuropathy (17.1% in KRd and 17.0% in Rd). There appeared to be a higher rate of high
blood pressure in the KRd arm (14.3 percent vs. 6.9 percent). Patient-reported health-
related quality of life was higher in the KRd arm as compared to the Rd arm.
Evaluating the role of doublet therapy in relapsed and refractory multiple myeloma, the
ENDEAVOR trial was a large phase III comparison of the second-generation proteasome
inhibitor carfilzomib in combination with dexamethasone to bortezomib and
dexamethasone [Dimopoulos et al. J Clin Oncol. 2015:33(suppl; abstr LBA8509)]. It
demonstrated that carfilzomib at a dose of 56 mg/m2 resulted in a PFS of 18.7 months
compared with 9.4 months for bortezomib (1.3 mg/m2) and dexamethasone. The benefit
was seen in both patients who had prior exposure to bortezomib (15.6 months vs. 8.1
months) and no prior bortezomib exposure (not reached vs. 11.2 months). This benefit
was also maintained across age subgroups. The median duration of treatment was 40
weeks for carfilzomib/dexamethasone versus 27 weeks for bortezomib and
dexamethasone. A total of 23% of patients receiving carfilzomb and dexamethasone
required a dose reduction due to toxicity compared with 47% of patients receiving
bortezomib and dexamethasone. Treatment with carfilzomib was associated with higher
rates of hypertension (25% vs. 9%) and somewhat higher rates of cardiac failure (8% vs.
3%).

Question 31.9 The correct answer is A.


MYD88 is a protein that interacts in toll-like receptor and IL-1 signaling and has been
implicated in prolonged B-cell survival. Activating point mutations in MYD88 such as the
L265P mutation have been implicated in the pathogenesis of Waldenstrom’s
macroglobulinemia (WM) [Hunter et al. Blood. 2014;123(11):1637–1646]. This study,
likewise, found frequent activating CXCR4 mutations in patients with WM. These
mutations in CXCR4 are similar to germline mutations detected in patients with the
WHIM syndrome (warts, hypogammaglobulinemia, infection, and myelokathexis). At
least 30 different WHIM mutations have been described in WM, and in vitro tumor cells
engineered to express CXCR4 WHIM receptors have been shown to have decreased
ibrutinib-related apoptosis [Cao et al. Leukemia. 2015;29:169–176]. Ibrutinib has been
recently investigated in WM and approved by the FDA for treatment of these patients.
This approval was based on findings from a phase II study of ibrutinib in 63 patients with
symptomatic WM who had received at least one prior treatment. This study found an
overall response rate (ORR) of 90.5% and a median time to response of 4 weeks among
the study population [Treon et al. NEJM. 2015:372:1430–1440]. Estimated progression-
free survival and overall survival at 2 years were 69% and 95%, respectively. Responses
to ibrutinib were found to differ by MYD88 and CXCR4 mutational status. The highest
response rates were among patients with MYD88L265P mutations who were CXCR4 wild-
type, with a 100% ORR in this population. This was followed by patients who had
MYD88L265P mutations who were CXCR4WHIM mutated, with an 85.7% ORR.
MYD88L265P and CXCR4 wild-type patients had a 71.4% ORR. As such, best responses are
seen in MYD88L265P mutated patients who are CXCR4 wild-type.

Question 31.10 The correct answer is A.


The optimal treatment of amyloidosis is not clear at this time and treatment on clinical
trials is often prioritized when possible. Up to eight cycles of cyclophosphamide,
bortezomib, and dexamethasone (CyBorD) have been found in a series of 43 patients to
have an overall hematologic response rate of 81.4%, including a complete response (CR)
rate of 41.9% and VGPR of 51.4% [Venner et al. Blood. 2011;119(19):4387–4390]. This
has established CyBorD as a key therapy for patients with AL amyloidosis.
Autologous stem cell transplantation has been investigated in phase II studies and has
been found to produce long-term durable remissions in AL amyloidosis patients [Gertz et
al. Bone Marrow Transplant. 2004;32(4):149–154]. It remains a mainstay of treatment for
patients who are eligible. Determination of this eligibility is largely a case-by-case
decision, but general guidelines include the following: appropriate physiologic age,
Troponon T <0.06 ng/mL, no more than two organs significantly involved, ECOG ≤2,
New York Heart Association functional class I or II, creatinine clearance ≥30 mL/min,
and NT-proBNP <5,000 ng/L. Given this patient’s elevated troponin and NT-proBNP, he is
ineligible for transplantation at this time.
Ixazomib, a novel oral proteasome inhibitor, has been investigated in
relapsed/refractory multiple myeloma as a single agent and as combination therapy (with
lenalidomide and dexamethasone). It has shown impressive outcomes in early trials. In a
phase I study of 60 patients who were heavily pretreated with a median of 4 prior lines of
therapy (88% with prior bortezomib), ixazomib had a response rate of 15% (PR or better)
with stable disease in 76% of patients [Richardson et al. Blood. 2014;124(7):1038–1046].
Investigation of ixazomib in combination with lenalidomide and dexamethasone in newly
diagnosed multiple myeloma patients revealed excellent tolerability with 37% achieving a
very good partial response or better.

Corresponding chapter in Cancer: Principles & Practice of Oncology, Tenth Edition: 112 (Plasma Cell Neoplasms).
32 Immunosuppression-Related Malignancies
Lee Ratner

QUESTIONS
Each of the numbered items below is followed by lettered answers. Select the ONE lettered answer
that is BEST in each case unless instructed otherwise.

Question 32.1 In human immunodeficiency virus (HIV)-infected patients, which of the


following malignancies is considered to be an acquired immunodeficiency syndrome (AIDS)-
defining cancer?
A. Colon cancer
B. Cervical cancer
C. Anal cancer
D. Penile cancer

Question 32.2 Cytotoxic chemotherapy is NOT well tolerated with which of the following
antiretroviral medications?
A. Zidovudine, nucleoside reverse transcriptase inhibitor therapy
B. Protease inhibitors
C. Raltegravir, integrase inhibitor
D. Fuzeon, HIV entry inhibitor

Question 32.3 What is the response rate to HAART therapy, in a treatment-naive patient
with favorable-risk Kaposi sarcoma (KS)?
A. 20%
B. 40%
C. 60%
D. 80%

Question 32.4 Which of the following is TRUE about the combination of rituximab with
chemotherapy for AIDS-associated lymphomas?
A. Adding rituximab to cyclophosphamide, doxorubicin, vincristine, and prednisone
(CHOP) chemotherapy in patients with CD4 <50/mm3 may result in higher rate of
neutropenic infections.
B. Rituximab is of no added benefit to CHOP or infusional etoposide, vincristine, and
doxorubicin, bolus cyclophosphamide, and daily prednisone (EPOCH) chemotherapy for
AIDS-associated diffuse large B-cell lymphoma (DLBCL).
C. Rituximab with chemotherapy is usually beneficial in plasmablastic lymphoma.
D. Rituximab with chemotherapy is usually beneficial in primary effusion lymphoma.

Question 32.5 Which is TRUE concerning primary central nervous system (CNS) lymphoma
in AIDS?
A. Positive cerebrospinal fluid (CSF) Epstein–Barr virus (EBV) polymerase chain reaction
(PCR) test and a consistent radiologic picture are sufficient to diagnose primary CNS
AIDS lymphoma.
B. In patients with CD4 count <50/mm3 and poor performance status, high dose
methotrexate should be instituted.
C. In patients with CD4 count >50/mm3 and good performance status, cranial
radiotherapy provides potentially curative therapy.
D. HAART has no role in the treatment of primary CNS lymphoma.

Question 32.6 A 38-year-old man with HIV infection presents with 3-month history of
weight loss and night sweats. He is not on antiretroviral therapy and his last CD4 count 3
months ago was 300/mm3. On examination, he has multiple enlarged cervical lymph nodes.
His hemoglobin is 10 g/dL, white blood cell count is 3.6 × 103 /mm3, and platelet count is
190 × 103/mm3. Serum LDH is 300. Infectious workup is negative. CT of the neck and chest
demonstrates diffuse cervical and mediastinal lymphadenopathy. You suspect lymphoma and
arrange for an excisional biopsy of a neck lymph node. Which of the following lymphomas is
a non-AIDS defining cancer?
A. Hodgkin lymphoma
B. Diffuse large B-cell lymphoma
C. Burkitt lymphoma
D. Primary CNS lymphoma

Question 32.7 Which of the following statements is CORRECT regarding KS?


A. Tumor, node, metastasis (TNM) system is useful for staging KS.
B. Extent of tumor and AIDS-related systemic illnesses is useful in stratifying patients with
KS into prognostic risk groups.
C. Response Evaluation Criteria in Solid Tumors (RECIST) is useful in assessing KS
response to therapy.
D. All of the above are correct.

Question 32.8 How does the presentation of HIV-associated Hodgkin lymphoma (HL) differ
from that of HL in immunocompetent patients?
A. Patients with HIV-associated HL present at an older age.
B. B symptoms are rare in patients with HIV-associated HL.
C. Extranodal sites are less frequently involved in patients with HIV-associated HL.
D. Mediastinal involvement is less frequent in patients with HIV-associated HL.

Question 32.9 Which of the following is CORRECT regarding anogenital cancers in patients
infected with HIV?
A. In HIV-infected women with preinvasive cervical neoplasia, standard therapy results in
a comparable rate of recurrence, compared to their immunocompetent counterparts.
B. In patients with a CD4 count of less than 200/mm3, who are treated with chemotherapy
and radiation for invasive anal cancer, the side effect profile is similar to that of HIV-
negative patients
C. In patients with HIV, anal cancer and cervical cancer are associated with high-risk
subtypes of human papilloma virus (HPV) infection in the large majority of individuals.

Question 32.10 Which of the following viruses are implicated in the development of cancers
in patients with HIV infection?
A. EBV
B. Hepatitis C virus
C. Human herpes virus-8 (HHV-8)
D. All of the above

Question 32.11 A 43-year-old HIV-positive man presents with multiple pigmented skin
nodules, hemoptysis and dyspnea without fever or chills, a bloody pleural effusion, and
diffuse adenopathy. Biopsy of a 4-cm supraclavicular lymph node is most likely to show:
A. Kaposi Sarcoma.
B. Non-Hodgkin Lymphoma.
C. Tuberculosis.
D. Hodgkin Lymphoma.

Question 32.12 A 45-year-old HIV-positive man on HAART with CD4 240, presents with an
anal mass and 3-cm R inguinal node, and biopsy of the anal mass and inguinal node reveal
moderately differentiated squamous cell carcinoma. PET examination showed no other
evidence of disease. The most appropriate initial treatment is:
A. Cisplatin–Fluorouracil therapy
B. Mitomycin–Fluorouracil therapy concurrent with radiation
C. Abdominoperineal resection
D. Radiation therapy alone

Question 32.13 Sorafenib therapy for hepatocellular carcinoma is well tolerated with which
of the following antiretroviral combinations?
A. Truvada and ritonavir boosted prezista
B. Truvada and Raltegravir
C. Atripla
D. Stribild
ANSWERS

Question 32.1 The answer is B.


Cervical cancer in a patient with HIV is considered to be an AIDS-defining cancer, whereas
penile cancer, colon cancer, and anal cancer fall in the non–AIDS-defining cancer category.

Question 32.2 The answer is A.


Zidovudine is poorly tolerated with cytotoxic chemotherapy because of myelosuppression.
Protease inhibitors are generally well tolerated with chemotherapy, although they may
affect metabolism of many drugs through effects on cytochrome P450-associated enzymes.
The integrase inhibitor raltegravir, coreceptor inhibitor, maraviroc, and fusion inhibitor,
fuzeon are also well tolerated with chemotherapy.

Question 32.3 The answer is D.


In patients with favorable-risk KS (confined to skin or lymph nodes, CD4 count >200, no
B symptoms or opportunistic infections, and good performance status) who are treatment-
naive, response rates of up to 80% have been reported with HAART therapy. However, it
is rare for patients with extensive poor-risk KS (tumor associated edema or ulceration,
extensive oral or gastrointestinal involvement, CD4 <200, opportunistic infections, B
symptoms, HIV-related illness, poor PS) to respond to HAART alone. In addition,
worsening of KS may occur with initiation of HAART, due to an immune reconstitution
inflammatory response.

Question 32.4 The answer is A.


Although the role of rituximab with chemotherapy in non–HIV-infected patients has been
documented, its role in AIDS lymphoma is more controversial. Although several studies
have suggested that there is a benefit to the addition of rituximab to CHOP or EPOCH
chemotherapy, in patients with CD4 less than 50 to 100/mm3 there are reports of excess
neutropenic fevers and infections. It has been suggested that this adverse result can be
ameliorated with prophylactic quinolone antibiotics. Rituximab is usually not beneficial
for treatment of plasmablastic lymphomas and primary effusion lymphomas, because
CD20 is usually not expressed on the malignant cells.

Question 32.5 The answer is A.


CSF EBV PCR is a rather sensitive (>80%) and highly specific test (>95%) for diagnosis
of primary CNS lymphoma, and together with a consistent radiologic picture, brain biopsy
may not be required to establish the diagnosis. In patients with low CD4 counts or poor
performance status, cranial radiotherapy is the treatment of choice for primary CNS
lymphoma, because opportunistic infections cause much greater morbidity and mortality
than recurrent lymphoma. In contrast, for patients with higher CD4 counts and good
performance status, high-dose systemic methotrexate therapy with leucovorin rescue may
be associated with improved responses, as reported in non–HIV-associated primary CNS
lymphoma.
Question 32.6 The answer is A.
Hodgkin disease is a non–AIDS-defining cancer, whereas Burkitt lymphoma, DLBCL, and
primary CNS lymphoma are considered to be AIDS-defining cancers.

Question 32.7 The answer is B.


TNM staging is not useful in KS. The most widely used staging system is the TIS system,
which attempts to stratify risk of poor prognosis in patients with KS. It is based on the
extent of tumor involvement (T), immune status of the patient (I), and the presence of
AIDS-related system illness. In the era of HAART, assessing the immune system (CD4
count) does not provide prognostic information in patients with HIV sensitive to HAART,
although it is useful in patients with multidrug resistant HIV. RECIST criteria are not
useful in assessing KS response to therapy. The AIDS Clinical Trials Group Oncology
Committee criteria for assessing response include a 50% decrease in the total number of
lesions, a 50% decrease in the area of measured cutaneous lesions, or a flattening of 50%
of nodular lesions.

Question 32.8 The answer is D.


Patients with HIV-associated HL present at a younger age, B symptoms are more frequent,
and they usually have higher-stage disease at presentation. Mediastinal involvement is
less frequent in patients with HIV-associated HL, whereas extranodal involvement is more
frequent.

Question 32.9 The answer is C.


Cervical cancer and anal cancer in patients infected with HIV appear to be related to HPV
infection. In women who are newly diagnosed with HIV, cervical cancer screening is
recommended. If preinvasive cervical neoplasia is found, standard therapy (cryotherapy,
laser therapy, cone biopsy, and loop excision) should be used. However, the recurrence
rate in patients with HIV infection is twice that of HIV-seronegative women. In HIV-
infected patients with anal cancer (CD4 <200/mm3), who are treated with chemotherapy
and radiation, treatment-related toxicities (diarrhea, cytopenias) are also significantly
higher than those in HIV-negative patients with anal cancer. High risk HPV subtypes
associated with anogenital cancers includes types 16 and 18.

Question 32.10 The answer is D.


EBV is involved in the pathogenesis of HL and the EBV genome has been detected in
tumor nuclei. Hepatitis C and HIV coinfection is common and leads to increased risk of
cirrhosis and hepatocellular carcinoma. HHV-8 is implicated in the development of KS and
primary effusion lymphomas.

Question 32.11 The answer is A


Pigmented skin lesions in an HIV-positive individual may be due to purpura, angioma,
dermatofibroma, nevi, melanoma, bacillary angiomatosis, or Kaposi sarcoma (KS). KS is
also associated with lymphadenopathy and pulmonary disease, including interstitial or
bronchial nodules or pleural lesions. Although a lymph node larger than 2 cm is suspicious
for lymphoma, given the entire clinical picture, KS is more likely. The lack of fevers
would make tuberculosis less likely. Although Hodgkin lymphoma is approximately 10-
fold more frequent in HIV-positive than HIV-negative individuals, pleural effusions are
uncommon without bulky mediastinal adenopathy, and are rarely hemorrhagic.

Question 32.12 The answer is B


The patient has stage IIIB anal carcinoma. Anal carcinoma is approximately 100-fold more
frequent in HIV-positive than HIV-negative individuals. Approximately half of such
individuals will achieve 5-year tumor-free survival after treatment with fluoropyrimidine-
based chemoradiotherapy using either cisplatin or mitomycin as part of the chemotherapy
typically provided on weeks 1 and 5 of therapy. Chemoradiotherapy results in superior
results as compared to radiation or chemotherapy alone. Patients who fail to achieve
complete remission with this approach, may be candidates for abdominoperineal
resection.

Question 32.13 The answer is B.


Sorafenib therapy is superior to best supportive care for patients with good performance
status and Child-Pugh A cirrhosis and hepatocellular carcinoma. Sorafenib is metabolized
predominantly by the CYP3A4 pathway. Other substrates include erlotinib, temsirolimus,
and sunitinib. Protease inhibitors, like darunavir and ritonavir, are strong CYP3A4
inhibitors. Cobicistat, a component of Stribild, which also includes elvitegravir, tenofovir,
and emtricitabine, is also a potent CYP3A4 inhibitor. Efavirenz, a nonnucleoside reverse
transcriptase inhibitor component of atripla is a moderate inducer of CYP3A4 activity. In
contrast, integrase inhibitor, raltegravir, and nucleoside and nucleotide analogs like those
in truvada, have little effect on CYP3A4 activity.

Corresponding chapters in Cancer: Principles & Practice of Oncology, Tenth Edition: 117 (HIV-Associated Malignancies) and
118 (Transplantation-Related Malignancies).
33 Stem Cell Transplantation
Brian Hess and Rizwan Romee

QUESTIONS
Each of the numbered items below is followed by lettered answers. Select the ONE lettered answer
that is BEST in each case unless instructed otherwise.

Question 33.1 The curative potential of allogeneic stem cell transplantation (allogeneic SCT)
may be derived from which of the following?
A. High-dose chemotherapy
B. Graft-versus-leukemia (GVL) effect
C. Graft-versus-host effect
D. A and B

Question 33.2 The allogeneic SCT conditioning regimen serves to achieve which of the
following?
A. Provide immunosuppression to prevent rejection
B. Provide immunosuppression to prevent graft-versus-host disease (GVHD)
C. Eradicate malignant cells
D. A and C

Question 33.3 A 54-year-old male is diagnosed with AML with complex cytogenetics. After
achieving complete remission with standard 7+3 chemotherapy he is evaluated for
allogeneic SCT from his HLA-matched brother. Which of the following regimens is considered
myeloablative for his upcoming transplant?
A. Cyclophosphamide 60 mg/kg/d intravenously (IV) × 2 days plus 1,200 cGy total body
irradiation (TBI)
B. Cyclophosphamide 60 mg/kg/d IV × 2 days plus busulfan 3.2 mg/kg/d IV × 4 days
C. Fludarabine 30 mg/m2 × 3 days plus 200 cGy TBI
D. A and B

Question 33.4 Which conditioning regimen leads to improved disease-free survival in


chronic myeloid leukemia (CML)?
A. Cyclophosphamide 60 mg/kg/d IV × 2 days plus busulfan 3.2 mg/kg/d IV × 4 days
B. 1,200 cGy TBI and cyclophosphamide 60 mg/kg/d IV × 2 days
C. Chemotherapy and radiation-based conditioning are equivalent
D. Fludarabine 30 mg/kg/d IV × 5 plus busulfan 3.2 mg/kg/d IV × 2 plus ATG 2.5
mg/kg/d IV × 4

Question 33.5 Which of the following drugs used in the conditioning regimens for stem cell
transplant (SCT) is the most common cause of acute cardiac toxicity?
A. Cytarabine
B. Fludarabine
C. Cyclophosphamide
D. Busulfan

Question 33.6 A 64-year-old female is diagnosed with multiple myeloma. She has
experienced a very good partial response after four cycles of induction therapy but her
creatinine remains elevated at 3.9 g/dL and her creatinine clearance remains less than 15
mL/min. What is the most appropriate management in regards to her conditioning regimen
for upcoming autologous stem cell transplant?
A. Melphalan 200 mg/m2
B. Dose reduce melphalan to 140 mg/m2
C. Dose reduce melphalan to 100 mg/m2
D. Her elevated creatinine is a contraindication to autologous transplant

Question 33.7 The GVL effect after transplant is most pronounced in which of the following
malignancies?
A. Acute myelogenous leukemia (AML)
B. Acute lymphoblastic leukemia (ALL)
C. Chronic-phase CML
D. Accelerated-phase CML

Question 33.8 Which of the following increases the risk of relapse after allogenic stem cell
transplantation?
A. Human leukocyte antigen (HLA)-mismatched transplant
B. CD34 cell dose >2 × 106 but <5 × 106/kg recipient body weight
C. Acute and chronic GVHD
D. Use of T-cell–depleted graft

Question 33.9 Which of the following therapies in a patient with multiple myeloma would
potentially hinder stem cell mobilization and collection for future autologous stem cell
transplant?
A. Melphalan
B. Long-term use of lenalidomide
C. Velcade
D. A and B

Question 33.10 Sinusoidal obstruction syndrome (SOS) of the liver is associated with which
of the following factors?
A. TBI 12 Gy conditioning
B. Oral busulfan conditioning
C. Advanced age
D. All of the above

Question 33.11 Which of the following factors is associated with increased risk of
engraftment failure in allogeneic stem cell transplantation?
A. T-cell depletion of the product
B. Less than 1 × 106 CD34+ cells/kg recipient body weight
C. HLA-mismatched stem cell donor
D. All of the above

Question 33.12 In which setting is the risk of CMV reactivation the highest after allogeneic
SCT?
A. CMV-positive donor to a CMV-negative recipient
B. CMV-negative donor to a CMV-positive recipient
C. CMV-negative donor to a CMV-negative recipient
D. CMV-positive donor to a CMV-positive recipient

Question 33.13 Peripheral blood mobilized stem cell allografts, compared with bone
marrow, are associated with which of the following?
A. Shorter period of neutropenia
B. Shorter period to platelet recovery
C. Equivalent T-cell numbers
D. Equivalent incidence of acute GVHD
E. A, B, and D

Question 33.14 For which of the following patients with an HLA-matched donor is
allogeneic SCT most appropriate?
A. A 35-year-old man with AML with inversion 16 in CR1 after 7+3
B. A 40-year-old woman with AML and complex cytogenetics in first clinical remission
(CR)
C. A 48-year-old woman with diffuse large B-cell lymphoma with chemosensitive relapse
after initial remission of 18 months
D. A 55-year-old man with RAI stage III chronic lymphocytic leukemia (CLL) in first
remission

Question 33.15 Which of the following are potential complications of donor lymphocyte
infusion (DLI)?
A. Bone marrow aplasia
B. GVHD flare
C. SOS
D. A and B
Question 33.16 Factors that influence the choice of conditioning regimen in allogeneic stem
cell transplantation include:
A. Underlying malignancy and prior treatment.
B. Patient age and comorbidity.
C. Donor:recipient HLA compatibility.
D. All of the above.

Question 33.17 Late complications after allogeneic SCT include:


A. Skeletal complications, including osteoporosis and avascular necrosis.
B. Secondary malignancies.
C. Endocrine failure.
D. All of the above.

Question 33.18 What is the most common cause of late hemorrhagic cystitis in patients
undergoing allogeneic SCT?
A. Cyclophosphamide
B. BK virus
C. EBV
D. CMV

Question 33.19 Autologous stem cell transplantation is indicated in certain situations for
which of the following solid tumors?
A. Testicular cancer
B. Neuroblastoma
C. Breast cancer
D. A and B

Question 33.20 Graft manipulation ex vivo to purge contaminating tumor cells is a strategy
that has been successful in reducing the rate of relapse after autologous stem cell
transplantation.
A. True
B. False

Question 33.21 Which of the following should be considered for autologous stem cell
transplant?
A. DLBCL that has relapsed after first-line R-CHOP chemotherapy
B. Follicular lymphoma that has relapsed after first-line Bendamustine and Rituxan
C. ALK-positive T-cell lymphoma in first remission
D. Nodal Marginal zone lymphoma that has progressed after a 6-month period of
observation

Question 33.22 A mismatch of which of the following HLA genes would NOT adversely
influence the outcome in an HLA mismatched unrelated donor allogeneic SCT?
A. HLA-A
B. HLA-B
C. HLA-C
D. HLA-DQB1
E. HLA-DRB1

Question 33.23 Which of the following cells present in the stem cell graft are thought to
play a major role in mediating GVL effect seen in allogeneic stem cell transplants?
A. Donor T lymphocytes
B. Donor B cells
C. Donor NK cells
D. A and C

Question 33.24 A 42- year-old male is undergoing transplant for relapsed AML. He has no
matched related or unrelated donors available and thus he is undergoing referral for a
haploidentical transplant from his brother. Which of the following would be appropriate in
order to decrease the likelihood of acute GVHD in this setting?
A. Post transplant Cytoxan 50 mg/m2 on day +3 and day +4
B. Reduce dose of calcineurin inhibitor started on day 0
C. Reduced intensity conditioning
D. Posttransplant Rituximab 375 mg/m2 on day +4
ANSWERS

Question 33.1 The answer is D.


High-dose chemotherapy (and/or radiation) conditioning regimens have significant
antitumor activity and are typically used in younger patients with aggressive hematologic
malignancies. Historically, it was believed that the curative potential of allogeneic SCT
was derived entirely from the conditioning regimen, and that the donor graft served only
as hematopoietic “rescue.” It is now known that the graft can mediate a powerful
immunologic “graft-versus-leukemia (GVL) effect” that can contribute significantly to the
curative potential of the transplant. GVHD represents the deleterious reverse of GVL, an
immunologic attack on normal tissue.

Question 33.2 The answer is D.


The primary objectives of allogeneic transplant conditioning are to provide sufficient
immunosuppression of the host immune system to permit donor engraftment, and
particularly with fully myeloablative regimens, to achieve significant cytoreduction of the
tumor cells by incorporating agents with known activity against the underlying
malignancy.

Question 33.3 The answer is D.


High-dose cyclophosphamide in combination with 1,200 to 1,350 cGy TBI or busulfan
13.8 mg/m2 IV constitute the two most commonly used myeloablative conditioning
regimens for allogeneic SCT. Fludarabine 30 mg/m2 × 3 days and low dose TBI is a
common nonmyeloablative regimen.

Question 33.4 The answer is C.


Two randomized clinical trials have failed to demonstrate differences in disease-free
survival among patients with CML conditioned with either cyclophosphamide and
busulfan or cyclophosphamide and TBI. Although the use of reduced-intensity conditioning
regimens (option D) has shown promise in older patients with CML, there are insufficient
data comparing such approaches with traditional myeloablative regimens to recommend
their routine use in younger patients who are candidates for myeloablative conditioning.

Question 33.5 The answer is C.


Although acute cardiac toxicity, characterized by cardiomyopathy, arrhythmia, or
pericarditis, may be seen with cytarabine, this toxicity is most commonly caused by
cyclophosphamide. Busulfan is not commonly associated with cardiac toxicity but may
cause VOD and pneumonitis. Fludarabine is not associated with cardiac cytotoxicity.

Question 33.6 The answer is B.


Patients with advanced renal impairment (CR >3 g/dL) or undergoing hemodialysis
should be considered for dose reduction in melphalan conditioning. This is however not an
absolute contraindication to transplant and it is recommended that the melphalan be dose
reduced to 140 mg/m2.

Question 33.7 The answer is C.


It was primarily on the basis of retrospective studies of outcomes among patients with
chronic-phase CML that the GVL effect was first recognized, and a majority of CML
patients treated with DLI for posttransplant relapse achieve a durable molecular
remission. In contrast to CML, only a minority of patients with relapsed acute leukemia
(AML or ALL) or more advanced CML have durable long-term responses with DLI.

Question 33.8 The answer is D.


Posttransplant disease relapse risk is inversely related to the incidence of acute and
chronic GVHD. T-cell depletion, although an effective strategy for lowering the risk of
GVHD, confers increased risk of disease relapse.

Question 33.9 The answer is D.


Therapies such as melphalan, long-term use of lenalidomide, or radiation to significant
volume of the bone marrow can all hinder future stem cell mobilization for autologous
stem cell transplant. Lenalidomide that is used as induction therapy in multiple myeloma
for a limited number of cycles preceding transplant does not significantly alter stem cell
collection.

Question 33.10 The answer is D.


The incidence of SOS, thought to result from chemotherapy or radiation-induced injury to
the small venules in the liver, is associated with advanced age, cyclophosphamide or oral
busulfan conditioning, and pre-existing liver disease.

Question 33.11 The answer is D.


T cells in the graft are important for mediating stem cell engraftment and therefore the
use of T-cell depleted (or CD34+ cell selected) grafts are associated with an increased
risk of engraftment failure. Reduced CD34+ cell dose, use of HLA mismatched donor
(haploidentical donor transplants or mismatched cord blood transplants) and prior allo-
immunization (multiparous females and patients who have received multiple blood
products prior to transplantation) are also associated with increased risk of engraftment
failure in HSCT recipients.

Question 33.12 The answer is B.


The risk of developing CMV infection with a CMV-negative donor and recipient is
negligible. Among CMV-positive recipients, a CMV-positive donor confers a lower risk of
reactivation because of the transfer of pre-existing CMV immunity to the recipient,
compared with a CMV-naive donor.

Question 33.13 The answer is E.


Mobilization and harvest of hematopoietic stem cells from peripheral blood with G-CSF
leads to significantly higher numbers of CD34+ cells and results in a shorter time to
engraftment of both neutrophils and platelets, compared with bone marrow. Despite
significantly higher numbers of T cells, there was no evidence of a significant increase in
the incidence of acute GVHD in a randomized trial of peripheral blood mononuclear cell
(PBMC) versus bone marrow, although a trend toward increased chronic GVHD in PBMC
recipients was observed.

Question 33.14 The answer is B.


Because of its substantial treatment-related morbidity and mortality, allogeneic stem cell
transplantation is generally reserved for patients with hematologic malignancies with a
low likelihood of cure with conventional chemotherapy and/or autologous SCT. Patients
with AML with inversion 16 and translocation (8;21) in remission have a favorable
prognosis for cure with high-dose cytarabine consolidation and are generally only
considered for allogeneic SCT in the event of relapse. In contrast, patients with AML with
high-risk cytogenetics, including deletions of chromosomes 5 and 7, and complex
karyotypes, are rarely cured with conventional chemotherapy, and should be referred for
consideration of allogeneic SCT when in first complete remission (CR1). Relapsed patients
with intermediate-grade non-Hodgkin lymphoma who respond to salvage chemotherapy
may be cured with autologous stem cell transplantation with a relatively low risk of
treatment-related morbidity and mortality. Because of the generally indolent nature of
CLL, initial remissions may persist for years. Although potentially curative for CLL, the
early mortality risk associated with allogeneic SCT is generally not justified in this
scenario, although it may be appropriate in selected patients with more advanced,
multiply relapsed disease.

Question 33.15 The answer is D.


DLIs by design result in the infusion of large numbers of effector T cells with the goal of
augmenting an alloimmune response directed against residual host-derived tumor cells. An
unintended negative consequence, however, is the simultaneous infusion of alloreactive T-
cell clones directed at normal tissue that may result in a significant flare in GVHD. In the
case of relapsed CML, a substantial percentage of host hematopoiesis may be derived
from the residual leukemic clone at relapse, in which case the development of the desired
GVL effect may be associated with transient marrow aplasia. SOS is typically observed
during the first 2 to 3 weeks posttransplant and is thought to result from endothelial
damage caused by the conditioning regimen.

Question 33.16 The answer is D.


The choice of conditioning regimen is driven by multiple factors including, for both
autologous and allogeneic SCT recipients, the underlying malignancy and its predicted
responsiveness to various therapeutic agents. Similarly, the age and comorbidities of the
recipient may dictate the use of a regimen with a lower predicted toxicity profile. For
recipients of allogeneic SCT, donor:recipient HLA mismatching may influence the use of a
more immunosuppressive conditioning regimen to lower the risk of graft rejection.
Question 33.17 The answer is D.
Allogeneic SCT recipients remain at risk for various long-term complications, including
skeletal complications (especially with chronic steroids), infertility, secondary
malignancies, and endocrine failure, for which ongoing follow-up and monitoring are
indicated.

Question 33.18 The answer is B.


Unlike the early hemorrhagic cystitis that is usually caused by high-dose
cyclophosphamide, the late counterpart is usually secondary from BK viral infection.

Question 33.19 The answer is D.


Autologous stem cell transplantation has a role in the treatment of patients with various
solid tumors including neuroblastoma, Ewing sarcoma, and testicular cancer. Since
randomized clinical trials did not show survival benefit with autologous stem cell
transplantation in breast cancer patients, this approach should be used only in the context
of clinical trials.

Question 33.20 The answer is B.


Attempts to purge tumor cells from autologous hematopoietic stem cell grafts, using
either negative or positive selection strategies, have not been proven to reduce the risk of
relapse or prolong overall survival.

Question 33.21 The answer is A


The standard of care for patients with DLBCL in first relapse is autologous STC. Follicular
lymphomas are often not referred for autologous stem cell transplant as there are many
other, less toxic options available (i.e., chemoimmunotherapy; Ibrutinib–Rituxan;
Lenalidomide–Rituxan). T cell lymphomas are often referred to transplant in first
remission. This however does not apply to the subset of T-cell lymphomas that are ALK+,
as they tend to have a better prognosis. Patients with nodal marginal zone lymphoma
often have a very indolent course and rarely are considered for autologous stem cell
transplant.

Question 33.22 The answer is D


Answer D: The preferred donor for any recipient will have a match of HLA-A, -B, -C, and -
DRB1. Recent data also supports favorable outcomes when matched at HLA-DPB1 locus
however there is no data to support matching at HLA-DQB1 is associated with a survival
benefit in allogeneic SCT recipients.

Question 33.23 Answer D


Donor T lymphocytes and NK cells are thought to mediate the GVL effect through a
number of mechanisms, including the T-cell recognition of the minor histocompatibility
antigens on leukemic cells. Donor KIR and recipient HLA genotypes seem to play a major
role in mediating GVL effect of the donor derived NK cells in allogeneic SCT. Donor B
cells are thought to be involved in the pathogenesis of chronic graft-versus-host disease
but there is no evidence that they have a role in the GVL effect.

Question 33.24 Answer A


Answer A: Numerous studies have shown that use cyclophosphamide early after allogeneic
SCT (on days +3 and +4) is effective in reducing the incidence of acute and chronic
graft-versus-host disease in haploidentical transplants. Decreasing the dose of the patient’s
calcineurin inhibitor such as tacrolimus would likely increase the risk for acute GVHD.
Posttransplant rituximab does not decrease acute GVHD after allogeneic SCT.

Corresponding chapters in Cancer: Principles & Practice of Oncology, Tenth Edition: 130 (Autologous Stem Cell
Transplantation) and 131 (Allogeneic Stem Cell Transplantation).
34 Oncologic Emergencies
Preet Paul Singh

QUESTIONS
Each of the numbered items below is followed by lettered answers. Select the ONE lettered answer
that is BEST in each case unless instructed otherwise.

Questions 34.1–3 A 48-year-old White man presents with a history of progressive facial
swelling, fullness in the head and shortness of breath for 1 month. He has a 40 pack-year
smoking history. On examination, the patient has venous distention of the neck and chest
wall. Laboratory test results revealed normal blood counts, basic metabolic profile, and liver
functions. Computed tomography (CT) scan revealed an 8 × 6-cm lung mass with multiple
liver lesions.

Question 34.1 What is the most likely cause of his condition?


A. Small cell lung cancer
B. Hodgkin lymphoma
C. Diffuse large B-cell lymphoma
D. Superior vena cava (SVC) thrombosis

Question 34.2 Biopsy was performed and showed small cell lung cancer. Which is the best
initial modality of treatment for this patient?
A. Surgery
B. Radiation therapy
C. Chemotherapy
D. SVC stent

Question 34.3 What is the most likely location for the primary tumor?
A. Peripheral left lung
B. Peripheral right lung
C. Central left lung
D. Central right lung

Question 34.4 Which among the following is the earliest fundoscopic sign for increased
intracranial pressure?
A. Papilledema with blurring of the disc margins
B. Foster–Kennedy syndrome
C. Disc hemorrhage
D. Absence of venous pulsations within the center of the optic disc

Questions 34.5–6 A 57-year-old man presents with severe back pain and bilateral leg
weakness for 3 days. Magnetic resonance imaging (MRI) of the spine reveals metastatic
lesion at T10 vertebral body with significant cord compression.

Question 34.5 Which of the following are most likely primary cancers in this patient?
A. Lung cancer and breast cancer
B. Lung cancer and lymphoma
C. Colon cancer and lymphoma
D. Colon cancer and prostate cancer

Question 34.6 What is the most appropriate initial therapy for this patient with malignant
spinal cord compression?
A. Chemotherapy
B. Radiation therapy
C. Strontium-89
D. Dexamethasone

Question 34.7 In patients with increased intracranial pressure, which of the following signs
indicates midbrain dysfunction?
A. Midsize pupils unresponsive to light
B. Coma
C. Absence of oculocephalic reflex
D. Ataxic breathing

Question 34.8 Which of the following laboratory abnormalities are associated with tumor
lysis syndrome? (Select two correct responses)
A. Hypokalemia
B. Hypophosphatemia
C. Hypocalcemia
D. Hyperuricemia

Question 34.9 A 28-year-old man presents with newly diagnosed acute myeloid leukemia.
He has a white cell count of 140,000/mm3, glucose of 96 mg/dL, sodium of 138 mEq/L,
potassium of 4 mEq/L, creatinine of 1.1 mg/dL, blood urea nitrogen (BUN) of 15 mg/dL, uric
acid of 4.8 mg/dL, albumin of 5.2 g/dL, and calcium of 10.2 mg/dL. He is started on
induction chemotherapy, aggressive intravenous fluids and allopurinol. Twelve hours later,
the patient developed cardiac arrest not responsive to cardiopulmonary resuscitation and
died. What is the most likely cause of his death?
A. Leukostasis
B. Severe hyperkalemia
C. Pulmonary embolism
D. Chemotherapy-induced cardiac arrhythmia

Questions 34.10–11 A 40-year-old woman presents with fever and headache 3 days after
receiving chemotherapy. Her blood pressure is 130/85 and heart rate 101. Laboratory tests
revealed hemoglobin of 8.5 g/dL, WBC of 11,500/mm3, platelets of 69,000/mm3, sodium of
142 mEq/L, potassium of 4.3 mEq/L, bicarbonate of 24 mEq/L, BUN of 44 mg/dL, creatinine
of 1.8 mg/dL, albumin of 3.8 g/dL, total bilirubin 2.4 mg/dL, and direct bilirubin 0.3 mg/dL.
Her prothrombin time and partial thromboplastin time were normal. Peripheral blood smear
showed several fragmented red blood cells and helmet cells. Direct Coombs test is negative.

Question 34.10 What is the most likely diagnosis?


A. Disseminated intravascular coagulopathy
B. Malignant hypertension
C. Brain metastases
D. Hemolytic uremic syndrome

Question 34.11 Which of the following chemotherapy drugs are implicated in the etiology
of this syndrome? (Select two correct responses)
A. Paclitaxel
B. Mitomycin
C. Gemcitabine
D. Pemetrexed

Question 34.12 A 63-year-old man with metastatic squamous cell carcinoma of the lung
presents with constipation and nausea for 3 days. Laboratory tests revealed hemoglobin of
13.5 g/dL, WBC of 12,500/mm3, platelets of 155,000/mm3, sodium of 146 mEq/L, potassium
of 4.8 mEq/L, bicarbonate of 28 mEq/L, BUN of 44 mg/dL, creatinine of 1.7 mg/dL, albumin
of 2.8 g/dL, and calcium of 13.1 mg/dL. What is the most appropriate therapy, in addition to
intravenous fluids?
A. Gallium nitrate
B. Furosemide
C. Bisphosphonates
D. Calcitonin

Question 34.13 Denosumab is approved for prevention of skeletal-related events in patients


with bone metastases from solid tumors. Which of the following describes the mechanism of
action of denosumab?
A. Inhibits osteoclasts by binding RANKL (receptor activator of nuclear factor-κΒ ligand)
to prevent RANKL activation by RANK.
B. Inhibits active form of vitamin D, 1,25-dihydroxyvitamin D and prevents intestinal
calcium absorption
C. Activates osteoblasts by binding RANKL (receptor activator of nuclear factor-κΒ ligand)
D. Impede protein prenylation and bone resorption by osteoclasts by inhibiting the
mevalonate pathway.

Question 34.14 Which of the following findings are required for diagnosis of syndrome of
inappropriate antidiuretic hormone secretion (SIADH)? (Select two correct responses)
A. Decreased effective serum osmolality (<275 mOsm/kg of water)
B. Inappropriate urinary concentration: Urine osmolality >100 mOsm/kg of water with
normal renal function
C. Clinical evidence of hypovolemia (orthostasis, tachycardia, decreased skin turgor, dry
mucous membranes)
D. Urine sodium <10 mmol/L in the face of normal salt and water intake

Question 34.15 Rasburicase, indicated for treatment of hyperuricemia, is a recombinant


version of which of the following enzymes involved in purine metabolism?
A. Xanthine oxidase
B. Urate oxidase
C. Guanase
D. Hypoxanthine phosphoribosyltransferase
ANSWERS

Question 34.1 The answer is A.


Malignant disease is the most common cause of SVC syndrome. Among the malignancies,
the most common is small cell lung cancer, followed by squamous cell lung cancer and
lymphoma in most series. Diffuse large B-cell lymphoma and lymphoblastic lymphoma are
the most common among lymphomas that cause SVC syndrome. Although Hodgkin
disease frequently involves mediastinum, it rarely causes SVC syndrome. Breast cancer is
the most common metastatic cancer that causes SVC syndrome. Nonmalignant conditions
causing SVC syndrome are being more frequently observed due to increased use of
intravascular devices such as catheters and pacemakers that can lead to SVC thrombosis.

Question 34.2 The answer is C.


Combination chemotherapy with an appropriate regimen is the treatment of choice for
Small cell lung cancer and non-Hodgkin lymphoma. Radiation therapy of the lesion and
adjacent nodal areas may enhance control after initial response to chemotherapy. Surgery
is not a good option for this patient with extensive-stage small cell lung cancer, and an
SVC stent may be used in case of lack of response or relapsed obstruction after initial
therapy. Although radiation therapy could be used for immediate symptomatic relief,
combination chemotherapy is highly effective and preferable as initial therapy, avoiding
the large radiation field and also addressing the metastatic lesions. Non–small cell lung
cancer causing superior vena cava syndrome is best treated with radiation therapy.

Question 34.3 The answer is D.


Small cell lung cancer is frequently central in location. The SVC extends from the junction
of the right and left innominate veins to the right atrium and is completely encircled by
lymph node chains draining from the right thoracic cavity and lower part of the left
thorax. Approximately 80% of the tumors causing SVC syndrome are located in the right
lung. In the absence of mediastinal lymph node enlargement, the most likely cause of the
SVC is direct extension from the centrally located right-sided tumor.

Question 34.4 The answer is D.


Absence of venous pulsations within the center of the optic disc is usually one of the
earliest signs of increased intracranial pressure, with papilledema with blurring of the
optic disc and hemorrhage occurring at later stages. The Foster–Kennedy syndrome—optic
nerve atrophy as a result of a sphenoid wing meningioma and contralateral papilledema
from increased ICP—is rarely seen in the days of improved neuroimaging methods and
earlier diagnosis.

Question 34.5 The answer is B.


Although most patients with malignant spinal cord compression (MSCC) have a history of
malignancy, approximately 20% develop this complication at the initial presentation.
Breast cancer is a common cause of MSCC during the course of the disease, but this
manifestation rarely occurs at presentation. MSCC occurring late in the course of the
disease is common in breast cancer. In this male patient, breast cancer would be unlikely.
Colon cancer is an uncommon cause of MSCC and prostate cancer usually affects the
lumbosacral spine. The most common types of cancer that account for MSCC are breast,
prostate, lung cancer, and lymphoma.

Question 34.6 The answer is D.


Treatment with corticosteroids should be initiated immediately when MSCC is suspected.
Corticosteroids not only facilitate pain management but also reduce vasogenic cord edema
and may prevent additional damage to the spinal cord from decreased perfusion. After an
initial intravenous bolus, doses of up to 10 mg every 6 hours are most commonly used.
Systemic chemotherapy is an appropriate treatment only for patients with MSCC caused
by highly chemosensitive tumors such as non-Hodgkin lymphoma or germ cell tumors.
Radiation therapy is commonly employed as subsequent therapy for control of MSCC, but
should not preclude early corticosteroid therapy. Strontium-89 is used as palliative
treatment for widely metastatic bone metastases from prostate cancer.

Question 34.7 The answer is A.


Midsize and nonresponding pupils indicate midbrain dysfunction, whereas coma is seen
with damage to the mesencephalic reticular activating system. Ataxic breathing may be
observed with cerebellar tonsil herniation and indicates damage to the medullary
respiratory center. Absence of the oculocephalic reflex (doll’s head maneuver) and
horizontal eye movements in response to caloric stimulation of the vestibular system are
signs of pontine damage.

Question 34.8 The answers are C and D.


Cell death in tumor lysis syndrome leads to the release of intracellular potassium,
phosphate, uric acid, and other purine metabolites overwhelming the kidney’s capacity for
clearance with resultant hyperkalemia, hyperphosphatemia and secondary hypocalcemia,
and hyperuricemia.

Question 34.9 The answer is B.


Despite vigorous hydration, the most likely cause of death soon after initiating
chemotherapy is tumor lysis syndrome (TLS). Among the metabolic complications of
tumor lysis syndrome, hyperkalemia caused by the release of intracellular potassium from
tumoral cells represents the most important cause of early mortality. Leukostasis,
pulmonary embolism, and cardiac arrhythmia are possible causes that are far less likely in
this patient.

Question 34.10 The answer is D


This patient has hemolytic uremic syndrome (HUS), which is a microvascular disorder
characterized histopathologically by disseminated microthrombi occluding the
microvasculature. The clinical manifestations of HUS include the classic pentad of
microangiopathic hemolytic anemia, thrombocytopenia, fever, renal failure, and
neurologic deficits. Severe disseminated intravascular coagulopathy (DIC) can cause
similar presentation, but normal coagulation studies are unlikely in DIC. Her blood
pressure is normal which makes malignant hypertension unlikely. Brain metastases can
lead to headache but are not associated with the rest of the abnormalities.

Question 34.11 The answers are B and C.


Chemotherapy drugs that have been implicated to cause HUS are mitomycin-C, 5-
fluoruracil, cisplatin, and gemcitabine. Gemcitabine-associated HUS reflects the
widespread use of this agent. Other agents that have been implicated in the etiology of
this syndrome include bleomycin, cytosine arabinoside, daunomycin, deoxycoformycin,
estramustine, and methyl-CCNU.

Question 34.12 The answer is C


This patient has hypercalcemia related to malignancy. Intravenous hydration is the most
important initial step. Together with hydration, bisphosphonates are the cornerstone of
therapy for malignancy-associated hypercalcemia. Bisphosphonates are also very effective
and well tolerated. Furosemide can be considered as it may enhance the renal calcium
excretion, but is not as important as bisphosphonates. Gallium nitrate is usually given by
continuous infusion for 5 days, with a gradual reduction in the calcium during the infusion
time. Calcitonin is safe and relatively nontoxic, and may result in a rapid decrease in
calcium levels within 2 to 6 hours. It can be utilized in hypercalcemia refractory to
bisphosphonates, but requires frequent administration. Tachyphylaxis can also develop
with calcitonin.

Question 34.13 The answer is A


Denosumab is a fully human monoclonal antibody with a high affinity and specificity for
RANKL (receptor activator of nuclear factorκΒ ligand). RANKL is part of a cytokine
system that belongs to the tumor necrosis factor superfamily. The components include the
ligand RANKL, its specific receptor RANK (receptor activator of nuclear factor-κΒ) and
OPG, a soluble “decoy” receptor. By binding to RANK, RANKL enhances bone resorption
by increasing osteoclast formation, inhibiting osteoclast apoptosis and increasing
osteoclast activity. The activity of RANKL is counterbalanced physiologically by
circulating OPG. Denosumab binds RANKL (similar to OPG) and inhibits RANK induced
activation of RANKL, thus suppressing osteoclasts. Nitrogen-containing bisphosphonates
(aminobisphosphonates), including pamidronate, ibandronate, and zoledronate, impede
protein prenylation and bone resorption by osteoclasts by inhibiting the mevalonate
pathway (farnesyl diphosphate synthase) disrupting the signaling functions of key
regulatory proteins.

Question 34.14 The answers are A and B


SIADH is defined as a hypo-osmolar or dilutional hyponatremia with excessive natriuresis.
Hyponatremia occurs because ADH secretion continues even after plasma osmolality falls
below the threshold for ADH release. Patients are clinically euvolemic, and presence of
hypovolemia or hypervolemia should prompt alternate diagnosis. Decreased effective
serum osmolality (<275 mOsm/kg of water) with excessive natriuresis (Urine sodium
>20 to 30 mmol/L) and inappropriate urinary concentration (Urine osmolality >100
mOsm/kg of water) are hallmarks of SIADH. SIADH is commonly reported with small cell
lung cancer, head and neck carcinomas, hematologic malignancies, and non–small cell
lung cancer.

Question 34.15 The answer is B


Rasburicase is a recombinant version of urate oxidase, an enzyme that metabolizes uric
acid to allantoin. Because urate oxidase degrades uric acid rather than prevent its
synthesis, as does allopurinol, rapid reduction in uric acid occurs following its
administration without the accumulation of precursors. It has a quick onset of action and
studies in adults and children have shown uric acid levels fall to 0.5 to 1 mg/dL within 4
hours of rasburicase injection with low levels maintained throughout treatment.
Allopurinol lowers uric acid by inhibiting xanthine oxidase, the enzyme responsible for
converting hypoxanthine to xanthine and xanthine to uric acid. Guanse and Hypoxanthine
phosphoribosyltransferase are other enzymes involved in purine metabolism.

Corresponding chapter in Cancer: Principles & Practice of Oncology, Tenth Edition: 119 (Superior Vena Cava Syndrome), 120
(Increased Intracranial Pressure), 121 (Spinal Cord Compression), and 122 (Metabolic Emergencies).
INDEX

A
AAW. See Antiandrogen withdrawal (AAW)
Abiraterone, 82, 86, 128
Abscopal effect of radiation, 70, 75
Absolute risk reduction, 44, 48
ABT-737, 17, 24
Accelerated fractionation, 75–76
ACOG. See American College of Obstetricians and Gynecologists (ACOG)
ACP. See American College of Physicians (ACP)
Acquired immunodeficiency syndrome (AIDS)
AIDS-defining cancer, 327, 330
CNS lymphoma in, 328, 330
non–AIDS-defining cancer, 328, 330
Acute lymphoblastic leukemia (ALL)
CNS prophylaxis in, 297, 302
imatinib in, 297, 302
maintenance therapy for, 297, 302
Acute megakaryocytic leukemia, mediastinal NSGSTs and, 142, 148
Acute myeloid leukemia (AML), 293, 299
BEP and, 217, 221
consolidation therapy in, 296, 302
core-binding factor, 293, 299
cytopathology, 293, 299
DNMT3A mutations in, 293, 299
in elderly, treatment of, 294, 295, 300, 301
induction therapy in, 294, 299
monosomal karyotype, 294, 300
nucleophosmin 1 (NPM1) mutation in, 298, 303
poor prognostic factor in, 294, 300
prognostic factors for, 294, 300
residual AML blasts, treatment of, 297, 302
Acute promyelocytic leukemia (APL)
arsenic salvage therapy in, 295, 300
ATRA and headache, 296, 301
and differentiation syndrome, 296, 301
fresh-frozen plasma in, 295, 301
relapsed, treatment in, 296, 301–302
risk of relapse in, 296, 301
ADCC. See Antibody-dependent cell-mediated toxicity (ADCC)
Adenocarcinomas
endometrioid, 225, 232
esophageal, 151, 156
of gastroesophageal (GE) junction, 152, 157
of pancreas, 165, 174
small bowel, 183, 186
Adjuvant chemotherapy
purpose of, 98, 102
relapses in early-stage colon cancer treated with, 98, 103
Adoptive cell therapy, 92, 95
purpose of, 91, 95
Afatinib, 131, 136
AIDS. See Acquired immunodeficiency syndrome (AIDS)
AIDS-associated lymphomas, combination of rituximab with chemotherapy for, 327, 330
Akt, 12
Alcohol consumption, and risk of cancer, 35, 40
Alemtuzumab, 302–303
ALK. See Anaplastic lymphoma kinase (ALK)
ALL. See Acute lymphoblastic leukemia (ALL)
Allele-specific oligonucleotide-based quantitative polymerase chain reaction (ASO-PCR), 321
Allogeneic stem cell transplantation, 333, 338
candidate for, 335, 339–340
choice of conditioning regimen, 335, 340
cyclophosphamide and cardiac toxicity, 334, 338
graft-versus-host disease, 337, 341
HLA mismatched unrelated donor allogeneic SCT, 336, 341
late complications after, 336, 340
late hemorrhagic cystitis in, causes of, 336, 340
myeloablative conditioning regimens in, 333, 338
objectives of, 333, 338
risk of CMV reactivation after, 335, 339
All-trans retinoic acid (ATRA), topical, 55
5-alpha-steroid reductase inhibitors, in prostate cancer, 53, 56
Alpha-Tocopherol, Beta-Carotene Trial (ATBC), 55
Alveolar rhabdomyosarcoma, 270
Alveolar soft part sarcoma
cytogenetic abnormality with, 268, 273
sunitinib for, 268, 273
American College of Obstetricians and Gynecologists (ACOG), 45, 48
American College of Physicians (ACP), 45, 48
AML. See Acute myeloid leukemia (AML)
Amyloidosis, 318, 324–325
Anal cancer, early localized, therapy for, 192, 196
Anaplastic large-cell ymphoma, 284, 289
Anaplastic lymphoma kinase (ALK), 84
Anaplastic oligodendroglioma, IDH1 mutations in, 258, 263
Androgen receptor, 206, 213
Angiogenesis, 2, 7, 38
Angiogenesis inhibitors, and apoptosis induction, 17, 24
Angiosarcoma, and risk of nodal metastases, 266, 271
Anthracycline cardiotoxicity, 80, 85
Antiandrogen withdrawal (AAW), 205, 212
Antiangiogenesis agents, 82, 87
Antiangiogenic therapy, side effects of, 19, 26
Antibody-dependent cell-mediated toxicity (ADCC), 87
Anti-CD19 CAR T cells, immunotherapy with, 92, 95
Antimitogenic signaling, activation of, 16, 22–23
APC gene mutation, 195
APL. See Acute promyelocytic leukemia (APL)
Apoptosis, 7, 16, 23
and lysosome activation, 18, 24
Aromatase inhibitors, use of, in breast cancers, 250, 254
Arsenic trioxide, 301
Asparaginase and methotrexate, 82, 86
ASPIRE trial, 323
Aspirin, in colorectal cancer, 52, 56
Astrocytomas, 260
alterations of p53 gene in, 255, 260
grade II, 257, 261
grade II, somatic mutation in, 259, 264
Ataxia-telangiectasia mutated (ATM) kinase, 39
Atelomeric chromosomes, 12
ATR/CHK1 signaling, 4, 10
Aurora kinase B, role of, 4, 10
Autoantibodies in CLL, 308, 313
Autoimmune adverse events, 89, 93
Autologous stem cell transplantation, 320–322, 324–325
DLBCL in first relapse and, 336, 341
in multiple myeloma, 316, 320–322
neuroblastoma and, 336, 340
testicular cancer and, 336, 340
Autophagy, 7, 17, 24
5-Azacytidine, 3, 9
5-Aza-2′-deoxycytidine, 9

B
Bacillus Calmette–Guérin (BCG) vaccine, intravesical, 201, 208
Bad, 24
BAP1 tumor suppressor gene, mutations in, 133, 137
Barrett esophagus, 151, 156
high-grade dysplasia in, 152, 157
Basal cell nevus syndrome, mutations in PTCH1 gene and, 276, 280
Base excision repair (BER), 4, 10
Bayesian methods, in planning phase I and II trials, 110, 116
B-cell lymphomas, 284, 289
Bcl-2 inhibitors, 17, 24
BCS. See Breast-conserving surgery (BCS)
Benign teratoma, 141, 147
BEP. See Bleomycin, etoposide, and cisplatin (BEP)
BER. See Base excision repair (BER)
Berry formulations, 53, 57
Beta-human chorionic gonadotropin (hCG), 231, 237
Bevacizumab, 18, 25, 87, 102
for recurrent GBM, 257, 261
Bicalutamide, 128
discontinuation of, 205, 212
Bilateral salpingo-oophorectomy (BSO), in BRCA carriers, 61, 66
Biliary cancers, adjuvant therapy for, 168, 176–177
Bim, 17, 24
Birt–Hogg–Dubé syndrome, 208
Bisphosphonates
for malignancy-associated hypercalcemia, 345, 349
nitrogen-containing, 349
for pain associated with bone metastases, 251, 254
and risk of recurrent breast cancer, 53, 57
Bladder cancer
cisplatin-based multiagent chemotherapy for, 201, 208–209
combined radiation and chemotherapy in, 202, 209
intravesical therapy, 201, 208
muscle-invasive, 208–209
neoadjuvant chemotherapy for, 202, 209
Schistosoma haematobium and, 204, 211
tumor suppressor genes in pathogenesis of, 201, 208
urinary diversion after cystectomy, options for, 202, 209
Bleomycin, etoposide, and cisplatin (BEP)
for granulosa cell tumors, 244
for nonseminomatous germcell tumor, 216, 217, 220, 221
Bone marrow and GI precursor cells, 99, 104
Bortezomib, 24, 87, 290
anticancer activity of, 98, 103
for multiple myeloma, 82, 86
Brachytherapy, 72, 77
BRAF inhibitors, 81, 86
BRAF V600E mutations
and resistance to cetuximab in colorectal cancer, 99, 103
treatment in, 81, 86
BRCA1 and BRCA2 gene mutations, 62, 66–67
BRCA2 gene variant, and genetic testing, 61, 66
BRCA1 mutation, and risk of ovarian cancer, 243
Breakage–fusion–bridge cycles, 5, 12
Breast cancer
ACS guidelines for MRI screening, 248, 252
adjuvant tamoxifen and, 250, 253
annual mammography and MRI at age 30, 44, 48
aromatase inhibitors and, 250, 254
BRCA1-associated, 247, 252
core needle biopsy for diagnosis of, 248, 252
Cowden syndrome and, 248, 252
DCIS, 249, 253
Gail model, 249, 253
by germline mutations, 247, 252
Her2/neu amplification in, 3, 10
incidence of, 32
inflammatory, 251, 254
Li–Fraumeni syndrome and, 247, 252
local recurrence after breast-conserving therapy, 249, 253
Luminal A subtype, 247, 252
male, 251, 254
obesity and physical activity as risk factors for, 36, 41
oncotype DX testing in, 248, 252
oophorectomy before menopause and risk of, 248, 253
preoperative chemotherapy for HER2+ breast cancer, 250, 253
P1 trial, 253
routine surveillance for early stage cancer patients, 251, 254
screening for, 44, 45, 48
stage IIA, 249, 253
STAR trial, 253
tamoxifen and raloxifen, contraindications to use of, 248, 252
taxanes, use of, 250, 254
Breast-conserving surgery (BCS)
contraindication to, 249, 253
local recurrences after, 249, 253
Breast self-examination (BSE), 44, 48
Bruton tyrosine kinase (BTK) inhibitor, 314
BSE. See Breast self-examination (BSE)
Burkitt lymphoma, 184, 187
endemic and sporadic, 283, 289
translocation t(8;14) in, 283, 289

C
CA 19-9, 171
CA-125, and ovarian cancer, 240, 243
Cabazitaxel, 81, 85
Cadherins, 20, 27
Calcitonin, 349
Calcium, and recurrent colorectal adenoma, 52, 56
Calretinin, 133, 137
Cancer. See also specific type
cell culture experiments in preclinical studies of, 1, 7
cytogenetic abnormalities in, 4, 10–11
gene amplification in, 3, 9–10
genetic mutations in, 1, 7
as hereditary disease, 60, 65
progression of, 2, 7
risk of
alcohol consumption and, 35, 40
dietary influence on, 35, 40
fruit and vegetable consumption and, 36, 40
specific nutrients and, 36, 40–41
strategy to promote apoptosis in, 5, 12
Cancer immunotherapy, approaches to, 90, 93
Cancer vaccines, treatment with, 90, 91, 94–95
Carbamazepine, 84
Carboplatin and paclitaxel, for NSCLC with squamous histology, 130, 134–135
Carcinogenic effects of chemicals, 34, 38–39
Carcinoids of small bowel, 183, 186
Carfilzomib, 86
in multiple myeloma, 317, 323–324
Carney triad, and GIST, 183, 186
Carotene and Retinol Efficacy Trial (CARET), 55
CARs, 95
Case-control studies, 29, 31
Caspases, role in apoptosis, 98, 102
Castrate-resistant prostate cancer (CRPC), 211, 212
Catenins, 27
CD 10, 222
CD 20, 222
CD 30, 222
CDH1 carriers, optimal surgical therapy for, 54, 57–58
CDH1 gene, mutations in, 154, 159
CDK2, 213
CDKs. See Cyclin-dependent kinases (CDKs)
CD117 (c-Kit) stain, 266, 271
CEBPA gene, 299
Celecoxib, 56, 87
Cell culture experiments, in studies of cancer cells, 1, 7
Cell cycle, 1, 7
DNA damage checkpoints, 2, 8
regulation of, 7
Cell-cycle exit, 15, 22
Cell lysis, 24
Cellular assays, 7
Central nervous system (CNS) prophylaxis, 297, 302
Central nervous system (CNS) tumor
contrast-enhanced MRI images, 256, 261
primary CNS lymphoma, 256, 261
spinal ependymoma, 256, 260
Cerebrospinal fluid (CSF) Epstein–Barr virus (EBV) polymerase chain reaction (PCR) test,
328, 330
Cervical cancer
cervical conization, 227, 233
epidemiology of, 226, 232–233
extracervical involvement, 227, 233
in HIV-infected patients, 327, 330
locoregionally advanced, treatment of, 228, 234
recurrence after combined chemoradiotherapy, 228, 234
screening for, 45, 48–49
small stage IB1 tumors, therapy for, 227, 233
spread of, 227, 233
staging of, 227, 233
Cetuximab, 136
in colorectal cancer, 100, 104
in HNSCC, 121, 125, 126
Charged particle beams, 73, 77
Chemical carcinogens, 34, 35, 38–39
Chemotherapy. See also specific type of cancer
in incurable, advanced, metastatic disease, goals of, 97, 102
indicator of effectiveness of, 98, 102–103
and radiation therapy, combining of, 71, 76
Child–Pugh C cirrhosis and unresectable HCC, 170, 179
Cholangiocarcinoma, risk factor for, 167, 176
Chondrosarcomas, 268, 272
Chromosomal rearrangements, 12
Chronic lymphocytic leukemia (CLL)
autoimmune complications in, 308, 313
ibrutinib in, 309, 314
IWCLL revised criteria, 308, 313–314
rituximab in, 309, 314
Chronic myeloid leukemia (CML)
adherence to therapy in, 308, 313
BCR-ABL fusion proteins in, 307, 312
blast-phase transformation, mutations in, 307, 312
conditioning regimens in, 333, 338
criteria for diagnosis of accelerated phase, 307, 312
cytogenetic response in, 305, 310
GVL effect in, 334, 338
hematopoietic cell transplantation (HCT) in, 307, 312
imatinib in, 306, 310–312
minimal residual disease in, 310
ponatinib in, 306, 311
poor prognosis in, factors associated with, 308, 313
relapse on therapy in, 306, 311
Sokal risk score, 305, 310
T315I mutation in, 306, 311
Cigarette smoking, and lung cancer, 33, 37
Circulating tumor cells (CTCs), 20, 27
extravasation of, 20, 27
Cisplatin radiosensitization, 76
Clinical trials, 107, 113
Bayesian methods in planning of, 110, 116
planning with interim analyses, 111, 116
power and hypothesis testing in, 110, 115
survival data. analysis of, 111, 116
CLL. See Chronic lymphocytic leukemia (CLL)
Clonidine, for tobacco dependence, 51, 55
Clonorchis sinensis, 211
ClonoSEQ MRD assessment, 321
CML. See Chronic myeloid leukemia (CML)
C-myc amplification, in small cell lung cancer, 3, 9–10
60Cobalt teletherapy unit, 75
Cobicistat, 332
Cohort studies, 29, 31
Colitis from T-cell activation, 89, 93
Colon cancer, obesity and physical activity as risk factors for, 36, 41
Colorectal cancer (CRC), 9. See also Hereditary nonpolyposis colon cancer (HNPCC)
BRAF V600E in, 193, 197
colonoscopy for screening for, 46, 49
endoscopy with polypectomy for, 46, 49
HNPCC and, 194
incidence of, 32
maintenance therapy with capecitabine and bevacizumab, 193, 196–197
microsatellite instability (MSI) and, 189, 194
resistance to cetuximab in, 99, 103
resistant to treatment with cetuximab, 192, 196
stage II, high-risk features for, 193, 197
stage III, adjuvant therapy for, 190, 195
Compton scattering, 76
Computed tomography contrast perfusion, 100, 104–105
Conditioning regimen, choice of, 335, 340
Confidence intervals, 29, 31
Confounding factors and interaction, 29, 31
Conventional 3 + 3 phase I designs, 107, 113
Core-binding factor (CBF) AML, 293, 299
Cowden syndrome, 186
and risk of breast cancer, 248, 252
Craniopharyngioma, sexual dysfunction in, 258, 263
CRC. See Colorectal cancer (CRC)
Crizotinib, 136
Crizotinib resistant tumors, treatment for, 131, 136
CROSS trial, 157–158
CRPC. See Castrate-resistant prostate cancer (CRPC)
CT colonography, 46, 49
CTCs. See Circulating tumor cells (CTCs)
CTLA-4, 91, 94
and inhibition of T cells, 90, 94
CXCL12, 20, 27
CXCR4, 20, 27
Cyclin-dependent kinase 4, 7
Cyclin-dependent kinases (CDKs), 7, 15, 22
Cyclins, 22
B-type, 15, 22
Cyclophosphamide, cardiac toxicity by, 334, 338
Cytokeratin, 222
Cytostatic agents, 114
early-phase trials of, 109, 114
Cytotoxic chemotherapy, zidovudine and, 327, 330

D
Daratumumab, 322
Dasatinib, 310
side effects of, 306, 311
Death receptor pathway, 17, 24
Denosumab, mechanism of action of, 345, 349
Depsipeptide, 9
DES. See Diethylstilbestrol (DES)
Desmoid tumors, 189, 194
Dexamethasone
in ALL induction therapy, 297, 302
in malignant spinal cord compression, 344, 348
DIC. See Disseminated intravascular coagulopathy (DIC)
Dicentric chromosome, 12
Diet-derived natural products, 53, 57
Diethylstilbestrol (DES), 234
exposure in utero and clear cell vaginal cancer, 228, 234
Differentiation syndrome, 296, 301
Diffuse alveolar hemorrhage, induction therapy in AML and, 294, 299
Diffuse large B-cell lymphoma (DLBCL), 284, 289, 347
advanced-stage, 286, 290
extranodal NK/T-cell, 287, 291
germinal center B-cell (GCB), 286, 290
R-CHOP chemotherapy for, 286, 290
Dihydropyrimidine dehydrogenase (DPD), 195
deficiency and risk for 5FU toxicities, 190, 195
deficiency of, 80, 84
Disseminated intravascular coagulopathy (DIC), 348
DLBCL. See Diffuse large B-cell lymphoma (DLBCL)
DLI. See Donor lymphocyte infusion (DLI)
DLL4, 26
DNA double-strand breaks
dose-related cellular response to, 70, 74
mechanisms of repair for, 69, 74
radiation-induced, 69, 74
DNA hypomethylating agents, 3, 9
DNA mismatch repair (MMR) gene, germline mutations in, 189, 194
genetic counseling and testing of at-risk family members, 190, 194–195
DNMT3A gene, 299
Donor lymphocyte infusion (DLI), 335, 340
Dose-dense chemotherapy, 99, 103
Double-strand breaks (DSBs), 10. See also DNA double-strand breaks
DPD. See Dihydropyrimidine dehydrogenase (DPD)
Drop-ins and drop-outs, effect of, 43, 47
DSBs. See Double-strand breaks (DSBs)
Dutasteride, and prostate cancer, 53, 56
Dynamic-contrast magnetic resonance imaging, 100, 104–105

E
EBV. See Epstein-Barr virus (EBV)
E-cadherin, 20, 27
E-cadherin mutation testing, 154, 159
Ecologic studies, 30, 32
Efavirenz, 332
Effect modification. See Confounding factors and interaction
EGFR mutations, 129, 134
EGFR–tyrosine kinase inhibitors (TKIs), 131, 134, 135
ELOQUENT-2 trial, 322
Elotuzumab, 322
EML4-ALK translocation, 134
ENDEAVOR trial, 324
Endometrial cancer
hysterectomy for grade 2 endometrioid cancer, 230, 236
IIIC1 stage, 231, 236
radiation and chemotherapy for, 231, 237
risk factors for, 226, 232
risk factors for development of, 230, 236
type I, 225, 232
type II, 225, 232
Endometrioid adenocarcinomas
genetic changes in, 225, 232
risk factors for, 225, 232
Enzalutamide, 128
EORTC 40983 study, 196
Ependymoma, 260
adjuvant radiation therapy after resection in, 258, 263
radiation therapy for, 257, 262
surgical resection for, 262
Epidermal growth factor receptor (EGFR), role of, in head and neck SCC (HNSCC), 119, 124
Epigenetic therapy, 81, 86
Epithelial mesotheliomas, 133, 137
Epstein–Barr virus (EBV), 34, 38, 256, 261
Epstein–Barr virus latent gene EBNA, 89, 93
Erlotinib, 85, 97, 102, 136, 145
resistance to, 129, 134
Esophageal adenocarcinoma, risk factors for, 151, 156
Esophageal cancer, 151, 152, 156–157
molecular biology of, 152, 157
perioperative chemotherapy in, 153, 158
European Organization for Research and Treatment of Cancer (EORTC) trial, 261
Ewing sarcoma, 268, 273
and cytogenetic abnormalities, 265, 270
Excisional biopsy, 252
Excision repair mechanisms, 4, 10
Exclusive inhibitors, 87
EXTREME trial, 125, 128
Ezrin, 27

F
Familial adenomatous polyposis (FAP), 54, 58
Familial cancer syndromes, germ line mutations in, 1, 7
Familial malignant melanoma syndrome, 23
Fanconi anemia
defects in, 10
Fanc genes, abnormalities in, 4, 10
FAP. See Familial adenomatous polyposis (FAP)
Fibrolamellar HCC, 168, 177
Finasteride, and prostate cancer, 53, 56
Fluorescence in situ hybridization (FISH), 299
5-Fluorouracil (5-FU), 21, 28, 80, 84
and leucovorin, 155, 160
and mitomycin for anal cancer, 192, 196
FOLFIRINOX, in adenocarcinoma of pancreas, 165, 174
FOLFOX, in pancreatic cancer, 174
Folic acid, in children with lymphoblastic leukemia, 97, 102
Follicular lymphoma, 284, 289, 341
Follicular thyroid cancer, 67
Foster–Kennedy syndrome, 347
FOXA1, 213
5-FU. See 5-Fluorouracil (5-FU)
Fumarate hydratase gene, 208
Furosemide, 349
Fusion PET-MRI, in multiple myeloma, 319

G
Gail model, breast cancer, 249, 253
Gallbladder cancer
and cholecystectomy, 168, 176
T2 NX (stage II), 167, 176
Gallium nitrate, 349
Gardasil vaccine for HPV, 33, 37–38
Gastric cancer, 151, 154, 156, 158–159
diagnostic modalities in, 154, 159
early-onset diffuse, 154, 159
metastatic, combination chemotherapy for, 155, 160
Gastroesophageal reflux disease (GERD), 151, 156
Gastrointestinal stromal tumors (GISTs), 181, 185, 271
criteria predictive of behavior of, 269, 273
familial and genetic syndromes with, 183, 186
imatinib for, 267, 271
imatinib-resistant, 182, 185
KIT mutations in, 185
regorafenib for, 182, 186
from small intestine, 183, 187
sunitinib therapy for, 182, 185
GBM. See Glioblastoma multiforme (GBM)
Gefitinib (Iressa), 12
Gemcitabine, 76, 97, 102
and cisplatin, 169, 178
and hemolytic uremic syndrome, 345, 348
Gemtuzumab ozogamicin, 301–302, 303
Gene amplification, 3, 9–10
Gene array technology, 3, 8–9
Genetic testing, noninformative, 62, 66, 67
Genome-wide association studies (GWAS), 30, 32
GERD. See Gastroesophageal reflux disease (GERD)
German Rectal Cancer Study, 195–196
Germ-cell cancers, 97, 102
Germ cell tumors of ovary, 242, 245
Germinal center B-cell (GCB) diffuse large B-cell lymphoma, 286, 290
Gestational trophoblastic disease (GTD) with complete mole, 226, 232
Gestational trophoblastic neoplasia (GTN), 231, 237
methotrexate for low-risk disease, 231, 237
Giant cell tumor (GCT), of bone, 268, 272
GISTs. See Gastrointestinal stromal tumors (GISTs)
Glasgow prognostic score (GPS), 34, 38
Glioblastoma, 260
Glioblastoma multiforme (GBM), 255, 256, 260
concurrent brain radiation with temozolomide in, 258, 263
dysregulated core pathways in, 259, 263
gene expression profiling, 263
mesenchymal group and loss of NF1 expression, 258, 263
radiation therapy for, 257, 261–262
recurrent, 257, 261
Gompertzian kinetics, 99, 103
Graft manipulation ex vivo to purge tumor cells, 336, 341
Graft-versus-leukemia (GVL) effect, 338
in chronic-phase CML, 334, 338–339
donor T lymphocyte and, 336, 341
NK cells and, 336, 341
Granulosa cell tumors, 223
of ovary, 241, 244
G1/S phase, cell cycle, 2, 8
G1/S-phase transition, cyclins E and A accumulation for, 15, 22
GTN. See Gestational trophoblastic neoplasia (GTN)
Guanase, 349
GVL effect. See Graft-versus-leukemia (GVL) effect
Gynecologic Oncology Group (GOG), 231, 237

H
Hairy cell leukemia (HCL), 308, 313
BRAF V600E mutation in, 309, 314
Hayflick phenomenon, 4, 11
HCC. See Hepatocellular carcinoma (HCC)
hCG. See Beta-human chorionic gonadotropin (hCG)
HCL. See Hairy cell leukemia (HCL)
HDGC. See Hereditary diffuse gastric cancer syndrome (HDGC)
Head and neck squamous cell carcinoma (HNSCC)
adjuvant chemotherapy in, 121, 126
cetuximab in, 121, 126
combinations of chemotherapy agents, 120, 125
concurrent chemotherapy and radiation therapy in, 121, 125
epidermal growth factor receptor in, role of, 119, 124
HPV-related, 120, 124
induction chemotherapy in, 122, 127–128
locally advanced, 122, 127
neck dissections, complications from, 120, 125
risk factors for, 119, 124
second primary cancers with, 122, 127
stage IVA, 120, 124
Helicobacter pylori infection, 151, 156
and risk of intestinal-type gastric adenocarcinoma, 53, 57
Hemolytic uremic syndrome (HUS), 345, 348
gemcitabine and, 345, 348
mitomycin and, 345, 348
Hemorrhagic cystitis, 80, 84
Hepatitis B virus, 177
Hepatitis C infection, 169, 177
Hepatitis C virus, 34, 38
Hepatitis D virus (HDV), 38
Hepatoblastoma, 168, 177
Hepatocellular carcinoma (HCC), 34, 38, 166, 175
clinical suspicion for, 168, 177
fibrolamellar, 168, 177
hypoglycemia and, 166, 175
liver transplantation in, criteria for, 166, 175
paraneoplastic syndromes with, 166, 175
risk factors for, 167, 175
screening and prevention for, 167, 175–176
staging systems for, 166, 175
surveillance for, 170, 178
Hereditary cancer syndromes, expression in, 60, 64
Hereditary diffuse gastric cancer syndrome (HDGC), 61, 65–66, 159
informative genetic test, 61, 66
noninformative genetic test, 66
Hereditary leiomyomatosis and renal cancer syndrome, 208
Hereditary nonpolyposis colon cancer (HNPCC), 9, 10, 159
and colorectal cancer, 194
diagnosis of, 190, 194
microsatellite instability (MSI) by mutations in DNA MMR genes, 194
MSH2 gene and, 190, 195
Hereditary nonpolyposis colorectal cancer syndrome, 230, 236
Hereditary papillary renal cancer, 207, 208
MET mutations in, 199, 207
Her2/neu amplification, in breast cancer, 3, 10
HER2 testing, 153, 158
HevyLite assay, 319
Hierarchy model of tumor heterogeneity, 21, 27
HIF. See Hypoxia-inducible factor (HIF)
High-grade dysplasia, in Barrett esophagus, 152, 157
Hilar cholangiocarcinoma, 167, 176
HIV. See Human immunodeficiency virus (HIV)
HIV-associated Hodgkin lymphoma (HL), 328, 331
HIV-infected patients
anogenital cancers in, 328, 331
cervical cancer in, 327, 330
stage IIIB anal carcinoma, treatment for, 329, 331–332
viruses in development of cancers in, 329, 331
HNPCC. See Hereditary nonpolyposis colon cancer (HNPCC)
HNSCC. See Head and neck squamous cell carcinoma (HNSCC)
Hodgkin lymphoma, 327, 330, 347
in pregnancy, 288, 292
Homologous recombination (HR), 69, 74
Hoosier Oncology Group (HOG), 136
Hormonal therapy in ovarian cancer, 240, 244, 245
Hormone replacement therapy (HRT), decline in use of, 30, 32
HOVON-65/GMMG-HD4 trial, 320
HPV. See Human papilloma virus (HPV)
HPV-associated vulvar cancer, 228, 235
HPV vaccines, 233
Human cancers, cytogenetic abnormalities in, 4, 10–11
Human cells, expressed genes in, 1, 7
Human Genome Project, 1, 7
Human herpesvirus-8, and primary effusion lymphoma, 284, 289
Human immunodeficiency virus (HIV), 227, 233
Human papillomavirus (HPV), 226, 232
prevention and treatment of infection, 226, 233
Human papilloma virus (HPV)
related HNSCC, 120, 124
vaccine, 90, 93
HUS. See Hemolytic uremic syndrome (HUS)
Hyperfractionation, 71, 75
Hyperkalemia, tumor lysis syndrome and, 344, 348
Hypoxanthine phosphoribosyltransferase, 349
Hypoxia-inducible factor (HIF), 208
pathway, 85
and tumor angiogenesis, 200, 208

I
IBC. See Inflammatory breast cancer (IBC)
Ibrutinib, 290
in chronic lymphocytic leukemia, 309, 314
IGRT. See Image-guided radiation therapy (IGRT)
IL-2 for melona, 90, 93–94
Image-guided radiation therapy (IGRT), 78
Imatinib, 12, 24, 81, 85, 100, 104, 145, 181, 185
in chronic myeloid leukemia, 306, 310–312
Immune lymphocytes, 89, 93
Immunotherapy to destroy cancer cells, mechanism for, 89, 93
IMRT. See Intensity-modulated radiation therapy (IMRT)
Inflammation, role of, in carcinogenesis, 34, 38
Inflammatory breast cancer (IBC), 251, 254
INK family proteins, 23
Intensity-modulated radiation therapy (IMRT), 71, 76, 78
and risk of xerostomia, 122, 127
Intention-to-treat analysis, 111, 116
Intention to treat (ITT) analysis, 99, 103
Interferon (IFN)-α, 16, 22
Intergroup Trial, 155, 160
International Gastric Cancer Linkage Consortium (IGCLC), 159
International Germ Cell Consensus Classification, 219, 222
International Myeloma Working Group, 319
Intracranial pressure, increased
and midbrain dysfunction, 344, 348
signs of, 343, 347
Intraepithelial neoplasias, 52, 55
Intravesical therapy, for bladder cancer, 201, 208
Ionizing radiation (IR)
and DNA damage/repair process, 69, 74
repair of damage from, 35, 39
risk of cancer from, 35, 39
Ipilimumab, and nivolumab, 91, 94
Ipilimumab-related endocrinopathy, 278, 281
IR. See Ionizing radiation (IR)
Irinotecan, and neutropenia, 191, 195
Isochromosome 12p (i[12p]), 147
Isocitrate dehydrogenase 1 (IDH1) gene, 260
Ixazomib, 325

K
Kaposi sarcoma (KS), 329, 331
and HAART therapy, 327, 330
staging system for, 328, 331
Ketoconazole, 84
Kidney cancer, anti-VEGF drugs in, 81, 85
Klinefelter syndrome
and germcell tumor, 216, 220–221
mediastinal NSGSTs and, 142, 148
KRAS mutation, 84
KS. See Kaposi sarcoma (KS)

L
Lapatinib, 81, 85
Large B-cell lymphoma, 219, 223
Laryngeal squamous cell carcinoma (SCC)
adjuvant chemotherapy in, 121, 126
concurrent chemotherapy and radiation therapy in, 119, 121, 124, 126
Lead-time bias, 43, 47, 49
Leiomyosarcoma
intra-abdominal, 266, 271
lung metastases, 267, 271
recurrence of, 266, 270
Lenalidomide, 334, 339
Length bias, 47
Leukoplakia, 52, 55
Leuprolide, 128
Leydig cell tumor, 223
Li–Fraumeni syndrome, 23, 186, 247, 252, 255, 260
and soft tissue sarcomas, 265, 270
Linear accelerator (LINAC), modern, 70, 75
Liposarcoma, high-grade extremity, 266, 271
Liver, sinusoidal obstruction syndrome of, 334, 339
Liver transplantation, 169, 178
Lovastatin, 56
LS. See Lynch syndrome (LS)
Lumbar puncture, 285, 289–290
Luminal A breast cancer, 247, 252
Lung Adjuvant Cisplatin Evaluation (LACE) meta-analysis, 137
Lung cancers
CT-guided biopsy, 129, 134
genomic profile of, 134
tumors from never smokers, 134
tumors from smokers, 129, 134
incidence of, in women, 32
and malignant spinal cord compression, 344, 347
platinum-based doublet chemotherapy, 130, 134
Lymphoma
adult, 283, 289
anaplastic large-cell, 284, 289
B-cell, 284, 289
Burkitt, 283, 289
childhood, 283, 289
International Prognostic Index (IPI) score for, 284, 289
and malignant spinal cord compression, 344, 347
mantle cell, 285, 290
primary effusion, 284, 289
T-cell neoplasms, 284, 289
Lynch II syndrome, 230, 236
Lynch syndrome (LS), 54, 57, 59, 64
autosomal-dominant inheritance in, 64
cancer risk from, 64
and cancer screening, 60, 64
and cervical cancer, 63, 67
and endometrial cancer risk, 54, 57
gastric cancers and, 154, 159
and genetic testing, 60, 65
MSH2 gene mutation, inheritance of, 64

M
Malabsorption after Whipple surgery, 164, 172–173
Male breast cancer, 251, 254
Male urethral cancers, risk factors for, 201, 209
Malignant mesotheliomas
epithelial, 133, 137
immunohistochemistry markers in, 133, 137
poor prognostic factors in, 133, 137
Malignant spinal cord compression (MSCC), 347
causes of, 344, 347
corticosteroids in, 344, 348
Malignant teratoma, 147
Mammography, 48
Mantle cell lymphoma, 285, 286, 290
Maximum tolerated dose, 113
Mayo Stratification for Myeloma and Risk-adapted Therapy (mSMART) criteria, 319–320
MBL. See Monoclonal B lymphocytosis (MBL)
MDS. See Myelodysplastic syndrome (MDS)
Mediastinal germ cell tumors, 141, 146–147
Mediastinal seminoma, 142, 148
cisplatin plus etoposide for, 143, 148
Medullary thyroid carcinoma (MTC), and preoperative basal calcitonin levels, 54, 57
MEK mutations, 79, 84
Melanoma
adjuvant stereotactic radiosurgery for, 277, 281
anatomical sites for, 276, 280
BRAF V600E mutation in, 275, 280
brain MRI for suspected solitary intracranial metastasis, 276, 280
Breslow thickness, 276, 280
cutaneous, 275, 280
genetic counseling, 275, 280
grade III autoimmune colitis, 278, 281
incidence of, 30, 32
ipilimumab in metastatic melanoma, 278, 281
risk factor for, 275, 280
stage IIIC, 277, 281
tumor biopsy and brain MRI examination, 277, 281
wide excision and sentinel lymph node mapping, 276, 280
Melphalan, 334, 339
Memorial Sloan Kettering Cancer Center (MSKCC) nomogram, for sarcoma-specific mortality,
266, 270
Meningioma, 255, 260
grade II resection for, 257, 262
6-Mercaptopurine (6MP), 80, 84
Mesna
hemorrhagic cystitis and, 80, 84
mechanism of action of, 269, 273
Meta-analysis, 112, 117
Metastasis progression genes, 20, 26
Metastasis, risk of, 19, 26
Metformin, 57
Methotrexate
for gestational trophoblastic neoplasia, 231, 237
high-dose, for immunocompetent patients with primary CNS lymphoma, 258, 262–263
Methylation, 12
Methyl guanine methyl transferase (MGMT) gene methylation, 261
Metronomic low-dose chemotherapy, 19, 26
Microarray technology, 2, 8
MicroRNA (miRNA), 2, 8, 16, 23
Microsatellite instability (MSI), 10, 189, 194
in colon cancer, 3, 10
and 5-fluorouracil chemotherapy, 3, 9
in type I endometrial cancers, 232
Minimum Information About A Microarray Experiment (MIAME), 2, 8
miRNA array signatures, 8
Mismatch repair machinery of cell, 4, 10
Missense variants, 66
Mitomycin, and hemolytic uremic syndrome, 345, 348
Mitosis, 15, 22
Mitoxantrone, 102
MMR mutation testing, 190, 194
Modified radical neck dissection (MRND), 125
Monoclonal B lymphocytosis (MBL), 309, 314
Monosomal karyotype, 294, 300
MOSAIC study, 195
MSCC. See Malignant spinal cord compression (MSCC)
MSI. See Microsatellite instability (MSI)
Mucosal-associated lymphoid tissue (MALT) lymphoma, 183, 186
Muir–Torre syndrome, 194
Multiple myeloma
antimyeloma therapy for, 315, 319
autologous stem cell transplantation in, 316, 320–322
carfilzomib therapy in, 317, 323–324
conditioning regimen for autologous stem cell transplant, 334, 338
depth of response and prolonged survival in, link between, 316, 320–321
diagnosis of, 319
FISH and cytogenetic studies, 315, 319–320
genomic sequencing in, 316, 320
monoclonal antibodies under investigation in, 322
MRD assessment, 321
panibinostat in, 317, 323
prognosis in, 319
risk classification system, 315, 319–320
Myasthenia gravis, 139, 144
Mycosis fungoides, 287, 291
MYD88, 324
Myeloablative transplantation in CR1, 297, 303
Myelodysplastic syndrome (MDS), 295, 301
MYH-associated polyposis, 192, 195, 196
Myxoid liposarcoma, 270

N
Nasopharyngeal carcinomas, 126–127 121
subtypes of, 126
treatment for, 127
National Comprehensive Cancer Network, 194
National Lung Screening Trial (NLST) eligibility criteria, 46, 49
Nbk/Bik, 24
NBS1 gene, 16, 23
Necitumumab, 136
Necrosis, 24–25
Negative predictive value (NPV), 47
Neoadjuvant chemotherapy, 98, 102
Neuroblastoma, N-myc amplification in, 3, 9
Neurofibromatosis, and GIST, 183, 186
Neurofibromatosis type 2 (NF2), 255, 260
Neurofibromatosis type I, 270
Next-generation sequencing (NGS), 321
NGS. See Next-generation sequencing (NGS)
Nicotine replacement therapy (NRT), 51, 55
Nijmegen breakage syndrome 1, 23
Nijmegen disease, 16, 23
Nilotinib, 310
Nitrogen-containing bisphosphonates, 349
Nivolumab, 91, 94
ipilimumab and, 91, 94
in lung cancer, 91, 94
N-myc amplification, in neuroblastoma, 3, 9
Nodal Marginal zone lymphoma, 341
Nodular lymphocyte-predominant Hodgkin lymphoma (NLPHL), 288, 291
Non-Hodgkins lymphoma, 23
Nonhomologous end-joining (NHEJ) pathway, 69, 74
Non–HPV-associated invasive squamous vulvar cancer, 228, 235
Noninformative censoring, 111, 117
Nonseminomatous germ cell tumor (NSGCT), 141, 146, 220
bleomycin, etoposide, and cisplatin (BEP) for, 216, 217, 220, 221
chemotherapy in, 215, 218, 220, 222
and CNS metastases, 215, 220
in first relapse, 219, 223
and hyperthyroidism, 219, 222–223
late relapse, 217, 221
mediastinal, 146–149, 216, 220–221
risk factor for recurrence of, 218, 222
and surgical resection, 216, 220
Non–small cell lung cancer (NSCLC)
adjuvant chemotherapy after resection for, 130, 135
concurrent chemoradiation for locally advanced NSCLC, 131, 135
concurrent definitive cisplatin and etoposide therapy for, 132, 136
with nonsquamous histology, 134
pemetrexed maintenance therapy in, 130, 135
ROS1 translocations in, 132, 136
with squamous histology, 130, 134–135
Nonsteroidal anti-inflammatory drugs (NSAIDs), in colorectal cancer, 52, 56
Nortriptyline, for tobacco dependence, 51, 55
NRIP1, 213
NRT. See Nicotine replacement therapy (NRT)
NS5A protein product of HCV genome, 169, 177
NSCLC. See Non-small cell lung cancer (NSCLC)
NSGCT. See Nonseminomatous germ cell tumor (NSGCT)
Nucleotide excision repair (NER) pathway, 4, 10
Nurse’s Health Study, 56

O
OCT3/4, 222
Octreotide, 140, 145
Ofatumumab, and rituximab, 83, 87
Oligodendroglioma
allelic loss of chromosome 1p and 19q in, 256, 260
IDH1 mutations in, 258, 263
prognosis of, 256, 261
Oncogenes, 1, 7
Opisthorchis viverrini, 211
Oral cavity SCC
stage II (T2N0M0), close monitoring in, 123, 128
stage IVA (T4aN1M0), surgery for, 120, 125
Osteosarcomas, 268, 272
treatment of, 265, 270
Ovarian cancer, 239, 243
advanced-stage, and risk of relapse, 241, 244
Ashkenazi-Jewish ancestry and, 239, 240, 243, 244
BRCA1 or BRCA2 mutations and, 240, 243, 244
CA-125 and, 240, 243
clinical surveillance in, 242, 245
elevated CA-12, and hormonal therapy, 240, 244
germ cell tumors, 242, 245
granulosa cell tumors, 241, 244
intraperitoneal chemotherapy, use of, 242, 245
nodal metastases, 241, 244
nulliparity and risk of, 240, 243
oral contraceptives and, 241, 243, 244
recurrent, and platinum-based chemotherapy, 242, 245
screening for, 45, 48
stage IA grade 1 mucinous tumor, 239–240, 243
tumor of low malignant potential, therapy for, 240, 243
Overdiagnosis, 45, 49
Oxygen enhancement ratio (OER), 71, 75

P
p53, 99, 104
alterations of, in astrocytoma, 255, 260
mutations of, 3, 9
and senescence, 5, 12
urothelial carcinoma of bladder, alterations in, 202, 210
Paclitaxel radiosensitizing, 76
Pair production, 76
Palbociclib, 100, 104
Pancoast tumor of lung, therapy for, 132, 136
Pancreatic cancer, 163, 171
adjuvant therapy for, 164, 173
familial, BRCA2 gene mutation in, 165, 174
KRAS mutation and development of, 163, 171–172
locally advanced, 164, 173
multiphase CT scan for, 164, 172
nab-paclitaxel and gemcitabine in, combination of, 165, 173
OFF chemotherapy regimen in, 165, 174
p16-mediated CDK inhibition and, 166, 174
risk factors for, 165, 174
stage III, T4N1M0, 163, 172
telomere shortening and, 166, 174
TGF-β receptors in, underexpression of, 166, 174
tobacco smoke and, 171
Pancreatic enzyme supplementation, 164, 172–173
Pancreatic intraepithelial neoplasms (PanINs), 171
Panibinostat, adverse events from, 317, 323
Parosteal osteosarcoma, 268, 272
Parotid gland cancers, treatment for, 122, 127
Partial response, definition of, 98, 103
Patched gene 1, 276, 280
Pazobanib, hepatotoxicity from, 269, 273
Pazopanib, 79, 84
PCNSL. See Primary CNS lymphoma (PCNSL)
PD-1, 91, 94
Pemetrexed, 21, 28
Penetrance, 64
reduced, 59, 64
Penile carcinoma, 206, 213
Peptidome, 3, 9
Pericytes, 18, 25
Peripheral blood mobilized stem cell allografts, 335, 339
Peritoneal mesothelioma, 133, 137–138
Peutz–Jeghers syndrome, 63, 68
Phase III trials, 109, 114–115
Phase II trials, 108, 113
early elimination of ineffective drugs, 108, 113–114
two-stage design, 108, 114
and welfare of patients, 108, 114
Phase I trials, 107, 113
limitations of, 107, 113
Phase 0 trials, 112, 117
Phenytoin, 79, 84
Photoelectric effect, 72, 76
PI3-kinase pathway, 28
Pilocytic astrocytoma, 256, 261
p16INK4a, 1, 7
and senescence, 5, 12
Placental alkaline phosphatase (PLAP), 222
Platinum-based doublet chemotherapy, for metastatic disease, 130, 134
Platinum-resistant tumor cells, 80, 85
Pleural mesothelioma
cisplatin and pemetrexed for, 133, 138
unresectable, 133, 137
Polymerase chain reaction (PCR), 299
Poly (ADP-ribose) polymerase (PARP) inhibition, 82, 86
Positive predictive value (PPV), 47
decrease in, 43, 47
Posttransplant disease relapse risk, 334, 339
Prednisone, 82, 86
Primary CNS lymphoma (PCNSL), 287, 291
methotrexate and rituximab for, 288, 291
Primary (de novo) glioblastoma multiforme (GBM), 255, 260
Primary intestinal lymphomas and secondary intestinal lymphomas, distinction between, 183,
186–187
Primary leptomeningeal lymphoma, 291
Prognostic biomarker, 79, 84
Programmed-cell death pathway, 100, 104
Prolymphocytic leukemia (PLL), 313
Prophylactic total gastrectomy, 57–58
Prostate cancer
adjuvant radiotherapy in, 204, 211
androgen ablation and risk of osteoporosis, 204, 211–212
androgen ablation in high-risk patients, 203, 211
androgen receptor and, 206, 213
antiandrogen withdrawal response, 204, 211
chromosomal translocations involving TMPRSS2 in, 201, 209
high-risk features and therapy, 203, 210
metastasis to liver, 20, 27
nonmetastatic CRPC, 205, 212
and osteoblastic bone metastases, 20, 27
predictors of cancer on biopsy, 203, 210
PTEN and, 206, 212–213
radical retropubic prostatectomy and complications, 204, 211
risk factors, 205, 212
risk factors for, 203, 210
surveillance in, 205, 212
Proteome, 2, 8
Proteomic signatures, 9
Proteomic technologies, 3, 8–9
Protonation, 8
Protooncogene, 23
definition of, 2, 7
Pulmonary carcinoid tumors, 131, 135
Puma, 24

Q
Quiescence, 5, 11

R
Radiation
adverse effects of, 77
depth dose characteristics of, 72, 77
subacute toxicity from, 72, 77
tissue effects from, 71, 76
Radiation effects, role of oxygen in, 71, 75
Radiation-induced cell death, 70, 75
Radiation-induced pneumonitis, 77
Radiation-induced sarcomas, 265, 270
Radiation Therapy Oncology Group (RTOG 9704), 173
Radical neck dissection (RND), 125
Radiotherapy, for localized mediastinal seminoma, 142, 148
Raloxifene, 52, 56
and risk of stroke, 53, 56
Raltitrexed, 174
Ramucirumab (IMC-1121B), 155, 161
Randomized assignment to treatment, 109, 115
Randomized discontinuation design, 100, 104
RANK-ligand inhibitors, for pain associated with bone metastases, 251, 254
Rasburicase, 346, 349
R-CHOP, 263
Real-time quantitative polymerase chain reaction (RQ-PCR), 305, 310
REAL-2 trial, 155, 160
Receptor phosphotyrosine phosphatases (RPTPs), 6, 13
Receptor tyrosine kinases, 6, 12, 13
RECIST criteria, 100, 104
Rectal cancer
T3+/N0-2, 191, 195–196
transanal excision, 191, 196
Reduced penetrance, 59, 64
REGARD study, 160–161
Regorafenib, for GIST, 182, 186
Relative risk reduction, 48
Renal cell cancer, 199, 207
Memorial Sloan Kettering Cancer Center (MSKCC) risk factors, 200, 207
prognostic factors, 200, 207
recurrent metastatic disease, 200, 207
subtypes, 207
temsirolimus in, 200, 208
Renal pelvis urothelial cancers
radical nephroureterectomy and bladder cuff resection for, 202, 210
Replicative senescence, 16, 23
Repopulation, during fractionated radiotherapy, 70, 74–75
Retinoblastoma, 270
Retinoids and beta-carotene, and skin cancer incidence in xeroderma pigmentosum, 52, 55
RET protooncogene, germ line mutations in, 54, 58
Rituximab, in chronic lymphocytic leukemia, 309, 314
Rofecoxib, 56
RTOG 91-11, 124, 126
RUNX1, 299

S
Salivary gland cancers, 128
expression of androgen receptor in, 123, 128
histologic types, 128
Salpingo-oophorectomy, 54, 57
Sarcomas
Ewing sarcoma, and cytogenetic abnormalities, 265, 270
MSKCC nomogram for sarcoma-specific mortality, 266, 270
radiation-induced, 265, 270
soft tissue, 265, 270
Schistosoma haematobium, bladder cancer by, 204, 211
SCLC. See Small cell lung cancer (SCLC)
Selection bias, 30, 31, 47, 49
Selective neck dissection (SND), 125
SELECT trial, 55
Selenium supplementation, and nonmelanoma skin cancer, 52, 55
Seminoma, chemotherapy for, 217, 221
Senescence, 2, 7
and quiescence, 11
as tumor suppressor mechanism, 5, 12
Sensitivity of test, 47
Sequenced human genome, analysis of, 1, 7
Sex cord–stromal tumors, 244
Sexual dimorphism, pharmacokinetic, 79, 84
Sexual dysfunction, in craniopharyngioma, 258, 263
SIADH. See Syndrome of inappropriate antidiuretic hormone secretion (SIADH)
Simon phase 2.5 designs, 109, 114
Sinusoidal obstruction syndrome (SOS), 334, 339
Sipuleucel-T, 91, 95
SIRIUS trial, 322
Small bowel adenocarcinomas, 183, 186
Small cell lung cancer (SCLC)
C-myc amplification in, 3, 9–10
combination chemotherapy for, 343, 347
genomic characterization of, 132, 137
limited-stage, management of, 130, 135
location for primary tumor, 343, 347
and SVC syndrome, 343, 347
Smoking
and lung cancer, 33, 37
and radiotherapy-associated toxicities, 51, 55
SN-38, 195
and glucuronidation, 195
and neutropenia, 191, 195
Soft tissue sarcomas, 265, 270
extremity, 269, 273
of head and neck, 271
locally recurrent, 267, 272
metastatic, 267, 272
chemotherapy treatment of, 267, 272
nonmetastatic, independent predictors of prognosis of, 267, 271
staging of, 266, 270
Sokal risk score, 305, 310
Sorafenib, 17, 24
for hepatocellular carcinoma, 329, 332
Southwest Oncology Group (SWOG), 136
Specificity of test, 47
Spermatic cord liposarcomas, 223
Spinal ependymoma, 256, 260
Splenic marginal zone lymphoma (SMZL), 285, 290
Src, 8
Stage shift, 48, 49
Standard fractionation for radiation therapy, 76
Statins, and cancer risk, 53, 56
Stem cell engraftment, 335, 339
Stem cell transplantation, 333–341. See also Allogeneic stem cell transplantation; Autologous
stem cell transplantation
Stochastic model of tumor heterogeneity, 21, 27
Strontium-89, 348
Suberoylanilide hydroxamic acid (SAHA), 9
Sulindac, 56
Sunitinib-associated hypothyroidism, 182, 185–186
Sunitinib, for alveolar soft part sarcoma, 268, 273
Superior vena cava syndrome, in thymic carcinoma, 140, 145–146
Superior vena cava (SVC) thrombosis, 347
Surveillance, Epidemiology, and End Results (SEER) Program, 30, 32
Survival data. analysis of, 111, 116, 117
Survival time after diagnosis, 48
Syndrome of inappropriate antidiuretic hormone secretion (SIADH), 346, 349
Synovial sarcoma, 270

T
Tamoxifen, 52, 56
for breast cancer, 252–253
TAMs. See Tumor-associated macrophages (TAMs)
Tartrate-resistant acid phosphatase (TRAP) testing, 313
Taxanes, 17, 24
T-cell depletion, and risk of engraftment failure, 335, 339
T-cell–mediated immunity, 89, 93
Telomerase, 5, 11, 23
Telomerase-null mice, 5, 12
Telomeres, 11
Temozolomide, 102
Temsirolimus, 21, 28, 207
Testicular cancer. See also Nonseminomatous germcell tumor (NSGCT)
hCG in, 219, 222–223
immunohistochemical proteins and, 218, 222
large B-cell lymphoma, 219, 223
long-term sequelae of cisplatin-based chemotherapy for, 219, 223
risk factors for, 218, 222
The Cancer Genome Atlas (TCGA), 263
Therapeutic equivalence trials, 110, 115
confidence interval, 110, 116
Thiopurine methyltransferase (TPMT) deficiency, 80, 84
Thymic carcinoid, and multiple endocrine neoplasia type 1, 141, 146
Thymoma, 139, 144
classification schemes and staging systems for, 144–145
incidence of, 144
molecularly targeted agents in, 140, 145
paraneoplastic syndromes with, 140, 145
predictor of long-term prognosis for people with, 139, 144
stage IV thymoma, systemic treatment for, 140, 145
Thyroid function tests, 278, 281
Time to tumor progression, as end point in clinical trials, 101, 105
T315I mutation, 84
TLS. See Tumor lysis syndrome (TLS)
T790M mutation, 84, 99, 103
Tobacco dependence, second-line pharmacotherapy for, 51, 55
Tobacco-induced cancers, 33, 37
Transarterial chemoembolization (TACE), 178
Transformation, 38
Transforming growth factor (TGF)-β, 16, 22
Trastuzumab, 153, 158
risk factors for cardiac dysfunction with, 250, 254
Treatment planning process, 72, 77
TSP-1, 26
Tumor angiogenesis, 18, 25
IL-6/8 and, 19, 26
Tumor-associated macrophages (TAMs), 20, 26–27
Tumor cells, and metastases, 19, 26
Tumor colonies, growth of, 18, 25
Tumor heterogeneity
hierarchy model of, 21, 27
stochastic model of, 21, 27
Tumor lysis syndrome (TLS), 344, 348
and hyperkalemia, 344, 348
Tumor suppressor genes, 1, 7, 16, 23
Tumor vasculature, 18, 25
Turcot syndrome, 222, 255, 260
Type II programmed cell death, 17, 24
Tyrosine kinases, 6, 12

U
UK-ABC-02 trial, 178
Ulcerative colitis, 277, 281
Ultraviolet (UV) light, 35, 40
Undifferentiated pleomorphic sarcoma (UPS) of soft tissue, 272
United States Preventive Services Task Force (USPSTF), 45, 48
Urate oxidase, 349
Uridine diphosphate glucuronosyltransferase (UGT) 1A1, 84
U.S. Preventive Services Task Force (USPSTF), 44, 47
Uterine corpus cancers, 230, 236

V
Vaginal cancers, 228, 234
stage II, 228, 234
Vandetanib, 85
Varenicline (Chantix), side effects of, 51, 55
Vascular endothelial growth factor (VEGF), 19, 25–26
in CTC extravasation, 20, 27
H1F1α and, 19, 26
Vasohibin, 26
VEGF. See Vascular endothelial growth factor (VEGF)
Vemurafenib (PLX4032), 21, 28
Veterans Administration Laryngeal Study Group Trial, 126
VHL gene, 200, 208
Villous lymphocytes, 285, 290
Vincristine, doxorubicin, and cyclophosphamide (VAC), 268, 273
Viral oncogenes (v-oncogenes), 8
Vismodegib, 279, 281
Vitamin B12 and folic acid supplementation, for hematologic toxicities with pemetrexed, 80,
85
Von Hippel–Lindau disease, 200, 208
Von Hippel–Lindau (VHL) tumor suppressor gene, loss of function in, 85
Vorinostat, 87
Vulvar cancer
HPV-associated, 228, 235
invasive, 228, 229, 235
local recurrence of, 229, 235
lymph node involvement and patient’s ultimate survival, 229, 235
non-HPV-associated, 228, 235
repeat resection for recurrent cancers, 229, 235
staging for, 229, 235

W
Waldenstrom’s macroglobulinemia (WM), 324
Warburg effect, 18, 25
Well-differentiated squamous cell carcinoma, thymic carcinoma, 141, 146
WHIM syndrome (warts, hypogammaglobulinemia, infection, and myelokathexis), 324
WHI trial, 56
WM. See Waldenstrom’s macroglobulinemia (WM)
Women’s Health Initiative, 56, 57

X
Xerostomia, 122, 127
X-rays, effects of, 69, 74

Z
Zidovudine, and cytotoxic chemotherapy, 327, 330

Вам также может понравиться